Você está na página 1de 272

01

FÍSICA II
TERMOMETRIA

TERMOLOGIA PRINCIPAIS ESCALAS TERMOMÉTRICAS


A Termologia é a parte da física que estuda os fenômenos Existem diversas escalas termométricas com valores diferentes
relacionados com a temperatura. O calor é uma forma de energia para os pontos fixos. Abaixo veremos as escalas mais importantes:
que possui grande importância na tecnologia. Temperatura e
calor são grandezas extremamente interligadas, logo, o estudo de Escala Celsius (°C)
temperatura será fundamental para entender o estudo do calor. A escala Celsius, antes denominada de centígrada, é a escala
Serão estudados os cinco tópicos da termologia: termometria, oficial utilizada no Brasil. Seus valores são os que mais aparecem
calorimetria, dilatação, gases perfeitos e termodinâmica. Ao
Reprodução proibida Art. 184 do CP.

em problemas. Seus pontos fixos são 0 °C (ponto do gelo) e 100 °C


estudarmos esses tópicos, estaremos estudando a medição (ponto do vapor).
da temperatura, o calor e sua propagação, e outros fenômenos
térmicos. Escala Fahrenheit (°F)
É a escala utilizada em países de língua inglesa. Seus pontos
ENERGIA TÉRMICA, CALOR E EQUILÍBRIO do gelo e vapor são 32 °F e 212 °F, respectivamente.
TÉRMICO.
Energia térmica é o nome dado à energia associada à agitação • CURIOSIDADES
das moléculas de um corpo. Quando dois ou mais corpos de
1. Dizem que Fahrenheit também utilizou os valores
diferentes temperaturas entram em contato, por exemplo, há uma
“0” e “100” na sua escala: o Zero teria sido para o dia
transferência de energia do corpo de maior temperatura para o
mais frio de sua cidade naquele ano e o 100 para a
corpo de menor temperatura, isso é um processo espontâneo. Essa
temperatura da Sra. Fahrenheit.
energia em trânsito é chamada de CALOR.
2. Observe que qualquer que tenha sido o critério
A transferência de calor ocorre enquanto as temperaturas
de utilização dos valores 0 e 100, não há nisso
são diferentes e cessa no momento que se igualam, ou seja,
nenhuma fundamentação científica, e, mesmo
quando atingirem o EQUILÍBRIO TÉRMICO. No nosso dia a dia
assim, pode-se dizer que cada metade do povo da
o termo CALOR e TEMPERATURA são bastante utilizados, mas
Terra usa uma dessas duas escalas.
quase sempre de maneira não condizente com suas definições na
ciência. “Estou com calor”, “O fogo transferiu temperatura para a 3. Somente com Lord Kelvin, você verá a seguir, surge
panela”, são alguns exemplos da maneira errada de utilização. A uma escala com base científica: Kelvin deduz a
TEMPERATURA é a grandeza física que pode ser encarada como a menor temperatura do universo (- 273 oC) aí coloca o
medida do estado de agitação das moléculas que constituem um valor zero (Zero Absoluto): ausência de vibração nas
determinado corpo e o CALOR é uma energia em trânsito, portanto moléculas do corpo (a essa temperatura todos os
jamais estará contido em um corpo. átomos e moléculas parariam por completo) e usa a
mesma graduação da escala Celsius.
As diferentes unidades de temperatura decorrem das diferentes
Escalas Termométricas que ainda são utilizadas até hoje.

• TERMOMETRIA Escala Kelvin (K)


Esta escala, também conhecida como escala absoluta, não produz
Chamamos de Termometria a parte da Física responsável pelo leituras negativas. A unidade de temperatura do SI é o Kelvin (não se
estudo da medida e da variação da temperatura.
fala grau Kelvin), nela o ponto do gelo é 273 K e o ponto do vapor 373 K
(lembre-se que Kelvin usou a mesma graduação do Celsius).
PONTOS FIXOS A grande maioria dos problemas de termometria se refere às
Para podermos comparar as diversas escalas termométricas, conversões das temperaturas de uma escala para a outra. Iremos
existem certos estados térmicos com características específicas, nos basear nos pontos fixos e nas variações de temperatura.
denominados pontos fixos.
Existem dois pontos fixos tradicionais:
1o) ponto do gelo → temperatura de fusão do gelo;
2o) ponto do vapor → temperatura de ebulição da água.

Os pontos fixos são utilizados como parâmetros para


relacionarmos as diversas escalas termométricas. Os pontos fixos
considerados nas diversas escalas são obtidos sob pressão, ao
nível do mar, que é de 1 atm.
Acima estão representadas as escalas Celsius, Fahrenheit e Kelvin.
A temperatura é entendida como o grau de agitação das Utilizando proporções para as escalas Celsius, Fahrenheit teremos:
moléculas de um sistema, portanto o Lorde Kelvin usou o grau de
agitação nulo como o zero de sua escala. O zero kelvin é conhecido C−0 F − 32 C F − 32 C F − 32
como o zero absoluto, pois representaria uma situação na qual não = → = →=
100 − 0 212 − 32 100 180 5 9
há movimento de partículas num sistema.

PRÉ-VESTIBULAR PROENEM.COM.BR 177


FÍSICA II 01 TERMOMETRIA

Entre Celsius e Kelvin a conversão ficaria:

Quando a transformação é entre Fahrenheit e Kelvin, temos:


Reprodução proibida Art. 184 do CP.

Podemos observar as proporções entre as escalas estudando


suas relações graficamente, marcando os pontos fixos e utilizando
a semelhança de triângulos:
VARIAÇÃO ENTRE AS ESCALAS
Podemos observar que entre os pontos fixos das escalas
Celsius e Kelvin existem 100 unidades, portanto a variação de
temperatura que ocorrer em uma delas a outra também sofrerá a
mesma mudança. Isso não acontece com a escala Fahrenheit, pois
entre seus pontos fixos temos uma diferença de 180 graus, sendo
assim, a cada grau ou unidade que variamos nas escalas Celsius e
Kelvin, a escala Fahrenheit mudará em 1,8°.

EXERCÍCIOS
PROTREINO
01. O jogador de futebol Neymar vai visitar a Inglaterra, ao arrumar
suas malas pesquisa a temperatura que vai enfrentar durante sua
visita, num site informativo encontra algo em torno de 86°F.
Julgue se Neymar deve levar roupas de pesadas (casacos) ou
roupas leves (regatas e shorts). Justifique.

02. Um pesquisador do Proenem observa uma variação de 12°C


em um experimento de termodinâmica.
Calcule essa variação na escala Fahrenheit.

178 PROENEM.COM.BR PRÉ-VESTIBULAR


01 TERMOMETRIA FÍSICA II

03. Durante uma gravação nos estúdios do Proenem foi verificado d) a criança está com temperatura de 38,8 °C indicando estado
uma temperatura de 283 K. Aplique as relações de transformações febril.
entre as escalas e calcule o valor dessa temperatura, em graus Celsius. e) a indicação do termômetro traduz um quadro e hipotermia de
34,8 °C
04. Em uma confraternização no Rio Grande do Sul foi servido
churrasco, com batatas fritas, arroz, farofa de bacon e molho à 03. Antônio, um estudante de Física, deseja relacionar a escala
campanha. Celsius (°C) com a escala de seu nome (°A) Para isso, ele faz leituras
de duas temperaturas com termômetros graduados em °C e em
°A. Assim, ele monta o gráfico abaixo. Qual a relação termométrica
entre a temperatura da escala Antônio e da escala Celsius?
Reprodução proibida Art. 184 do CP.

No instante do corte da carne foi constatado que ela estava a 75°C,


C
a) A = C + 40 d) A = + 90
25 minutos depois foi constatado que ela estava 25°C. Calcule o 4
valor dessa variação de temperatura, em Kelvin.
C 10C
b) A = –100 e) A = – 40
05. Um universitário deseja construir sua própria escala de 2 9
temperatura, uma escala X. Foi considerado o ponto de fusão do
gelo a 0°X e o ponto de ebulição da água é de 60°X. Formule uma
relação de conversão entre a escala X e a escala Celsius. c) A = 2C – 80

04. A massa de ar frio polar continua influenciando o tempo em


Santa Catarina. Nesta segunda-feira (13), Urupema voltou a registrar
EXERCÍCIOS a temperatura mais baixa do ano até agora no estado [...]. Há uma

PROPOSTOS semana, a cidade da Serra amanhece com temperaturas negativas.


G1. Urupema volta a registrar a temperatura mais baixa do ano.
Disponível em: <http://g1.globo.com/sc/santa-catarina/noticia/2016/06/urupema-
volta-registrar-temperaturamais-baixa-do-ano-em-sc.html>.
01. Vários turistas frequentemente têm tido a oportunidade Acesso em: 13 jun. 2016 (Adaptado).
de viajar para países que utilizam a escala Fahrenheit como
referência para medidas da temperatura. Considerando-se Suponha que o termômetro utilizado na cidade de Urupema,
que quando um termômetro graduado na escala Fahrenheit no estado do Paraná, tenha sua escala termométrica de leitura
em Fahrenheit (°F) indicando uma temperatura de 15,8 °F. Se o
assinala 32 °F essa temperatura corresponde ao ponto termômetro possuísse escala termométrica em graus Celsius,
de gelo, e quando assinala 212 °F trata-se do ponto de quanto estaria marcando?
vapor. Em um desses países, um turista observou que um a) -7 °C. d) -10 °C.
termômetro assinalava temperatura de 74,3 °F. Assinale a
b) -8 °C. e) -9 °C.
alternativa que apresenta a temperatura, na escala Celsius,
correspondente à temperatura observada pelo turista. c) -29 °C.

a) 12,2 °C.
05. Ainda existem discordâncias sobre o local ideal para mensurar
b) 18,7 °C. a temperatura corporal. Pode ser axilar, bucal, timpânico,
c) 23,5 °C. esofágico, nasofaringeano, vesical e retal. Os locais habitualmente
d) 30 °C. mensurados são
e) 33,5 °C. • Axilar: temperatura normal encontra-se entre 35,5 a 37,0 °C
com média de 36,0 °C a 36,5 °C.
02. Um pediatra brasileiro está fazendo especialização médica em • Bucal: temperatura normal encontra-se entre 36 a 37,4 °C.
Londres. Ao medir a temperatura de uma criança com suspeita de • Retal: temperatura normal encontra-se entre 36 a 37,5 °C.
infecção, obtém em seu termômetro clínico a indicação 101,84 °F. Transformando esses valores para escala Kelvin, a temperatura
Sobre esta temperatura é correto afirmar que: normal, na região bucal, encontra-se entre:
Fonte: Disponível em: <http://fisiologia.med.up.pt/Textos_Apoio/outros/ Termorreg.pdf>.
a) este valor é preocupante, pois equivale a uma febre de 39,6 °C. Acesso em: 10 jun. 2014. (adaptado)
b) o valor correspondente é 37,8 °C não sendo considerado como
febre. a) 308,0 a 311,5 d) 309,0 a 310,5
c) este valor corresponde a 37 °C sendo a temperatura corpórea b) 308,5 a 310,0 e) 310,2 a 310,4
normal do ser humano. c) 309,0 a 310,4

PRÉ-VESTIBULAR PROENEM.COM.BR 179


FÍSICA II 01 TERMOMETRIA

06. Uma temperatura é tal que 18 (dezoito) vezes o seu valor na 11. (IFSUL 2017) Ao atender um paciente, um médico verifica
escala Celsius é igual a -10 (menos dez) vezes o seu valor na que, entre outros problemas, ele está com temperatura de
escala Fahrenheit. Determine essa temperatura. 37,5°C e deixa-o em observação no posto de saúde. Depois
a) 8 °F b) 16 °F c) 32 °F d) 64 °F e) 128 °F de uma hora, examina-o novamente, medindo a temperatura e
observa que ela aumentou 2°C.
07. No mapa abaixo, está representada a variação média da O valor dessa variação de temperatura, na escala Fahrenheit, e a
temperatura dos oceanos em um determinado mês do ano. Ao temperatura final, na escala Kelvin, são respectivamente iguais a
lado, encontra-se a escala, em graus Celsius, utilizada para a a) 3,6°F e 233,5 K. c) 35,6°F e 233,5 K.
elaboração do mapa. b) 35,6°F e 312,5 K. d) 3,6°F e 312,5 K.

12. (MACKENZIE 2019) SONHOS SOB CHAMAS


Reprodução proibida Art. 184 do CP.

Na madrugada da sexta feira do dia 08 de fevereiro de 2019, dez


sonhos deixaram de existir sob as chamas do Ninho do Urubu,
Determine, em Kelvins, o módulo da variação entre a maior e a
centro de treinamento do Clube de Regatas do Flamengo, no Rio
menor temperatura da escala apresentada.
de Janeiro. Eram adolescentes, aspirantes a craques de futebol,
a) 1,5 K b) 3,5 K c) 4,5 K d) 6 K e) 8 K que dormiam no alojamento do clube e foram surpreendidos pelas
chamas advindas do aparelho de ar condicionado que, em poucos
08. Um ramo importante da Física, ligado à Termologia é a criogenia, minutos, fizeram a temperatura local atingir valores insuportáveis
cuja finalidade é conseguir temperaturas extremamente baixas com ao ser humano. Essa temperatura na escala Celsius tem a sua
as mais diferentes aplicações. A obtenção de temperaturas reduzidas correspondente na escala Fahrenheit valendo o seu dobro,
é utilizada, por exemplo, na conservação de produtos alimentícios, adicionado de catorze unidades.
no transporte de gêneros perecíveis, na preservação de tecidos, Com bases nos dados fornecidos, é correto afirmar que o valor
dos componentes do sangue e de outras partes do corpo humano absoluto da temperatura citada vale
para posterior utilização. Em Biologia e Veterinária, a criogenia
a) 162 b) 194 c) 273 d) 363 e) 294
está associada à conservação do sêmen de animais para uso em
fertilização. A manutenção de sêmen bovino em temperatura da
13. (IFCE 2019) Um termômetro com defeito está graduado na
ordem de 73° kelvin é fundamental para preservar suas características,
escala Fahrenheit, indicando 30°F para o ponto de fusão do gelo e
a fim de que o processo de inseminação artificial tenha sucesso. (...).
214°F para o ponto de ebulição da água. A única temperatura neste
(Adaptado de FERRARO, N. G.; SOARES, P. A. de Toledo; FOGO. Ronaldo. Física Básica. 3.
ed. vol. único. São Paulo: Atual. 2009. p. 273).
termômetro medida corretamente na escala Celsius é
a) 158 b) 86 c) 122. d) 50. e) 194.
Com base nas informações apresentadas no texto acima, indique
qual o valor correspondente à temperatura de 73° kelvin nas 14. (IFCE 2019) Uma temperatura na escala Fahrenheit é expressa
escalas Celsius e Fahrenheit, respectivamente, para manutenção por um número que é o triplo do correspondente na escala Celsius.
de sêmen bovino. Essa temperatura na escala Fahrenheit é
a) -220 °C e - 380 °F d) -220 °C e 400 °F a) 20 b) 60 c) 40 d) 80 e) 100
b) -200 °C e 360 °F e) -240 °C e -420 °F
15. (IFSUL 2018) Num recipiente com água, dois termômetros
c) -200 °C e -328 °F
determinam, simultaneamente, a temperatura, sendo um deles
graduado em graus Fahrenheit e o outro em graus Celsius. A
09. Largamente utilizados na medicina, os termômetros clínicos diferença entre as leituras dos dois termômetros é 100,0.
de mercúrio relacionam o comprimento da coluna de mercúrio
com a temperatura. Sabendo-se que quando a coluna de mercúrio Com base nas informações fornecidas, é correto afirmar que a
atinge 2,0 cm, a temperatura equivale a 34 °C e, quando atinge 14 temperatura da água contida no recipiente, em graus Fahrenheit, é
cm, a temperatura equivale a 46 °C. Ao medir a temperatura de um a) 85,0 b) 185,0 c) 100,0 d) 180,0
paciente com esse termômetro, a coluna de mercúrio atingiu 8,0 cm.
A alternativa correta que apresenta a temperatura do paciente, em 16. Pernambuco registrou, em 2015, um recorde na temperatura
°C, nessa medição é após dezessete anos. O estado atingiu a média máxima de 31°C
segundo a Agência Pernambucana de Águas e Clima (APAC). A falta
a) 36 b) 42 c) 38 d) 40 e) 50
de chuvas desse ano só foi pior em 1998 – quando foi registrada
a pior seca dos últimos 50 anos, provocada pelo fenômeno “El
10. (ifce 2019) Qualquer pessoa pode construir sua própria escala Niño”, que reduziu a níveis críticos os reservatórios e impôs o
de temperaturas. Suponha que a escala Nunes seja construída racionamento de água. Novembro foi o mês mais quente de 2015,
levando em consideração os valores 7°N e 27°N para os pontos de aponta a APAC. Dos municípios que atingiram as temperaturas
fusão e ebulição da água, respectivamente. Se existir, a temperatura mais altas esse ano, Águas Belas, no Agreste, aparece em primeiro
coincidente na escala Nunes e Celsius será lugar com média máxima de 42°C.
a) 4,25 b) 8,75 c) 3,75 d) 2,25 e) 1,75 (Fonte: g1.com.br).

180 PROENEM.COM.BR PRÉ-VESTIBULAR


01 TERMOMETRIA FÍSICA II

Utilizando o quadro abaixo, que relaciona as temperaturas em °C Para operar, o Slide deve ter a sua estrutura metálica interna
(graus Celsius), °F (Fahrenheit) e K (Kelvin), podemos mostrar que as resfriada a temperaturas baixíssimas, alcançadas com nitrogênio
temperaturas médias máximas, expressas em K para Pernambuco líquido. Daí a “fumaça” que se vê nas imagens, que, na verdade, é o
e para Águas Belas, ambas em 2015, foram, respectivamente, nitrogênio vaporizando novamente devido à temperatura ambiente
e que, para permanecer no estado líquido, deve ser mantido a
aproximadamente -200 graus Celsius. Então, quando o nitrogênio
acaba, o skate para de “voar”.
Reprodução proibida Art. 184 do CP.

a) 300 e 317 d) 242 e 232


b) 273 e 373 e) 254 e 302
c) 304 e 315
Com relação ao texto, a temperatura do nitrogênio líquido, -200°C
17. Um estudante monta um dispositivo termométrico utilizando que resfria a estrutura metálica interna do Slide, quando convertida
uma câmara, contendo um gás, e um tubo capilar, em formato de para as escalas Fahrenheit e Kelvin, seria respectivamente:
“U”, cheio de mercúrio, conforme mostra a figura. O tubo é aberto
em uma das suas extremidades, que está em contato com a a) -328 e 73 c) -392 e -473
atmosfera. b) -392 e 73 d) -328 e -73

19. O uso do chuveiro elétrico representa uma parcela significativa


do gasto com energia elétrica em uma casa. Nos dias de maior
insolação, este gasto pode ser diminuído com o uso de aquecedores
solares de água.
Um modelo simples e de baixo custo, construído com garrafas
plásticas de refrigerante e caixas de leite, substitui com bastante
eficiência painéis solares produzidos industrialmente.
Observe a fotografia de um desses painéis em que sua capacidade
de aquecimento está sendo testada.

Inicialmente a câmara é imersa em um recipiente contendo água e gelo


em fusão, sendo a medida da altura h da coluna de mercúrio (figura)
de 2 cm. Em um segundo momento, a câmara é imersa em água em
ebulição e a medida da altura h da coluna de mercúrio passa a ser de
27 cm. O estudante, a partir dos dados obtidos, monta uma equação
que permite determinar a temperatura do gás no interior da câmara
(θ) em graus Celsius, a partir da altura h em centímetros. (Considere a
temperatura de fusão do gelo 0°C e a de ebulição da água 100°C).
Assinale a alternativa que apresenta a equação criada pelo
estudante. Nessa construção, dezoito canos de PVC, dispostos um ao lado
do outro, estão conectados em seus extremos por dois canos
a) θ=2h horizontais. Cada um dos dezoito canos é envolvido por garrafas
27h com o fundo cortado. Dentro de cada garrafa há uma peça obtida
b) θ= do corte de embalagens de leite, pintada na cor preto-fosco.
2
Desempenhando a função de reservatório de água, foi usada uma
c) θ=4h - 8 caixa de isopor.
d) θ=5h2 – 20 Os dois canos horizontais estão em alturas diferentes e ambos
se conectam ao reservatório de água. O cano horizontal superior
se conecta ao reservatório em um ponto mais acima do ponto de
18. O Slide, nome dado ao skate futurista, usa levitação magnética
conexão do cano horizontal inferior.
para se manter longe do chão e ainda ser capaz de carregar o peso
de uma pessoa. É o mesmo princípio utilizado, por exemplo, pelos A água preenche todo o sistema, que funciona automaticamente sem
trens ultrarrápidos japoneses. o auxílio de bombas hidráulicas, uma vez que a água na tubulação do
painel, aquecida pelo Sol, torna-se menos densa e sobe. Ao procurar
uma posição mais elevada, ela flui para o reservatório.

PRÉ-VESTIBULAR PROENEM.COM.BR 181


FÍSICA II 01 TERMOMETRIA

Na fase de testes deste painel, a água atingiu, em pouco mais de


GABARITO
uma hora, a temperatura de 45°C (muito alta para um banho). Como
o aquecedor de baixo custo funcionou surpreendentemente bem, EXERCÍCIOS PROPOSTOS
seu construtor desejou divulgar os resultados na internet, tendo o 01. C 05. C 09. D 13. D 17. C
cuidado de transcrever essa temperatura para a escala Fahrenheit, 02. D 06. B 10. B 14. D 18. A
com o intuito de que um internauta, acostumado com esta escala, 03. C 07. E 11. D 15. B 19. D
também fosse capaz de entender rapidamente a informação. 04. E 08. C 12. D 16. C 20. D
Desse modo, a temperatura que deverá ser divulgada na internet EXERCÍCIOS DE APROFUNDAMENTO
será, em graus Fahrenheit, aproximadamente, 01. 197,6°F 02. Não 03. 72°F 04. F = 16°F 05. K+F=249

Lembre-se que a equação de conversão entre as escalas Celsius ANOTAÇÕES

t t − 32
(tC) e Fahrenheit (tF) é dada por c = F
5 9

a) 9 d) 113
Reprodução proibida Art. 184 do CP.

b) 49 e) 240
c) 81

20. Durante uma corrida de Formula Indy ou de Fórmula 1, os


pilotos ficam sujeitos a um microambiente quente no cockpit que
chega a atingir 50°C gerado por diversas fontes de calor (do Sol, do
motor, do terreno, do metabolismo cerebral, da atividade muscular
etc.). Essa temperatura está muito acima da temperatura corporal
média tolerável, por isso, eles devem se manter sempre com bom
condicionamento físico.
As corridas de Fórmula Indy são mais tradicionais nos EUA, onde se
adota a leitura da temperatura na escala Fahrenheit.
Baseado nas informações apresentadas no texto, é correto afirmar
que a temperatura do cockpit que um carro de Fórmula Indy chega
a atingir durante a corrida, em grau Fahrenheit, é
Dados:
–– Temperatura de fusão do gelo = 32°F
–– Temperatura de ebulição da água = 212°F
a) 32. b) 50. c) 82. d) 122. e) 212.

EXERCÍCIOS DE
05. APROFUNDAMENTO
01. Um mecânico, medindo a temperatura de um dispositivo do
motor do carro de um turista americano, usou um termômetro
cuja leitura digital foi de 92°C. Para que o turista entendesse
melhor a temperatura, o mecânico teve de converter a unidade de
temperatura para Fahrenheit.
Qual foi o valor da temperatura após esta conversão?

02. Uma pessoa colocou um termômetro graduado na escala


Fahrenheit e verificou que sua temperatura era de 97,7 °F.
Considerando que a temperatura normal do corpo humano é de
36,5 °C, responda se esta pessoa estava com febre.

03. Um corpo sofre um aquecimento de 40°C. Se este aquecimento


fosse acompanhado pela escala fahrenheit qual seria a variação
nesta escala?

04. Uma temperatura é tal que 18 (dezoito) vezes o seu valor na


escala Celsius é igual a –10 (menos dez) vezes o seu valor na
escala Fahrenheit. Determine essa temperatura.

05. No interior de um freezer (congelador doméstico), a


temperatura se mantém a – 20ºC. Quanto valeria a soma algébrica
das indicações de dois termômetros graduados nas escalas
Fahrenheit e Kelvin, após o equilíbrio térmico ser estabelecido, se
ambos fossem colocados no interior desse congelador?

182 PROENEM.COM.BR PRÉ-VESTIBULAR


02
FÍSICA II
DILATAÇÃO TÉRMICA

DILATAÇÃO TÉRMICA A expressão citada anteriormente combinada com a expressão


∆L = L – Lo nos leva à equação L=L0(1+α∆θ), que nos permite
Sabemos que a temperatura de um corpo é a medida do calcular o comprimento final. Nossa experiência mostra que os
grau de agitação de suas moléculas. Quando a temperatura do alunos erram menos quando trabalham com as duas primeiras
corpo aumenta, essa agitação também aumenta e com isso o equações separadas.
espaçamento médio entre as moléculas tende a se torna maior.
Portanto, cresce o volume ocupado pelo corpo, o que chamamos
de DILATAÇÃO. É fácil concluir que se a temperatura diminuir, as
moléculas tendem a ficar mais próximas, diminuindo espaçamento
Reprodução proibida Art. 184 do CP.

entre elas e provocando o que chamamos de CONTRAÇÃO.


Existem substâncias, como por exemplo, a água, que demonstram
um comportamento anômalo em certas variações de temperatura,
mas abordaremos esse caso mais à frente.
Vale ressaltar que o volume do corpo sofre alteração com a
mudança de temperatura, mas a massa do corpo permanece
inalterada, logo, a densidade também passa a depender da
temperatura.
Volume e densidade são inversamente proporcionais.

m
↓d= Lâminas Bimetálicas
v↑
Uma lâmina bimetálica é construída a partir de duas lâminas
metálicas com coeficientes de dilatação diferentes. Na figura
Se um corpo sofre dilatação sua densidade diminui, se o corpo quando as lâminas são aquecidas, considerando a1 > a2, a lâmina do
sofre contração sua densidade aumenta. metal 1 sofre uma dilatação maior que a lâmina do metal 2, gerando
uma deflexão tomando a forma de um arco. No resfriamento
também ocorre uma deflexão, porém com uma curvatura contrária
DILATAÇÃO LINEAR da observada no aquecimento.
Dilatação (ou Contração) Térmica é aquela que ocorre devido
a uma variação na sua temperatura. É pouco usada a expressão
contração, mas quando se encontra uma dilatação negativa é
fato que o corpo contraiu. Quando esta dilatação se faz sensível
em apenas uma das dimensões do corpo dizemos que houve uma
Dilatação Linear.
Consideremos uma barra de comprimento inicial igual a L0 a uma
temperatura θ0. Se aquecermos esta barra a uma temperatura θ, ela
aumentará seu comprimento de L0 atingindo um comprimento L.
A variação de comprimento ΔL é proporcional ao comprimento
inicial L0 e à variação de temperatura

ΔL = L0 · α · Δθ DILATAÇÃO SUPERFICIAL
Dizemos que um corpo sofreu uma dilatação superficial
quando a dilatação se faz sensível em apenas duas dimensões. Na
A constante de proporcionalidade α que nos permite escrever figura abaixo, a placa possuía uma área igual a A0. Ao ser aquecida,
a equação acima é uma característica do material com o qual se sua dimensão passará a ser A.
está trabalhando e leva o nome de Coeficiente de Dilatação Linear.
A tabela abaixo mostra alguns valores de coeficientes de dilatação
linear. Verifique porque sua unidade é o oC-1.

Material (°C-1)

Chumbo 27.10-6
Alumínio 22.10-6 A expressão que calcula a variação da área da placa é bem
parecida com a expressão da dilatação linear. Podemos calcular a
Cobre 17.10 -6
variação da área pela fórmula:
Vidro comum 9.10-6
∆A=A0·β·∆θ

PRÉ-VESTIBULAR PROENEM.COM.BR 183


FÍSICA II 02 DILATAÇÃO TÉRMICA

Combinando com ∆A = A – A0 encontramos A=A0(1+β∆θ),


continuamos sugerindo que trabalhe apenas com as duas
primeiras equações.
A constante de proporcionalidade β que proporciona a primeira
equação é denominada Coeficiente de Dilatação Superficial, e é
possível mostrar que sua relação com β é: β=2α.
Reprodução proibida Art. 184 do CP.

Dilatação em cavidades
No aquecimento, orifícios encontrados em placas ou blocos
aumentarão de tamanho e, no resfriamento diminuirão de
tamanho. Tudo acontecendo como se a placa ou o bloco tivessem
os buracos preenchidos do mesmo material existente ao seu redor.
Veja o exemplo na imagem:

Extravasamento
Agora temos um líquido que preenche por completo um
recipiente e ambos são aquecidos. O líquido sofrerá uma dilatação
volumétrica e uma parte acabará sendo derramada para fora do
recipiente. Essa porção que extravasou é chamada de dilatação
aparente, pois para saber a dilatação real do líquido ainda
deveríamos levar em conta a dilatação do recipiente. Logo:

Volume inicial do líquido = Volume inicial do recipiente

Quando um objeto sofre uma dilatação térmica,


quaisquer buracos existentes no objeto também se dilatam
(a dilatação foi exagerada na imagem).

DILATAÇÃO VOLUMÉTRICA
Na dilatação volumétrica não podemos desprezar nenhuma Variação Percentual
dimensão, as três são relevantes fazendo com que o corpo aumente o Pode-se perguntar a variação percentual das dimensões de
seu volume. Ocorre por exemplo nos fluidos e sólidos como: líquidos, um corpo ao sofrer uma variação de temperatura, para responder
gases, esferas, cubos, etc. Agora a constante de proporcionalidade devemos usar as seguintes relações:
será o COEFICIENTE DE DILATAÇÃO VOLUMÉTRICO γ, onde γ=3α.

184 PROENEM.COM.BR PRÉ-VESTIBULAR


02 DILATAÇÃO TÉRMICA FÍSICA II

1) ∆L% =
∆L
⋅ 100
DILATAÇÃO ANÔMALA DA ÁGUA
L0 para variação percentual na Quando a água sofre um aquecimento de 0°C a 4°C ao invés
ou dilatação linear. de sofrer uma dilatação ela sofre uma contração, a partir de 4°C
o aquecimento volta a provocar dilatação. Com isso, a 4°C a água
∆L% = α ⋅ ∆θ ⋅ 100 possui sua maior densidade e isso foi fundamental para que os
lagos não congelassem em regiões de inverno rigoroso, mantendo
∆A a vida aquática por toda a estação. Se água não possuísse esse
2) ∆A% = ⋅ 100
A0 para variação percentual na comportamento diferente, talvez não teríamos vida subaquática
ou nas regiões próximas aos polos da Terra. Quando a temperatura
dilatação superficial.
da água atinge os 4°C ela se deposita no fundo do lago e na
∆A% = β ⋅ ∆θ ⋅ 100 parte superior se formam camadas cada vez mais espessas
de gelo, dificultando ainda mais a transferência de calor. Esse
∆V comportamento de contração, mesmo que apenas no intervalo de
3) ∆V% = ⋅ 100 0°C a 4°C é o que chamamos de comportamento anômalo da água.
V0 para variação percentual na
Reprodução proibida Art. 184 do CP.

ou dilatação volumétrica.
  ∆V% = γ ⋅ ∆θ ⋅ 100

Observe os exercícios resolvidos para entender as aplicações


de variação percentual em questões.

EXERCÍCIO RESOLVIDO

01. Uma barra sofre uma variação percentual de 0,4% de Isso é decorrente das ligações químicas na molécula da água,
comprimento ao sofrer um aquecimento de 200°C. Calcule o mas esse assunto deixaremos para os químicos por enquanto.
coeficiente de dilatação linear dessa barra.
A seguir o gráfico do volume ocupado pela temperatura:
Resolução:

∆L% = α ⋅ ∆θ ·100
0,4 = α ⋅ 200 ⋅ 100
0,4 = α ⋅ 20 000
0,4
α=
20 000
4 ⋅ 10−1
α= = 2 ⋅ 10−5
2 ⋅ 104
α = 2 ⋅ 10−5°C−1

EXERCÍCIOS
02. Uma placa metálica de 20,000 cm² sofre uma dilatação
de 0,004 cm² ao sofrer um aquecimento. Calcule a variação PROTREINO
percentual da área.
Resolução: 01. Uma barra de cobre de 20 cm é colocada num forno a 40°C.
Após o forno ser ligado ela chega a uma temperatura de 240°C.
∆A Calcule o comprimento da barra após o aquecimento. Adote:
∆A% = ⋅ 100
A0
αcobre= 17·10−6°C−1
0,004
∆A%
= ⋅ 100
20 02. Um cubo mágico fabricado para possuir medidas de 10 cm de
lado a 30°C, é levado para uma competição num dia de verão. No
0,4 interior do local, que está ocorrendo a competição de cubo mágico,
∆A% =
20 a temperatura medida é de 40°C. Calcule o volume do cubo, em
cm³, nesse ambiente. Considere que o cubo está em equilíbrio
∆A% =
0,02%
térmico com o ambiente e adote: αcubo= 7·10−5°C−1

03. Uma barra sofre uma variação percentual de 0,125% de


comprimento ao sofrer um aquecimento de 100°C. Calcule o
coeficiente de dilatação linear dessa barra.

PRÉ-VESTIBULAR PROENEM.COM.BR 185


FÍSICA II 02 DILATAÇÃO TÉRMICA

04. Uma mesa sofre um aquecimento conforme gráfico a seguir: a) 1000 c) 1050
b) 1025 d) 1075
17
Pressão (atm)
02. Uma esfera de aço tem volume de 1.000 cm³ em uma temperatura
16
A
15

de 20 °C. Este material possui um coeficiente de dilatação linear


14
13

médio de 1,2 x10-5C-1. A esfera é aquecida até 220 °C.


12
11
10
9
8 Nestas condições, a dilatação sofrida pela esfera após o
7
6 aquecimento, em cm³ é
5
4
3
B a) 3,6. b) 6,0. c) 4,8. d) 7,2. e) 2,4.
2
1
Volume (L)
0

-1
0 0 0 0 0 0 0 0
03. Num laboratório, um grupo de alunos registrou o comprimento
L de uma barra metálica, à medida que sua temperatura T
aumentava, obtendo o gráfico abaixo:
Reprodução proibida Art. 184 do CP.

Analisando as informações do gráfico, calcule:


a) A variação de área;
b) O coeficiente de dilatação superficial;
c) A variação percentual da área;

05. Num laboratório, um grupo de universitários registrou a área de


uma chapa, à medida que sua temperatura aumentava. Os dados
obtidos foram reunidos no gráfico a seguir:
Área (m²)

100,25
Pela análise do gráfico, o valor do coeficiente de dilatação do metal é
a) 1,05 · 10–5 ºC –1 d) 1,22 · 10–5 ºC –1
b) 1,14 · 10 –5
ºC –1
e) 1,25 · 10–5 ºC–1
c) 1,18 · 10 –5
ºC –1

04. (IFCE 2019) Em uma atividade de laboratório, um aluno do IFCE


dispõe dos materiais listados na tabela a seguir. Se o professor
100,00
pediu a ele que selecionasse, dentre as opções, aquele material que
possibilita maior dilatação volumétrica para uma mesma variação
(ºC) de temperatura e um mesmo volume inicial, a escolha correta seria
12 212

O professor ao observar o gráfico pediu aos seus alunos que, Coeficiente de dilatação linear (α)
Material
usando a tabela abaixo, identificassem o material que é feita a em °C-1
chapa. Aço 1,1·10-5

Coeficiente de Alumínio 2,4·10-5


Material
Dilatação Linear (°C-1) Chumbo 2,9·10-5
A 3,50.10-6 Cobre 1,7·10-5
B 6,25.10 -6
Zinco 2,6·10-5
C 12,50.10 -6
a) alumínio. d) cobre.
Aplicando seus conhecimentos sobre dilatação, determine o b) chumbo. e) zinco.
material que é feito a chapa. c) aço.

05. O piso de concreto de um corredor de ônibus é constituído


de secções de 20 m separadas por juntas de dilatação. Sabe-se
que o coeficiente de dilatação linear do concreto é 12 x 10–6°C–
EXERCÍCIOS 1, e que a variação de temperatura no local pode chegar a 50 °C
PROPOSTOS entre o inverno e o verão. Nessas condições, a variação máxima
de comprimento, em metros, de uma dessas secções, devido à
dilatação térmica, é
01. Um portão de chapa de ferro de 4 m de largura possui um vão
de 48 mm entre si e o batente a uma temperatura de 25 °C. Qual a) 1,0 · 10–2 d) 4,8 · 10–4
a temperatura máxima, em ºC que o portão pode atingir sem que b) 1,2 · 10–2 e) 6,0 · 10–4
fique enroscado no batente? c) 2,4 · 10 –4

Dado: coeficiente de dilatação linear do ferro igual a 12 · 10-6 ºC-1.

186 PROENEM.COM.BR PRÉ-VESTIBULAR


02 DILATAÇÃO TÉRMICA FÍSICA II

06. Uma chapa retangular, de lados 20 cm e 10 cm feita de 11. Um cientista está à procura de um material que tenha um
um material cujo coeficiente de dilatação linear é igual a coeficiente de dilatação alto. O objetivo dele é produzir vigas
22 · 10–6 °C–1 tem um furo no seu centro, cujo diâmetro é desse material para utilizá-las como suportes para os telhados
5 cm à 25 ºC. Se a chapa for aquecida até 125 °C afirma-se que a das casas. Assim, nos dias muito quentes, as vigas dilatar-se-iam
área do furo bastante, elevando o telhado e permitindo uma certa circulação
a) diminui e que o diâmetro passa a ser 4,985 cm. de ar pela casa, refrescando o ambiente. Nos dias frios, as vigas
encolheriam e o telhado abaixaria, não permitindo a circulação de
b) não se altera e que o diâmetro continua sendo 5,000 cm. ar. Após algumas experiências, ele obteve um composto com o
c) aumenta e que o diâmetro passa a ser 5,011 cm. qual fez uma barra. Em seguida, o cientista mediu o comprimento
d) diminui e que o diâmetro passa a ser 4,890 cm. L da barra em função da temperatura T e obteve o gráfico a seguir:
e) aumenta e o diâmetro passa a ser 5,022 cm.

07. Um cubo regular homogêneo de aresta 20,0 cm está


inicialmente a 20,0 °C. O coeficiente de dilatação linear médio do
material com que foi fabricado é 2,00 ·10–5 °C–1. Aquecendo-se
Reprodução proibida Art. 184 do CP.

uniformemente o cubo com uma fonte de calor constante durante


50,0 s a temperatura se eleva para 120,0 °C. A dilatação ocorrida
em uma das superfícies do cubo é:
a) 4,00 · 10–1 cm2 d) 16,00 · 10–1 cm2
b) 8,00 · 10–1 cm2 e) 20,00 · 10–1 cm2
c) 12,00 · 10–1 cm2
Analisando o gráfico, é correto afirmar que o coeficiente de
08. Uma placa de vidro possui as dimensões de
dilatação linear do material produzido pelo cientista vale:
1,0 m x 1,0 m x 1,0 cm quando está à temperatura ambiente. Seu
coeficiente de dilatação linear é 9 ·10–6 °C–1. Se a placa sofrer uma a) α = 2 · 10-5 °C-1. d) α = 5 · 10-5 °C-1.
variação de temperatura de 10 °C de quanto será a variação de b) α = 3 · 10 °C .
-3 -1
e) α = 6 · 10-4 °C-1.
volume da placa, em cm3 c) α = 4 · 10 °C .
-4 -1

a) 7,3 · 10–11 d) 9,0 · 10–1


b) 7,3 · 10–7 e) 2,7 12. Uma barra metálica de 1m de comprimento é submetida a um
c) 9,0 · 10 –3 processo de aquecimento e sofre uma variação de temperatura. O
gráfico abaixo representa a variação ∆l, em mm, no comprimento
da barra, em função da variação de temperatura ∆T em °C.
09. O comprimento ℓ de uma barra de latão varia, em função da
temperatura θ, segundo o gráfico a seguir.

Qual é o valor do coeficiente de dilatação térmica linear do material


de que é feita a barra, em unidades 10-6/°C?
Assim, o coeficiente de dilatação linear do latão, no intervalo de 0
a) 0,2 b) 2,0 c) 5,0 d) 20 e) 50
°C a 100 °C, vale:
a) 2,0 . 10–5 /°C d) 2,0 . 10–4 /°C 13. Seja um anel metálico construído com um fio muito fino. O
b) 5,0 . 10 /°C
–5
e) 5,0 . 10–4 /°C material tem coeficiente de dilatação linear e sofre uma variação
c) 1,0 . 10–4 /°C de temperatura ∆T. A razão entre o comprimento da circunferência
após o aquecimento e o comprimento inicial é
10. Um recipiente de vidro de capacidade 2,0.102 cm3 está a) α∆T
completamente cheio de mercúrio, a 0 °C. Os coeficientes de
dilatação volumétrica do vidro e do mercúrio são, respectivamente, 1
4,0 ·10-5 °C-1 e 1,8 ·10-4 °C-1. Aquecendo o conjunto a 100°C, o b)
(1+ α∆T )
volume de mercúrio que extravasa, em cm3, vale
a) 2,8 · 10–4 d) 2,8 · 10–1 1
c)
b) 2,8 · 10 α∆T
–3
e) 2,8
c) 2,8 · 10–2 d) 1+ α∆T

PRÉ-VESTIBULAR PROENEM.COM.BR 187


FÍSICA II 02 DILATAÇÃO TÉRMICA

14. Quanta energia deve ser dada a uma panela de ferro o gráfico concluímos que o nível de água no copo irá
de 300 g para que sua temperatura seja elevada em a) diminuir, se a temperatura do sistema diminuir.
100 °C? Considere o calor específico da panela como
c = 450 J/ kg °C. b) diminuir, independentemente de a temperatura do sistema
aumentar ou diminuir.
a) 300 J c) 750 J e) 13500 J
c) transbordar, independentemente de a temperatura do sistema
b) 450 J d) 1750 J aumentar ou diminuir.
d) transbordar, somente se a temperatura do sistema aumentar.
15. O piso de concreto de um corredor de ônibus é constituído de
secções de 20m separadas por juntas de dilatação. Sabe-se que o
coeficiente de dilatação linear do concreto é 12 x 10-6°C-1, e que a 18. (IFSUL 2018) Um aparelho eletrônico mal desenhado tem dois
variação de temperatura no local pode chegar a 50°C entre o inverno parafusos presos a partes diferentes que quase se tocam em seu
e o verão. Nessas condições, a variação máxima de comprimento, interior, como mostra a figura abaixo.
em metros, de uma dessas secções, devido à dilatação térmica, é
a) 1,0 · 10-2 d) 4,8 · 10-4
Reprodução proibida Art. 184 do CP.

b) 1,2 · 10 -2
e) 6,0 · 10-4
c) 2,4 · 10 -4

16. (MACKENZIE 2019) Desertos são locais com temperaturas


elevadas, extremamente áridos e de baixa umidade relativa do ar.
O deserto do Saara, por exemplo, apresenta uma elevada amplitude
térmica. Suas temperaturas podem ir de -10°C até 50°C ao longo
de um único dia. Os parafusos de aço e latão têm potenciais elétricos diferentes e,
caso se toquem, haverá um curto-circuito, danificando o aparelho.
O intervalo inicial entre as pontas dos parafusos é de 5µm a
27°C. Suponha que a distância entre as paredes do aparelho
não seja afetada pela mudança na temperatura. Considere,
para a resolução, os seguintes dados: αlatão =19 × 10−6 °C−1;
αaço = 11× 10−6 °C−1; 1µm = 10−6 m.
Nessas condições, a temperatura em que os parafusos se tocarão
é de
a) 34,0°C b) 32,0°C c) 34,4°C d) 7,4°C

19. (IFSUL 2018) Um estudante mede o comprimento de uma


haste de cobre com uma trena de aço a 20,0°C e encontra 95,0 cm.
Suponha que, ao realizar nova medida, a haste de cobre e a trena
estejam a uma temperatura de -15,0°C.
Dados:
Uma chapa de ferro, cujo coeficiente de dilação linear é igual
–– Coeficiente de dilatação superficial do cobre:
a 1,2·10-5°C-1 é aquecida sendo submetida a uma variação de
temperatura, que representa a amplitude térmica do deserto do −6 −1
34,0 × 10 °C ;
Saara, no exemplo dado anteriormente.
–– Coeficiente de dilatação volumétrica do aço:
Considerando sua área inicial igual a 5m2, o aumento de sua área,
−6 −1
em m2, é de 33,0 × 10 °C .
a) 2,0·10-6 c) 3,6·10-3 e) 3,6·10-6 Considerando as condições e os dados informados no enunciado,
b) 4,0·10-3 d) 7,2·10-3 o valor da medida encontrada pelo estudante será
a) 0,9503 m. c) 0,9497 m.
17. (IFSUL 2019) Um copo de vidro de 50 g de massa possui 100 b) 0,9301 m. d) 0,9603 m.
g de água que o preenche até a “boca”. O sistema encontra-se
inicialmente em equilíbrio térmico a uma temperatura de 4°C. O 20. (UEFS 2018) A figura representa duas barras metálicas, A e B, de
gráfico mostra como se comporta o volume do vidro e da água em espessura e largura desprezíveis, que apresentam, à temperatura
função da temperatura. inicial θ0, comprimentos iniciais L0 e 2·L0, respectivamente.

De acordo com o comportamento anômalo da água ou analisando Quando essas barras sofreram uma mesma variação de

188 PROENEM.COM.BR PRÉ-VESTIBULAR


02 DILATAÇÃO TÉRMICA FÍSICA II

temperatura ∆θ, devido à dilatação térmica, elas passaram a medir 03. (UFOP 2010) Um recipiente, cujo volume é exatamente 1.000
LA e LB. Sendo αA e αB os coeficientes de dilatação térmica linear de cm3, à temperatura de 20°C, está completamente cheio de glicerina
a essa temperatura. Quando o conjunto é aquecido até 100°C, são
A e B, se α A = 2 ⋅ αB , então entornados 38,0 cm3 de glicerina.
a) LB-LA<0 Dado: coeficiente de dilatação volumétrico da glicerina
b) LB-LA=LA = 0,5 x 10-3 ºC-1.
c) LB-LA=L0 Calcule:
d) LB-LA>L0 a) a dilatação real da glicerina;
e) LB-LA<L0 b) a dilatação do frasco;
c) o valor do coeficiente de dilatação volumétrica do recipiente.

EXERCÍCIOS DE 04. (UFPR 2019) A dilatação térmica linear sofrida por um objeto
em forma de barra feito de um dado material foi investigada por um
05. APROFUNDAMENTO estudante, que mediu o comprimento T. da barra em função de sua
Reprodução proibida Art. 184 do CP.

temperatura T. Os dados foram dispostos no gráfico apresentado


01. (UERJ 2016) Fenda na Ponte Rio-Niterói é uma junta de a seguir.
dilatação, diz CCR
De acordo com a CCR, no trecho sobre a Baía de Guanabara, as
fendas existem a cada 400 metros, com cerca de 13cm de abertura.
oglobo.com, 10/04/2014.

Admita que o material dos blocos que constituem a Ponte Rio-


Niterói seja o concreto, cujo coeficiente de dilatação linear é igual
a 1 x 10–5°C–1.
Determine a variação necessária de temperatura para que as duas Com base nos dados obtidos nesse gráfico, determine o
bordas de uma das fendas citadas na reportagem se unam. comprimento final Lf de uma barra feita do mesmo material
que a barra utilizada para a obtenção do gráfico acima, tendo
02. (UFJF-PISM 2 2015) O gráfico abaixo mostra o comprimento comprimento L0=3,00 m em T0=20°C, após sofrer uma variação
de temperatura de modo que sua temperatura final seja Tf=70°C.
de um bastão feito de um material desconhecido em função da
temperatura. A 0°C o comprimento inicial do bastão é 200mm. A
05. (UERJ 2010) A figura a seguir representa um retângulo formado
tabela ao lado mostra os coeficientes de dilatação linear de alguns
por quatro hastes fixas.
materiais.

Considere as seguintes informações sobre esse retângulo:


–– sua área é de 75 cm2 à temperatura de 20°C;
Coeficiente de
–– a razão entre os comprimentos l0A e l0B é igual a 3;
Material dilatação linear
(em °C–1) –– as hastes de comprimento l0A são constituídas de um
mesmo material, e as hastes de comprimento l0B de
Latão 20 x 10 –6
outro;
–– a relação entre os coeficientes de dilatação desses
Vidro comum 8 x 10–6
dois materiais equivale a 9.
Vidro pirex 5 x 10–6 Admitindo que o retângulo se transforma em um quadrado à
temperatura de 320°C, calcule, em °C-1, o valor do coeficiente de
Porcelana 3 x 10–6 dilatação linear do material que constitui as hastes menores.

Concreto 12 x 10–6

Com base nesses dados, responda o que se pede.


a) De que material o bastão é feito? Justifique sua resposta com
cálculos.
b) Qual é o comprimento do bastão a uma temperatura de 210°C?

PRÉ-VESTIBULAR PROENEM.COM.BR 189


FÍSICA II 02 DILATAÇÃO TÉRMICA

GABARITO
EXERCÍCIOS PROPOSTOS
01. B 05. B 09. A 13. D 17. C
02. D 06. C 10. E 14. E 18. C
03. E 07. D 11. E 15. B 19. C
04. B 08. E 12. D 16. D 20. C

EXERCÍCIOS DE APROFUNDAMENTO
01. 32,5°C

02.
a) 5 · 10-6 °C-1 b) L = 200,21 mm

03.
a) 40 cm³ b) 2 cm³ c) 2,5 · 10-5 °C-1

04. Lf = 3,03 m
Reprodução proibida Art. 184 do CP.

−2 −1
05. αB = 1× 10 °C

ANOTAÇÕES

190 PROENEM.COM.BR PRÉ-VESTIBULAR


03 CALORIMETRIA CONCEITOS E FÍSICA II

FONTES DE CALOR

O CALOR As correntes de convecção consistem na constante troca do ar


quente, que mais “leve” tende a subir, com o ar frio, mais “pesado”,
Já sabemos que a temperatura mede o grau de agitação das irá descer.
moléculas. Para que um corpo altere o grau de agitação de suas
O refrigerador utiliza as correntes de convecção para manter
moléculas, ou seja, altere sua temperatura, é necessário a troca
todo recipiente gelado.
de energia. A essa energia térmica que se propaga, chamamos de
calor. O refrigerador na parte superior resfria o ar da parte superior,
que fica mais “pesado” que o ar na parte inferior e dessa forma,
Calor é a energia térmica em trânsito.
ele desce e o ar quente (menos frio) mais “leve” sobe. Este ar ao
Reprodução proibida Art. 184 do CP.

• Quando um corpo cede calor, sua temperatura diminui, atingir as camadas superiores se esfria e torna a descer, ao mesmo
portanto, ele resfria. tempo, o ar que havia descido se aquece e volta a subir e isto se
• Quando um corpo recebe calor, sua temperatura aumenta, torna um ciclo. Exemplo: o ar condicionado resfriando um cômodo.
portanto, ele aquece.
Observe a imagem abaixo:

A unidade usual de quantidade de calor é a caloria: uma caloria


(1 cal) é a quantidade de calor necessária para elevar a temperatura
de 1g de água desde 14,5oC até 15,5oC sob pressão normal. No SI, a Observe que o ar condicionado instalado na parte superior do
energia é expressa em joule(J) e a relação entre elas é: ambiente favorece a convecção, visto que o ar frio tende a descer. Por
outro lado, o aquecedor deve ser instalado na parte inferior do ambiente
1 cal ≅4,18J para favorecer a convecção, visto que o ar quente tende a subir.

Brisas Marinhas
PROCESSOS DE PROPAGAÇÃO No litoral, durante o dia, a brisa sopra do mar para o litoral e,
O calor pode se propagar por três processos: condução,
à noite, do litoral para o mar. A areia tem calor específico muito
convecção e irradiação.
pequeno em relação ao da água (para massas iguais, a areia
necessita de menos energia para sofrer a mesma variação de
CONDUÇÃO temperatura), por este motivo se aquece e se resfria em um
Pelo processo da condução, o calor é transmitido de partícula intervalo de tempo menor.
para partícula. A partícula que recebe energia aumenta seu grau
de agitação e esse grau de agitação é passado para as partículas
vizinhas. Não há transporte de matéria. Exemplo: uma chama
aquecendo uma barra metálica.

Durante o dia, o ar quente próximo à areia sobe, criando uma


Convecção zona de baixa pressão em terra e alta pressão sobre o mar, isso
A convecção é uma forma de transmissão de calor que ocorre provoca o deslocamento do ar frio que se encontra sobre a água
em fluidos em geral e se baseia nas correntes de convecção com para a praia. Durante a noite, o ar que está sobre o solo resfria-
transmissão de matéria. se mais do que o ar que está sobre o mar criando uma zona de
O ar frio é mais “pesado”, ou seja, tem uma densidade maior, alta pressão em terra e baixa pressão sobre o mar, provocando o
logo em um recipiente contendo ar quente e frio, o ar frio ficará deslocamento do ar frio que se encontra sobre a areia para a água.
sempre na parte inferior.

PRÉ-VESTIBULAR PROENEM.COM.BR 191


FÍSICA II 03 CALORIMETRIA CONCEITOS E FONTES DE CALOR

Irradiação EQUAÇÃO FUNDAMENTAL DA


Também pode ser chamada de radiação e é o calor se CALORIMETRIA
propagando por meio de ondas eletromagnéticas.
Chamamos quantidade de calor sensível àquela quantidade
As ondas eletromagnéticas se diferenciam das ondas
de calor que provoca no corpo uma determinada variação de
mecânicas, pois não precisam de meio material para se propagarem.
Devido a isto, o calor pode se propagar do Sol até a Terra, pois como temperatura.
no espaço há vácuo, o calor não pode se propagar por condução e
nem por convecção, só restando a irradiação. A irradiação é o único Q
A combinação das equações C = e C = m·c nos leva à
processo pelo qual o calor pode se propagar no vácuo. ∆θ
equação:
Q = m · c · ∆θ
PROEXPLICA
Conhecida como Equação Fundamental da Calorimetria.
Garrafa Térmica ou Vaso de Dewar é um recipiente criado para Observe que ela permite calcular a quantidade de calor cedida
Reprodução proibida Art. 184 do CP.

manter a temperatura em seu interior o mais constante possível. ou recebida (Q), a quantidade de calor sensível, por certa quantidade
de massa (m) de uma substância, desde que ela tenha sofrido uma
variação de temperatura (∆θ).

CALOR LATENTE (L)


Experimentalmente observou-se que em determinadas
temperaturas existem certas quantidades de calor que o corpo
recebe ou cede e sua temperatura não varia, esta quantidade
de calor provoca mudança no estado físico da matéria e foi
denominada quantidade de calor latente. Empiricamente viu-se
Toda garrafa térmica possui características estruturais
que ela pode ser determinada pela expressão:
importantes para evitar as trocas de calor com o meio externo.
• As paredes internas são de vidro. Por ser mau Q=m·L
condutora dificulta as trocas por condução e
também são espelhadas a fim de refletir as ondas Onde Q é a quantidade de calor necessária para provocar a
eletromagnéticas e diminuir as trocas por radiação. mudança de estado físico (latente), m é a massa da substância e
• Entre as duas paredes de vidro, há uma região de L, característica da substância e da mudança de estado que ela irá
vácuo, dificultando ainda mais as trocas por condução. sofrer, denomina-se Calor Latente de Mudança de Estado.

• Para evitar as trocas por convecção, basta fechar a No caso da água:


garrafa, já que sendo a temperatura interna constante, • Calor latente de fusão do gelo: L f = 80 cal/g
não haverá surgimento de correntes. • Calor latente de solidificação da água: L s = –80 cal/g
• Calor latente de vaporização da água: L v = 540 cal/g
• Calor latente de liquefação do vapor de água: L l = –540 cal/g
CAPACIDADE TÉRMICA Observe que se um processo é o contrário do outro os valores
Define-se como capacidade térmica (C) de um corpo a razão
são iguais, mudando apenas o sinal, que é para ficar claro se a
entre a quantidade de calor recebida por ele e a variação de
temperatura causada por esta quantidade de calor. substância deve receber ou ceder calor para que o processo ocorra.

Quando se diz que a capacidade térmica de um corpo é 200cal/


o
C queremos dizer que para sua temperatura aumentar um grau MUDANÇAS DE ESTADO FÍSICO
Celsius ele deve receber 200 cal.
Toda substância possui três estados físicos: sólido, líquido e
Q gasoso, quando um corpo passa de um estado para outro, dizemos
C=
∆θ que ele realizou uma mudança de estado, e, para isto, ele deve
ceder ou receber uma certa quantidade de calor latente.
CALOR ESPECÍFICO
Investigando como a massa de uma mesma substância
interfere no aquecimento de uma substância, não foi difícil concluir
que para corpos de mesma substância a razão entre a capacidade
térmica do corpo e sua massa se mantinha sempre o mesmo valor,
que passou a ser denominado calor específico da substância que
constitui o corpo (c):

C
C = ou como cos tuma ser escrito C m ·c
m A mudança de estado físico de uma substância depende não
só da temperatura, mas também depende da pressão submetida.
Sua unidade mais usual é o cal/g C que deve ser interpretado
o

como sendo a quantidade de calor necessária para produzir uma Portanto, uma substância pura pode ser analisada através de seu
variação de 1 oC em 1 g da substância. diagrama de fases, um gráfico p x T.

192 PROENEM.COM.BR PRÉ-VESTIBULAR


03 CALORIMETRIA CONCEITOS E FONTES DE CALOR FÍSICA II

EXERCÍCIOS
PROTREINO
01. Determine a energia necessária para transformar 100 gramas
de gelo a -20°C em água a 50°C à pressão atmosférica.
cal 0,5cal
=
Dados: cágua líquida 1,0
=  ; cgelo =   ;L
  fusão 80 cal / g
g°C g°C
02. Utilizando os dados da questão anterior, faça a curva de
• Ponto T - ponto triplo, onde há a convivência dos três estados. aquecimento que ilustre a transformação de 100 gramas de gelo a
• Curva 1 - curva da fusão, pontos sobre a curva caracterizam -20°C em água a 50°C à pressão atmosférica.
a convivência dos estados líquido e sólido.
• Curva 2 - curva da valorização, pontos sobre a curva 03. Uma fonte térmica fornece 16000 cal a uma barra de 1 Kg,
caracterizam a convivência dos estados líquido e vapor. fazendo-a aquecer 80°C. Usando os dados da tabela, identifique
Reprodução proibida Art. 184 do CP.

• Curva 3 - curva da sublimação, pontos sobre a curva qual é o material que constitui a barra.
caracterizam a convivência dos estados sólido e vapor.
Material Calor específico (Cal/g°C)
Para a água, o diagrama de fases possui uma pequena
diferença, isso se dá por seu comportamento anômalo entra a A 0,2
temperatura de 0°C à 4ºC. O volume da água dilata quando sua
temperatura diminui nesse intervalo, com isso a curva que separa
B 0,4
os estados sólido e líquido é diferente quando comparada à curva C 0,6
de uma substância pura.
D 0,8

04. A panela de pressão é comumente usada para cozinhar mais


rapidamente alimentos que em recipientes abertos demorariam
muito para ficar prontos.
O esquema da panela de pressão e um diagrama de fase da água
são apresentados a seguir.

Na tabela abaixo temos algumas temperaturas de fusão e


ebulição.
Explique a vantagem do uso da panela de pressão no cozimento de
Substância Temperatura de Temperatura de alimentos e a que isto se deve.
Fusão Ebulição
05. O gráfico abaixo indica o comportamento térmico de 5g de uma
Água 0 ºC 100 ºC substância que, ao receber calor de uma fonte, passa integralmente
Álcool - 11,4 ºC 78 ºC da fase sólida para a fase líquida.
Mercúrio 39 ºC 357 ºC Calcule o calor específico da substância na fase sólida e o calor
latente de fusão dessa substância, em cal/g.
Zinco 420 ºC 907 ºC
Alumínio 660 ºC 2330 ºC

Se durante a mudança de estado físico a pressão permanece


constante, a temperatura permanece constante.
A seguir mostramos um gráfico de aquecimento da água,
desde o estado sólido até o estado gasoso.

PRÉ-VESTIBULAR PROENEM.COM.BR 193


FÍSICA II 03 CALORIMETRIA CONCEITOS E FONTES DE CALOR

07. Observe no diagrama as etapas de variação da temperatura e


de mudanças de estado físico de uma esfera sólida, em função do
calor por ela recebido. Admita que a esfera é constituída por um
EXERCÍCIOS metal puro.
PROPOSTOS
01. Você vai acampar por três dias e leva bujõezinhos de gás de
2 kg; o calor de combustão do GLP (gás liquefeito de petróleo) é
600 cal/g. Suponha que não haja perdas. Você utilizará o gás para
aquecer 10 L de água, desde 16 °C até 100 °C, por dia. O número de
bujões necessários será:
Dados: densidade da água = 1 kg/L:
a) 2 b) 1 c) 5 d) 4 e) 3
Reprodução proibida Art. 184 do CP.

02. Uma barra de alumínio de 400 g recebe 4 400 cal de uma fonte
de calor. Sabendo que a temperatura inicial do bloco é 20°C e que o
calor específico do alumínio é 0,22 cal/g · °C, podemos afirmar que Durante a etapa D, ocorre a seguinte mudança de estado físico:
a temperatura final da barra, em graus Celsius, será: a) fusão. d) vaporização.
a) 10 b) 40 c) 50 d) 70 e) 90 b) sublimação. e) solidificação.
c) condensação.
03. Para elevar a temperatura de 200 g de uma certa substância,
de calor específico igual a 0,6 cal/g°C, de 20 °C para 50 °C, será 08. Analise as proposições e indique a verdadeira.
necessário fornecer-lhe uma quantidade de energia igual a:
a) Calor e energia térmica são a mesma coisa, podendo sempre
a) 120 cal c) 900 cal e) 3600 cal ser usados tanto um termo quanto o outro, indiferentemente.
b) 600 cal d) 1800 cal b) Dois corpos estão em equilíbrio térmico quando possuem
quantidades iguais de energia térmica.
04. Uma mesma quantidade de calor Q é fornecida a massas c) O calor sempre flui da região de menor temperatura para a de
iguais de dois líquidos diferentes, 1 e 2. Durante o aquecimento, os maior temperatura.
líquidos não alteram seu estado físico e seus calores específicos
permanecem constantes, sendo tais que c1 = 5c2. Na situação d) Um corpo somente possui temperatura maior que a de um
acima, os líquidos 1 e 2 sofrem, respectivamente, variações de outro quando sua quantidade de energia térmica também é
temperatura ∆T1 e ∆T2, tais que DT1 é igual a: maior que a do outro.

a) ∆T2/5 d) 5 ∆T2/2 e) Calor é energia térmica em trânsito, fluindo espontaneamente


da região de maior temperatura para a de menor temperatura.
b) 2 ∆T2/5 e) 5 ∆T2
c) ∆T2 09. A energia necessária para aquecer uma certa massa de água é
a mesma nos seguintes casos:
05. A utilização do termômetro, para a avaliação da temperatura
a) 2 Kg, de 20 °C para 23 °C, ou de 3 Kg, de 20 °c para 23 °C
de um determinado corpo, é possível porque, após algum tempo de
contato entre eles, ambos adquirem a mesma temperatura. b) 1 Kg, de 20 °C para 21 °C, ou 2 Kg, de 20 °C para 22 °C
Neste caso, é válido dizer que eles atingem a (o): c) 2 Kg, de 20 °C para 23 °C, ou 3 Kg, de 20 °C para 22 °C
a) equilíbrio térmico. d) 1 kg, de 20 °C para 21 °C, ou 3 Kg, de 20 °C para 23 °C
b) ponto de condensação. e) 2 kg de 40 °C, para 42 °C, a 5 kg de 10 °C para 11 °C
c) coeficiente de dilatação máximo.
10. Qual a quantidade de calor que devemos fornecer a
d) mesma capacidade térmica.
200g de gelo que estão a -20 °C para transformar em água a
e) mesmo calor específico. 50 °C?
Considere: Cgelo = 0,5/(g . ºC); Cagua = 1cal/(g . ºC); Cfusao = 80cal/g
06. Em um estudo sobre fenômenos térmicos, foram avaliados
quatro objetos distintos, cujos valores de massa m de quantidade a) 28 Kcal. c) 16 Kcal. e) 18 Kcal.
de calor Q e de variação de temperatura ∆θ estão apresentados na b) 26 Kcal. d) 12 Kcal.
tabela abaixo.
11. Ana, em sua casa de praia, deseja ferver 2 litros de água numa
Objeto m (g) Q (cal) ∆θ(°c) chaleira de alumínio de 500 g, ambos na temperatura ambiente de
25°C. No entanto, seu botijão de gás natural possui apenas 1% da
I 20 100 10
sua capacidade total.
II 30 120 20 Considerando a perda de calor para o meio ambiente de 35%, a
III 60 150 10 quantidade de gás disponível é:
– Considere: Densidade da água = 1 g/cm3
IV 40 180 15
– Calor específico da água = 1,0 cal/g°C
Com base nesses dados, o objeto com o maior calor específico
– Calor específico do alumínio = 0,2 cal/g°C
está identificado pelo(s) seguinte(s) número(s):
– Capacidade total do botijão = 13 kg ou 31 litros
a) I. b) II. c) III. d) IV. e) I e IV
– Calor de combustão do gás natural = 12.000 kcal/kg

194 PROENEM.COM.BR PRÉ-VESTIBULAR


03 CALORIMETRIA CONCEITOS E FONTES DE CALOR FÍSICA II

a) Suficiente, afinal ela necessita de aproximadamente 10 gramas. b)


b) Suficiente, afinal ela necessita de aproximadamente 20 gramas.
c) Suficiente, afinal ela necessita de aproximadamente 30 gramas.
d) Insuficiente, já que ela precisa de 200 gramas.
e) Insuficiente, já que ela precisa de 300 gramas.

12. Para responder à questão, considere as informações e as


afirmativas sobre o gráfico a seguir.
c)
O gráfico abaixo representa a temperatura (T) em função da
quantidade de calor fornecido (Q) para uma substância pura de
massa igual a 0,1 kg inicialmente na fase sólida (trecho a).
Reprodução proibida Art. 184 do CP.

d)

I. A temperatura de fusão da substância é 30°C.


e)
II. O calor específico da substância na fase sólida é constante.
III. Ao longo de todo o trecho b, a substância encontra- se
integralmente na fase líquida.
Está/Estão correta(s) apenas a(s) afirmativa(s)
a) I. b) II. c) I e II. d) I e III. e) II e III.

13. Um recipiente de capacidade térmica desprezível contém 100


g de gelo à temperatura de -10,0°C. O conjunto é aquecido até a
14. Um bloco de gelo a -30°C repousa sobre uma superfície de
temperatura de +10°C através de uma fonte térmica que fornece
plástico com temperatura inicial de 21°C. Considere que esses
calor à razão constante de 1.000 cal/min.
dois objetos estejam isolados termicamente do ambiente, mas
que haja troca de energia térmica entre eles. Durante um intervalo
de tempo muito pequeno comparado ao tempo necessário para
que haja equilíbrio térmico entre as duas partes, pode-se afirmar
corretamente que
a) a superfície de plástico tem mais calor que o bloco de gelo e há
transferência de temperatura entre as partes.
b) a superfície de plástico tem menos calor que o bloco de gelo e
há transferência de temperatura entre as partes.
c) a superfície de plástico tem mais calor que o bloco de gelo e há
transferência de energia entre as partes.
d) a superfície de plástico transfere calor para o bloco de gelo e há
cal diferença de temperatura entre as partes.
Dados: calor específico do gelo  : cg = 0,50
g ⋅°C
cal 15. Um forno de micro-ondas produz ondas eletromagnéticas
calor específico da água: ca = 1,0
g ⋅°C que aquecem os alimentos colocados no seu interior ao provocar
cal a agitação e o atrito entre suas moléculas. Se colocarmos no
calor latente de fusão do gelo: Lf = 80
g interior do forno um copo com 250g de água a 15ºC, quanto
A temperatura do conjunto (θ) em função do tempo (t⁡) de tempo será necessário para aquecê-lo a 80ºC? Suponha que
aquecimento é melhor representado por as micro-ondas produzam 13000cal/min na água e despreze a
capacidade térmica do copo.
a) Dado: calor específico sensível da água: 1,0 cal/gºC.
a) 1,25 s
b) 25,0 s
c) 50,0 s
d) 75,0 s

PRÉ-VESTIBULAR PROENEM.COM.BR 195


FÍSICA II 03 CALORIMETRIA CONCEITOS E FONTES DE CALOR

16. (UEL 2019) Numa sala com temperatura de 18°C, estão 20. (FEEVALE 2017) Enquanto você está fazendo esta prova do
dispostos um objeto metálico e outro plástico, ambos com vestibular, está transferindo energia do seu corpo para o ambiente
a mesma temperatura desse ambiente. Um indivíduo com por meio da dissipação de calor. Essa dissipação poderá ocorrer
temperatura corporal média de 36°C segura esses objetos, um em por quais mecanismos de transporte?
cada mão, simultaneamente. Neste caso, é correto afirmar que há
a) Dissipação volumétrica, radiação e convecção.
rápida transferência de calor
b) Condução, convecção e dissipação fractal.
a) da mão para o objeto metálico e lenta da mão para o plástico, por
isso a sensação de frio maior proveniente do objeto metálico. c) Convecção, condução e radiação.
b) do objeto metálico para a mão e lenta do plástico para a mão, d) Radiação corpuscular, convecção e contração.
por isso a sensação de frio maior proveniente do plástico. e) Convecção, condução e capilarização.
c) da mão para o plástico e lenta da mão para o objeto metálico,
por isso a sensação de frio maior proveniente do plástico.
d) do plástico para a mão e lenta do objeto metálico para a mão, por EXERCÍCIOS DE
APROFUNDAMENTO
isso a sensação de calor maior proveniente do objeto metálico.
05.
Reprodução proibida Art. 184 do CP.

e) da mão para o plástico e lenta da mão para o objeto metálico, por


isso a sensação de calor maior proveniente do objeto metálico.
01. (UEL 2018) Messias está preparando um almoço e deseja gelar
10 latas da sua bebida preferida. Ele então as coloca dentro de uma
17. (ENEM PPL 2018) Duas jarras idênticas foram pintadas, uma de
caixa com isolamento térmico perfeito e sobre elas despeja gelo
branco e a outra de preto, e colocadas cheias de água na geladeira.
que está a uma temperatura de 0 ºC. Considerando que as trocas
No dia seguinte, com a água a 8°C, foram retiradas da geladeira
de calor se dão, única e exclusivamente, entre o gelo e as latas,
e foi medido o tempo decorrido para que a água, em cada uma
pode-se afirmar que o módulo do calor perdido pelas latas é igual
delas, atingisse a temperatura ambiente. Em seguida, a água das
ao módulo do calor recebido pelo gelo.
duas jarras foi aquecida até 90°C e novamente foi medido o tempo
decorrido para que a água nas jarras atingisse a temperatura Sabendo que a temperatura inicial das latas é de 20 ºC, que a
ambiente. capacidade térmica de cada lata é de 400 cal/ ºC e que o calor
latente de fusão do gelo é de 80cal/g, responda aos itens a seguir.
Qual jarra demorou menos tempo para chegar à temperatura
ambiente nessas duas situações? Determine a quantidade de calor extraído das latas até elas atingirem
a temperatura de 0 ºC . Justifique sua resposta, apresentando os
a) A jarra preta demorou menos tempo nas duas situações
cálculos envolvidos na resolução deste item.
b) A jarra branca demorou menos tempo nas duas situações.
Calcule a massa de gelo necessária para baixar a temperatura das
c) As jarras demoraram o mesmo tempo, já que são feitas do latas para 0 ºC. Justifique sua resposta, apresentando os cálculos
mesmo material. envolvidos na resolução deste item.
d) A jarra preta demorou menos tempo na primeira situação e a
branca, na segunda. 02. (UERJ 2015) Um corpo de massa igual a 500g, aquecido por
e) A jarra branca demorou menos tempo na primeira situação e a uma fonte térmica cuja potência é constante e igual a 100cal/min,
preta, na segunda. absorve integralmente toda a energia fornecida por essa fonte.
Observe no gráfico a variação de temperatura do corpo em função
do tempo.
18. (UFJF-PISM 2 2017) A garrafa térmica de uma determinada
marca foi construída de forma a diminuir as trocas de calor com
o ambiente que podem ocorrer por três processos: condução,
convecção e radiação. Dentre as suas várias características,
podemos citar:
I. a ampola interna da garrafa é feita de plástico.
II. a ampola possui paredes duplas, e entre essas paredes, é feito
vácuo.
III. a superfície interna da ampola é espelhada.
Assinale a alternativa que corresponde ao processo que se quer
evitar usando as características citadas acima.
a) I – radiação; II – condução e convecção; III – convecção. Calcule o calor específico da substância da qual o corpo é
b) I – condução e radiação; II – convecção; III – condução. composto, bem como a capacidade térmica desse corpo.
c) I – convecção; II – condução; III – radiação.
03. (UERJ 2014) A energia consumida por uma pessoa adulta em
d) I – condução; II – condução e convecção; III – radiação.
um dia é igual a 2 400 kcal.
e) I – radiação; II – condução e convecção; III – radiação.
Determine a massa de gelo a 0°C que pode ser totalmente liquefeita
19. (UECE 2017) Considere o enunciado de uma lei da pela quantidade de energia consumida em um dia por um adulto.
termodinâmica, que diz “se dois corpos estiverem em equilíbrio Em seguida, calcule a energia necessária para elevar a temperatura
térmico com um terceiro, estarão em equilíbrio térmico entre si”. dessa massa de água até 30°C.
Assim, é correto afirmar que no equilíbrio térmico
a) os três corpos devem estar em temperaturas distintas. 04. (UFPE 2013) O calor necessário para fundir uma certa massa
b) não há fluxo de calor entre os três corpos. de uma substância é igual ao calor necessário para aumentar em
c) os três corpos necessariamente têm a mesma energia interna. 30 K a temperatura da mesma massa da substância multiplicado
por uma constante A. Se A=2,5, quanto vale a razão Lf/c, em K, entre
d) há sempre fluxo de calor entre os três corpos.
o calor de fusão Lf e o calor específico c desta substância?

196 PROENEM.COM.BR PRÉ-VESTIBULAR


03 CALORIMETRIA CONCEITOS E FONTES DE CALOR FÍSICA II

05. (UFPE 2012)


Dados:
Aceleração da gravidade: 10m/s2
Densidade do mercúrio: 13,6 g/cm3
Pressão atmosférica: 1,0.105. N/m2
Constante eletrostática: K0=1/4πo 9,0·109N·M2/C2
O gálio (Ga) é um metal cuja temperatura de fusão, à pressão
atmosférica, é aproximadamente igual a 30 °C. O calor específico
médio do Ga na fase sólida é em torno de 0,4 kJ/(kg.ºC) e o calor
latente de fusão é 80 kJ/kg. Utilizando uma fonte térmica de 100
W, um estudante determina a energia necessária para fundir
completamente 100 g de Ga, a partir de 0ºC. O gráfico mostra a
variação da temperatura em função do tempo das medições
realizadas pelo estudante. Determine o tempo total tT que o
Reprodução proibida Art. 184 do CP.

estudante levou para realizar o experimento. Suponha que todo o


calor fornecido pela fonte é absorvido pela amostra de Ga. Dê a sua
resposta em segundos.

GABARITO

EXERCÍCIOS PROPOSTOS
01. E 05. A 09. C 13. A 17. A
02. D 06. A 10. A 14. D 18. D
03. E 07. D 11. B 15. D 19. B
04. A 08. E 12. A 16. A 20. C

EXERCÍCIOS DE APROFUNDAMENTO
01.
a) 8000 cal b) m = 1000 g

02.
a) c = 0,15 cal/g°C b) C = 75 cal/°C

03. m = 30 Kg e Q = 9000 cal

04. 75 K

05. 92 segundos

ANOTAÇÕES

PRÉ-VESTIBULAR PROENEM.COM.BR 197


FÍSICA II 03 CALORIMETRIA CONCEITOS E FONTES DE CALOR

ANOTAÇÕES
Reprodução proibida Art. 184 do CP.

198 PROENEM.COM.BR PRÉ-VESTIBULAR


04 CALORIMETRIA EQUILÍBRIO FÍSICA II

TÉRMICO

SISTEMA TERMICAMENTE ISOLADO 02. Muitas pessoas gostam de café, mas não o apreciam
Dizemos que um sistema é termicamente isolado quando muito quente e têm o hábito de adicionar um pequeno cubo
este não troca calor com o meio externo. Na prática, não existem de gelo para resfriá-lo rapidamente. Deve-se considerar que
sistemas perfeitamente isolados (calorímetro ideal), porém, há os a xícara tem capacidade térmica igual a 30 cal/ºC e contém
calorímetros que reduzem bastante as trocas de calor com meio inicialmente 120 g de café (cujo calor específico é igual ao
ambiente, e, para nosso nível, a não ser que se diga ao contrário, ele da água, 1 cal/g.ºC) a 100 ºC, e que essa xícara encontra-se
será considerado ideal. em equilíbrio térmico com o líquido. Acrescentando-se uma
O sistema termicamente isolado é denominado adiabático. pedra de gelo de 10 g, inicialmente a 0 ºC, sendo que o calor
Reprodução proibida Art. 184 do CP.

Se em um calorímetro colocarmos duas substâncias com latente de fusão do gelo vale 80 cal/g, após o gelo derreter e
temperaturas diferentes θ1 e θ2 haverá trocas de calor até que as todo o sistema entrar em equilíbrio térmico, desprezando-se
temperaturas dos dois corpos se tornem iguais. Dizemos, nesse caso, as perdas de calor para o ambiente, a temperatura do café
que eles atingiram o Equilíbrio Térmico. será igual a
O corpo que tiver temperatura maior cede calor para o corpo que a) 86,15 ºC. c) 93,75 ºC.
tiver temperatura menor; quem cede calor, sua temperatura abaixa b) 88,75 ºC. d) 95,35 ºC.
e quem recebe calor, sua temperatura aumenta, até que seja atingido
o equilíbrio térmico, quando cessa a troca de calor entre os corpos. Resolução:
Analisando os sinais na Equação Fundamental da calorimetria,
concluímos que a quantidade de calor cedida terá sinal de menos Dados: Cxícara = 10 cal/°C; mcafé = 120 g; mgelo = 10 g;
enquanto que a quantidade de calor recebida terá sinal de mais, porém Lgelo = 10 cal/g; cágua = 1 cal/g°C.
serão iguais em módulo. Daí o Princípio Geral das Trocas de Calor:
O calor liberado pelo café e pela xícara deve derreter o gelo
Quando dois ou mais corpos trocam calor entre si, em um e esquentar a água do gelo até a temperatura de equilíbrio.
sistema termicamente isolado, o somatório das quantidades Sendo um sistema termicamente isolado, temos:
de calor trocadas até que se atinja o equilíbrio térmico é nulo.
Q xícara + Q café + Q fusão + Q água =
0⇒
Em termos Matemáticos: ∑ QCedido + ∑ QRecebido =0 Cxícara (T − 100) + mcafé cágua (T − 100) + mgelo Lfusão + mgelocágua (T − 0) =
0⇒
30(T − 100) + 120 × 1× (T − 100) + 10(80) + 10 × 1× (T) = 0 ⇒
Notação
3T − 300 + 12T − 1200 + 80 + T =⇒
0 16T =
1420 ⇒ T =
88,75°C
Calor cedido Q < 0
Calor recebido Q > 0 Letra B.

Vejamos alguns exemplos:

EXERCÍCIOS
EXERCÍCIO RESOLVIDO
PROTREINO
01. (PUCRJ 2010) Um cubo de gelo dentro de um copo com
água resfria o seu conteúdo. Se o cubo tem 10 g e o copo com 01. Para as questões abaixo, adote:vv
água tem 200 ml e suas respectivas temperaturas iniciais são
0 °C e 24 °C, quantos cubos de gelo devem ser colocados para Calor específico da água 1,0 cal/g°C
baixar a temperatura da água para 20 °C? (Considere que o
calor específico da água é ca = 1,0 cal/(g °C), o calor latente de Calor específico do gelo 0,5 cal/g°C
fusão do gelo L = 80 cal/g, e a densidade da água, d = 1 g/ml). Calor latente de fusão 80 cal/g
a) 1 b) 2 c) 3 d) 4 e) 5 Calor latente de vaporização 540 cal/g
Resolução:
Em um calorímetro ideal mistura-se 200 gramas de água a 40°C
Dados: mcubo = 10 g; Lgelo = 80 cal/g; mág = 200 g;
com 200 gramas de água a 60°C. Calcule a temperatura do sistema
T0 = 24°C; T = 20°C; cág = 1 cal/g.°C.
após atingir o equilíbrio térmico.
Módulo da quantidade calor liberada pela água para o
resfriamento desejado: 02. Em um calorímetro ideal mistura-se 400 gramas de água a
|Qág| = mág cág |∆T| = 200 (1) |20 – 24| = 800 cal. 20°C com 600 gramas de água a 80°C. Calcule a temperatura do
Quantidade de calor necessária para fundir um cubo de sistema após atingir o equilíbrio térmico.
gelo:
Qcubo = mcubo Lgelo = 10 (80) = 800 cal. 03. Um boco metálico com 100 g a 224°C é introduzido num
calorímetro de capacidade térmica desprezível que contém 400g
Como |Qág| = Qcubo, concluímos que basta um cubo de gelo de água a 20°C. Calcule o calor específico do metal que constitui o
para provocar o resfriamento desejado da água. bloco, sabendo que o equilíbrio térmico se estabelece a 24°C.
Letra A.

PRÉ-VESTIBULAR PROENEM.COM.BR 199


FÍSICA II 04 CALORIMETRIA EQUILÍBRIO TÉRMICO

04. Considere duas amostras, X e Y, de materiais distintos, sendo a Dados da água:


massa de X igual a quatro vezes a massa de Y. –– Calor específico no estado sólido: 0,5 cal/g.°C
As amostras foram colocadas em um calorímetro e, após o sistema –– Calor específico no estado líquido: 1,0 cal/g.°C
atingir o equilíbrio térmico, determinou-se que a capacidade –– Calor latente de fusão do gelo: 80 cal/g
térmica de X corresponde ao dobro da capacidade térmica de Y.
–– Adote 1 cal = 4 J
Admita que CX e CY sejam os calores específicos, respectivamente,
de X e Y. a) 4 b) 8 c) 10 d) 80 e) 120

Determine a razão cX . 04. Um estudante irá realizar um experimento de física e precisará


cY de 500 g de água a 0 °C. Acontece que ele tem disponível somente
05. Mistura-se 200 gramas de água a 60°C com 100 gramas de um bloco de gelo de massa igual a 500 g e terá que transformá-lo
gelo a -20°C em um calorímetro ideal. Calcule a temperatura após em água. Considerando o sistema isolado, a quantidade de calor,
o sistema atingir o equilíbrio térmico. em cal, necessária para que o gelo derreta será:
Dados: calor de fusão do gelo = 80 cal/g.°C
Reprodução proibida Art. 184 do CP.

a) 40 c) 4.000 e) 400.000
b) 400 d) 40.000
EXERCÍCIOS
PROPOSTOS 05. O gráfico abaixo indica o comportamento térmico de 10 g
de uma substância que, ao receber calor de uma fonte, passa
integralmente da fase sólida para a fase líquida.
01. Sabe-se que um líquido possui calor específico igual a 0,58 cal/g.°C.
Com o intuito de descobrir o valor de seu calor latente de vaporização,
foi realizado um experimento onde o líquido foi aquecido por meio de
uma fonte de potência uniforme, até sua total vaporização, obtendo-
se o gráfico abaixo. O valor obtido para o calor latente de vaporização
do líquido, em cal/g está mais próximo de:

O calor latente de fusão dessa substância, em cal/g, é igual a:


a) 70
b) 80
c) 90
d) 100
a) 100 e) 110
b) 200
c) 540 06. Um corpo A homogêneo, de massa 200g varia sua temperatura
de 20 °C para 50 ºC ao receber 1200 calorias de uma fonte térmica.
d) 780
Durante todo o aquecimento, o corpo A se mantém na fase sólida.
e) 1000 Um outro corpo B homogêneo, constituído da mesma substância
do corpo A tem o dobro da sua massa. Qual é, em cal/gºC o calor
02. Um buffet foi contratado para servir 100 convidados em específico da substância de B?
um evento. Dentre os itens do cardápio constava água a 10 °C. a) 0,1 b) 0,2 c) 0,6 d) 0,8 e) 1,6
Sabendo que o buffet tinha em seu estoque 30 litros de água a
25 °C, determine a quantidade de gelo, em quilogramas, a 0 °C
07. Dois blocos A e B ambos do mesmo material, são colocados
necessário para obter água à temperatura de 10 °C.
em contato no interior de um calorímetro ideal para que estejam
Considere que a água e o gelo estão em um sistema isolado. isolados de influências externas. Considerando o calor específico
Dados: do material (c) bem como considerando que a massa do bloco
B(mB) é igual ao dobro da massa do bloco A(mA), que a temperatura
–– densidade da água = 1g/cm³
inicial do bloco B(TB) é igual ao triplo da temperatura inicial do bloco
–– calor específico da água = 1 cal/g.°C A(TA) e que TA = 60 ºC pode-se afirmar que, quando alcançado o
–– calor de fusão do gelo = 80 cal/g.°C equilíbrio térmico do sistema, a temperatura de equilíbrio (Teq) será
–– calor específico do gelo = 0,5 cal/g.°C igual a:
a) 2 b) 3 c) 4 d) 5 e) 6 a) 420 ºC b) 60 ºC c) 180 ºC d) 140 ºC e) 120 ºC

03. Um chefe de cozinha precisa transformar 10 g de gelo a 08. Um objeto de metal de 2,0 kg a 90 ºC é submerso em 1,0 kg de
0 °C em água a 40 °C em 10 minutos. Para isto utiliza uma água (calor específico 1,0 cal/g. ºC) a 20 ºC. O sistema água-metal
resistência elétrica percorrida por uma corrente elétrica que atinge o equilíbrio térmico a 32 ºC. Nessas condições, afirma-se
fornecerá calor para o gelo. Supondo-se que todo calor fornecido que o calor específico do metal é: (Dado: 1 cal = 4,2J).
pela resistência seja absorvido pelo gelo e desprezando-se perdas a) 0,840 kJ/kg.K. d) 0,433 kJ/kg.K.
de calor para o meio ambiente e para o frasco que contém o gelo, b) 0,129 kJ/kg.K. e) 0,300 kJ/kg.K.
a potência desta resistência deve ser, em watts, no mínimo, igual a:
c) 0,512 k/J/kg.K.

200 PROENEM.COM.BR PRÉ-VESTIBULAR


04 CALORIMETRIA EQUILÍBRIO TÉRMICO FÍSICA II

09. Um Físico acorda pela manhã em um dia muito frio e vai logo 14. Clarice colocou em uma xícara 50 mL de café a 80 °C, 100 mL
preparar seu café. Para tanto, ele utiliza uma xícara de alumínio que de leite a 50 °C e, para cuidar de sua forma física, adoçou com 2 mL
tem massa igual a 200,0 g e está a uma temperatura igual a 5,0 ºC. de adoçante líquido a 20 °C. Sabe-se que o calor específico do café
Ele coloca dentro dessa xícara 300,0 g de café inicialmente a 90,0 vale 1 cal/(g.°C), do leite vale 0,9 cal/(g.°C), do adoçante vale 2 cal/
ºC. Considerando as trocas de calor apenas entre a xícara e o café e (g.°C) e que a capacidade térmica da xícara é desprezível.
sabendo-se que o calor específico do alumínio é 0,2 cal/g.ºC e que
o calor específico do café é 1,0 cal/g.ºC, qual é a temperatura final
do conjunto (xícara e café) ao atingir o equilíbrio térmico?
a) 85,0 ºC c) 75,0 ºC e) 65,0 ºC
b) 80,0 ºC d) 70,0 ºC​​​​​​

10. A perspectiva de uma pessoa que usa uma garrafa térmica é que
esta não permita a troca de calor entre o meio ambiente e o conteúdo
da garrafa. Porém, em geral, a própria garrafa já provoca uma pequena
redução de temperatura quando nela colocamos um líquido quente,
Reprodução proibida Art. 184 do CP.

como o café, uma vez que a capacidade térmica da garrafa não é nula.
Numa garrafa térmica que está a 24 ºC colocam-se 500 g de
água (c=1cal/g ºC) a 90 ºC e, após algum tempo, nota- se que a
Considerando que as densidades do leite, do café e do adoçante
temperatura estabiliza em 84 ºC. Pode-se afirmar que a capacidade
sejam iguais e que a perda de calor para a atmosfera é desprezível,
térmica desta garrafa é, em cal/ ºC.
depois de atingido o equilíbrio térmico, a temperatura final da
a) 5 b) 6 c) 50 d) 60 e) 100 bebida de Clarice, em °C, estava entre
1 a) 75,0 e 85,0. d) 45,0 e 54,9.
11. O conteúdo de uma garrafa térmica tem de seu volume
2 3 b) 65,0 e 74,9. e) 35,0 e 44,9.
preenchido com água à temperatura T0 e preenchido com água
T 3 c) 55,0 e 64,9.
à temperatura 0 . A temperatura de equilíbrio, em função de T0, é
2
T 15. Dona Joana é cozinheira e precisa de água a 80 ºC para sua
a) 0
2 receita. Como não tem um termômetro, decide misturar água fria,
T0 que obtém de seu filtro, a 25 ºC, com água fervente. Só não sabe
b) em que proporção deve fazer a mistura. Resolve, então, pedir ajuda
2
a seu filho, um excelente aluno em física. Após alguns cálculos,
T0 em que levou em conta o fato de morarem no litoral, e em que
c)
2 desprezou todas as possíveis perdas de calor, ele orienta sua mãe a
2T0 misturar um copo de 200 mL de água do filtro com uma quantidade
d) de água fervente, em mL, igual a
3
a) 800.
e) T0
b) 750.
12. Em um calorímetro são colocados 100 g de gelo a 0°C e 200 g c) 625.
de água a 40°C. d) 600.
Calcule, em °C, a temperatura final do sistema, supondo o e) 550.
calorímetro perfeitamente isolado.
16. (MACKENZIE 2019) Nas engenharias metalúrgica, mecânica e
Dados: Cágua 1,00
= =
cal
; Lgelo 80
cal de materiais, o processo de têmpera é muito utilizado para conferir
g °C g dureza aos materiais. Esse processo consiste em submeter o
material a um resfriamento brusco após aquecê-lo acima de
a) 40 b) 20 c) 10 d) 5 e) 0 determinadas temperaturas. Isso causa o surgimento de tensões
residuais internas, provocando um aumento da dureza e resistência
13. Uma quantidade de 750 mL de água a 90°C é paulatinamente do material.
resfriada até chegar ao equilíbrio térmico com o reservatório que
Nos laboratórios da Universidade Presbiteriana Mackenzie um
a contém, cedendo um total de 130 kcal para esse reservatório.
aluno deseja realizar a têmpera de uma barra de ferro, cuja massa
Sobre a água ao fim do processo, é correto afirmar que
vale 1000 g. A peça é então colocada em um forno de recozimento
1,0 cal durante o tempo suficiente para que ocorra o equilíbrio térmico.
Considere: calor específico da água líquida cágua =
g °C Em seguida é retirada e rapidamente imersa em um tanque com
0,55 cal 10.000 g de óleo, cujo calor específico sensível vale 0,40 cal/g°C.
–– calor específico do gelo cgelo =
g °C Sabendo-se que o calor específico sensível do ferro tem valor
80 cal aproximado de 0,11 cal/g°C, e que a temperatura do óleo muda
–– calor latente de solidificação da água CL = de 28°C para 38°C, a temperatura do forno no momento em que a
g
1,0 g barra é retirada vale aproximadamente, em °C
–– densidade da água líquida ρágua =
mL a) 100
a) a água se encontra inteiramente em forma de gelo.
b) 200
b) a água se encontra a uma temperatura de 0 °C.
c) a água se encontra inteiramente em estado líquido. c) 300

d) a temperatura final da água é de 4 °C. d) 400


e) há uma mistura de gelo e água líquida. e) 500

PRÉ-VESTIBULAR PROENEM.COM.BR 201


FÍSICA II 04 CALORIMETRIA EQUILÍBRIO TÉRMICO

17. (MACKENZIE 2019) Anelise lava a sua garrafa térmica com


água filtrada, à temperatura de 20°C Coloca então, na garrafa, uma EXERCÍCIOS DE
porção de 200 g de café que acabara de coar, a uma temperatura
inicial θ0 Considerando-se a capacidade térmica da garrafa
05. APROFUNDAMENTO
100 cal/°C, o calor específico sensível do café 1,0 cal/g°C e, após
01. (UFJF 2006) Um bloco de chumbo de 6,68 kg é retirado de um
algum tempo, a temperatura de equilíbrio do sistema garrafa/café
forno a 300°C e colocado sobre um grande bloco de gelo a 0°C.
ter atingido 60°C, pode-se afirmar que o valor de θ0, em °C, é
Supondo que não haja perda de calor para o meio externo, qual é a
a) 30 b) 40 c) 60 d) 70 e) 80 quantidade de gelo que deve ser fundida?
Dados: calor específico do gelo a 0°C = 2100 J/(kg.K)
18. (EEAR 2019) Duas porções de líquidos A e B, de substâncias
–– calor latente de fusão do gelo = 334 x 103 J/kg calor
diferentes, mas de mesma massa, apresentam valores de calor
específico do chumbo = 230 J/(kg.K)
específico respectivamente iguais a 0,58 cal/g ·°C e 1,0 cal/g ·°C.
Se ambas receberem a mesma quantidade de calor sem, –– calor latente de fusão do chumbo = 24,5 x 103 J/kg
contudo, sofrerem mudanças de estado físico, podemos afirmar –– temperatura de fusão do chumbo = 327°C
corretamente que:
Reprodução proibida Art. 184 do CP.

a) a porção do líquido A sofrerá maior variação de temperatura do 02. (UFRJ 1998) Num calorímetro de capacidade térmica
que a porção do líquido B. desprezível que contém 60g de gelo a 0°C, injeta-se vapor d’água a
b) a porção do líquido B sofrerá maior variação de temperatura do 100°C, ambos sob pressão normal.
que a porção do líquido A. Quando se estabelece o equilíbrio térmico, há apenas 45g de água
c) as duas porções, dos líquidos A e B, sofrerão a mesma variação no calorímetro. O calor de fusão do gelo é 80 cal/g, o calor de
de temperatura. condensação do vapor d’água é 540 cal/g e o calor específico da
água é 1,0 cal/g°C.
d) as duas porções, dos líquidos A e B, não sofrerão nenhuma
variação de temperatura. Calcule a massa do vapor d’água injetado.

19. (IFSUL 2019) O gráfico a seguir representa a variação de 03. (UFRJ 2004) Em um calorímetro de capacidade térmica
temperatura ∆T, em função da quantidade de calor Q, transferidas a desprezível, há 200 g de gelo a -20°C. Introduz-se, no calorímetro,
dois sistemas A e B, que apresentam a mesma massa cada um deles. água a 20°C. O calor latente de solidificação da água é - 80
cal/g e os calores específicos do gelo e da água (líquida) valem,
respectivamente, 0,50 cal/g·°C e 1,0 cal/g·°C.
Calcule o valor máximo da massa da água introduzida, a fim de que,
ao ser atingido o equilíbrio térmico, haja apenas gelo no calorímetro.

04. Usando o conceito de “ar quente” e “ar frio”, explique porque o


congelador de uma geladeira deve ser colocado na parte de cima
do aparelho.

05. (UFSC 1996) Coloca-se um cubo de gelo de massa 50 g e


temperatura 0°C dentro de um copo, contendo 200 g de água a 70 °C.
Considerando a ocorrência de trocas de energia apenas entre o gelo
e a água, determine a temperatura final de equilíbrio térmico, em °C.
De acordo com o gráfico, concluímos que a capacidade térmica do Dados:
corpo A (CA), em relação à capacidade térmica do corpo B (CB) é calor de fusão do gelo = 80 cal/g.
a) duas vezes maior. d) quatro vezes menor. calor específico da água = 1,0 cal/g°C.
b) quatro vezes maior. e) oito vezes maior.
c) duas vezes menor. GABARITO
EXERCÍCIOS PROPOSTOS
20. (EEAR 2019) A figura a seguir mostra a curva de aquecimento de 01. B 05. A 09. B 13. A 17. E
uma amostra de 200 g de uma substância hipotética, inicialmente 02. D 06. B 10. C 14. C 18. A
a 15°C, no estado sólido, em função da quantidade de calor que
03. B 07. D 11. D 15. E 19. C
esta recebe.
04. D 08. D 12. E 16. D 20. B

EXERCÍCIOS DE APROFUNDAMENTO
01. m = 1,38 Kg
02. m = 5 g
03. 20 g
04. O ar quente sobe para o congelador, torna-se frio e desce para refrigerar as demais
partes.
05. 40°C

Determine o valor aproximado do calor latente de vaporização da


substância, em cal/g.
a) 10 b) 20 c) 30 d) 40 e) 60

202 PROENEM.COM.BR PRÉ-VESTIBULAR


05
FISICA II
REVISÃO - CALORIMETRIA

Nesse capítulo vamos ser breves e sucintos, reunindo tudo de ÁGUA


mais importante já estudado até aqui sobre energia térmica. Lfusão= 80 cal/g
Energia térmica é a média das energias cinéticas das Lvaporização = 540 cal/g
moléculas de um corpo, Quando há transferência de energia
térmica denominamos de Calor, o sempre flui de forma espontânea
da região de maior temperatura para região de menor temperatura. TROCAS DE CALOR
Quando as temperaturas dos corpos se igualam, ocorre o equilíbrio Quando dois ou mais corpos entram em contato, uma
térmico e a transferência de calor se encerra. transferência de energia, espontânea, do corpo de maior
Reprodução proibida Art. 184 do CP.

Existem três formas de ocorrer transferência de calor: temperatura para o corpo de menor temperatura é iniciada até que
CONDUÇÃO, CONVECÇÃO e IRRADIAÇÃO. o sistema atinja o equilíbrio térmico. Suponha que o conjunto esteja
no interior de um recipiente que evita a troca de energia entre o
Condução: calor transferido de molécula por molécula, sem sistema e o meio externo, mantendo-o isolado. Chamaremos esse
que haja movimento de matéria. recipiente de ADIABÁTICO.
Convecção: calor transferido por diferença de pressão e Todo calor cedido por um corpo será recebido por outro, uma
densidade, com movimento de matéria. vez que a entrada e a saída de energia está impossibilitada de
Irradiação: calor transferido sob a forma de onda eletromagnética, ocorrer graças ao recipiente adiabático. Portanto, temos:
única maneira de transferir calor através do vácuo.
ΣQcedido+ΣQrecebido=0
CALOR ESPECÍFICO
Propriedade da substância, taxa que indica quanto de energia é EXERCÍCIOS
necessário para aumentar em 1°C a temperatura de 1g da substância.
PROTREINO
C
c = ou como cos tuma ser escrito C m·c
m 01. Colocam-se 500 g de ferro (c=0,1 cal/g°C) a 42°C num
recipiente de capacidade térmica desprezível contendo 500g de
CAPACIDADE TÉRMICA água (c=1cal/g°C) a 20°C. Calcule a temperatura quando o sistema
atinge o equilíbrio térmico.
Propriedade do corpo que depende da substância constituinte
e de sua massa. É uma taxa que indica a quantidade de energia
02. Um bloco de alumínio (c = 0,22 cal/g°C) de massa 100g é
necessária para aumentar em 1°C a temperatura desse corpo.
deixado no interior de um forno até entrar em equilíbrio térmico
com ele. Logo ao ser retirado, é colocado em 4400g de água (c
Q
C= = 1,0 cal/g°C) a 30°C. A temperatura de equilíbrio térmico é 32°C.
∆θ Calcule a temperatura do forno.

03. Num recipiente de capacidade térmica 30 cal/°C há 20g de


CALOR SENSÍVEL um líquido de calor específico 0,5 cal/g°C, a 60°C. Colocando-se
Responsável pela variação da temperatura sem que haja nesse líquido 10 g de gelo em fusão, calcule a temperatura final de
mudança no estado físico. equilíbrio, admitindo-se que o sistema está termicamente isolado
do ambiente. Dados (calor latente de fusão do gelo 80 cal/g e calor
Q=m·c·∆θ específico da água 1 cal/g°C).

04. Num bloco de gelo em fusão faz-se uma cavidade onde são
CALOR LATENTE colocados 80 g de um metal de calor específico 0,03 cal/g°C
Responsável pela mudança de estado físico sem que haja a 200°C. Calcule a massa de água que se forma até o equilíbrio
variação de temperatura. térmico. Adote: calor latente de fusão do gelo é 80 cal/g.

Q = m·L 05. Misturam-se, num calorímetro de capacidade térmica


desprezível, 200g de gelo a 0°C com 200g de água a 40°C. Sendo
80 cal/g o calor latente de fusão do gelo e 1,0 cal/g°C o calor
específico da água, determine:
a) a temperatura de equilíbrio térmico.
b) a massa de gelo que se funde.

PRÉ-VESTIBULAR PROENEM.COM.BR 203


FÍSICA II 05 REVISÃO - CALORIMETRIA

Considerando os aspectos termodinâmicos dos fenômenos


envolvidos, analise as afirmações:
EXERCÍCIOS I. Os aquecedores devem ser mantidos próximos ao piso do
ambiente, porque a condutividade térmica do ar é maior
PROPOSTOS quando próxima à superfície da Terra.
II. Energia é transferida continuamente entre o corpo e as suas
01. Uma ferramenta de corte a raio laser consegue cortar vizinhanças por meio de ondas eletromagnéticas.
vários materiais, como aço carbono, aço inoxidável, alumínio, III. O ato de encolher-se permite às pessoas diminuir sua área
titânio, plásticos, etc. Supondo, numa situação idealizada para exposta ao ambiente e, consequentemente, diminuir a perda
fins de simplificação, que o material sólido a ser cortado estava de energia.
exatamente na sua temperatura de transição do estado sólido para
o líquido; além disso, que o laser foi aplicado e liquefez o material Está(ão) correta(s)
nos trechos em que esteve em contato com ele, porém, sem a) apenas I. d) apenas II e III.
aumentar a temperatura do material nesses trechos. Pode-se dizer b) apenas II. e) I, II e III.
que o laser transferiu para o material uma quantidade de energia
c) apenas I e III.
Reprodução proibida Art. 184 do CP.

associada diretamente
a) ao calor específico do material.
06. Em 2010 o Prêmio Nobel de Física foi dado a dois cientistas de
b) ao calor latente de fusão do material. origem russa, André Geim e Konstantin Novoselov, por descobrirem
c) à capacidade térmica do material. em 2004 o grafeno, uma forma revolucionária do grafite. O grafeno
apresenta vários aspectos positivo para a tecnologia de hoje, sendo
d) ao módulo de compressibilidade do material.
uma delas o melhor condutor de calor. Analise as afirmações
e) ao número de moles do material. abaixo sobre os processos de propagação de calor.
I. Convecção: é o processo de transmissão de energia térmica
02. É possível passar a matéria do estado sólido diretamente para feita de partícula para partícula sem que haja transporte de
o gasoso, evitando a fase líquida. Tal fenômeno físico se verifica matéria de uma região para outra.
comumente no gelo seco e na naftalina, mas também pode ocorrer
com a água, dependendo das condições de temperatura e pressão. II. Condução: é o processo de transmissão de energia térmica
A essa passagem dá-se o nome de feita por meio do transporte da matéria de uma região para
outra.
a) condensação. d) vaporização.
III. Radiação: é o processo que consiste na transmissão de energia
b) sublimação. e) calefação. térmica por meio de ondas eletromagnéticas. Ocorre tanto no
c) fusão. vácuo quanto em outros meios materiais.
Analisando as afirmações, é CORRETO apenas o que se afirma em:
03. A elevação da temperatura das águas de rios, lagos e mares
a) I. d) I e III.
diminui a solubilidade do oxigênio, pondo em risco as diversas
formas de vida aquática que dependem desse gás. Se essa b) II. e) II e III.
elevação de temperatura acontece por meios artificiais, dizemos c) III.
que existe poluição térmica. As usinas nucleares, pela própria
natureza do processo de geração de energia, podem causar esse 07. O gráfico mostra como varia a temperatura em função do
tipo de poluição. tempo de aquecimento de um líquido, inicialmente a 20 ºC.
Que parte do ciclo de geração de energia das usinas nucleares está
associada a esse tipo de poluição?
a) Fissão do material radioativo.
b) Condensação do vapor d‘água no final do processo.
c) Conversão de energia das turbinas pelos geradores.
d) Aquecimento da água líquida para gerar vapor d‘água.
e) Lançamento do vapor d‘água sobre as pás das turbinas.

04. No senso comum, as grandezas físicas calor e temperatura


geralmente são interpretadas de forma equivocada. Diante disso, a
linguagem científica está corretamente empregada em
a) “Hoje o dia está fazendo calor.”
b) “O calor está fluindo do fogo para a panela.” A partir da análise deste gráfico, pode-se concluir que o líquido
c) “A temperatura está alta, por isso estou com muito calor.” a) entra em ebulição a uma temperatura de 80 ºC.
d) “O gelo está transmitindo temperatura para água no copo.” b) inicia a vaporização a uma temperatura de 60 ºC.
e) “Estou ganhando muita temperatura do ambiente.” c) transforma-se em gás a uma temperatura de 20 ºC.
d) permanece como líquido a uma temperatura de 70 ºC.
05. O inverno é caracterizado pela ocorrência de baixas
temperaturas, especialmente nas regiões ao sul do Brasil. Por essa e) N.D.A.
razão, é alto o índice de incidência de doenças respiratórias, de
modo que a primeira recomendação é manter-se abrigado sempre
que possível e agasalhar-se adequadamente.

204 PROENEM.COM.BR PRÉ-VESTIBULAR


05 REVISÃO - CALORIMETRIA FÍSICA II

08. Uma quantidade de água líquida de massa m = 200 g, a uma d) O ar que está sobre a água se esfria, criando um centro de alta
temperatura de 30 ºC, é colocada em um calorímetro junto a 150 g pressão que atrai massas de ar continental.
de gelo a 0 ºC. Após atingir o equilíbrio, dado que o calor específico e) O ar sobre o solo, mais quente, é deslocado para o mar,
da água é ca = 1,0 cal/(g . ºC) e o calor latente de fusão do gelo é equilibrando a baixa temperatura do ar que está sobre o mar.
L = 80 cal/g, calcule a temperatura final da mistura gelo + água.
a) 10 ºC b) 15 ºC c) 0 ºC d) 30 ºC e) 60 ºC 11. Um líquido é aquecido através de uma fonte térmica que
provê 50,0 cal por minuto. Observa-se que 200 g deste líquido se
09. Considere que um bloco de gelo, inicialmente a 0 ºC, seja aquecem de 20,0 °C em 20,0 min.
aquecido a uma taxa constante. Um tempo t é necessário para Qual é o calor específico do líquido, medido em cal/(g °C)?
transformar o bloco de gelo completamente em vapor d’água a
a) 0,0125 b) 0,25 c) 5,0 d) 2,5 e) 4,0
t
100 ºC. O que se tem após o tempo ?
2 12. Em um calorímetro são colocados 2,0 kg de água, no estado
Considere: calor latente de fusão do gelo = 80 cal/g; calor líquido, a uma temperatura de 0 °C. A seguir, são adicionados 2,0 kg
específico da água = 1 cal/g.°C e calor latente de vaporização da de gelo, a uma temperatura não especificada. Após algum tempo,
água = 540 cal/g.
Reprodução proibida Art. 184 do CP.

tendo sido atingido o equilíbrio térmico, verifica-se que a temperatura


a) Água a uma temperatura entre 0 ºC e 100 ºC. da mistura é de 0 ºC e que a massa de gelo aumentou em 100 g.
b) Apenas gelo a 0 ºC. Considere que o calor específico do gelo (c = 2,1 kJ/kg.°C) é a
metade do calor específico da água e que o calor latente de fusão
c) Uma mistura de água e vapor a 100 ºC.
do gelo é de 330 kJ/kg; e desconsidere a capacidade térmica do
d) Uma mistura de gelo e água a 0 ºC. calorímetro e a troca de calor com o exterior.
e) Apenas vapor a 100 °C. Nessas condições, a temperatura do gelo que foi inicialmente
adicionado à água era, aproximadamente,
10. (ENEM 2002) Numa área de praia, a brisa marítima é uma a) 0 °C. c) - 3,9 °C. e) - 7,9 °C.
consequência da diferença no tempo de aquecimento do solo
e da água, apesar de ambos estarem submetidos às mesmas b) - 2,6 °C. d) - 6,1 °C.
condições de irradiação solar. No local (solo) que se aquece mais
rapidamente, o ar fica mais quente e sobe, deixando uma área de 13. Um engenheiro testa materiais para serem usados na fabricação
baixa pressão, provocando o deslocamento do ar da superfície que da carroceria de um automóvel. Entre outras propriedades, é
está mais fria (mar). desejável a utilização de materiais com alto calor específico. Ele
verifica que, para aumentar em 3 °C a temperatura de 32 g do
material A, é necessário fornecer 24 cal de calor a esse material.
Para obter o mesmo aumento de temperatura em 40 g do material
B, é preciso 24 cal. Já 50 g do material C necessitam 15 cal para
sofrer o mesmo acréscimo de temperatura. Os calores específicos
dos materiais A, B e C são respectivamente:
a) CA = 0,25 cal/g°C ; CB = 0,20 cal/g°C ; CC = 0,10 cal/g°C
b) CA = 0,20 cal/g°C ; CB = 0,35 cal/g°C ; CC = 0,15 cal/g°C
c) CA = 0,30 cal/g°C ; CB = 0,10 cal/g°C ; CC = 0,20 cal/g°C
d) CA = 0,35 cal/g°C ; CB = 0,20 cal/g°C ; CC = 0,10 cal/g°C
e) CA = 0,10 cal/g°C ; CB = 0,30 cal/g°C ; CC = 0,25 cal/g°C
À noite, ocorre um processo inverso ao que se verifica durante o dia.
14. Um trocador de calor consiste em uma serpentina, pela qual
circulam 18 litros de água por minuto. A água entra na serpentina
à temperatura ambiente (20°C) e sai mais quente. Com isso,
resfria-se o líquido que passa por uma tubulação principal, na
qual a serpentina está enrolada. Em uma fábrica, o líquido a ser
resfriado na tubulação principal é também água, a 85°C, mantida
a uma vazão de 12 litros por minuto. Quando a temperatura de
saída da água da serpentina for 40°C, será possível estimar que a
água da tubulação principal esteja saindo a uma temperatura T de,
aproximadamente,

Como a água leva mais tempo para esquentar (de dia), mas
também leva mais tempo para esfriar (à noite), o fenômeno noturno
(brisa terrestre) pode ser explicado da seguinte maneira:
a) O ar que está sobre a água se aquece mais; ao subir, deixa uma
área de baixa pressão, causando um deslocamento de ar do
continente para o mar.
b) O ar mais quente desce e se desloca do continente para a água,
a qual não conseguiu reter calor durante o dia.
a) 75°C d) 45°C
c) O ar que está sobre o mar se esfria e dissolve-se na água;
forma-se, assim, um centro de baixa pressão, que atrai o ar b) 65°C e) 35°C
quente do continente. c) 55°C

PRÉ-VESTIBULAR PROENEM.COM.BR 205


FÍSICA II 05 REVISÃO - CALORIMETRIA

15. Os trajes de Neoprene, um tecido emborrachado e isolante A alternativa correta que mostra o gráfico da temperatura T da
térmico, são utilizados por mergulhadores para que certa água após a mistura em função do tempo t é:
quantidade de água seja mantida próxima ao corpo, aprisionada
nos espaços vazios no momento em que o mergulhador entra c)
a)
na água. Essa porção de água em contato com o corpo é por ele
aquecida, mantendo assim uma temperatura constante e agradável
ao mergulhador. Suponha que, ao entrar na água, um traje retenha
2,5 L de água inicialmente a 21°C. A energia envolvida no processo
de aquecimento dessa água até 35°C é
Dados: densidade da água = 1 kg/L
calor específico da água = 1 cal/(g.°C) b) d)

a) 25,5 kcal. c) 40,0 kcal. e) 70,0 kcal.


b) 35,0 kcal. d) 50,5 kcal.
Reprodução proibida Art. 184 do CP.

16. (IFSUL 2018) Na refrigeração de motores de automóveis, a


substância refrigerante tanto pode ser o ar como a água.
Dados: 20. (EEAR 2018) Um corpo absorve calor de uma fonte a uma taxa
–– calor específico do ar=0,25 cal/g·°C. constante de 30 cal/min e sua temperatura (T) muda em função do
–– calor específico da água= 1,00 cal/g·°C. tempo (t) de acordo com o gráfico a seguir.
A razão entre a massa de ar e a massa de água, para proporcionar a
mesma refrigeração no motor de um automóvel, deverá ser igual a
a) 0,25 b) 1,00 c) 1,20 d) 4,00

17. (UFRGS 2018) Uma quantidade de calor Q=56.100,00 J é


fornecida a 100 g de gelo que se encontra inicialmente a -10°C.
Sendo
–– o calor específico do gelo cg=2,1J/(g°C),
–– o calor específico da água ca=4,2J/(g°C) e A capacidade térmica (ou calorífica), em cal/°C, desse corpo, no
–– o calor latente de fusão CL=330,0J/g, intervalo descrito pelo gráfico, é igual a
a temperatura final da água em °C é, aproximadamente, a) 1 b) 3 c) 10 d) 30
a) 83,8. b) 60,0. c) 54,8. d) 50,0. e) 37,7.

18. (UPF 2018) Um professor de Física, ao final de seu dia de EXERCÍCIOS DE


APROFUNDAMENTO
trabalho, resolve preparar um banho de banheira e deseja que
sua água esteja exatamente a 38°C. Entretanto, ele se descuida e 05.
verifica que a temperatura da água atingiu 42°C. Para solucionar o
problema, o professor resolve adicionar água da torneira, que está 01. (FUVEST 1997) Um pesquisador estuda a troca de calor entre um
a 18°C. Considerando que há, na banheira, 60 litros de água, e que bloco de ferro e certa quantidade de uma substância desconhecida,
haja trocas de calor apenas entre a água quente e a água fria, qual dentro de um calorímetro de capacidade térmica desprezível
será o volume de água, em litros, que ele deverá acrescentar na (ver Figura 1). Em sucessivas experiências, ele coloca no
banheira para atingir a temperatura desejada? calorímetro a substância desconhecida, sempre no estado sólido
à temperatura T0 = 20°C, e o bloco de ferro, a várias temperaturas
a) 12 b) 20 c) 18 d) 16 e) 6 iniciais T, medindo em cada caso a temperatura final de equilíbrio
térmico Te. O gráfico da Figura 2 representa o resultado das
19. (ACAFE 2018) Em quase todos os hospitais e algumas experiências. A razão das massas do bloco de ferro e da substância
residências é comum o uso de chuveiros misturadores de água desconhecida é mf/ms = 0,8. Considere o valor do calor específico
quente e fria como na figura abaixo. do ferro igual a 0,1 cal/(g°C). A partir destas informações, determine
para a substância desconhecida:

Suponha que a torneira quente forneça 50 gramas de água por


segundo a temperatura de 50°C e a torneira fria forneça 100
gramas de água por segundo a temperatura de 20°C. Considere
também que não há trocas de calor entre a água e o ambiente e as
águas das duas torneiras se misturam rapidamente.

206 PROENEM.COM.BR PRÉ-VESTIBULAR


05 REVISÃO - CALORIMETRIA FÍSICA II

a) a temperatura de fusão, Tfusão. Após um certo período de tempo, foram misturados 200 mL de leite
b) o calor específico, cs, na fase sólida. a 20°C a 100mL do café preparado, agora a 80°C, em uma caneca
c) o calor latente de fusão L. de porcelana de capacidade térmica 100 cal/°C, inicialmente a
20°C. Considerando os calores específicos da água, do café e do
02. (UNESP 2000) A figura mostra as quantidades de calor Q leite iguais a 1cal/(g·cal/°C), as densidades da água, do café e
absorvidas, respectivamente, por dois corpos, A e B, em função de
do leite iguais a 1 kg/L, que 1 cal/s=4 W e desprezando todas as
suas temperaturas.
perdas de calor para o ambiente, calcule:
a) a potência, em W, da chama utilizada para aquecer a água para
fazer o café.
b) a temperatura, em °C, em que o café com leite foi ingerido,
supondo que o consumidor tenha aguardado que a caneca e
seu conteúdo entrassem em equilíbrio térmico.
Reprodução proibida Art. 184 do CP.

GABARITO
EXERCÍCIOS PROPOSTOS
01. B 05. D 09. C 13. A 17. D
a) Determine a capacidade térmica CA do corpo A e a capacidade
02. B 06. C 10. A 14. C 18. A
térmica CB do corpo B, em J/°C.
03. B 07. B 11. B 15. B 19. A
b) Sabendo que o calor específico da substância de que é feito o corpo 04. B 08. C 12. E 16. D 20. D
B é duas vezes maior que o da substância de A, determine a razão
mA/mB entre as massas de A e B. EXERCÍCIOS DE APROFUNDAMENTO
01.
03. (PUCRJ 2000) Devido a um resfriado, um homem de 80 kg a) 60°C; b) 0,28 cal/°C; c) 20 cal/g
tem temperatura do corpo igual a 39°C ao invés da temperatura 02. a) CA = 7,5 J/°C; CB = 5 J/°C; b) mA/mB = 3
normal de 37°C. Supondo que o corpo humano seja constituído
03. ∆Q = 160.000 cal
basicamente de água, qual a quantidade de calor produzida pelo
corpo para causar este aumento de temperatura? 04.
a) A: 3 cal/°C e 0,03 cal/g°C; B: 8 cal/°C e 0,05 cal/g°C b) A: 90 cal; B: 240 cal
Calor específico da água = 1 cal/g°C.
05. 35°C

04. (UFG 2001) No diagrama Q×t, estão representadas as ANOTAÇÕES


quantidades de calor absorvidas por duas substâncias, A e B, cujas
massas são, respectivamente, iguais a 100g e 160g, em função da
temperatura. Considere 0°C a temperatura inicial das substâncias.

a) Determine as capacidades térmicas e os calores específicos


de A e B.
b) Determine as quantidades de calor absorvidas por A e B,
quando ambas estiverem à temperatura t, indicada no gráfico.

05. (UNIFESP 2018) Para a preparação de um café, 1L de água é


aquecido de 25°C até 85°C em uma panela sobre a chama de um
fogão que fornece calor a uma taxa constante. O gráfico representa
a temperatura (θ) da água em função do tempo, considerando que
todo o calor fornecido pela chama tenha sido absorvido pela água.

PRÉ-VESTIBULAR PROENEM.COM.BR 207


FÍSICA II 05 REVISÃO - CALORIMETRIA

ANOTAÇÕES
Reprodução proibida Art. 184 do CP.

208 PROENEM.COM.BR PRÉ-VESTIBULAR


06
FÍSICA II
LEI GERAL DOS GASES

TEORIA CINÉTICA DOS GASES p1 ·V1 p2 ·V2


=
A teoria cinética dos gases perfeitos é responsável pela T1 T2
determinação do comportamento microscópico de um gás
chamado de ideal ou perfeito. Os postulados que serão enunciados É importante lembrar ao aluno mais uma vez que a Lei Geral
abaixo podem ser aplicados a gases reais em altas temperaturas e dos Gases poderá ser utilizada sempre que não existir alteração do
baixas pressões, gás rarefeito, que nessas condições apresentam número de mols, a massa não se altera.
um comportamento aproximado de um gás ideal.
Os postulados da teoria cinética dos gases enunciam: TRANSFORMAÇÕES GASOSAS
Reprodução proibida Art. 184 do CP.

• uma massa gasosa é constituída por um número Existem algumas transformações pelas quais os gases podem
muito grande de partículas, as moléculas que estão passar que são especiais em relação às variáveis de estado: a
em movimento desordenado colidem entre si e com as pressão (p), o volume (V) e a temperatura (T).
paredes do recipiente;
Veremos, agora, essas transformações particulares.
• a soma dos volumes das moléculas é desprezível diante
do volume total do gás, isto é, o tamanho da molécula é
desprezível diante da distância entre as moléculas; Isotérmica
• as moléculas colidem elasticamente; Transformação Isotérmica é aquela na qual a temperatura
absoluta do gás se mantém constante. Boyle-Mariotte estudaram o
• as forças intermoleculares, a não ser devido aos choques,
comportamento de um gás numa situação isotérmica, concluindo:
são desprezíveis, logo, podemos afirmar que as moléculas
realizam um movimento retilíneo uniforme (MRU) entre Numa transformação isotérmica de determinada massa de
duas colisões. gás, pressão (p) e volume (V) são inversamente proporcionais.
Podemos fazer algumas afirmações sobre as grandezas Matematicamente:
fundamentais que envolvem o estudo de um gás perfeito. p.V = k (k constante que só depende do gás).
• Volume: um gás sempre ocupa todo o volume do recipiente
Se o gás passa de um determinado estado denominado 1 para
que o contém, logo, o volume de um gás ideal é o mesmo
do recipiente. outro denominado 2, teremos:
• Temperatura: como já vimos, a temperatura expressa O gráfico abaixo mostra o comportamento do gás em uma
o grau de agitação das moléculas. Nas expressões que transformação isotérmica:
veremos a seguir, a temperatura deverá ser colocada
sempre em Kelvin, conhecida como escala absoluta. p1V1=p2V2
• Pressão: a pressão é devido às inúmeras colisões das
moléculas do gás com as paredes do recipiente.
É bom ressaltar que os fenômenos termodinâmicos que vamos
estudar correspondem a transformações quase estáticas, ou seja,
ocorrem tão lentamente que o equilíbrio termodinâmico é atingido
em quase todo instante.

LEI GERAL DOS GASES

PROEXPLICA

1ª) A curva que representa duas grandezas inversamente


proporcionais denomina-se hipérbole, ela é simétrica
em relação à bissetriz do primeiro quadrante.

A Lei Geral dos Gases é obtida através de uma combinação 2ª) No caso da hipérbole com as variáveis p e V, será
das leis de Boyle-Mariott, de Gay-Lussac e Charles que se chamada de isoterma, já que todos os seus pontos
referem às transformações isotérmica, isobárica e isométrica, estão na mesma temperatura.
respectivamente. 3ª) Num mesmo par de eixos com mais de uma isoterma,
Se em uma transformação qualquer, um gás passa das aquela que estiver mais afastada dos eixos terá seus
condições p1, V1 e T1 para outras p2, V2 e T2, sem que exista variação pontos numa temperatura maior.
de massa, a Lei Geral dos Gases enuncia a relação:

PRÉ-VESTIBULAR PROENEM.COM.BR 209


FÍSICA II 06 LEI GERAL DOS GASES

Como pressão pode ser dada pela razão entre força aplicada e
área de contato, um corpo de massa constante sobre um êmbolo
Um reservatório térmico mantém a temperatura constante que pode se mover sem atrito, mantém a força sobre a área desse
Reprodução proibida Art. 184 do CP.

de qualquer substância que permanecer em contato com ele. êmbolo, resultando assim em uma pressão invariável.
Podemos imaginar um forno que mantém sua temperatura como
um bom exemplo de reservatório térmico. Além disso, uma parede Isométrica, Isovolumétrica ou Isocórica
diatérmica é aquela que permite a troca de calor entre os corpos, Transformação Isométrica é aquela que ocorre com seu
mas não permite passar massa. volume se mantendo constante, tanto Gay-Lussac como Charles
estudaram a transformação isométrica e concluíram:
Numa transformação isométrica, a pressão (p) e a temperatura
Isobárica absoluta (T) são grandezas diretamente proporcionais.
Transformação Isobárica é a que ocorre com a pressão se Matematicamente:
mantendo constante. Gay-Lussac estudando-a concluiu:
p
Numa transformação isobárica o volume (V) e a temperatura = k (k constante que só depende do gás).
T
absoluta (T) são diretamente proporcionais.
Matematicamente:
Passando o gás de um estado denominado 1 para outro
denominado 2, teremos:
V
= k (k constante que só depende do gás).
T

Passando o gás de um estado denominado 1 para outro


denominado 2, teremos:

PROEXPLICA

Valem, aqui, as três observações feitas na transformação


isobárica.

PROEXPLICA

1ª) O gráfico de uma proporcionalidade direta é uma reta


passando pela origem.
2ª) O tracejado da reta próximo da origem se faz, pois não
se tem como atingir o zero absoluto.
3ª) Se no eixo horizontal, das temperaturas, a unidade
fosse °C a reta não passaria mais pela origem,
Admitindo que a dilatação térmica do recipiente é desprezível e
tenderia a passar pelo valor – 273 °C.
que o gás ocupa todo o volume desse recipiente, mantendo fixo o
êmbolo, teremos um volume constante.

210 PROENEM.COM.BR PRÉ-VESTIBULAR


06 LEI GERAL DOS GASES FÍSICA II

Adiabática Uma barra de 560g de ferro possui 10 mols do elemento.


Na transformação adiabática, o gás não troca calor com 560
=n = 10mols
o meio externo, se torna um sistema isolado termicamente. 56
Inclusive, o recipiente que evita a troca de calor é chamado de
recipiente adiabático. A lei que rege uma transformação adiabática
será estudada em Termodinâmica. Veremos que apesar de não
existir trocas de calor com meio externo, haverá uma mudança de EXERCÍCIOS
temperatura.
PROTREINO
EQUAÇÃO DE CLAPEYRON
01. Um gás ideal inicialmente à temperatura de 77°C e volume
Quando em uma transformação ocorrer mudança no número de
0,04 m³ é submetido a uma transformação isobárica, elevando seu
mols do gás, ou seja, na massa de gás envolvido, então a Lei Geral
volume para 0,08 m³. Nessas condições, calcule sua temperatura
dos Gases se torna ineficiente e precisaremos de uma função mais
final, em °C.
abrangente. O físico francês Emile Clapeyron, estabeleceu uma
Reprodução proibida Art. 184 do CP.

Função de Estado dos Gases Ideais, uma equação que descreve o


comportamento desses gases quando sofrem alterações (ou não) em 02. Um gás ideal inicialmente à temperatura de 27°C e volume
seu(sua): volume, pressão, temperatura e número de mols (massa). 0,01 m³ é submetido a uma transformação isocórica, elevando sua
temperatura para 77°C. Calcule a razão entre a pressão final e a
P⋅V pressão inicial.
=R (para 1 mol)
T
P⋅V 03. Um gás ideal inicialmente à 0,04 m³ e pressão 2 atm é submetido
= 2 ⋅R (para 2 mols)
T a uma transformação isotérmica, diminuindo sua pressão até 0,5
P⋅V atm. Calcule o volume, em m³, nessa nova pressão.
= 3R (para 3 mols)
T
P⋅V 04. O gráfico abaixo relaciona o volume e a temperatura a que certa
= nR → P ⋅ V = n ⋅R ⋅ T
T massa de gás ideal possui ao sofrer transformações.
A constante R, conhecida por constante geral dos gases, pode Volume
ser obtida utilizando os dados de um gás nas CNTP`s (Condições
normais de temperatura e pressão): B C

P = 1 atm
V = 22,4L (para cada mol)
T= 273K

P ⋅ V 1⋅ 22,4
= = 0,082
T 273
A
R = 0,082 atm.L/mol.K D

Temperatura

NÚMERO DE MOLS
O mol é uma unidade de medida utilizada para determinar a
Classifique o tipo de transformação entre os trechos:
quantidade de partículas, como átomos ou moléculas. Assim como
uma dezena de laranjas representa 10 unidades de laranja e uma AB: __________________ Resposta: Isobárica
dúzia de bananas representa 12 unidades de banana, 1 mol de BC: __________________ Resposta: Isovolumétrica
átomos representa 6,02 . 1023 (Constante de Avogadro) unidades CD: __________________ Resposta: Isotérmica
desse átomo. Na tabela periódica podemos encontrar a massa
molar (ou massa atômica) dos elementos, que consiste na massa 05. O gráfico abaixo apresenta a transformação isotérmica de um
existente em cada mol daquela substância. Exemplo: gás ideal.

Pressão (atm)

17
16
A
15
14
13
12
11
10
9
8
7
6
5
4
B
3
2

m → massa total da substância 1


Número de mols ← n =
M → massa molar
0 5 10 15 20 25 30 35 40 45 55
-1
Volume (L)

Exemplo: Alguns dados do gráfico foram apagados. Calcule o volume quando


MFe ≈ 56g
a pressão é de 3 atm.

PRÉ-VESTIBULAR PROENEM.COM.BR 211


FÍSICA II 06 LEI GERAL DOS GASES

Escolha a alternativa que identifica de forma correta as


transformações sofridas por esse gás, representadas,
respectivamente, em I e II.
EXERCÍCIOS
a) Isobárica e isocórica. d) Isocórica e isobárica.
PROPOSTOS b) Isotérmica e isocórica. e) Isocórica e isotérmica.
c) Isotérmica e isobárica.
01. Uma certa quantidade de gás ideal ocupa inicialmente um
volume V0 com pressão P0.
06. Um mol de gás ideal, inicialmente num estado A, ocupa o
Se sobre esse gás se realiza um processo isotérmico dobrando sua volume de 5,6 litros. Após sofrer uma transformação isotérmica,
pressão para 2 P0. Qual será o volume final do gás? é levado ao estado B. Sabendo que em B o gás está nas CNTP
a) V0/3 b) V0/2 c) V0 d) 2V0 e) 3V0 (condições normais de temperatura e pressão), podemos afirmar
que em A:
02. A pressão do ar no interior dos pneus é recomendada pelo
fabricante para a situação em que a borracha está fria. Quando o
Reprodução proibida Art. 184 do CP.

carro é posto em movimento, os pneus se aquecem, seus volumes


têm alterações desprezíveis e ocorrem variações nas pressões
internas dos mesmos. Considere que os pneus de um veículo
tenham sido calibrados a 17 °C com uma pressão de 1,6 ·105 N/m2.
Após rodar por uma hora, a temperatura dos pneus chega a 37 °C.
A pressão no interior dos pneus atinge um valor aproximado de:
a) 1,8x105 N/m2 d) 8,7x105 N/m2
b) 3,7x10 N/m
5 2
e) 9,1x105N/m2
c) 7,8x10 N/m
5 2

03. No estudo da termodinâmica dos gases perfeitos, o


a) a pressão é desconhecida e não pode ser determinada com os
comportamento do gás é analisado através das suas propriedades
dados disponíveis.
macroscópicas, levando em conta as grandezas físicas a ele
associadas. Essas grandezas, denominadas variáveis de estado, b) a pressão é 1,0 atmosfera.
são: temperatura, volume e pressão. Em geral, quando determinada c) a pressão é 2,0 atmosferas.
massa de gás perfeito sofre uma transformação, pelo menos duas d) a pressão é 4,0 atmosferas.
dessas grandezas sofrem variações.
e) a pressão é 5,6 atmosferas.
Analise as seguintes afirmativas referentes às transformações
termodinâmicas em um gás perfeito:
07. Considere um gás ideal em um recipiente mantido a temperatura
I. Quando determinada massa de gás perfeito sofre uma constante e com paredes móveis, de modo que se possa controlar
transformação isotérmica, sua pressão é inversamente seu volume. Nesse recipiente há um vazamento muito pequeno,
proporcional ao volume por ele ocupado. mas o volume é controlado lentamente de modo que a razão entre
II. Quando determinada massa de gás perfeito sofre uma o número de moles de gás e seu volume se mantém constante.
transformação isobárica, seu volume é diretamente Pode-se afirmar corretamente que a pressão desse gás
proporcional a sua temperatura absoluta. a) é crescente.
III. Quando determinada massa de gás perfeito sofre uma b) é decrescente.
transformação isométrica, sua pressão é inversamente
c) varia proporcionalmente ao volume.
proporcional a sua temperatura absoluta.
d) é constante.
Está(ão) correta(s) apenas a(s) afirmativa(s)
e) é inversamente proporcional à temperatura.
a) I. c) I e II. e) I, II e III.
b) III. d) II e III. 08. Certo gás, considerado ideal, com massa 34 g, está contido
em um recipiente de 12,3 litros, sob pressão de 4 atm a 27 °C.
04. Um gás ideal inicialmente à temperatura de 27 °C e volume de Considerando apenas as massas atômicas dadas pela tabela a
0,02 m³ é submetido a uma transformação isobárica, elevando seu seguir, assinale o gás contido no recipiente:
volume para 0,06 m³. Nessas condições, é possível afirmar que sua
temperatura final é, em °C de: Elemento Massa Atômica
a) 627 b) 81 c) 900 d) 1.173 e) 300 H 1

05. Nos gráficos I e II abaixo, p representa a pressão a que certa C 12


massa de gás ideal está sujeita, T a sua temperatura e V o volume N 14
por ela ocupado.
O 16

Dado: R – 0,082 atm.litro/(mol.K)


a) CH4 b) C2H6 c) CO3 d) NH3 e) N2

212 PROENEM.COM.BR PRÉ-VESTIBULAR


06 LEI GERAL DOS GASES FÍSICA II

09. Nos últimos anos, o gás natural (GNV: gás natural veicular)
vem sendo utilizado pela frota de veículos nacional, por ser viável
economicamente e menos agressivo do ponto de vista ambiental.
O quadro compara algumas características do gás natural e da
gasolina em condições ambiente.

Densidade (kg/m3) Poder calorífico (kj/kg)


GNV 0,8 50.200
Gasolina 738 46.900

Apesar das vantagens no uso de GNV, sua utilização implica Em (I), a pressão vale p e a temperatura é de 40 K. Em (II), a
algumas adaptações técnicas, pois, em condições ambientes, o temperatura é de 30 K e a nova pressão é dada por
VOLUME de combustível necessário, em relação ao de gasolina, 3p
Reprodução proibida Art. 184 do CP.

para produzir a mesma energia, seria: a) p d)


4.
a) muito maior, o que requer um motor muito mais potente. b) 2p. e)
4p
b) muito maior, o que requer que ele seja armazenado a alta 3.
pressão. p
c)
2.
c) igual, mas sua potência será muito menor.
d) muito menor, o que o torna o veículo menos eficiente. 13. Um frasco para medicamento com capacidade de 50 mL,
e) muito menor, o que facilita sua dispersão para a atmosfera. contém 35 mL de remédio, sendo o volume restante ocupado por
ar. Uma enfermeira encaixa uma seringa nesse frasco e retira 10
10. Considere o diagrama de fases adiante, em que p representa a mL do medicamento, sem que tenha entrado ou saído ar do frasco.
Considere que durante o processo a temperatura do sistema tenha
pressão e θ a temperatura absoluta da substância.
permanecido constante e que o ar dentro do frasco possa ser
considerado um gás ideal.

É correto afirmar que:


a) a curva TC representa a solidificação da substância.
b) acima de θc o sistema é tetrafásico.
c) gás é um estado da substância que se consegue liquefazer por
compressão isotérmica.
Na situação final em que a seringa com o medicamento ainda
d) gás é um estado da substância que não pode se tornar líquido
estava encaixada no frasco, a retirada dessa dose fez com que
por compressão isotérmica.
a pressão do ar dentro do frasco passasse a ser, em relação à
e) no diagrama está representada uma isoterma. pressão inicial,
a) 60% maior. d) 40% menor.
11. Considere um gás ideal em um recipiente mantido a temperatura
constante e com paredes móveis, de modo que se possa controlar b) 40% maior. e) 25% menor.
seu volume. Nesse recipiente há um vazamento muito pequeno, c) 60% menor.
mas o volume é controlado lentamente de modo que a razão entre
o número de moles de gás e seu volume se mantém constante. 14. Um gás ideal possui, inicialmente, volume V0 e encontra-se sob
Pode-se afirmar corretamente que a pressão desse gás uma pressão p0. O gás passa por uma transformação isotérmica, ao
final da qual o seu volume torna-se igual a V0/2. Em seguida, o gás
a) é crescente.
passa por uma transformação isobárica, após a qual seu volume é
b) é decrescente. 2V0. Denotando a temperatura absoluta inicial do gás por T0, a sua
c) varia proporcionalmente ao volume. temperatura absoluta ao final das duas transformações é igual a:
d) é constante. a) T0/4 b) T0/2 c) T0 d) 2T0 e) 4T0

12. A figura abaixo ilustra um experimento realizado sem troca 15. Utilizando-se de uma bomba pneumática com corpo de volume
de calor com o meio externo no qual um cilindro com um êmbolo 0,15 litro, Tiago "enche" um pneu de volume constante e igual a 4,5 litros,
móvel contém um gás considerado ideal e é levado da configuração ao nível do mar. Após bombear 30 vezes, com a plenitude de volume da
(I) para a (II). bomba, a pressão do ar contido no pneu vai de 1 atm até o valor que ele
deseja. Considerando o ar como gás ideal, e que sua temperatura não
variou durante o processo, a pressão final do pneu será de:
a) 1,5 atm c) 2,5 atm e) 4,0 atm
b) 2,0 atm d) 3,0 atm

PRÉ-VESTIBULAR PROENEM.COM.BR 213


FÍSICA II 06 LEI GERAL DOS GASES

16. (EEAR 2019) Um cilindro dotado de um êmbolo contém 03. (UERJ 2011) Um professor realizou com seus alunos o seguinte
aprisionado em seu interior 150 cm3 de um gás ideal à temperatura experimento para observar fenômenos térmicos:
controlada de 22°C e à pressão de 2Pa. Considere que o êmbolo do • colocou, inicialmente, uma quantidade de gás ideal em um
cilindro pode ser movido por uma força externa, de modo que o gás recipiente adiabático;
seja comprimido a um terço de seu volume inicial, sem, contudo,
variar a sua temperatura. Nessas condições, determine em Pascal • comprimiu isotermicamente o gás à temperatura de 27°C,
(Pa) a nova pressão à qual o gás estará submetido. até a pressão de 2,0 atm;

a) 2 b) 3 c) 6 d) 9 e) 12 • liberou, em seguida, a metade do gás do recipiente;


• verificou, mantendo o volume constante, a nova
17. (UEMG 2019) Antes de viajar, o motorista calibrou os pneus temperatura de equilíbrio, igual a 7°C.
do seu carro a uma pressão de 30 psiquando a temperatura dos Calcule a pressão do gás no recipiente ao final do experimento.
pneus era de 27°C. Durante a viagem, após parar em um posto de
gasolina, o motorista percebeu que os pneus estavam aquecidos. 04. (UFPE 2010) Um operário está fazendo manutenção em uma
Ao conferir a calibragem, o motorista verificou que a pressão dos plataforma marítima de petróleo na profundidade de 50 m, quando
pneus era de 32 psi. uma pequena bolha de ar, de volume Vi, é liberada e sobe até a
Reprodução proibida Art. 184 do CP.

Considerando a dilatação do pneu desprezível e o ar dentro dos superfície. O aumento da pressão em função da profundidade está
pneus como um gás ideal, assinale a alternativa que MELHOR representado no gráfico a seguir.
representa a temperatura mais próxima dos pneus. Considerando o gás da bolha como ideal e que a temperatura da
a) 29°C b) 38°C c) 47°C d) 52°C e) 62 °C água não varia entre a superfície e a profundidade de 50 m, calcule
a razão Vf/Vi entre o volume final Vf da bolha e o volume inicial Vi.
18. (UPF 2019) Considerando que o volume de um gás ideal é
V1=0,5 m3 na temperatura T1=0°C e pressão P1, podemos afirmar
que, na pressão P2=0,5 P1 e T2=10 T1, o volume do gás, em m3, será
a) 1 b) 5 c) 20 d) 10 e) 0,1

19. (MACKENZIE 2018) Um gás perfeito, que tem um volume de


12,0 , encontra-se no interior de um frasco sob pressão de 3,00
atm e com temperatura de 200 K. Inicialmente, o gás sofre uma
transformação isotérmica, de tal forma que sua pressão passa a ser
de 9,00 atm, a seguir, o gás sofre uma transformação segundo a lei
de Gay-Lussac, atingindo uma temperatura de 500 K. Os volumes,
05. Um gás ideal sofre uma compressão isobárica tal que seu
após as duas transformações, respectivamente, são iguais a volume se reduz a 2/3 do inicial.
a) 10,0  e 4,00 . Se a temperatura inicial do gás era de 150°C, determine a
b) 4,00  e 2,00  temperatura final, em °C,
c) 10,0  e 2,00 
GABARITO
d) 2,00  e 4,00 
EXERCÍCIOS PROPOSTOS
e) 4,00  e 10,0  01. B 05. D 09. A 13. D 17. C
02. A 06. D 10. D 14. E 18. D
20. (UPF 2018) Dois mols de um gás ideal, inicialmente sob 03. C 07. D 11. D 15. B 19. E
pressão de 1,01·105Pa, temperatura de -10°C e volume de 4 m3, 04. A 08. D 12. C 16. C 20. E
são submetidos a uma transformação isobárica, elevando seu EXERCÍCIOS DE APROFUNDAMENTO
volume até 8 m3. 01. 8. 02. 2. 03. 0,92 atm. 04. 0,6. 05. 9°C.
Nessas condições, é possível afirmar que a temperatura final do
ANOTAÇÕES
gás, em graus Celsius, é de:
a) 526 b) 131,5 c) 20 d) 10 e) 253

EXERCÍCIOS DE
05. APROFUNDAMENTO
01. Um gás ideal passa por uma transformação termodinâmica em
que sua pressão dobra, seu número de moléculas triplica, e seu
volume é multiplicado por um fator de 12. Nessa transformação,
qual a razão entre as temperaturas absolutas final e inicial do gás?

02. Sabe-se que a pressão que um gás exerce sobre um recipiente


é decorrente dos choques de suas moléculas contra as paredes do
recipiente.
Diminuindo em 50% o volume do recipiente que contém um gás
ideal, sem alterar sua temperatura, estabeleça a razão entre a
pressão final e a pressão inicial.

214 PROENEM.COM.BR PRÉ-VESTIBULAR


07
FÍSICA II
LEI DE CLAPEYRON

EQUAÇÃO DE CLAPEYRON
No módulo anterior estudamos as transformações gasosas p·V
Pela equação de Clapeyron: P.V = n.R.T; n =
nas quais a massa do gás (número de mols) se mantinha R·T
Como: n = n1+n2
constante. Coube a Clapeyron desenvolver uma equação aplicável
às situações nas quais a massa do gás varia.
Seu raciocínio foi simples: p·V p1 ·V1 p2 ·V2
Teremos: = +
Pela equação geral dos gases: R·T R1·T1 R2 ·T2
Reprodução proibida Art. 184 do CP.

p ·V Como R é o termo comum, pode ser eliminado:


=k
T
(k constante que depende do gás com que se trabalha). p·V p ·V p ·V
= 1 1+ 2 2
T T1 T2
Estudando várias situações ele pode substituir a constante
k pelo produto do número de mols do gás (n) vezes uma nova
constante (R), só que esta constante se aplica a todos os gases, As condições finais da mistura é a soma das condições dos
ela fica conhecida como Constante Universal dos Gases. E onde gases componentes.
aparece a massa?
Lembre-se que:
EXERCÍCIOS
número de mols =
massa
massa molar
PROTREINO
A equação de Clapeyron pode ser escrita como: 01. Calcule o número de mols de um gás que ocupa volume de 41
p · V= n · R · T litros. Este gás está a uma pressão de 2 atm e a uma temperatura
Em unidades usuais: de 100K. (Dado: R = 0,082 atm.L/mol.K)
atm·L
R = 0,082 02. Calcule o volume, em litros, aproximadamente, de um mol de um
mol·K
gás ideal, cujas condições estejam na CNTP, ou seja, a temperatura
Em unidades SI: à 0°C e a pressão a 1 atm. (Dado: R = 0,082 atm.L/mol.K)

J 03. Um certo gás, cuja massa vale 140g, ocupa um volume


R = 8,31
mol·K de 41 litros, sob pressão 2,9 atmosferas a temperatura
de 17°C. Considere o número de Avogadro 6,0·1023 e a
Mas não se preocupe, pois em todos os vestibulares, se for constante universal dos gases perfeitos R=0,082 atm.L/mol.K.
preciso usar qualquer um dos valores, estes serão informados. Nessas condições, calcule o número de moléculas continuadas no
gás é aproximadamente de:
É interessante ressaltar que o número de moléculas de
1 mol vale 6,02 · 1023, como foi dito no módulo anterior.Além disso,
que esse número de moléculas é uma constante universal. Se 04. Uma quantidade de matéria igual a 5mol de gás ideal a uma
possuirmos um mesmo volume de gases diferentes, a uma mesma temperatura de 27ºC ocupa um volume de 16,4 litros. Calcule a
temperatura e pressão, eles terão um mesmo número de moléculas, pressão exercida por essa quantidade de gás é: (Dado: R: 0,082
isso para qualquer gás ideal, sem dúvida algo surpreendente e nem atm .L/mol.K)
um pouco óbvio.
05. Um gás ideal está confinado em um recipiente cúbico de aresta
igual a 0,5 m. A pressão exercida sobre as paredes do recipiente
MISTURA DE GASES PERFEITOS corresponde a 59760 Pa. Sabendo que a temperatura do gás é de
Considere dois recipientes separados por uma válvula. O 150 K, determine o número de moléculas contidas no recipiente.
primeiro recipiente possui volume V1 e número de mols igual a n1, Dado: Considere R = 8,3 (J/mol.K)
o segundo recipiente possui volume V2 e n2 mols. Ao abrirmos a
válvula, ocorrerá uma mistura dos dois gases, sendo que o número
de mols final será a soma do número de mols em cada recipiente:

EXERCÍCIOS
PROPOSTOS
01. A figura abaixo apresenta um diagrama Pressão x Volume.
Nele, os pontos M, N e R representam três estados de uma mesma
amostra de gás ideal.

PRÉ-VESTIBULAR PROENEM.COM.BR 215


FÍSICA II 07 LEI DE CLAPEYRON

06. O gráfico da pressão (P) em função do volume (V) no desenho


abaixo representa as transformações sofridas por um gás ideal. Do
ponto A até o ponto B, o gás sofre uma transformação isotérmica;
do ponto B até o ponto C, sofre uma transformação isobárica e do
ponto C até o ponto A, sofre uma transformação isovolumétrica.
Considerando TA, TB, e TC as temperaturas absolutas do gás nos
pontos A, B e C, respectivamente, pode-se afirmar que:
Reprodução proibida Art. 184 do CP.

Assinale a alternativa que indica corretamente a relação entre as


temperaturas absolutas TM, TN, TR dos respectivos estados M, N e R.
a) TA = TB e TB < TC
a) TR<TM>TN
b) TA = TB e TB > TC
b) TR>TM>TN
c) TA = TC e TB > TA
c) TR=TM>TN
d) TA = TC e TB < TA
d) TR<TM<TN
e) TA = TB = TC
e) TR=TM<TN
07. 0,5 moles de um gás ocupam um volume V de 0,1 m³ quando a
02. Considere um gás ideal que passa por dois estados, através de uma temperatura de 300 K. Qual é a pressão do gás a 300 K?
um processo isotérmico reversível. Sobre a pressão P e o volume V
Considere R = 8,3 J/ mol K.
desse gás, ao longo desse processo, é correto afirmar-se que
a) 830 Pa d) 12450 Pa
a) PV é crescente de um estado para outro.
b) 1245 Pa e) 18300 Pa
b) PV é constante.
c) 1830 Pa
c) PV é decrescente de um estado para outro.
d) PV é inversamente proporcional à temperatura do gás
08. Na superfície da Terra, a pressão, a temperatura e a densidade
e) PV é nulo. do ar (considerado um gás ideal) foram medidas por aparelhos que
forneceram os seguintes valores, respectivamente, 754 mm de Hg,
03. Em um reator nuclear, a energia liberada na fissão de 1 g de 17°C e 1,30 kg/m³. A uma altitude de 10 km, a pressão do ar aferida
urânio é utilizada para evaporar a quantidade de 3,6 ·104 kg de água foi 230 mm de Hg e a temperatura foi 43°C negativos. A densidade
a 227 ºC e sob 30 atm, necessária para movimentar uma turbina do ar, em kg/m³, medida nesta altitude foi de:
geradora de energia elétrica. Admita que o vapor d’água apresenta a) 0,75 b) 0,30 c) 0,15 d) 0,50 e) 0,65
comportamento de gás ideal. O volume de vapor d’água, em litros,
gerado a partir da fissão de 1 g de urânio, corresponde a:
09. Considere que a bola tenha um volume de 4 × 10-3 m³ e que
a) 1,32⋅105 d) 7,42⋅108 a pressão do ar, no seu interior, seja de 5 × 105 N/m², quando a
b) 2,67⋅106 e) 9,2⋅108 temperatura for de 27°C. Sabendo que o valor da constante
c) 3,24⋅107 universal dos gases é R = 8,31 J/mol K e que o ar, nessas condições,
comporta-se, aproximadamente, como gás ideal, a quantidade de
ar dentro da bola, em mol, é de, aproximadamente,
04. A bola utilizada em uma partida de futebol é uma esfera de
diâmetro interno igual a 20 cm. Quando cheia, a bola apresenta, a) 0,8
em seu interior, ar sob pressão de 1,0 atm e temperatura de 27 OC. b) 1,25
Considere π = 3, R = 0,080 atm·L·mol1·k-1 e, para o ar, comportamento c) 8,8
de gás ideal e massa molar igual a 30 g.mol-1. d) 80
No interior da bola cheia, a massa de ar, em gramas, corresponde a: e) 160
a) 2,5 b) 5,0 c) 7,5 d) 10,0 e) 15,0
10. Um mol de gás ideal, à pressão de 16,6 atm, ocupa uma caixa
05. Um motorista calibra os pneus de seu carro com uma pressão cúbica cujo volume é de 0,001 m3. Qual a temperatura do gás e a
de 30 libras/pol2 a uma temperatura de 27ºC. Após uma viagem, força que o gás exerce sobre a tampa quadrada da caixa?
a temperatura deles subiu para 47ºC. Desprezando-se a variação (Considere 1,0 atm = 1,0 ·105 Pa, R = 8,3 J/mol K)
de volume dos pneus e sabendo-se que 10% da massa de ar
contida em um dos pneus escapou pela válvula durante a viagem, a) 100 K e 8,3 · 103 N
a pressão do ar neste pneu, ao término desta viagem, em libras/ b) 100 K e 16,6 · 103 N
pol2 é de, aproximadamente c) 166 K e 8,3 · 103 N
a) 25 d) 32 d) 200 K e 16,6 · 103 N
b) 26 e) 48 e) 200 K e 8,3 · 103 N
c) 29

216 PROENEM.COM.BR PRÉ-VESTIBULAR


07 LEI DE CLAPEYRON FÍSICA II

11. TEXTO PARA A PRÓXIMA QUESTÃO: 13. Nos últimos anos, o gás natural (GNV: gás natural veicular)
Utilize as informações a seguir para responder à(s) questão(ões). vem sendo utilizado pela frota de veículos nacional, por ser viável
economicamente e menos agressivo do ponto de vista ambiental.
Novas tecnologias de embalagens visam a aumentar o prazo
de validade dos alimentos, reduzindo sua deterioração e mantendo O quadro compara algumas características do gás natural e da
a qualidade do produto comercializado. Essas embalagens podem gasolina em condições ambiente.
ser classificadas em Embalagens de Atmosfera Modificada
Tradicionais (MAP) e Embalagens de Atmosfera Modificada em
Equilíbrio (EMAP). As MAP são embalagens fechadas que podem
utilizar em seu interior tanto gases como He, Ne, Ar e Kr, quanto
composições deCO2 e O2 em proporções adequadas. As EMAP
também podem utilizar uma atmosfera modificada formada
por CO2 e O2 e apresentam microperfurações na sua superfície,
conforme ilustrado abaixo.
Apesar das vantagens no uso de GNV, sua utilização implica
algumas adaptações técnicas, pois, em condições ambiente, o
VOLUME de combustível necessário, em relação ao de gasolina,
Reprodução proibida Art. 184 do CP.

para produzir a mesma energia, seria


a) muito maior, o que requer um motor muito mais potente.
b) muito maior, o que requer que ele seja armazenado a alta
pressão.
c) igual, mas sua potência será muito menor.
d) muito menor, o que o torna o veículo menos eficiente.
e) muito menor, o que facilita sua dispersão para a atmosfera.

14. Analise a figura abaixo.

Admita que, imediatamente após a colocação do gás argônio em


uma embalagem específica, esse gás assume o comportamento Após uma lavagem, certa quantidade de vapor d’água, na
de um gás ideal e apresenta as seguintes características: temperatura inicial de 27 °C, permaneceu confinada no interior de
– Pressão =1 atm um tanque metálico. A redução da temperatura para 7,0 °C causou
condensação e uma consequente redução de 50% no número de
– Temperatura =300 K moléculas de vapor. Suponha que o vapor d’água se comporte
– Massa =0,16 g como um gás ideal ocupando um volume constante. Se a pressão
– R= 0,08 atm.L/k.mol inicial for 3,0·103 Pa, a pressão final, em quilopascal, será
Nessas condições, o volume, em mililitros, ocupado pelo gás na a) 1,4 b) 1,5 c) 2,0 d) 2,8 e) 2,9
embalagem é:
a) 96 b) 85 c) 77 d) 64 15. Na figura a seguir, temos um êmbolo de massa M que
se encontra em equilíbrio dentro de um recipiente cilíndrico,
termicamente isolado e que está preenchido por um gás ideal de
12. Uma panela de pressão com volume interno de 3,0 litros e
temperatura T.
contendo 1,0 litro de água é levada ao fogo. No equilíbrio térmico, a
quantidade de vapor de água que preenche o espaço restante é de
0,2 mol. A válvula de segurança da panela vem ajustada para que
a pressão interna não ultrapasse 4,1 atm. Considerando o vapor
de água como um gás ideal e desprezando o pequeno volume de
água que se transformou em vapor, calcule a temperatura, em 102
K, atingida dentro da panela.

Acima do êmbolo, o volume de gás é quatro vezes maior que o


abaixo dele, e as massas de cada parte do gás bem como suas
temperaturas são sempre idênticas. Se o êmbolo tiver sua massa
dobrada e não houver variações nos volumes e nas massas de
a) 4,0 d) 4,7 cada parte do gás, qual é a relação entre a nova temperatura, T’, e
a anterior de maneira que ainda haja equilíbrio? Despreze o atrito.
b) 4,2 e) 5,0
a) T’ = 3T/4 c) T’ = T e) T’ = 4T
c) 4,5
b) T’ = T/2 d) T’ = 2T

PRÉ-VESTIBULAR PROENEM.COM.BR 217


FÍSICA II 07 LEI DE CLAPEYRON

16. (UECE 2019) Considere dois balões infláveis, de propaganda, No primeiro tempo mostrado, há uma compressão máxima do gás
fabricados com tecido de poliéster inextensível. Um dos balões dentro do cilindro, o qual exerce uma pressão p sobre as paredes
tem iluminação interna feita com uma lâmpada incandescente, que do cilindro a uma temperatura T. No segundo tempo mostrado, o
dissipa muita energia por efeito Joule, e o outro com uma lâmpada volume ocupado pelo gás é máximo, 3 vezes maior que o anterior,
LED, de baixa dissipação se comparada à incandescente. exercendo uma pressão 3 vezes menor que p, a uma temperatura
Supondo que, após inflados com a mesma pressão, os balões 2 vezes maior que T. Durante a expansão volumétrica, a rápida
sejam vedados e não tenham vazamentos, é correto afirmar que, abertura de uma válvula de escape permitiu a liberação de certa
após ligadas as iluminações dos dois balões, quantidade de gás para a fonte fria. A relação entre o número (n1)
de mols do gás que havia no interior do cilindro no primeiro tempo
a) o balão com a lâmpada incandescente terá sua pressão interna e o número (n2) de mols do gás que permaneceu no cilindro no
menor que a do balão com LED. segundo tempo, n1/n2, é igual a
b) as temperaturas nos balões se manterão iguais, tendo em vista a) 12. b) 3. c) 4. d) 2,5. e) 2.
que as pressões iniciais eram idênticas.
c) o balão com a lâmpada incandescente terá sua temperatura 19. (UECE 2017) Considere dois sistemas compostos por gases
interna menor que a do balão com LED. ideais, com massas moleculares diferentes, cada um em um
Reprodução proibida Art. 184 do CP.

d) o balão com a lâmpada incandescente terá sua pressão interna recipiente com isolamento térmico. A pressão, o volume e a
maior que a do balão com LED. PV
temperatura são tais que é o mesmo para ambos.
RT
É correto afirmar que
17. (EEAR 2018) O gráfico que melhor representa a expansão de
uma amostra de gás ideal a pressão constante é: a) o número de moles de gás em cada recipiente é igual, assim
como as massas também são iguais.
Considere:
b) o número de moles de gás em cada recipiente é diferente, mas
1. a temperatura (K) dada em kelvin (K) e
as massas são iguais.
2. V= volume.
c) o número de moles de gás em cada recipiente é igual, mas as
a) b) massas são diferentes.
d) o número de moles de gás em cada recipiente é diferente,
assim como as massas são diferentes.

20. (UECE 2015) Considere um gás ideal em um recipiente mantido


a temperatura constante e com paredes móveis, de modo que se
possa controlar seu volume. Nesse recipiente há um vazamento
muito pequeno, mas o volume é controlado lentamente de modo
c) d) que a razão entre o número de moles de gás e seu volume se
mantém constante. Pode-se afirmar corretamente que a pressão
desse gás
a) é crescente.
b) é decrescente.
c) varia proporcionalmente ao volume.
d) é constante.

18. (INSPER 2018) A figura ilustra 2 instantes em que o pistão de


uma máquina térmica ocupa duas posições (tempos) de volumes EXERCÍCIOS DE
APROFUNDAMENTO
extremos.
05.

01. Analise a figura abaixo.

Após uma lavagem, certa quantidade de vapor d’água, na temperatura


inicial de 27 °C, permaneceu confinada no interior de um tanque
metálico. A redução da temperatura para 7,0 °C causou condensação
e uma consequente redução de 50% no número de moléculas de vapor.
Suponha que o vapor d’água se comporte como um gás ideal ocupando
um volume constante. Se a pressão inicial for 3,0·103 Pa, calcule a
pressão final, em quilopascal.

218 PROENEM.COM.BR PRÉ-VESTIBULAR


07 LEI DE CLAPEYRON FÍSICA II

02. Conforme mostra a figura abaixo, dois recipientes, A e B, 04. (UFMG 2013) Na figura está representado um pistão
termicamente isolados, de volumes iguais, estão ligados por um constituído de um cilindro e um êmbolo. O êmbolo, que pode se
tubo delgado que pode conduzir gases, mas não transfere calor. mover livremente, tem massa de 0,30 kg e uma área de seção
Inicialmente, os recipientes são ocupados por uma amostra de um transversal de 8,0 cm2.
certo gás ideal na temperatura T0 e na pressão P0. Considere que a
Esse pistão contém 4,0 × 10−3 mol de um gás ideal à temperatura
temperatura no recipiente A é triplicada, enquanto a do recipiente B
se mantém constante. Determine a razão entre a pressão final nos de 27°C. A pressão no ambiente é de 1,0 atm.
P
dois recipientes e a pressão inicial, ,
P0
Reprodução proibida Art. 184 do CP.

03. (FUVEST 2018) O motor Stirling, uma máquina térmica de alto


rendimento, é considerado um motor ecológico, pois pode funcionar a) DETERMINE o valor da força que o gás exerce sobre o êmbolo
com diversas fontes energéticas. A figura I mostra esquematicamente na situação de equilíbrio.
um motor Stirling com dois cilindros. O ciclo termodinâmico de Stirling, b) DETERMINE o valor da altura h em que o êmbolo se encontra
mostrado na figura II, representa o processo em que o combustível é nessa situação.
queimado externamente para aquecer um dos dois cilindros do motor,
Em seguida, o gás é aquecido até que sua temperatura atinja 57°C.
sendo que uma quantidade fixa de gás inerte se move entre eles,
expandindo-se e contraindo-se. c) DETERMINE o valor do deslocamento Dh do pistão devido a
Nessa figura está representado um ciclo de Stirling no diagrama esse aquecimento.
P × V para um mol de gás ideal monoatômico. No estado A, a
pressão é PA = 4 atm, a temperatura é T= 27° C e o volume é VA . 05. Considere que a bola tenha um volume de 4 · 10-3 m3 e que
1
A partir do estado A, o gás é comprimido isotermicamente até um a pressão do ar, no seu interior, seja de 5 · 105 N/m2 , quando a
terço do volume inicial, atingindo o estado B. Na isoterma T1, a temperatura for de 27°C.
quantidade de calor trocada é Q1 = 2.640 J, e, na isoterma T2 , é Sabendo que o valor da constante universal dos gases é R = 8,31 J/
Q2 = 7.910 J. mol K e que o ar, nessas condições, comporta-se, aproximadamente,
como gás ideal, calcule a quantidade de ar dentro da bola, em mol.

GABARITO
EXERCÍCIOS PROPOSTOS
01. E 05. C 09. A 13. B 17. A
02. B 06. A 10. D 14. A 18. E
03. B 07. D 11. A 15. D 19. C
04. B 08. D 12. E 16. D 20. D

EXERCÍCIOS DE APROFUNDAMENTO
01. 1,4kPa
02. 3/2
03.
a) Pela equação de Clayperon, temos:
PA ⋅ VA = n ⋅ R ⋅ TA
4 ⋅ VA =⋅
1 0,08 ⋅ 300
Determine
∴ VA =
6L
a) o volume VA , em litros;
b) Entre os estados A e B (com VB=VA/3 e TA = TB), temos:
b) a pressão PD, em atm, no estado T2 . PA ⋅ VA PB ⋅ VB
=
TA TB
c) a temperatura T2 .
4 ⋅ 6 = PB ⋅ 6 3
Note e adote: ∴PD = PB = 12 atm

Calor específico a volume constante: CV = 3 R 2 c) Entre os estados A e D (com VA= VD), temos:
Constante universal dos=
gases: R 8=
J (mol K) 0,08 atm  (mol K) PA ⋅ VA PD ⋅ VD
=
TA TD
0 °C =273 K 4 12
=
1atm = 105Pa 300 TD
∴ TD =
900 K
1m3 = 1.000 
04. a) Fgas = 83 N b) h = 0,12 m c) Dh = 0,012 m.
05. 20.102 = 2493.n → n = 0,80 mol

PRÉ-VESTIBULAR PROENEM.COM.BR 219


FÍSICA II 07 LEI DE CLAPEYRON

ANOTAÇÕES
Reprodução proibida Art. 184 do CP.

220 PROENEM.COM.BR PRÉ-VESTIBULAR


08
FÍSICA II
TERMODINÂMICA – 1ª LEI

ENERGIA INTERNA Se o volume final for maior que o volume inicial, o gás sofreu
uma expansão, o trabalho é positivo e dessa forma diremos que foi
A energia interna (U) de um gás perfeito é a energia cinética
realizado trabalho pelo gás.
total média de suas moléculas, conhecida como teoria cinética
dos gases. Experimentalmente é possível perceber que a energia Se o volume final for menor que o volume inicial, o gás sofreu
cinética das moléculas de diferentes gases depende apenas da uma compressão, o trabalho será negativo e diremos que o trabalho
temperatura e, claro, do número de moléculas existentes. foi realizado sobre o gás.
Podemos afirmar que um gás sofreu uma variação de sua Obviamente, não havendo variação de volume, não haverá
energia interna quando houver uma variação de temperatura. Para
Reprodução proibida Art. 184 do CP.

trabalho realizado.
um gás monoatômico vale a relação:

3 Resumindo:
U
∆= n·R·∆T
2 • Vf > Vl → ∆V > 0 → τ > 0 → Expansão: o trabalho foi realizado
pelo gás.
Para um gás diatômico, os graus de liberdade das moléculas
aumentam, alterando a energia cinética média das moléculas.
• Vf < Vl → ∆V < 0 → τ < 0 → Compressão: o trabalho foi
Agora, além dos movimentos de translação, as moléculas possuem
realizado sobre o gás.
movimento de rotação associados, contribuindo com o aumento
dessa energia. Com isso, para um gás diatômico, temos:
• Vf = Vl → ∆V = 0 → τ = 0 → Transformação Isométrica, não
5 há realização de trabalho.
U
∆= n·R·∆T
2
Outra maneira de se calcular o trabalho realizado ou sofrido por
Como a variação de energia interna de um gás é função da um gás é através do gráfico da pressão em função do volume (P x
variação de temperatura é bom que tenhamos estas relações: V). A área abaixo da curva representa o trabalho.
∆T > 0 → ∆U > 0
∆T = 0 → ∆U = 0
∆T < 0 → ∆U < 0

TRABALHO DE UM GÁS
Dizemos que um gás realizou trabalho ou sofreu trabalho
quando, contido em um recipiente, ele tem seu volume expandido
ou comprimido.

1ª LEI DA TERMODINÂMICA
A quantidade de calor trocado com o meio externo Q é igual à
soma algébrica do trabalho realizado pelo gás ou sobre gás com a
variação de energia interna do gás.
A primeira Lei da Termodinâmica ou Princípio da conservação
de energia afirma que um gás ao receber uma certa quantidade
de calor, que é uma forma de energia, utiliza esta energia de duas
O trabalho de um gás pode ser calculado pelo produto da pressão maneiras: realizando trabalho e variando sua energia interna. O
média exercida sobre o gás pela variação de volume, depois de ter sido
contrário também é válido, isto é, sofrendo trabalho e tendo sua
deduzido de sua definição mecânica como o produto da força paralela
pelo deslocamento, sendo no caso termodinâmico a força exercida energia interna reduzida, o gás libera energia ao meio externo.
pelo/sobre o gás e o deslocamento do êmbolo de um pistão. Algebricamente:

τ =F·d
mas F = p·A e d = ∆h ∆U = Q – τ
=τ p · A· ∆h
como ∆V= A · ∆h Sobre o calor trocado Q teremos também os sinais positivo e
τ p · ∆V
= negativo:
Q > 0 → calor recebido pelo gás.
Podemos perceber que o trabalho poderá ser positivo ou Q < 0 → calor cedido pelo gás.
negativo, dependendo da variação de volume.

PRÉ-VESTIBULAR PROENEM.COM.BR 221


FÍSICA II 08 TERMODINÂMICA – 1ª LEI

05. Considere uma certa massa de um gás ideal em equilíbrio


EXERCÍCIOS termodinâmico. Numa primeira experiência, faz-se o gás sofrer
PROTREINO uma expansão isotérmica durante a qual realiza um trabalho W e
recebe 150J de calor do meio externo. Numa segunda experiência,
faz-se o gás sofrer uma expansão adiabática, a partir das mesmas
01. Considere um gás diatômico (ar) de calor específico molar a
5 condições iniciais, durante a qual ele realiza o mesmo trabalho W.
volume constante CV = ·R , inicialmente à pressão atmosférica Calcule a variação de energia interna ∆U do gás nessa expansão
2
e a 27°C. Esse gás encontra-se contido dentro de um calorímetro adiabática.
de volume 24,7 litros e é, então, aquecido, a volume constante, até
aumentar sua temperatura em 150°C.
Dados: Patm = 1,01 · 105 Pa ; R = 8,31 J/mol.K
a) Calcule a pressão do gás ao fim do processo.
EXERCÍCIOS
b) Calcule a quantidade de calor absorvida pelo gás.
PROPOSTOS
Reprodução proibida Art. 184 do CP.

02. Em um motor de automóvel, a mistura de combustível com ar


é comprimida pelo pistão antes da ignição a uma taxa de 10,0 para 01. Um estudo do ciclo termodinâmico sobre um gás que está
1,0, ou seja, o volume final do cilindro é 10 vezes menor que o volume sendo testado para uso em um motor à combustão no espaço é
inicial, como mostrado na figura abaixo (figura fora de escala). mostrado no diagrama a seguir.

Considere que não haja trocas de calor entre a mistura no interior


do cilindro e sua vizinhança, que as dimensões do equipamento
não sofram variações significativas com a temperatura, e que a
mistura tenha comportamento semelhante ao de um gás ideal e
faça o que se pede.
a) Se a pressão inicial e a temperatura inicial valem 1,5 atm e
127°C respectivamente, e a pressão final é de 30 atm, calcule,
em graus Celsius, a temperatura da mistura na situação final. Se ∆Eint representava a variação de energia interna do gás, e Q
é o calor associado ao ciclo, analise as alternativas e assinale a
b) Explique a variação de temperatura sofrida pela mistura.
CORRETA.

03. Um gás ideal em equilíbrio termodinâmico tem pressão de a) ∆Eint = 0, Q > 0 d) ∆Eint < 0, Q > 0
1,0×105 N/m2, volume de 2,0×10−3 m3 e temperatura de 300 K. O b) ∆Eint = 0, Q < 0 e) ∆Eint = 0, Q = 0
gás é aquecido lentamente à pressão constante recebendo uma c) ∆Eint > 0, Q < 0
quantidade de 375 J de calor até atingir um volume de 3,5×10−3 m3,
no qual permanece em equilíbrio termodinâmico. 02. No estudo da termodinâmica dos gases perfeitos, são
a) Calcule a temperatura do gás em seu estado final de equilíbrio. parâmetros básicos as grandezas físicas quantidade de calor (Q),
trabalho (W) e energia interna (U), associadas às transformações
b) Calcule a variação da energia interna do gás entre os estados
que um gás perfeito pode sofrer.
inicial e final.
Analise as seguintes afirmativas referentes às transformações
04. A figura a seguir representa o gráfico p-V de um gás, suposto termodinâmicas em um gás perfeito:
ideal, que sofre primeiramente um processo isobárico, partindo I. Quando determinada massa de gás perfeito sofre uma
do ponto A para o ponto B, e depois um processo isovolumétrico, transformação adiabática, o trabalho (W) que o sistema
atingindo o ponto C, que se situa sobre a mesma isoterma que A. troca com o meio externo é nulo.
II. Quando determinada massa de gás perfeito sofre uma
transformação isotérmica, a variação da energia interna é
nula (∆U = 0 )
III. Quando determinada massa de gás perfeito sofre uma
transformação isométrica, a variação da energia interna
(∆U) sofrida pelo sistema é igual a quantidade de calor (Q)
trocado com o meio externo.
Está (ão) correta (s) apenas a(s) afirmativa (s)
a) I.
b) III.
Calcule c) I e II.
a) o trabalho realizado pelo gás ao final do processo ABC; d) II e III.
b) o calor recebido pelo gás ao final do processo ABC. e) I, II e III.

222 PROENEM.COM.BR PRÉ-VESTIBULAR


08 TERMODINÂMICA – 1ª LEI FÍSICA II

03. O processo de expansão ou compressão de um gás em um curto 08. De acordo com dados de um fabricante de fogões, uma panela
intervalo de tempo pode representar um processo termodinâmico com 2,2 litros de água à temperatura ambiente chega a 90 ºC em
que se aproxima de um processo adiabático. Como exemplo, pode- pouco mais de seis minutos em um fogão elétrico. O mesmo teste
se mencionar a expansão de gases de combustão em um cilindro foi feito em um fogão convencional, a GLP, sendo necessários 11,5
de motor de automóvel em alta rotação. minutos. Sobre a água aquecida, é correto afirmar que
É correto afirmar que, em um processo adiabático no sistema, a) adquiriu mais energia térmica no fogão convencional.
a) a temperatura é constante e o trabalho realizado pelo sistema b) adquiriu mais energia térmica no fogão elétrico.
é nulo. c) ganha a mesma energia térmica para atingir 90 ºC nas duas
b) não há transferência de calor. experiências.
c) a pressão e o volume são constantes. d) nos dois experimentos o ganho de energia térmica não depende
d) a energia interna é variável e a pressão é constante. da variação de temperatura sofrida.
e) a energia interna se mantém constante. e) a potência do fogão convencional é maior que do fogão elétrico.

04. Num sistema termodinâmico um gás ideal, ao receber 300 J do 09. Um gás ideal é submetido a um processo termodinâmico
Reprodução proibida Art. 184 do CP.

meio externo, realiza um trabalho de 200 J. É correto afirmar que: ABCD, conforme ilustra a figura a seguir.
a) a transformação é adiabática.
b) a temperatura do sistema aumentou.
c) o volume do gás permanece constante.
d) a variação de energia interna é negativa.
e) não há variação da energia interna.

05. Sob condições de pressão constante, certa quantidade de calor


Q, fornecida a um gás ideal monoatômico, eleva sua temperatura
em ∆T
Quanto calor seria necessário, em termos de Q, para concluir a Sabendo que o trabalho total associado a esse processo é igual a
mesma elevação de temperatura ∆T se o gás fosse mantido em 1050J, qual o trabalho no subprocesso BCD?
volume constante? a) 60 J
a) 3Q b) 5Q/3. c) Q. d) 3Q/5. e) 2Q/5. b) 340 J
c) 650 J
06. O gasto de energia pelo corpo humano depende da atividade d) 840 J
física em execução. Ficar sentado consome de 3 a 7 KJ/min, em pé
há um gasto de 6 a 10 KJ/min, caminhar consome de 5 a 22 KJ/min e) 990 J
e jogar voleibol faz uso de 14 a 39 KJ/min.
10. Na tabela abaixo, EH , EO , VH , VO são, respectivamente, as
Considerando as taxas máximas de consumo energético, pode-se 2 2 2 2
energias cinéticas médias e as velocidades médias das moléculas
dizer corretamente que as atividades que mais preservam recursos
de uma amostra de gás H2 e de outra, de gás O2, ambas em
energéticos no organismo são, em ordem crescente:
temperatura de 27ºc.
a) sentado, em pé, caminhada, voleibol.
b) voleibol, caminhada, em pé, sentado.
c) sentado, em pé, voleibol, caminhada. Temperatura Energia cinética Velocidade
Gás
d) voleibol, caminhada, sentado, em pé. (ºc) média média
e) em pé, voleibol, caminhando, sentado.

07. A energia interna de um gás perfeito (gás ideal) tem dependência H2 27 EH VH


2 2
somente com a temperatura. O gráfico que melhor qualifica essa
dependência é
O2 27 EO VO
2 2
a) c)

Assinale a alternativa que relaciona corretamente os valores das


energias cinéticas médias e das velocidades médias das moléculas
de H2 e de O2
a) EH > EO e VH > VO
2 2 2 2
b) EH < EO e VH < VO
2 2 2 2
b) d) c) EH = EO e VH > VO
2 2 2 2

d) EH = EO e VH = VO
2 2 2 2

PRÉ-VESTIBULAR PROENEM.COM.BR 223


FÍSICA II 08 TERMODINÂMICA – 1ª LEI

11. A figura representa uma transformação termodinâmica da 14. Certa quantidade de gás sofre três transformações sucessivas,
mudança do estado inicial A para o estado final B de uma massa A → B, B → C e C → A, conforme o diagrama p – V apresentado na
de gás ideal e pode ser feita pelo “caminho” I ou pelo “caminho” II. figura abaixo.
Uma análise do gráfico, associada aos conhecimentos de
termodinâmica, permite concluir:
a) A temperatura da massa de gás no estado A é maior do que
no estado B.
b) A variação da energia interna
do gás no “caminho” I é
maior do que no “caminho” II.
c) A quantidade de calor A respeito dessas transformações, afirmou-se o seguinte:
trocada pela massa de gás
no “caminho” I é igual a I. O trabalho total realizado no ciclo A→B, é nulo.
4,15·104J. II. A energia interna do gás no estado C é maior que no estado A.
Reprodução proibida Art. 184 do CP.

d) O trabalho realizado pela III. Durante a transformação A→B, o gás recebe calor e realiza
massa de gás no “caminho” II trabalho.
tem módulo igual a 6,0·103J. Está correto o que se afirma em:
e) A quantidade de calor a) I. b) II. c) III. d) I e II. e) II e III.
trocada pela massa de gás
no “caminho” II é da ordem
15. O gráfico representa, em um processo isobárico, a variação em
de 104J.
função do tempo da temperatura de uma amostra de um elemento
puro cuja massa é de 1,0 kg observada durante 9 minutos.
12. A primeira lei da termodinâmica para sistemas fechados foi
originalmente comprovada pela observação empírica, no entanto
é hoje considerada como a definição de calor através da lei da
conservação da energia e da definição de trabalho em termos de
mudanças nos parâmetros externos de um sistema.
Com base nos conhecimentos sobre a Termodinâmica, é correto
afirmar:
a) A energia interna de uma amostra de um gás ideal é função da
pressão e da temperatura absoluta.
b) Ao receber uma quantidade de calor Q igual a 48,0 J um gás
realiza um trabalho igual a 16,0 J tendo uma variação da O processo que ocorre na fase sólida envolve um trabalho total de
energia interna do sistema igual 64,0 J. 0,1 kJ. Nessa fase, a variação da energia interna da amostra é
c) Quando se fornece a um sistema certa quantidade de energia a) 6,1 kJ. b) 5,9 kJ. c) 6,0 kJ. d) -5,9 kJ. e) -6,1kJ.
Q, esta energia pode ser usada apenas para o sistema realizar
trabalho. 16. (EFOMM 2019) Um mol de um gás ideal monoatômico vai do
estado a ao estado c, passando pelo estado b com pressão, como
d) Nos processos cíclicos, a energia interna não varia, pois volume,
mostrado na figura abaixo. A quantidade de calor Q que entra no
pressão e temperatura são iguais no estado inicial e final. sistema durante esse processo é de aproximadamente:
e) A energia interna, o trabalho realizado e a quantidade de
calor recebida ou cedida independem do processo que leva o
sistema do estado inicial A até um estado final B.

13. Um fluido se expande do estado A para o estado B, como


indicado no diagrama da figura.

a) 4.000 J.
b) 5.000 J.
c) 6.000 J.
d) 7.000 J.
e) 8.000 J.

Analisando-se essas informações, é correto afirmar que o trabalho 17. (UPF 2018) São várias as reportagens veiculadas na mídia que
realizado nessa expansão, em kJ, é igual a mostram pessoas tentando construir um motor que não necessita
a) 2,3 b) 2,2 c) 2,1 d) 2,0 e) 1,9 fornecimento contínuo de energia externa para funcionar, ao que se

224 PROENEM.COM.BR PRÉ-VESTIBULAR


08 TERMODINÂMICA – 1ª LEI FÍSICA II

denomina de “moto perpétuo”. Essas máquinas têm como objetivo


gerar energia para manter o seu próprio movimento, bastando dar EXERCÍCIOS DE
um impulso inicial e o movimento se dará de forma perpétua.
Se essa máquina funcionasse, necessariamente se estaria violando a
05. APROFUNDAMENTO
a) Lei da Conservação de Energia. 01. (PUCRJ 2017) Seja um gás diatômico (ar) de calor específico
b) Primeira Lei de Newton. molar a volume constante CV = 5/2 R, inicialmente à pressão
atmosférica e a 27 °C, Esse gás encontra-se contido dentro de um
c) Lei da Conservação de Quantidade de Movimento. calorímetro de volume 24,7 litros e é, então, aquecido, a volume
d) Lei da Gravitação Universal. constante, até aumentar sua temperatura em 150 °C.
e) Equação geral dos gases. Dados: Patm=1,01 · 105 Pa;  R=8,31 J/mol · k
a) Calcule a pressão do gás ao fim do processo.
18. (UDESC 2018) Um gás ideal monoatômico, com n mols e
b) Calcule a quantidade de calor absorvida pelo gás.
inicialmente na temperatura absoluta T, sofre uma expansão
adiabática até que sua temperatura fique a um terço de sua
temperatura inicial. 02. (PUCRJ 2016) Um ciclo termodinâmico, para um mol de um
Reprodução proibida Art. 184 do CP.

Logo, o gás: gás monoatômico, consiste em 4 processos: AB→isobárico; BC


a) absorveu uma quantidade de calor igual a nRT. →isocórico; CD→isobárico; DA→isocórico; representados no
diagrama PV da figura.
b) se expandiu isobaricamente.
Sabe-se que PA=3,0 ·105 Pa, PC=1,0 ·105 Pa, VD=8,3 · 10-3m3 , VB=2,0VA.
c) realizou trabalho liberando uma quantidade de calor igual a nRT. Considere a constante universal dos gases R = 8,3 J/K . mol.
d) se expandiu aumentando sua energia interna de nRT.
e) realizou trabalho e sua energia interna diminuiu de nRT.

19. (UPE-SSA 2 2018) Dois moles de um gás ideal podem ser


levados do estado inicial a um estado final por três processos
diferentes (1, 2 e 3), conforme ilustra o diagrama pressão p em
função do volume V a seguir. Então, acerca do calor absorvido pelo
gás, é CORRETO afirmar que

a) Calcule as temperaturas máxima e mínima em que opera o ciclo.


b) Calcule o trabalho realizado pelo gás em um ciclo.

03. (UEL 2015) Analise o gráfico a seguir, que representa uma


transformação cíclica ABCDA de 1 mol de gás ideal.

a) é maior no processo 1 que no processo 3.


b) é maior no processo 3 que no processo 1.
c) o maior calor absorvido ocorre no processo 3.
d) o menor calor absorvido ocorre no processo 2.
e) são iguais nos processos 1 e 3.

20. (UFRGS 2017) Observe a figura abaixo.


A figura mostra dois processos, I e II, em um diagrama pressão (P)
x volume (V) ao longo dos quais um gás ideal pode ser levado do a) Calcule o trabalho realizado pelo gás durante o ciclo ABCDA.
estado inicial f para o estado final f. b) Calcule o maior e o menor valor
temperatura da
Assinale a alternativa que preenche corretamente as lacunas do absoluta do gás no ciclo J
(considere).
R=8
enunciado abaixo, na ordem em que aparecem. K mol
De acordo com a 1ª Lei da Termodinâmica, a variação da energia Justifique sua resposta apresentando todos os cálculos
interna é __________ nos dois processos. O trabalho WI realizado no realizados.
processo I é __________ que o trabalho
WII realizado no processo II. 04. (UEMA 2015) No controle de qualidade de produção de seringa,
a) igual − maior para aplicação de injeção, fez-se o seguinte teste: escolheu-se
uma amostra da seringa fabricada e colocou-se 3,0 · 10-6 m3 de
b) igual − menor
determinado gás. Em seguida, levou-se o sistema para uma estufa
c) igual − igual em que o volume passou para 3,5 · 10-6 m3 ao atingir o equilíbrio
d) diferente − maior térmico.
e) diferente − menor Considerando que esse processo ocorreu sobre pressão constante
de 1,5 ·105 Pa, calcule, em joule, o trabalho realizado pelo sistema.

PRÉ-VESTIBULAR PROENEM.COM.BR 225


FÍSICA II 08 TERMODINÂMICA – 1ª LEI

05. (UFES 2015) A figura abaixo apresenta um conjunto de transformações termodinâmicas sofridas por um gás perfeito. Na transformação
1→2, são adicionados 200 J de calor ao gás, levando esse gás a atingir a temperatura de 60°C no ponto 2. A partir desses dados, determine
Reprodução proibida Art. 184 do CP.

a) a variação da energia interna do gás no processo 1→2;


b) a temperatura do gás no ponto 5;
c) a variação da energia interna do gás em todo o processo termodinâmico 1→5.

GABARITO
EXERCÍCIOS PROPOSTOS
01. B 05. D 09. E 13. C 17. A
02. D 06. B 10. C 14. E 18. E
03. B 07. A 11. E 15. B 19. A
04. B 08. C 12. D 16. D 20. B

EXERCÍCIOS DE APROFUNDAMENTO
01. a) P2=1,515·105 Pa b) 3116,25 J

02. a) TD = 600 K   b) W = 1660 J

03. a) 40J   b) 1,25K

04. τ = 0,075J

05. a) ∆U = 200J  b) T5 = 60 °C c) ∆U15 = 200J

ANOTAÇÕES

226 PROENEM.COM.BR PRÉ-VESTIBULAR


09
FÍSICA II
TRANSFORMAÇÕES GASOSAS

TRANSFORMAÇÕES GASOSAS ISOMÉRICA, ISOVOLUMÉTRICA


O primeiro princípio da Termodinâmica possui características OU ISOCÓRICA
importantes quando tratamos das transformações gasosas: Em uma transformação isométrica não há variação de volume,
não ocorre realização de trabalho (τ = 0).
ISOBÁRICA
ΔU = Q – τ → ΔU = Q – 0
Nas transformações isobáricas, a pressão do sistema é
constante e o volume (V) varia na razão direta da temperatura
absoluta (T), ou seja, quando a temperatura aumenta o volume ΔU= Q
aumenta na mesma proporção, quando a temperatura diminui
o volume também diminui, na mesma proporção. Fique atento,
pois em transformações isobáricas há duas situações a serem Dessa forma, quando um gás recebe calor do meio externo,
consideradas. sua energia interna aumenta (TEMPERATURA AUMENTA) e quando
I. Quando a temperatura absoluta do gás aumenta, o volume um gás cede calor ao meio externo, sua energia interna diminui
aumenta. Lembre-se que quando a temperatura do sistema (TEMPERATURA DIMINUI).
aumenta a energia interna U do sistema também aumenta. Gráfico P x V
A energia entra no sistema na forma de calor.

∆U > 0 → Q > τ

II. Quando a temperatura absoluta do gás diminui, o volume


diminui. Lembre-se que quando a temperatura do sistema
diminui a energia interna U do sistema também diminui. A
energia sai do sistema na forma de calor.
∆U < 0 → Q < τ

Observe o gráfico PxV para transformação isobárica

P
ISOTÉRMICA
A energia interna do gás depende da temperatura. Logo,
em uma transformação isotérmica, como não há variação de
p temperatura, não há variação de energia interna (ΔU = 0).

 ΔU = Q – τ → 0 = Q – τ → Q=τ
O sistema se comporta como um conversor de energia: se ele
recebe energia em forma de calor ele cede em forma de trabalho,
ou ao contrário, recebe trabalho e cede calor.
V
V Gráfico P x V

A área do gráfico PxV é numericamente igual ao trabalho, logo


na transformação isobárica o trabalho τ é dado por:

τ = p·∆V
Usando a Equação de Clapeyron, completamos a relação da
seguinte forma:

τ = p·∆V= n·R·∆T

Pela 1ª Lei da Termodinâmica temos que ∆U = Q — τgás

Para transformações isobáricas ∆U = Q — p·∆V

PRÉ-VESTIBULAR PROENEM.COM.BR 207


FÍSICA II 09 TRANSFORMAÇÕES GASOSAS

ADIABÁTICA CALORES ESPECÍFICOS


Em uma transformação adiabática o sistema não cede nem
receba calor durante o processo. DOS GASES IDEAIS
Lembrando que ∆U= Q – τ, nesse caso, como não há troca de Podemos observar que as variações de temperatura obtidas
calor com o meio externo Q = 0 durante as transformações isométrica e isobárica, ocorrem com
trocas de calores diferentes. Ou seja, uma massa de ar que sofre
Logo teremos uma mudança em sua temperatura precisa ceder ou receber calor
de maneiras distintas dependendo da transformação.
∆U = -τ
Isométrica: ∆Uv = Qv
É importante notar que as transformações adiabáticas podem Isobárica: ∆Up = QP — W
ocorrer de duas formas;
Supondo ∆T iguais: ∆Up = ∆Uv
• Expansão adiabática QV  Qp  
Sempre que ocorrer uma expansão adiabática o gás Qp  QV   (Qp  QV )
exercerá trabalho sobre o meio externo para aumentar
Qp  QV   (   p  V  nRT)
de volume, perceba que o gás “gasta” energia para
expandir, por isso a energia interna do gás diminui e m
Qp  QV  nRT (n 
)
consequentemente o gás resfria. M
Vemos um exemplo de expansão adiabática diariamente m
mCp T  m CV T  R T
quando usamos desodorantes spray. M
MCp  MCV  R
Cp  CV  R

TRANSFORMAÇÕES CÍCLICAS
Podemos afirmar que um gás sofreu uma transformação
cíclica quando seu estado final é igual ao seu estado inicial.
No gráfico abaixo, o gás sofreu os processos AB, BC, CD e
DA sendo seu estado inicial A igual ao seu estado final. Portanto,
ele retorna à mesma temperatura, pressão e volume que possuía
O conteúdo líquido passa pelo pequeno orifício e seu inicialmente. Para um gás ideal em um sistema fechado, a energia
vapor se expande muito rapidamente contra a atmosfera, interna depende da temperatura, por isso, em um ciclo podemos
exercendo trabalho sobre ela. O efeito perceptível disso é o afirmar que não há variação de energia interna (ΔU=0).
resfriamento do vapor, o que era esperado, já que em toda
expansão adiabática o gás, nesse caso vapor, resfria.
• Compressão adiabática.
Sempre que ocorrer uma compressão adiabática o meio
exercerá trabalho sobre o gás para diminuir o volume do
gás, perceba que o gás recebe energia do meio externo
na forma de trabalho τ, por isso, a energia interna do gás
aumenta e consequentemente ele esquenta.

Se o ciclo ocorre no sentido horário teremos o τ > 0, ou seja, o


Resumindo
sistema recebe calor e o transforma em trabalho. Se o ciclo ocorre
Expansão adiabática → Gás esfria (temperatura diminui) no sentido anti-horário teremos τ < 0, o trabalho realizado sobre o
Compressão adiabática → Gás esquenta (temperatura aumenta) sistema é convertido em calor.
Gráfico

p Isotermas

Trabalho = Área interna do ciclo


Processo adiabático

τ > 0 – sentido horário


A área no gráfico que caracteriza a expansão é maior que a
V
área que representa a compressão. Portanto, o trabalho realizado
A curva adiabática é uma hipérbole que intercepta as isotermas. pelo gás supera o trabalho realizado sobre o gás.

208 PROENEM.COM.BR PRÉ-VESTIBULAR


09 TRANSFORMAÇÕES GASOSAS FÍSICA II

τ < 0 – sentido anti-horário


A área no gráfico que caracteriza a compressão é maior que WC > WB > WA
a área que representa a expansão. Portanto, o trabalho realizado Perceba que o trabalho é função trajetória, ou seja, diferente
sobre o gás supera o trabalho realizado pelo gás. a energia interna, ele muda de acordo com a trajetória do
gráfico.
EXERCÍCIO RESOLVIDO

01. (FUVEST) No diagrama P x V da figura A, B e Crepresentam


transformações possíveis de um gás entre os estados I e II. EXERCÍCIOS
PROTREINO
01. Um gás ideal é comprimido por um agente externo, ao mesmo
tempo em que recebe calor de 250J de uma fonte térmica.
Sabendo-se que o trabalho do agente externo é de 500J calcule
variação de energia interna do gás.

02. Considere um gás confinado em um recipiente cilíndrico, de


paredes fixas, exceto pela tampa, que é composta por um êmbolo
móvel que exerce uma pressão constante 1 · 105 Pa sobre o gás.
Considere que o gás se expanda e seu volume sofra um aumento
de 2·10-3 m³, calcule o trabalho realizado pelo gás.
Com relação à variação ∆U da energia interna do gás e ao
trabalho W por ele realizado, entre esses estados, é correto 03. O diagrama abaixo representa a relação pressão x volume.
afirmar que P(N/m²)
a) ∆UA = ∆UB = ∆UC e WC > WB > WA
b) ∆UA > ∆UC > ∆UB e WC = WA < WB
800
c) ∆UA < ∆UB < ∆UC e WC > WB > WA
d) ∆UA = ∆UB = ∆UC e WC = WA > WB
e) ∆UA > ∆UB > ∆UC e WC = WB = WA
300

Resolução: A
Como ∆T = TII - TI é o mesmo para as três transformações, 2 5 V(m³)
devemos ter que:
Calcule o trabalho realizado em um ciclo completo.
∆UA = ∆UB = ∆UC, lembre-se que a energia interna U, é uma
função de ponto, ou seja, não depende da trajetória do gráfico, 04. Um mol de um gás ideal realiza o ciclo termodinâmico como
apenas dos pontos inicial e final. mostrado no gráfico pressão x volume a seguir. O ciclo é percorrido
E como os trabalhos são dados pelas áreas sob as curvas no sentido ABCA, onde A, B e C são os vértices de um triângulo
das transformações, de acordo com a figura abaixo, podemos retângulo.
concluir que:

Sabendo que a transformação A → B é isobárica e que a pressão no


ponto B é de 1 Pa, calcule o calor Q trocado pelo gás em um ciclo.

PRÉ-VESTIBULAR PROENEM.COM.BR 209


FÍSICA II 09 TRANSFORMAÇÕES GASOSAS

05. A imagem a seguir mostra uma pessoa assoprando sua mão de 03. (EN) Analise o gráfico a seguir.
duas formas diferentes.

O gráfico acima representa um gás ideal descrevendo um ciclo ABC


em um diagrama P x V. Esse ciclo consiste em uma transformação
isotérmica seguida de uma transformação isocórica e uma
isobárica.
Em um diagrama V x T qual gráfico pode representar o mesmo ciclo
ABC?
a) d)

A temperatura percebida muda da situação 1 para situação 2,


explique o motivo.

EXERCÍCIOS
PROPOSTOS
b) e)
01. (FAMERP) Certa massa de gás ideal sofre a transformação
cíclica 1 - 2 - 3 - 4 - 5 - 1 representada no diagrama de pressão (P)
e volume (V).

c)

O trecho em que a força exercida pelo gás realiza o maior trabalho é


a) 2 - 3
b) 4 - 5 04. (IFSUL) Abaixo temos o diagrama p x V onde estão
c) 3 - 4 representadas três transformações que levam um gás ideal do
estado inicial (i) para o estado final (f).
d) 1 - 2
e) 5 - 1

02. (EFOMM) Um gás ideal sofre uma expansão isotérmica,


seguida de uma compressão adiabática. A variação total da energia
interna do gás poderia ser nula se, dentre as opções abaixo, a
transformação seguinte fosse uma
a) compressão isocórica
b) expansão isocórica
c) expansão isobárica
d) compressão isobárica
e) compressão isotérmica

210 PROENEM.COM.BR PRÉ-VESTIBULAR


09 TRANSFORMAÇÕES GASOSAS FÍSICA II

Considerando o estudo das transformações gasosas, os três 06. (EN) Analise o gráfico abaixo.
processos aos quais o gás é submetido são, respectivamente
a) isobárico, isotérmico e isovolumétrico.
b) isovolumétrico, isobárico e isotérmico.
c) isotérmico, isobárico e isovolumétrico.
d) isovolumétrico, isotérmico e isobárico.

05. (PUC-PR) Uma forma de gelo com água a 25 ºC é colocada num


freezer de uma geladeira para formar gelo. O freezer está no nível
de congelamento mínimo, cuja temperatura corresponde a –18 ºC.
As etapas do processo de trocas de calor e de mudança de estado
da substância água podem ser identificadas num gráfico da
temperatura X quantidade de calor cedida.
Qual dos gráficos a seguir mostra, corretamente (sem considerar Se entre os estados A e B mostrados na figura, um mol de um gás
a escala), as etapas de mudança de fase da água e de seu ideal passa por um processo isotérmico. A(s) curva(s) que pode(m)
resfriamento para uma atmosfera? representar a função P = f(V) desse processo, é(são)
a)
a) 1 e 5 c) 3 e) 2 e 4
b) 2 d) 4

07. (UERN) O gráfico representa um ciclo termodinâmico:

b)

Os trabalhos realizados nas transformações AB, BC, CD e DA são,


respectivamente:

c) a) negativo, nulo, positivo e nulo.


b) positivo, nulo, negativo e nulo.
c) positivo, negativo, nulo e positivo.
d) negativo, negativo, nulo e positivo.

08. (FUVEST) Certa quantidade de gás sofre três transformações


sucessivas, A → B, B → C e C → A, conforme o diagrama p – V
apresentado na figura abaixo.

d)

A respeito dessas transformações, afirmou-se o seguinte:


e)
I. O trabalho total realizado no ciclo ABCA é nulo.
II. A energia interna do gás no estado C é maior que no estado
A.
III. Durante a transformação A → B, o gás recebe calor e realiza
trabalho.
Está correto o que se afirma em:
a) I. d) I e II.
b) II. e) II e III.
c) III.

PRÉ-VESTIBULAR PROENEM.COM.BR 211


FÍSICA II 09 TRANSFORMAÇÕES GASOSAS

09. (IFSUL) No gráfico temos a representação da pressão “P” em a) A → B e C → D, pois em ambos ocorre realização de trabalho.
função do volume “V” para uma massa de gás perfeito. b) A → B e B → C, pois em ambos ocorre elevação da temperatura.
c) C → D, pois representa uma expansão adiabática e o sistema
realiza trabalho.
d) A → B, pois representa uma compressão adiabática em que
ocorre elevação de temperatura.
e) B → C, pois representa expansão isobárica em que o sistema
realiza trabalho e a temperatura se eleva.

12. (ESPCEX (AMAN)) Um gás ideal é comprimido por um agente


externo, ao mesmo tempo em que recebe calor de 300 J de uma
As sucessivas transformações gasosas representadas no gráfico fonte térmica.
ao lado: A → B; B → C e C → A, são, respectivamente, Sabendo-se que o trabalho do agente externo é de 600 J, então a
a) isocórica, isobárica e isotérmica. variação de energia interna do gás é
b) isobárica, isocórica e isotérmica. a) 900 J. c) 400 J. e) 300 J.
c) isotérmica, isobárica e isocórica. b) 600 J. d) 500 J.
d) isocórica, isotérmica e isobárica.
13. (UECE) Considere um gás confinado em um recipiente
cilíndrico, de paredes fixas, exceto pela tampa, que é composta por
10. (PUC-RS) Analise a figura abaixo, que representa transforma-
um êmbolo móvel que exerce uma pressão constante (P) sobre o
ções termo­dinâmicas às quais um gás ideal está submetido, e
gás. Caso o gás se expanda e seu volume sofra um incremento ∆V,
complete as lacunas do texto que segue.
em função de deslocamento do êmbolo, o trabalho realizado pelo
gás é
a) P/∆V. c) P∆V.
b) ∆V/P. d) -P∆V.

14. (ENEM 2ª APLICAÇÃO) O motor de combustão interna, utilizado


no transporte de pessoas e cargas, é uma máquina térmica
cujo ciclo consiste em quatro etapas: admissão, compressão,
explosão/expansão e escape. Essas etapas estão representadas
no diagrama da pressão em função do volume. Nos motores a
gasolina, a mistura ar/combustível entra em combustão por uma
De acordo com o gráfico, a temperatura do gás no estado A é centelha elétrica.
__________ do que a do estado B. A transformação BC é __________, e
o trabalho envolvido na transformação CD é __________ do que zero.
a) maior – isobárica – maior
b) menor – isométrica – maior
c) menor – isobárica – menor Para o motor descrito,
d) maior – isométrica – menor em qual ponto do ciclo
é produzida a centelha
e) menor – isobárica – maior elétrica?
a) A
11. (ENEM PPL) Rudolph Diesel patenteou um motor a combustão
interna de elevada eficiência, cujo ciclo está esquematizado no b) B
diagrama pressão x volume. O ciclo Diesel é composto por quatro c) C
etapas, duas das quais são transformações adiabáticas. O motor d) D
de Diesel é caracterizado pela compressão de ar apenas, com a
e) E
injeção de combustível no final.

15. (ENEM) O ar atmosférico pode ser utilizado para armazenar o


excedente de energia gerada no sistema elétrico, diminuindo seu
desperdício, por meio do seguinte processo: água e gás carbônico
são inicialmente removidos do ar atmosférico e a massa de ar
restante é resfriada até -198°C. Presente na proporção de 78%
dessa massa de ar, o nitrogênio gasoso é liquefeito, ocupando
um volume 700 vezes menor. A energia excedente do sistema
elétrico é utilizada nesse processo, sendo parcialmente recuperada
quando o nitrogênio líquido, exposto à temperatura ambiente, entra
em ebulição e se expande, fazendo girar turbinas que convertem
energia mecânica em energia elétrica.
MACHADO, R. Disponível em www.correiobraziliense.com.br.
Acesso em: 9 set. 2013 (adaptado).

No processo descrito, o excedente de energia elétrica é armazenado


pela
No ciclo Diesel, o calor é absorvido em: a) expansão do nitrogênio durante a ebulição.

212 PROENEM.COM.BR PRÉ-VESTIBULAR


09 TRANSFORMAÇÕES GASOSAS FÍSICA II

b) absorção de calor pelo nitrogênio durante a ebulição.


c) realização de trabalho sobre o nitrogênio durante a liquefação.
d) retirada de água e gás carbônico da atmosfera antes do
resfriamento.
e) liberação de calor do nitrogênio para a vizinhança durante a
liquefação.

16. (ENEM) Um sistema de pistão contendo um gás é mostrado


na figura. Sobre a extremidade superior do êmbolo, que pode
movimentar-se livremente sem atrito, encontra-se um objeto.
Através de uma chapa de aquecimento é possível fornecer calor ao
gás e, com auxílio de um manômetro, medir sua pressão. A partir Considerando não ter havido perda de energia, o trabalho realizado
de diferentes valores de calor fornecido, considerando o sistema pelas forças exercidas pelo gás no trecho AB dessa transformação
como hermético, o objeto elevou-se em valores ∆h, como mostrado foi de
no gráfico. Foram estudadas, separadamente, quantidades a) 600 J. c) 500 J. e) 800 J.
equimolares de dois diferentes gases, denominados M e V. b) 400 J. d) 1.100 J.

19. (UFRGS) Um gás ideal contido em um cilindro com pistão pode


ser levado de um estado inicial i até um estado final f, seguindo
dois processos distintos, I e II, conforme ilustrado na figura abaixo.

A diferença no comportamento dos gases no experimento decorre


do fato de o gás M, em relação ao V, apresentar
a) maior pressão de vapor. Os trabalhos WI e WII, realizados pelo gás nos processos I e II,
b) menor massa molecular. valem respectivamente

c) maior compressibilidade. a) 10 J e 30 J.

d) menor energia de ativação. b) 20 J e 20 J.

e) menor capacidade calorífica. c) 20 J e 30 J.


d) 30 J e 10 J.
17. (UNIOESTE) Em um sistema fechado, um gás ideal passa e) 30 J e 20 J.
lentamente de um estado inicial 1 para um estado final 2 devido
a uma expansão isotérmica. Assim, ao final deste processo 20. (PUCSP) A figura representa dois modos diferentes de um
termodinâmico, homem soprar uma de suas mãos.
a) o gás não terá absorvido energia na forma de calor uma vez
que a temperatura no estado 1 é igual à temperatura no estado
2.
b) o trabalho realizado pelo gás será igual à variação da energia
interna calculada entre o estado 2 e o estado 1.
c) o calor absorvido pelo gás será igual à variação da energia
interna calculada entre o estado 2 e o estado 1.
d) o trabalho realizado sobre o gás será igual à energia por ele
absorvida na forma de calor ao passar do estado 1 para o
estado 2.
e) o trabalho realizado pelo gás será igual à energia por ele
absorvida na forma de calor ao passar do estado 1 para o
estado 2.

18. (FAC. ALBERT EINSTEIN - MEDICINA) Para provocar a


transformação gasosa ABC, representada no diagrama P x V,
em determinada massa constante de gás ideal, foi necessário
fornecer-lhe 1.400 J de energia em forma de calor, dos quais 300
J transformaram-se em energia interna do gás, devido ao seu Considerando a segunda situação, o diagrama pressão (p) x volume
aquecimento nesse processo. (v) que melhor descreve a transformação AB que o ar soprado pelo
homem sofre é

PRÉ-VESTIBULAR PROENEM.COM.BR 213


FÍSICA II 09 TRANSFORMAÇÕES GASOSAS

a) d)

b) e)

03. (PUCRS) A observação de alguns corpos celestes tem se


tornado difícil em grandes centros urbanos, principalmente por
conta da poluição luminosa produzida. Os rastros luminosos
c) deixados no céu pelas estrelas cadentes, por exemplo, são mais
facilmente observados em locais ermos e distantes das cidades.
As estrelas cadentes são, na verdade, meteoros cujas velocidades
medidas são da ordem de milhares de quilômetros por hora.
Erroneamente se atribui o aquecimento das regiões próximas ao
meteoro ao atrito entre ele e a atmosfera, mas a principal razão
desse aquecimento é a __________ do ar atmosférico logo à frente
do meteoro.
a) compressão isobárica
b) compressão adiabática
EXERCÍCIOS DE c) expansão isobárica

APROFUNDAMENTO d) expansão adiabática

04. (FUVEST) O motor Stirling, uma máquina térmica de alto


rendimento, é considerado um motor ecológico, pois pode funcionar
01. (FUVEST) Um mol de um gás ideal monoatômico é resfriado
com diversas fontes energéticas. A figura I mostra esquematicamente
adiabaticamente de uma temperatura inicial T1 até uma temperatura
um motor Stirling com dois cilindros. O ciclo termodinâmico de
final T1/3.
Stirling, mostrado na figura II, representa o processo em que o
Com base nessas informações, responda: combustível é queimado externamente para aquecer um dos dois
a) O gás sofreu expansão ou compressão ao final do processo? cilindros do motor, sendo que uma quantidade fixa de gás inerte se
Justifique sua resposta. move entre eles, expandindo-se e contraindo-se.
b) Encontre o valor do trabalho realizado pelo gás nesse processo Nessa figura está representado um ciclo de Stirling no diagrama P
em termos da constante universal dos gases ideais R e de T1. x V para um mol de gás ideal monoatômico. No estado A, a pressão
c) Encontre a razão entre as pressões final e inicial do gás após é PA = 4 atm, a temperatura é T1 = 27°C e o volume é VA. A partir
o processo. do estado A, o gás é comprimido isotermicamente até um terço do
volume inicial, atingindo o estado B. Na isoterma T1, a quantidade
Note e adote: de calor trocada é Q1 = 2,640 J, e, na isoterma T2, é Q2 = 7.910 J.
Em um processo adiabático, não há troca de calor com o ambiente.
Energia interna por mol de um gás ideal monoatômico: U = 3RT/2.
Para o processo adiabático em questão, vale a relação PV5/3 =
constante.

02. (UNICAMP) Nas proximidades do Sol, a Sonda Solar Parker


estará exposta a altas intensidades de radiação e a altas
temperaturas. Diversos dispositivos serão usados para evitar o
aquecimento excessivo dos equipamentos a bordo da sonda, entre
eles um sistema de refrigeração. Um refrigerador opera através da
execução de ciclos termodinâmicos.
a) Considere o ciclo termodinâmico representado abaixo para um
gás ideal, em que V2 = 1,5V1 e T1 = 200K. Calcule a temperatura
T3.
b) A partir do gráfico, estime o módulo do trabalho realizado sobre Determine
o gás em um ciclo, em termos apenas de V1, V2, P1 e P4. a) o volume VA, em litros;
b) a pressão PD, em atm, no estado D;
c) a temperatura T2.
Considerando apenas as transformações em que o gás recebe
calor, determine

214 PROENEM.COM.BR PRÉ-VESTIBULAR


09 TRANSFORMAÇÕES GASOSAS FÍSICA II

d) a quantidade total de calor recebido em um ciclo, QR, em J. ANOTAÇÕES

Note e adote:
Calor específico a volume constante: Cv = 3R/2
Constante universal dos gases: R = 8J/(molK) = 0,08 atm/(molK)
0°C = 273 K
1 atm = 105 Pa
1m³ = 1.000 

05. (UNESP) A figura mostra uma máquina térmica em que a


caldeira funciona como a fonte quente e o condensador como a
fonte fria.

a) Considerando que, a cada minuto, a caldeira fornece, por meio


do vapor, uma quantidade de calor igual a 1,6 x 109 J e que o
condensador recebe uma quantidade de calor igual a 1,2 x 109
J, calcule o rendimento dessa máquina térmica.
b) Considerando que 6,0 x 103 kg de água de refrigeração fluem
pelo condensador a cada minuto, que essa água sai do
condensador com temperatura 20°C acima da temperatura de
entrada e que o calor específico da água é igual a 4,0 x 103 J/
(kg · °C), calcule a razão entre a quantidade de calor retirada
pela água de refrigeração e a quantidade de calor recebida pelo
condensador.

GABARITO
EXERCÍCIOS PROPOSTOS
01. D 05. A 09. A 13. C 17. E
02. D 06. B 10. E 14. C 18. C
03. A 07. B 11. E 15. C 19. C
04. B 08. E 12. A 16. E 20. D

EXERCÍCIOS DE APROFUNDAMENTO
01. a) o gás sofreu expansão.
b) τ = RT1

Pf 3
c) =
P1 27
02. a) T3 = T2 = 300 K
18
b) τ
= ( V2 − V1 )(P4 − P1 )
25
03. B
04. a) VA = 6L
b) PD = PB = 12 atm
c) TD = 900 K
d) Qrecebido = 15110 J
05. a) 25%
b) 0,4

PRÉ-VESTIBULAR PROENEM.COM.BR 215


FÍSICA II 09 TRANSFORMAÇÕES GASOSAS

216 PROENEM.COM.BR PRÉ-VESTIBULAR


10
FÍSICA II
TERMODINÂMICA (2ª LEI)

2ª LEI DA TERMODINÂMICA E AS ENUNCIADO DE KELVIN-PLANCK DA 2ª LEI DA TERMODINÂMICA


MÁQUINAS TÉRMICAS “É impossível uma máquina térmica que opere em ciclos
Uma característica que pode ser observada em uma máquina converter energia térmica integralmente em trabalho, ou seja,
térmica é a transformação de energia térmica em energia o rendimento de uma máquina térmica jamais será de 100%”.
mecânica. Até hoje, mesmo com os avanços da tecnologia, ainda
fazemos uso desse tipo de aparato, mas foi no século XVIII que elas
iniciaram um papel importante para os avanços da humanidade.
ENUNCIADO DE CLAUSIUS DA 2ª LEI DA TERMODINÂMICA
Foram diversas máquinas construídas, mas uma que chamou a
atenção foi a de James Watt, que com um bom rendimento para a “O calor flui espontaneamente do corpo de maior temperatura
época contribuiu para um fabuloso capítulo da história, a revolução para o corpo de menor temperatura”.
industrial inglesa.
Uma máquina térmica consiste em duas fontes que transferem Aparentemente parecem dois enunciados distintos, mas as
energia entre si por uma diferença de temperatura existente, porém, duas coisas querem dizer exatamente a mesma coisa. Se a energia
ao longo do processo parte dessa energia realiza trabalho, ou seja, térmica é transferida do corpo de maior temperatura para o corpo
é usada para gerar movimento. de menor temperatura, isso significa dizer que ao sair da fonte
quente esse calor necessariamente precisa chegar à fonte fria,
impossibilitando assim o rendimento de 100%.

 O CICLO DE CARNOT
Sadi Carnot, francês e engenheiro, morreu muito jovem de
cólera e isso quase destruiu todas as suas obras. Isso porque era
uma prática da época queimar todos os pertences de uma vítima
dessa patologia para evitar possíveis contágios. Mas um pequeno
 livro foi salvo e estudado por muitos físicos da época, como por
exemplo: Clausius e Joule. Nesse exemplar, Carnot explicava
sobre uma máquina teórica que, operando de forma cíclica, seria
detentora do maior rendimento possível. Esse aparato respeitaria
dois postulados:
1º Nenhuma máquina térmica operando em duas temperaturas
fixas pode ter o rendimento maior que a máquina teórica de Carnot
sob as mesmas temperaturas.
2º A máquina de Carnot não depende da substância que a faz
operar.
A energia gerada na fonte quente se desloca espontaneamente Para um gás ideal operando sob o ciclo de Carnot, temos:
em direção à fonte fria, onde parte dessa energia será rejeitada (não
aproveitada), a diferença entre a energia gerada e a rejeitada será o
trabalho realizado pela máquina, ou seja, a energia útil do processo.
Podemos calcular a eficiência de uma máquina (rendimento)
utilizando a definição do conceito, fazendo a razão entre a energia
aproveitada pela energia total fornecida, portanto:


 t Qq − Q f
=
Qq
  

η = (Qq — Qf)/Qq
Qf
1
 
Qq

Transformação A-B: Expansão isotérmica.


Observa-se que para o rendimento de uma máquina ser 100% Transformação B-C: Expansão adiabática.
(η = 1) o calor rejeitado para a fonte fria deve ser nulo, o que é Transformação C-D: Compressão isotérmica.
impossível, uma vez que a energia térmica só se transfere da fonte Transformação D-A: Compressão adiabática.
quente graças à certeza de existir uma fonte fria.

PRÉ-VESTIBULAR PROENEM.COM.BR 217


FÍSICA II 10 TERMODINÂMICA (2ª LEI)

Na máquina ideal de Carnot, a razão entre os calores trocados é


igual à razão entre as temperaturas absolutas das fontes térmicas.

Qf Tf
=
Qq Tq

Tf
1
 
Tq

REFRIGERADOR
Uma geladeira transfere calor do meio de menor temperatura
(fonte fria) para o meio de maior temperatura (fonte fria), isso não
ocorre espontaneamente como vimos na 2ª lei da termodinâmica,
portanto é necessário fornecer energia à máquina, ou seja, realizar
um trabalho sobre ela. De modo que um refrigerador é caracterizado
como uma máquina térmica inversa. No ciclo de Diesel não há centelha, a mistura de combustível e
ar entra em combustão através de uma compressão que provoca
um aumento significativo da temperatura.


Eficiência ou Coeficiente de performance:

e
Qf calor retirado da fonte fria TRANSFORMAÇÕES REVERSÍVEIS E
W trabalho realizado pelo motor
IRREVERSÍVEIS
Qf Chamamos de transformação reversível aquela em que, após
e sua conclusão, o sistema pode retornar às suas condições iniciais
Qq  Qf pelo mesmo caminho, ou seja, voltando gradativamente pelos
mesmos estados intermediários até o estado inicial, sem
É importante ressaltar que a energia depositada na fonte interferência externa.
quente é maior que a energia retirada da fonte fria, já que será a
soma do calor retirado (Qf) e do trabalho realizado pelo motor (t). Chamamos de transformação irreversível quando não satisfaz
às condições citadas acima.
Portanto, em um dia de verão, se você resolve abrir a geladeira
para refrescar sua cozinha, estará cometendo um grave erro. O Vemos que na natureza a maioria dos processos são
motor estará depositando mais energia térmica pela parte traseira irreversíveis, imagine um gás que se expande em uma sala ou
do que retirando-a pela parte frontal. que se mistura com outro gás, ele não voltaria ao volume inicial ou
se separaria do outro gás espontaneamente para voltar a condição
inicial.
OUTROS CICLOS
Outros ciclos de transformações bastante usuais são os de Otto
e o de Diesel, usados pelos motores de automóveis. No ciclo de Otto,
ENTROPIA
uma mistura de combustível (gasolina) e ar é admitida no interior do Em 1865, Rudolf Clausius usou a palavra entropia (S) para
motor, em seguida, essa mistura sofre uma explosão através de uma analisar sistemas termodinâmicos irreversíveis. Imagine um
centelha que provoca a expansão capaz de realizar trabalho. gás que se expande livremente no vácuo, este gás não exerce
trabalho em nada para expandir e também não recebe energia
de nenhuma forma para executar a expansão, esse gás não cede
nem recebe energia de nenhuma forma, logo ∆U = 0, Q = 0 e τ =
0. Esse gás realizou espontaneamente uma expansão no espaço,
porém ele jamais retornaria ao volume inicial espontaneamente,

218 PROENEM.COM.BR PRÉ-VESTIBULAR


10 TERMODINÂMICA (2ª LEI) FÍSICA II

seria necessário exercer trabalho sobre o gás para comprimi-


lo, isso daria energia para ele e consequentemente o aqueceria, EXERCÍCIOS
aumentando sua energia interna U, perceba que a expansão livre
é um processo irreversível, percebemos que as partículas desse
PROTREINO
sistema ficaram em uma disposição mais desordenada após a
expansão, Clausius estabeleceu a ideia de entropia como sendo 01. É muito comum receber notícias que mostram pessoas
uma medida de desordem do sistema e os sistemas físicos tendem tentando construir um motor que não necessite de uma fonte
espontaneamente para estados cada vez mais desordenados. de energia externa para funcionar, ao que se denomina de “moto
Segundo Clausius, quando estudamos a entropia como uma perpétuo”. Essas máquinas visam gerar energia para manter seu
grandeza física na forma potencial o importante não é saber o próprio movimento, bastando dar um impulso inicial para que seu
seu valor absoluto e sim a sua variação, no exemplo acima da movimento se perpetue infinitamente.
expansão livre, não nos interessa o valor absoluto da entropia do
sistema antes ou depois, o que nos interessa é o quanto o grau de Explique o porquê desse tipo de máquina não poder ser construída
desordem variou durante a transformação. e qual Lei física ela violaria.
Assim, Clausius definiu que a variação de entropia (∆S) de um
sistema, quando se agrega uma quantidade de calor (∆Q), mediante 02. O ciclo de Carnot está representado na imagem abaixo
um processo reversível (à temperatura constante T) é dada por

∆Q
∆S =
T

PROEXPLICA

Você pode estar se perguntando, se a maior parte dos


processos na natureza são irreversíveis por que Clausius
definiu a relação acima mediante um processo reversível?
Veja essa afirmação
Para determinar a variação de entropia que ocorre em um
processo irreversível, substituímos esse processo por um Classifique as transformações de A para B, de B para C, de C para
processo reversível que envolva os mesmos estados inicial e D e de D para A.
final e calculamos a variação de entropia para esse processo
reversível. 03. Considere que a máquina de Carnot que recebe 400J do
Perceba que se os estados inicial e final forem idênticos, para fins reservatório quente na etapa AB (Q1=400J) e cede 320J ao
de ensino médio, podemos usar a relação acima para calcular a reservatório frio na etapa CD (Q2=–320J)
variação de entropia em processos irreversíveis também.

Veja no exemplo abaixo como esse tema foi cobrado no Enem.

EXERCÍCIO RESOLVIDO

01. (ENEM 2ª APLICAÇÃO 2016) Até 1824 acreditava-se que


as máquinas térmicas, cujos exemplos são as máquinas a
vapor e os atuais motores a combustão, poderiam ter um
funcionamento ideal. Sadi Carnot demonstrou a impossibili-
dade de uma máquina térmica, funcionando em ciclos entre
duas fontes térmicas (uma quente e outra fria), obter 100%
de rendimento. Calcule a energia que essa máquina converte em trabalho e
Tal limitação ocorre porque essas máquinas determine o rendimento.
a) realizam trabalho mecânico.
b) produzem aumento da entropia. 04. Uma máquina frigorífica gasta 40 J de energia na forma de
trabalho para cada 100 J de calor que ela extrai da fonte fria. Calcule
c) utilizam transformações adiabáticas.
o coeficiente de performance dessa máquina.
d) contrariam a lei da conservação de energia.
e) funcionam com temperatura igual à da fonte quente. 05. Um aparelho de ar-condicionado consegue remover calor da
Resolução: B fonte fria (espaço que está sendo refrigerado) num ritmo da ordem
As transformações ocorridas nas máquinas térmicas a vapor de 2700 J/s. Seu compressor gasta cerca de 600 J/s. Calcule o
são irreversíveis, produzindo aumento da entropia. desempenho (coeficiente de performance) para esse aparelho.

PRÉ-VESTIBULAR PROENEM.COM.BR 219


FÍSICA II 10 TERMODINÂMICA (2ª LEI)

05. (UDESC) Uma máquina a vapor foi projetada para operar entre
duas fontes térmicas, a fonte quente e a fonte fria, e para trabalhar
segundo o ciclo de Carnot. Sabe-se que a temperatura da fonte
EXERCÍCIOS quente é de 127 ºC e que a máquina retira, a cada ciclo, 600 J desta
PROPOSTOS fonte, alcançando um rendimento máximo igual a 0,25. O trabalho
realizado pela máquina, por ciclo, e a temperatura da fonte fria são,
respectivamente:
01. (UECE) Em um motor de carro o processo de combustão gera a) 240 J e 95 ºC
300 J de energia térmica. Deste valor, 200 J são perdidos sob a b) 150 J e 27 ºC
forma de calor. Qual a eficiência desse motor? c) 15 J e 95 ºC
a) 300 / 3. c) 200 / 3. d) 90 J e 27 ºC
b) 100 / 3. d) 500 / 2. e) 24 J e 0 ºC

02. (ENEM) Um motor só poderá realizar trabalho se receber 06. (UECE) Imagine um sistema termicamente isolado, composto
uma quantidade de energia de outro sistema. No caso, a energia por cilindros conectados por uma válvula, inicialmente fechada.
armazenada no combustível é, em parte, liberada durante a Um dos cilindros contêm um gás perfeito, mantido à pressão de 1
combustão para que o aparelho possa funcionar. Quando o atm, e no outro, tem-se vácuo. Abrindo-se a válvula
motor funciona, parte da energia convertida ou transformada na
combustão não pode ser utilizada para a realização de trabalho. a) o gás se expande e, assim, sua temperatura diminui.
Isso significa dizer que há vazamento da energia em outra forma. b) a entropia do sistema se mantém constante, pois não há troca
CARVALHO, A. X. Z. Física Térmica. Belo Horizonte: Pax, 2009 (adaptado). de calor.
c) a entropia do sistema aumenta, porque o processo é irreversível.
De acordo com o texto, as transformações de energia que ocorrem
d) a energia interna do gás diminui, porque sua pressão diminui.
durante o funcionamento do motor são decorrentes de a
e) a energia interna do gás aumenta, porque sua pressão aumenta.
a) liberação de calor dentro do motor ser impossível.
b) realização de trabalho pelo motor ser incontrolável.
07. (UECE) O processo de expansão ou compressão de um gás
c) conversão integral de calor em trabalho ser impossível. em um curto intervalo de tempo pode representar um processo
d) transformação de energia térmica em cinética ser impossível. termodinâmico que se aproxima de um processo adiabático. Como
exemplo, pode-se mencionar a expansão de gases de combustão
e) utilização de energia potencial do combustível ser incontrolável.
em um cilindro de motor de automóvel em alta rotação.
03. (UFU) Em um refrigerador, o fluido refrigerante passa por É correto afirmar que, em um processo adiabático no sistema,
processos termodinâmicos que permitem que o calor seja a) a temperatura é constante e o trabalho realizado pelo sistema
removido de um ambiente à baixa temperatura e levado para outro é nulo.
de temperatura maior. Nesse processo, ora o trabalho é realizado b) não há transferência de calor.
sobre o fluido refrigerante, ora é ele que realiza trabalho sobre o
c) a pressão e o volume são constantes.
meio.
d) a energia interna é variável e a pressão é constante.
Esquematicamente, as etapas de tais processos são representadas
a seguir.
08. (PUC - CAMPINAS) Um dispositivo mecânico usado para medir
o equivalente mecânico do calor recebe 250 J de energia mecânica
e agita, por meio de pás, 100 g de água que acabam por sofrer
elevação de 0,50 ºC de sua temperatura.
Adote 1 cal = 4,2 J e cágua = 4,2 J e cágua = 1,0 cal/g ºC.
O rendimento do dispositivo nesse processo de aquecimento é de
a) 16%.
b) 19%.
c) 67%.
d) 81%.
Nesse ciclo, ocorrem uma expansão adiabática e uma compressão e) 84%.
adiabática, respectivamente, entre:
a) 4 e 1; 2 e 3. c) 3 e 4; 1 e 2. 09. (UEL) Leia o texto a seguir e responda à questão.
b) 4 e 1; 1 e 2. d) 2 e 3; 3 e 4. A vida em grandes metrópoles apresenta atributos que
consideramos sinônimos de progresso, como facilidades de acesso
04. (IFSUL) Durante cada ciclo, uma máquina térmica absorve 500 aos bens de consumo, oportunidades de trabalho, lazer, serviços,
J de calor de um reservatório térmico, realiza trabalho e rejeita 420 educação, saúde etc. Por outro lado, em algumas delas, devido à
J para um reservatório frio. Para cada ciclo, o trabalho realizado e grandiosidade dessas cidades e aos milhões de cidadãos que ali
o rendimento da máquina térmica são, respectivamente, iguais a moram, existem muito mais problemas do que benefícios. Seus
a) 80 J e 16% habitantes sabem como são complicados o trânsito, a segurança
pública, a poluição, os problemas ambientais, a habitação etc.
b) 420 J e 8% Sem dúvida, são desafios que exigem muito esforço não só dos
c) 420 J e 84% governantes, mas também de todas as pessoas que vivem nesses
d) 80 J e 84% lugares. Essas cidades convivem ao mesmo tempo com a ordem
e o caos, com a pobreza e a riqueza, com a beleza e a feiura. A
tendência das coisas de se desordenarem espontaneamente é

220 PROENEM.COM.BR PRÉ-VESTIBULAR


10 TERMODINÂMICA (2ª LEI) FÍSICA II

uma característica fundamental da natureza. Para que ocorra a b) A eficiência máxima de uma máquina térmica que opere entre
organização, é necessária alguma ação que restabeleça a ordem. as temperaturas citadas é de 75%.
É o que acontece nas grandes cidades: despoluir um rio, melhorar c) Diminuindo pela metade as temperaturas citadas, o rendimento
a condição de vida dos seus habitantes e diminuir a violência, por máximo de uma máquina térmica que opere entre essas
exemplo, são tarefas que exigem muito trabalho e não acontecem temperaturas não é alterado.
espontaneamente. Se não houver qualquer ação nesse sentido, a
tendência é que prevaleça a desorganização. Em nosso cotidiano, d) Com a tecnologia moderna, é possível construir uma máquina
percebemos que é mais fácil deixarmos as coisas desorganizadas térmica que opere entre as temperaturas citadas com
do que em ordem. A ordem tem seu preço. Portanto, percebemos rendimento superior a 75%.
que há um embate constante na manutenção da vida e do universo e) Devido à segunda lei da termodinâmica, é impossível
contra a desordem. A luta contra a desorganização é travada a construir um dispositivo cujo único efeito seja converter calor
cada momento por nós. Por exemplo, desde o momento da nossa integralmente em trabalho.
concepção, a partir da fecundação do óvulo pelo espermatozoide,
nosso organismo vai se desenvolvendo e ficando mais complexo. 11. (FUVEST) O desenvolvimento de teorias científicas, geralmente,
Partimos de uma única célula e chegamos à fase adulta com tem forte relação com contextos políticos, econômicos, sociais
trilhões delas, especializadas para determinadas funções. e culturais mais amplos. A evolução dos conceitos básicos da
Entretanto, com o passar dos anos, envelhecemos e nosso corpo Termodinâmica ocorre, principalmente, no contexto
não consegue mais funcionar adequadamente, ocorre uma a) da Idade Média.
falha fatal e morremos. O que se observa na natureza é que a
manutenção da ordem é fruto da ação das forças fundamentais, b) das grandes navegações.
que, ao interagirem com a matéria, permitem que esta se organize. c) da Revolução Industrial.
Desde a formação do nosso planeta, há cerca de 5 bilhões de anos, d) do período entre as duas grandes guerras mundiais.
a vida somente conseguiu se desenvolver à custa de transformar
e) da Segunda Guerra Mundial.
a energia recebida pelo Sol em uma forma útil, ou seja, capaz de
manter a organização. Para tal, pagamos um preço alto: grande
parte dessa energia é perdida, principalmente na forma de calor. 12. (UDESC) Um gás ideal monoatômico, com n mols e inicialmente
Dessa forma, para que existamos, pagamos o preço de aumentar na temperatura absoluta T, sofre uma expansão adiabática até que
a desorganização do nosso planeta. Quando o Sol não puder sua temperatura fique a um terço de sua temperatura inicial.
mais fornecer essa energia, dentro de mais 5 bilhões de anos, não Logo, o gás:
existirá mais vida na Terra. Com certeza a espécie humana já terá a) absorveu uma quantidade de calor igual a nRT.
sido extinta muito antes disso.
b) se expandiu isobaricamente.
(Adaptado de: OLIVEIRA, A. O Caos e a Ordem. Ciência Hoje. Disponível em:
<http://cienciahoje.uol.com.br/colunas/fisica-sem-misterio/o-caos-ea- ordem>. c) realizou trabalho liberando uma quantidade de calor igual a
Acesso em: 10 abr. 2015.)
nRT.
Considerando a afirmação presente no texto “a tendência das d) se expandiu aumentando sua energia interna de nRT.
coisas de se desordenarem espontaneamente é uma característica e) realizou trabalho e sua energia interna diminuiu de nRT.
fundamental da natureza”, e com base nos conhecimentos sobre
as leis da termodinâmica, assinale a alternativa correta. 13. (UDESC) Considere as afirmações com relação à
a) Quando dois corpos com temperaturas diferentes são Termodinâmica.
colocados em contato, ocorre a transferência espontânea de I. A energia interna de uma dada quantidade de gás ideal depende
calor do corpo mais quente para o mais frio. apenas da temperatura.
b) O calor, gerado por um motor a explosão, pode ser convertido II. Quando um sistema pode ir do estado (1) para o estado (2) por
de maneira espontânea e integralmente em energia mecânica, vários processos diferentes, a quantidade de calor absorvida
elétrica, química ou nuclear. pelo sistema será a mesma para todos os processos.
c) O nitrogênio e o hélio misturados e contidos em um recipiente III. Quando um sistema pode ir do estado (1) para o estado (2) por
se separam de modo espontâneo após o equilíbrio térmico do vários processos diferentes, a variação da entropia do sistema
sistema. será a mesma para todos os processos.
d) Uma máquina térmica perfeita opera, na prática, em ciclos, IV. Quando um sistema pode ir do estado (1) para o estado (2) por
converte o calor integralmente em trabalho e é capaz de vários processos diferentes, a variação da energia interna do
funcionar como um motoperpétuo. sistema será a mesma para todos os processos.
e) As moléculas de tinta que tingem uma porção de água de V. Quando um sistema pode ir do estado (1) para o estado (2) por
maneira homogênea tendem a se agrupar espontaneamente vários processos diferentes, a quantidade de trabalho realizado
e com isso restaurar a gota de tinta original. sobre o sistema será a mesma para todos os processos.
Assinale a alternativa correta.
10. (IMED) Podemos considerar como máquina térmica qualquer
a) Somente as afirmativas I, II e IV são verdadeiras.
dispositivo que receba uma quantidade de calor Q1 e converta parte
b) Somente as afirmativas III e V são verdadeiras.
da energia recebida dessa maneira em trabalho mecânico W. O calor
não aproveitado, chamado Q2 = Q1 – W, é devolvido ao ambiente sem c) Somente as afirmativas II e V são verdadeiras.
ser aproveitado. Em relação a essas trocas de calor, definimos como d) Somente as afirmativas I, III e V são verdadeiras.
eficiência de uma máquina térmica a razão entre o trabalho mecânico e) Somente as afirmativas I, III e IV são verdadeiras.
W produzido e a quantidade de calor Q1 entregue à máquina. Em
particular, considere uma máquina térmica que opera entre as 14. (PUCSP) Um motor de potência 2,5 cv absorve 925 cal/s de
temperaturas 300 K e 1200 K. Sobre as informações acima descritas, uma fonte térmica quente, cuja temperatura é de 927°C. Sendo
a temperatura da fonte fria de 80,6°F, determine a razão entre o
assinale a alternativa INCORRETA.
rendimento de um motor de Carnot que operasse entre essas
a) Todas as máquinas térmicas devem satisfazer igualmente a mesmas fontes térmicas e o rendimento do referido motor.
primeira e a segunda lei da termodinâmica.
a) 0,75 b) 1,00 c) 1,50 d) 2,00

PRÉ-VESTIBULAR PROENEM.COM.BR 221


FÍSICA II 10 TERMODINÂMICA (2ª LEI)

15. (PUCRS) Em uma máquina térmica ideal que opere em ciclos, 18. (UPF) São várias as reportagens veiculadas na mídia que
todos os processos termodinâmicos, além de reversíveis, não mostram pessoas tentando construir um motor que não necessita
apresentariam dissipação de energia causada por possíveis efeitos dos fornecimento contínuo de energia externa para funcionar, ao que se
atritos internos nos mecanismos ou turbulências no fluido operador denomina de “moto perpétuo”. Essas máquinas têm como objetivo
da máquina. O ciclo de Carnot é um bom exemplo de processo gerar energia para manter o seu próprio movimento, bastando dar
termodinâmico idealizado, que apresentaria a maior eficiência um impulso inicial e o movimento se dará de forma perpétua.
possível na transformação de calor em trabalho útil. A eficiência para Se essa máquina funcionasse, necessariamente se estaria violando a
uma máquina de Carnot operando entre as temperaturas absolutas
de 300 K e 900 K seria de aproximadamente __________, e a entropia do a) Lei da Conservação de Energia.
sistema ficaria __________ durante o processo. b) Primeira Lei de Newton.
a) 66% – maior c) 33% – menor e) 100% – igual c) Lei da Conservação de Quantidade de Movimento.
b) 66% – igual d) 33% – maior d) Lei da Gravitação Universal.
e) Equação geral dos gases.
16. (PUCSP) O diagrama abaixo mostra um ciclo realizado por 1
mol de um gás monoatômico ideal. 19. (UEG) Em um livro com diagramação antiga era apresentado o
esquema a seguir, da troca de calor entre dois corpos A e B.

Nesse esquema o autor explica que “o calor espontaneamente não


pode ir de um corpo para outro de temperatura mais alta". Essa
afirmação está de acordo com a
a) transformação adiabática.
b) primeira Lei da Termodinâmica.
c) segunda Lei da Termodinâmica.
Determine, em porcentagem, o rendimento de uma máquina de d) propagação de calor por convecção.
Carnot que operasse entre as mesmas fontes térmicas desse ciclo. e) experimentação de Joule-Thompson.
a) 24 b) 35 c) 65 d) 76
20. (PUCRS) Numa turbina, o vapor de água é admitido a 800K e é
17. (IME) Um escritório de patentes analisa as afirmativas de um expulso a 400K. Se o rendimento real dessa turbina é 80% do seu
inventor que deseja obter os direitos sobre três máquinas térmicas reais rendimento ideal ou limite, fornecendo-se 100kJ de calor à turbina
que trabalham em um ciclo termodinâmico. Os dados sobre o calor ela poderá realizar um trabalho igual a
rejeitado para a fonte fria e o trabalho produzido pela máquina térmica a) 80kJ b) 60kJ c) 40kJ d) 20kJ e) 10kJ
– ambos expressos em Joules – encontram-se na tabela abaixo.
Trabalho EXERCÍCIOS DE
Máquina Térmica Calor Rejeitado [J]
Produzido [J]
APROFUNDAMENTO
A 4 60
B 15 30 01. (UFJF-PISM 2) Uma máquina a vapor é uma máquina térmica
C 8 12 que utiliza a pressão do vapor d’água. Considerando que o calor é
uma forma de energia, este pode produzir trabalho. Conforme as
As afirmativas do inventor são: leis da Termodinâmica, as máquinas a vapor operam em ciclos.
• Afirmativa 1: O rendimento das máquinas A e C são os James Watt (1736-1819) contribuiu de forma decisiva para a
mesmos para quaisquer temperaturas de fonte quente e Revolução Industrial (entre a 2ª metade do séc. XVIII e a 1ª metade
de fonte fria. do séc. XIX) nos processos de melhoria no motor a vapor. Mesmo
• Afirmativa 2: As máquinas A, B e C obedecem à Segunda após a invenção do motor a combustão no final do sec. XIX, ainda
Lei da Termodinâmica. hoje são utilizados motores térmicos – por exemplo, nas usinas
nucleares – para a geração de eletricidade.
• Afirmativa 3: Se o calor rejeitado nas três situações
acima for dobrado e se for mantida a mesma produção Analise as afirmações a seguir a respeito de máquinas a vapor,
de trabalho, a máquina B apresentará rendimento superior respondendo se são verdadeiras ou falsas, JUSTIFICANDO SUA
aos das máquinas A e C, supondo atendidos os princípios RESPOSTA DA MANEIRA MAIS OBJETIVA POSSÍVEL.
da termodinâmica.
Tomando sempre as temperaturas dos reservatórios das fontes
quente e fria das máquinas como 900 K e 300 K, está(ão) correta(s)
a(s) afirmativa(s):
a) 1, apenas.
b) 2, apenas.
c) 1, 2 e 3.
d) 1 e 3, apenas.
e) 2 e 3, apenas.

222 PROENEM.COM.BR PRÉ-VESTIBULAR


10 TERMODINÂMICA (2ª LEI) FÍSICA II

a) Considere que, em determinado momento, o volume do vapor 05. (FAMERP) A figura representa o diagrama de fluxo de energia
permanece constante, porque o êmbolo que pressiona o vapor de uma máquina térmica que, trabalhando em ciclos, retira
travou devido a uma falha mecânica. Nesse caso, conforme calor (Q1) de uma fonte quente. Parte dessa quantidade de calor
a 1ª lei da Termodinâmica, toda a energia obtida na forma de é transformada em trabalho mecânico (τ) e a outra parte (Q2)
calor é transformada em energia interna. transfere-se para uma fonte fria. A cada ciclo da máquina, Q1 e Q2
b) De acordo com a 2ª lei da Termodinâmica, as máquinas a são iguais, em módulo, respectivamente, a 4 x 103 J e 2,8 x 103 J.
vapor, no decorrer de um ciclo, transformam em trabalho todo
calor recebido da fonte quente, e a energia interna do vapor se
mantém constante.

02. (UFU) Em um motor de automóvel, a mistura de combustível


com ar é comprimida pelo pistão antes da ignição a uma taxa de
10,0 para 1,0, ou seja, o volume final do cilindro é 10 vezes menor
que o volume inicial, como mostrado na figura abaixo (figura fora
de escala).

Considere que não haja trocas de calor entre a mistura no interior


do cilindro e sua vizinhança, que as dimensões do equipamento
não sofram variações significativas com a temperatura, e que a
mistura tenha comportamento semelhante ao de um gás ideal e Sabendo que essa máquina executa 3.000 ciclos por minuto,
faça o que se pede. calcule:
a) Se a pressão inicial e a temperatura inicial valem 1,5 atm e a) o rendimento dessa máquina.
127°C respectivamente, e a pressão final é de 30 atm, calcule,
b) a potência, em watts, com que essa máquina opera.
em graus Celsius, a temperatura da mistura na situação final.
b) Explique a variação de temperatura sofrida pela mistura.
GABARITO
03. (UFPR) No desenvolvimento de uma certa máquina térmica, EXERCÍCIOS PROPOSTOS
o ciclo termodinâmico executado por um gás ideal comporta-se 01. B 05. B 09. A 13. E 17. D
como o apresentado no diagrama P x V (pressão x volume) a seguir. 02. C 06. C 10. D 14. C 18. A
03. C 07. B 11. C 15. B 19. C
04. A 08. E 12. E 16. D 20. C

EXERCÍCIOS DE APROFUNDAMENTO
01. a) Verdadeira: Travando-se o êmbolo, não há realização de trabalho (W = 0). Pela
conservação da energia, todo calor (Q) é transformado em energia interna (∆U). Aplicando
a 1ª Lei: ∆U = Q – W ⇒ ∆U = Q.
b) Falsa: a afirmação contraria a 2ª lei da Termodinâmica que, segundo o enunciado de
Kelvin-Planck: É impossível a construção de uma máquina que, operando em um ciclo
termodinâmico, converta toda a quantidade de calor recebido em trabalho.
02. a) T = 527°C
b) Como ∆U é proporcional a ∆T, este deve ter uma variação positiva.
03. a) WAB = 37,5·103 J
b) TC = 450 K
a) Qual o trabalho realizado pelo gás durante o processo AB? c) Como o processo é isotérmico não há variação de temperatura do sistema gasoso,
também não há variação da energia interna, que é nula.
b) Sabendo que a temperatura do gás no ponto B vale TB = 300K,
04. a) η = 0,25 ou 25%
determine a temperatura do gás no ponto C.
b) T2 = 27°C
c) O processo DA é isotérmico. Qual a variação de energia interna
05. a) η = 30%
do gás nesse processo?
b) Pot = 6·104 W

04. (UFPR) Uma máquina térmica teórica ideal teve um


dimensionamento tal que, a cada ciclo, ela realizaria trabalho de 50 ANOTAÇÕES
cal e cederia 150 cal para a fonte fria. A temperatura prevista para
a fonte quente seria de 127°C. Determine:
a) O rendimento dessa máquina térmica.
b) A temperatura prevista para a fonte fria, em graus Celsius.

PRÉ-VESTIBULAR PROENEM.COM.BR 223


FÍSICA II 10 TERMODINÂMICA (2ª LEI)

ANOTAÇÕES

224 PROENEM.COM.BR PRÉ-VESTIBULAR


11
FÍSICA II
ELETRIZAÇÃO E FORÇA ELÉTRICA

ELETRICIDADE ELETROSFERA
A parte da Física chamada de Eletricidade é muito importante. É uma região do espaço que circunda o núcleo onde são
Existem inúmeras questões em todas as provas de vestibular sobre encontrados os elétrons que se movimentam distribuídos em
o assunto, que se baseiam no estudo das cargas elétricas. níveis e subníveis de acordo com seu grau de energia.
A Eletricidade pode ser dividida em dois blocos: • Elétrons: Partículas que apresentam carga elétrica não
nula. Os elétrons são portadores de carga elétrica negativa.
• Eletrostática: estuda as cargas e seus efeitos produzidos
quando estão em repouso.
• Eletrodinâmica: estudo da corrente elétrica, ou seja, das Carga Elementar
cargas elétricas em movimento, dos circuitos e seus Através de algumas experiências importantes, podemos
componentes. ressaltar o experimento de Millikan por exemplo (a Gota de Millikan),
Existe também o Eletromagnetismo, que estuda os efeitos concluiu-se que o valor da carga de um próton e de um elétron
magnéticos gerados por correntes elétricas e o movimento de era o mesmo, e como são unidades básicas, ou seja, as menores
cargas dentro de campos magnéticos. possíveis, a carga, em módulo, de um próton e de um elétron foi
chamada de carga elementar.
O valor da carga elementar, representada por “e”, no Sistema
CARGA ELÉTRICA Internacional (SI), é dado por:
Próton → carga positiva.
Elétron → carga negativa. |e| = 1,6.10-19C
Nêutron → carga nula. A unidade de carga no SI é o coulomb(C), porém, é comum
encontrarmos seus submúltiplos:
MODELO ATÔMICO • mC: milicoulomb → 1 mC = 10-3 C
O modelo atômico mais aceito atualmente foi obtido já no • μC: microcoulomb → 1 μC = 10-6 C
século XX e teve contribuições de nomes importantes como • nC: nanocoulomb → 1 nC = 10-9 C
Bohr, Rutherford, Heinsenberg entre outros. Se o átomo fosse • pC: picocoulomb → 1 pC = 10-12C
do tamanho de um estádio de futebol, seu núcleo composto de
nêutrons e prótons seria do tamanho de uma bolinha de tênis. Em
volta do núcleo temos os elétrons que giram em um movimento Corpo Eletrizado
onde não se pode obter muitas informações simultâneas. Uma Um corpo se apresenta carregado, isto é, eletrizado, quando
espécie de nuvem eletrônica com camadas de energia definidas. possui quantidades diferentes de prótons e elétrons. Dessa forma,
caso o número de prótons (nP) seja maior que o número de elétrons
(nE), o corpo estará carregado positivamente e, do contrário, estará
negativo.
se np > ne → corpo eletrizado positivamente;
se np = ne → corpo eletricamente neutro;
se np < ne → corpo eletrizado negativamente.
Como um corpo só pode possuir um número inteiro de prótons
e elétrons, a carga elétrica de um corpo sempre será um múltiplo
da carga elementar, ou seja, a carga de um corpo será o produto do
número de prótons ou elétrons em excesso pela carga elementar .
Por isso dizemos que a carga elétrica é uma grandeza quantizada.
Observe a tabela:

n carga CARGA EM C
1 próton 1·e 1,6 · 10-19
2 prótons 2·e 3,2 · 10-19
3 elétrons -3 · e -4,8 · 10-19

NÚCLEO 4 prótons 4·e 6,4 · 10-19


É a parte central do átomo, praticamente toda massa do átomo 5 elétrons -5 · e -8 · 10-19
está concentrada no núcleo. No núcleo temos:
Podemos concluir que um corpo com n elétrons ou prótons em
• Prótons: Partículas que apresentam carga elétrica não excesso tem sua carga dada por:
nula. Os prótons são portadores de carga elétrica positiva.
• Nêutrons: Partículas que apresentam carga elétrica nula, +
=
Q − n⋅ e
ou seja, não geram nem sofrem a influência de ações
elétricas.

PRÉ-VESTIBULAR PROENEM.COM.BR 225


FÍSICA II 11 ELETRIZAÇÃO E FORÇA ELÉTRICA

Condutores e Isolantes Experiências mostraram quais corpos tinham tendência a


eletrizar-se positivamente e quais corpos tinham a tendência de
Condutores - Nos condutores eletrizados, o excesso de cargas
eletrizar-se negativamente.
se distribui na superfície externa do objeto.
Após essas experiências foi organizada uma série, conhecida
O condutor ideal é um material cujo portadores de cargas
como Série Triboelétrica.
elétricas se movimentam livremente pelo material, sem qualquer
oposição do meio natural. regra substância
vidro
Isolantes ou dielétricos - são materiais que impedem
mica
o deslocamento da carga elétrica em sua extensão. Como
exemplo, temos: os plásticos em geral, a madeira (seca), o vidro, lã
a borracha etc. pele de gato
seda
PRINCÍPIOS FUNDAMENTAIS algodão
ebonite
DA ELETROSTÁTICA cobre
• Princípio das ações: cargas elétricas de mesmo sinal se
enxofre
repelem e de sinais contrários se atraem.
É um fato bem conhecido que, se colocarmos próximos celuloide
dois corpos de mesma carga (positiva ou negativa),
Os materiais que vêm primeiro, ao se atritar com materiais que
eles irão se repelir, e se aproximarmos cargas de sinais
vêm em seguida, ficarão positivos. Por exemplo, atritando-se vidro
contrários, irão se atrair. A força com que duas cargas se
com seda, o vidro ficará positivo enquanto a seda ficará negativa.
atraem ou se repelem será estudada mais à frente com a
Lei de Coulomb.
• Princípio da conservação das cargas elétricas: num Eletrização por Contato
sistema eletricamente isolado, a carga elétrica total Na figura abaixo, o corpo A está carregado positivamente,
permanece constante. enquanto o corpo B está neutro. No intuito de se carregar o corpo
Este princípio não impede a passagem de elétrons de um B, colocaremos o corpo A em contato com o corpo B.
corpo para outro, porém, o somatório final permanecerá
constante. Veja o exemplo abaixo.
Duas cargas A e B possuem cargas +8 C e -4 C. Através
de um processo qualquer, elas trocaram cargas e ficaram
com cargas idênticas de 2 C.

+ + - + +
+ +
A - B - A B
+ +
O corpo A, inicialmente positivo, tinha um excesso de prótons;
+ + - + + ao encostar em B haverá uma passagem de elétrons de B para A, e
assim o corpo B também ficará positivo.
Q1 = 8C Q2 = - 4C Q1 = 2C Q2 = 2C

Podemos perceber que o somatório inicial de cargas é igual ao


somatório final, fazendo com que o princípio da conservação das
cargas se comprove.

PROCESSOS DE ELETRIZAÇÃO
Um corpo neutro, como já foi dito, possui o número de elétrons
igual ao número de prótons. Para eletrizá-lo devemos, basicamente, Na eletrização por contato, de esferas idênticas, as cargas
desfazer esta igualdade, ou seja, o corpo deve possuir número finais são as mesmas para as esferas em contato e será dada pelo
diferente de prótons e elétrons. Estudaremos, a seguir, os três somatório de todas as cargas dividido pelo número de esferas.
processos mais comuns para a eletrização de corpos: por atrito, Q1 + Q2 + Q3 …+ Qn
por contato ou por indução eletrostática. QF =
n
Exemplo:
Eletrização por Atrito Duas esferas idênticas possuem cargas iguais a -8 C e 6 C
Consiste em atritar dois corpos, inicialmente neutros, de quando são postas em contato. Qual será a carga final de cada
materiais diferentes. Na fase de atrito haverá passagem de elétrons esfera?
de um corpo para outro. Como ocorre passagem de elétrons de um -8c 6c -8c 6c
para outro, obviamente um corpo, ao final, estará positivo e o outro -8c 6c -8c 6c
negativo, com cargas de mesmo módulo.
Q1  Q2 8  6 2
QF     1C
2 2 2

-1c -1c
-1c -1c

226 PROENEM.COM.BR PRÉ-VESTIBULAR


11 ELETRIZAÇÃO E FORÇA ELÉTRICA FÍSICA II

Eletrização por Indução Densidade Volumétrica de Cargas


O último e mais complexo processo de eletrização é o de Quando o corpo carregado é um isolante ele pode ter cargas
indução eletrostática. Aproximamos a um corpo eletricamente acumuladas em seu interior, diferente de materiais condutores.
neutro, que será chamado de induzido, um outro carregado, Com a mobilidade de locomoção nos condutores, as cargas se
positivamente ou negativamente, que será chamado de indutor. depositam na parte mais externa do corpo. Portanto não faz
Imediatamente, os  elétrons do corpo neutro irão se aproximar sentido falar em densidade volumétrica de carga para um condutor
do lado onde foi colocado o corpo carregado, provocando uma apenas para um isolante.
polarização no corpo neutro.
Q
ρ=
V

Unidade: Coulomb/metro cúbico (C/m3)

Apesar disso, o corpo permanece neutro. Para que ocorra a Nos condutores carregados, a distribuição de cargas é sempre
eletrização, ligaremos o corpo neutro à Terra através de um fio mais intensa nas regiões pontiagudas; este fato é conhecido como
condutor. Devido à polarização ocorrida pela indução do corpo poder das pontas.
carregado positivamente, haverá transferência de elétrons da Terra
para o corpo neutro.
LEI DE COULOMB
CARGA ELÉTRICA PUNTIFORME
Chama-se de carga elétrica puntiforme quando uma
determinada carga elétrica se distribui em um corpo de dimensões
desprezíveis perante o sistema.
Em seguida, corta-se a ligação com a Terra a fim de se evitar Quando tivermos um corpo esférico não considerado um
que os elétrons possam voltar para a Terra. Agora, o indutor poderá ponto material (carga elétrica puntiforme), iremos considerar que
ser afastado, pois o induzido já está carregado. sua carga se encontra no centro do corpo para interações elétricas
externas a ele.

FORÇA ELÉTRICA
A Lei de Coulomb refere-se à força elétrica entre dois corpos
eletrizados. Vimos que dois corpos eletrizados com cargas de
sinais diferentes sofrem uma atração mútua, e quando possuem
mesmo sinal, sofrem uma repulsão.
Caso tivéssemos aproximado um corpo eletrizado negativa-
mente, o fluxo de elétrons sairia do corpo para a Terra e este
ficaria eletrizado positivamente. Logo, podemos concluir que na
eletrização por indução, as cargas finais do indutor e induzido
serão contrárias.
Quando a interação ocorre entre um corpo carregado e um
corpo neutro não puntiformes, os dois corpos sofrem atração com
Densidade Linear de Cargas força de mesmo módulo. Isso ocorre graças a uma polarização que
É possível determinar a quantidade de carga líquida que ocupa resulta nas cargas opostas mais próximas que as cargas iguais
certa unidade de comprimento, como em um fio ou em um cabo. gerando uma força de atração maior que a força de repulsão.
Essa razão é conhecida como densidade linear de cargas. Quando
a distribuição de carga é uniforme, temos uma densidade de carga Inicialmente
Carregado
constante por todo o corpo.
  neutro
F --- polarização
+
Q + F ++
λ= + + --
++
L + + datração
Unidade: Coulomb/metro (C/m) datração < drepulsão
drepulsão Fatração > Frepulsão
Densidade Superficial de Cargas
Quando as cargas em excesso se acomodam no corpo Pela 3ª Lei de Newton, a intensidade da força elétrica é igual
eletrizado, e isso ocorre na superfície externa do condutor, elas para as duas cargas e depende dos módulos das cargas e da
nem sempre se distribuem de modo uniforme. Consideremos um distância entre elas, sendo dada pela fórmula:
condutor com área externa A, em equilíbrio eletrostático, eletrizado
com uma carga Q. Chama-se densidade superficial média de
cargas a razão entre Q e A. Q1 ⋅ Q2
FEL= K ⋅
d2
Q
σ=
A Unidade: Newton (N)

Unidade: Coulomb/metro quadrado (C/m2)

PRÉ-VESTIBULAR PROENEM.COM.BR 227


FÍSICA II 11 ELETRIZAÇÃO E FORÇA ELÉTRICA

Sendo k a chamada constante eletrostática, que no vácuo e na


maioria dos problemas aparece com o valor de: EXERCÍCIOS

k = 9.109Nm2/C2 = 1/4πε0
PROTREINO
A letra grega épsilon (ε) representa a constante de permissivi- 01. Atrita-se um cano de PVC com uma capa de couro, inicialmente
dade do vácuo (8,85.10-12 C2/Nm2). neutros. Usando a série abaixo, indique o material que ganhou
elétrons.
GRÁFICO Fel X d
MATERIAL
Pele humana seca
Couro
Pele de coelho
Vidro
Cabelo humano
Nylon
Papel
Madeira
Como a relação entre o módulo força elétrica e a distância é
Borracha
inversamente proporcional ao quadrado, o gráfico da Fe em função
de d será uma hipérbole quadrática. Poliéster
É fácil observar no gráfico que quanto maior a distância entre Isopor
as cargas menor será a força elétrica entre elas, mas jamais será Polietileno
nula. A força elétrica é caracterizada por ter um alcance infinito. PVC
Teflon
PROEXPLICA
02. Dispõe-se de três esferas metálicas idênticas e isoladas umas
A descoberta dos raios catódicos gerou diversas discus- das outras. Duas delas (A e B) estão neutras e a terceira (C) está
sões no cenário científico do final do século XIX. A partir deles, eletrizada com a carga Q. Coloca-se C em contato sucessivamente
o elétron foi identificado como um componente fundamental com A e depois com B. Determine a carga final de C.
da matéria. A questão aborda duas características desses
raios: a massa e a carga elétrica. 03. Dispõe-se de três esferas metálicas idênticas e isoladas umas
das outras. Duas delas (A e B) estão neutras e a terceira (C) está
eletrizada com a carga Q. Coloca-se C em contato simultâneo com
A e B. Determine a carga final de C.
EXERCÍCIO RESOLVIDO
04. Um corpo inicialmente neutro é eletrizado com carga Q = 64µC.
01. (UERJ) Observe os esquemas a seguir, que representam Calcule o número de elétrons retirados do corpo.
experimentos envolvendo raios catódicos.
05. Duas cargas elétricas puntiformes e positivas estão eletrizadas
com cargas Q e 2Q. Sabendo-se que ao colocar as cargas no vácuo
a 2 metros de distância surge uma força de repulsão de módulo
72·10-3 N. Calcule o módulo de Q. Adote: k = 9·109 N·m2/C2.

EXERCÍCIOS
PROPOSTOS
01. (EEAR) Duas cargas são colocadas em uma região onde há
interação elétrica entre elas. Quando separadas por uma distância
d, a força de interação elétrica entre elas tem módulo igual a F.
Triplicando-se a distância entre as cargas, a nova força de interação
Desses experimentos resultou a descoberta de uma partícula elétrica em relação à força inicial, será
subatômica. As propriedades massa e carga elétrica c) aumentada 3 vezes.
a) diminuída 3 vezes.
dessa partícula apresentam, respectivamente, a seguinte
caracterização: b) diminuída 9 vezes. d) aumentada 9 vezes.
a) igual a zero; igual a zero.
02. (PUC-RJ) Duas cargas pontuais q1 e q2 são colocadas a uma
b) igual a zero; maior que zero. distância R entre si. Nesta situação, observa-se uma força de
c) diferente de zero; igual a zero. módulo F0 sobre a carga q2.
d) diferente de zero; menor que zero. Se agora a carga q2 for reduzida à metade e a distância entre as cargas
e) diferente de zero; maior que zero. for reduzida para R/4, qual será o módulo da força atuando em q2?
a) F0/32 d) 8F0
Gabarito: D
b) F0/2 e) 16F0
c) 2F0

228 PROENEM.COM.BR PRÉ-VESTIBULAR


11 ELETRIZAÇÃO E FORÇA ELÉTRICA FÍSICA II

03.(MACKENZIE) aterrada, e, a seguir, for desfeito o aterramento, a esfera ficará _____.


Por outro lado, se primeiramente o aterramento for desfeito e,
depois, a carga Q for afastada, a esfera ficará __________.
a) eletricamente neutra – positivamente carregada
b) eletricamente neutra – negativamente carregada
c) positivamente carregada – eletricamente neutra
d) positivamente carregada – negativamente carregada
e) negativamente carregada – positivamente carregada

Dois corpos eletrizados com cargas elétricas puntiformes +Q e –Q 07. (UFJF-PISM 3) Em uma experiência realizada em sala de
são colocados sobre o eixo x nas posições +x e –x, respectivamen- aula, o professor de Física usou três esferas metálicas, idênticas e
te. Uma carga elétrica de prova –q é colocada sobre o eixo y na numeradas de 1 a 3, suspensas por fios isolantes em três arranjos
posição +y, como mostra a figura acima. diferentes, como mostra a figura abaixo:
A força eletrostática resultante sobre a carga elétrica de prova
a) tem direção horizontal e sentido da esquerda para a direita.
b) tem direção horizontal e sentido da direita para a esquerda.
c) tem direção vertical e sentido ascendente.
d) tem direção vertical e sentido descendente.
e) é um vetor nulo.
Inicialmente, o Professor eletrizou a esfera 3 com carga negativa.
04. (ULBRA) Considere duas cargas, QA = 4μC e QB = 5μC, separadas Na sequência, o professor aproximou a esfera 1 da esfera 3 e elas
por 3 cm no vácuo. Elas são postas em contato e, após, separadas no se repeliram. Em seguida, ele aproximou a esfera 2 da esfera 1 e
mesmo local, por 1 cm. Qual o sentido e o valor da força eletrostática elas se atraíram. Por fim, aproximou a esfera 2 da esfera 3 e elas se
entre elas, após o contato? atraíram. Na tentativa de explicar o fenômeno, 6 alunos fizeram os
seguintes comentários:
6 Nm2
Considere:
= 1µC 10−= C, k 0 9x109 João: A esfera 1 pode estar eletrizada negativamente, e a esfera 2,
c2 positivamente.
a) Atração; 0,2 N. d) Repulsão; 0,2 N. Maria: A esfera pode estar eletrizada positivamente e a esfera 2
b) Atração; 2,5 N. e) Repulsão; 22,5 N. negativamente.
c) Atração; 200,0 N. Letícia: A esfera 1 pode estar eletrizada negativamente, e a esfera
2 neutra.
05. (UDESC) Uma das principais contribuições para os estudos Joaquim: A esfera 1 pode estar neutra e a esfera 2 eletrizada
sobre eletricidade foi a da definição precisa da natureza da força positivamente.
elétrica realizada, principalmente, pelos trabalhos de Charles Marcos: As esferas 1 e 2 podem estar neutras.
Augustin de Coulomb (1736-1806). Coulomb realizou diversos
Marta: As esferas 1 e 2 podem estar eletrizadas positivamente.
experimentos para determinar a força elétrica existente entre
objetos carregados, resumindo suas conclusões em uma relação Assinale a alternativa que apresenta os alunos que fizeram
que conhecemos atualmente como Lei de Coulomb. comentários corretos com relação aos fenômenos observados:
Considerando a Lei de Coulomb, assinale a alternativa correta. a) somente João e Maria. d) somente João, Letícia e Marcos.
a) A força elétrica entre dois corpos eletricamente carregados é b) somente João e Letícia. e) somente Letícia e Maria.
diretamente proporcional ao produto das cargas e ao quadrado c) somente Joaquim e Marta.
da distância entre estes corpos.
b) A força elétrica entre dois corpos eletricamente carregados 08. (IFSP) A tabela a seguir mostra a série triboelétrica.
é inversamente proporcional ao produto das cargas e Pele de coelho
diretamente proporcional ao quadrado da distância entre estes
corpos. Vidro
Cabelo humano
c) A força elétrica entre dois corpos eletricamente carregados é
diretamente proporcional ao produto das cargas e inversamente Mica
proporcional ao quadrado da distância entre estes corpos. Lã
d) A força elétrica entre dois corpos eletricamente carregados é Pele de gato
diretamente proporcional ao produto das cargas e inversamente Seda
proporcional a distância entre estes corpos. Algodão
e) A força elétrica entre dois corpos eletricamente carregados Âmbar
é diretamente proporcional a distância entre estes corpos e
Ebonite
inversamente proporcional ao produto das cargas.
Poliéster
06. (UFRGS) Uma carga negativa Q é aproximada de uma esfera Isopor
condutora isolada, eletricamente neutra. A esfera é, então, aterrada Plástico
com um fio condutor.
Através dessa série é possível determinar a carga elétrica adquirida
Assinale a alternativa que preenche corretamente as lacunas do por cada material quando são atritados entre si. O isopor ao ser
enunciado abaixo, na ordem em que aparecem. atritado com a lã fica carregado negativamente.
Se a carga Q for afastada para bem longe enquanto a esfera está

PRÉ-VESTIBULAR PROENEM.COM.BR 229


FÍSICA II 11 ELETRIZAÇÃO E FORÇA ELÉTRICA

O vidro ao ser atritado com a seda ficará carregado: Q Q


a) + ; ; zero; -7Q e -8Q.
a) positivamente, pois ganhou prótons. 2 2
b) positivamente, pois perdeu elétrons. Q Q
b) − ; − ; zero; -7Q e -8Q.
c) negativamente, pois ganhou elétrons. 2 2
d) negativamente, pois perdeu prótons. Q Q
c) − ; ; zero; -7Q e -8Q.
e) com carga elétrica nula, pois é impossível o vidro ser eletrizado. 4 2
Q Q
09. (PUC-RJ) Dois bastões metálicos idênticos estão carregados d) − ; ; zero; 7Q e 8Q.
4 2
com a carga de 9,0 µC. Eles são colocados em contato com um
terceiro bastão, também idêntico aos outros dois, mas cuja carga Q Q
e) ; ; zero; -7Q e -8Q.
líquida é zero. Após o contato entre eles ser estabelecido, afastam- 4 4
se os três bastões.
Qual é a carga líquida resultante, em µC, no terceiro bastão? 12. Quando atritamos uma régua de plástico com um pedaço de lã:
a) 3,0 c) 6,0 e) 18 I. Fazemos com que a régua de plástico fique carregada com
cargas elétricas e o pedaço de lã continue neutro eletricamente,
b) 4,5 d) 9,0 pois o papel da lã é de atritar a régua.
II. Fazemos com que a régua de plástico fique carregada com
10. (CPS) O transporte de grãos para o interior dos silos de
cargas elétricas e o pedaço de lã fique carregado com cargas
armazenagem ocorre com o auxílio de esteiras de borracha,
elétricas contrárias às da régua, pois há transferência de
conforme mostra a figura, e requer alguns cuidados, pois os
cargas de um material para o outro.
grãos, ao caírem sobre a esteira com velocidade diferente dela,
até assimilarem a nova velocidade, sofrem escorregamentos, III. Fazemos com que a régua de plástico fique carregada
eletrizando a esteira e os próprios grãos. Essa eletrização pode eletricamente com o mesmo tipo de cargas da lã, pois a
provocar faíscas que, no ambiente repleto de fragmentos de grãos transferência de cargas se dá de um objeto carregado para o
suspensos no ar, pode acarretar incêndios. outro.
IV. A régua de plástico e a lã ficam eletricamente neutros, pois o
processo de eletrização por atrito é o processo de indução de
cargas.
Está(ão) correta(s):
a) I. c) III. e) I e IV.
b) II. d) IV.

13. (FUVEST) Três pequenas esferas carregadas com carga


positiva Q ocupam os vértices de um triângulo, como mostra a
figura. Na parte interna do triângulo, está afixada outra pequena
esfera, com carga negativa q. As distâncias dessa carga às outras
três podem ser obtidas a partir da figura.
Nesse processo de eletrização, os grãos e a esteira ficam carrega-
dos com cargas elétricas de sinais
a) iguais, eletrizados por atrito.
b) iguais, eletrizados por contato.
c) opostos, eletrizados por atrito.
d) opostos, eletrizados por contato.
e) opostos, eletrizados por indução.

11. Um aluno do IFCE dispõe de quatro objetos esféricos idênticos


eletrizados conforme mostra a figura a seguir.

Sendo Q = 2 x 10-4C, q = -2 x 10-5C e d = 6m, a força elétrica


resultante sobre a carga q.
Note e adote:
Ele efetua os seguintes procedimentos: m2
A constante k0 da lei de Coulomb vale 9 × 109 N
1) toca C em B, com A mantida à distância, e em seguida C2
a) é nula.
separa C de B;
b) tem direção do eixo y, sentido para baixo e módulo 1,8N.
2) toca C em A, com B mantida à distância, e em seguida
separa C de A; c) tem direção do eixo y, sentido para cima e módulo 1,0N.
3) toca A em B, com C mantida à distância, e em seguida d) tem direção do eixo y, sentido para baixo e módulo 1,0N.
separa A de B. e) tem direção do eixo y, sentido para cima e módulo 0,3N.
É correto afirmar-se que a carga final das esferas A e D e a soma
das cargas das quatro esferas após os procedimentos realizados
pelo aluno valem, respectivamente,

230 PROENEM.COM.BR PRÉ-VESTIBULAR


11 ELETRIZAÇÃO E FORÇA ELÉTRICA FÍSICA II

14. Uma carga q0 é colocada em uma posição fixa. Ao colocar uma 17. (PUCRS) Para responder à questão a seguir considere as
carga q1 = 2q0 a uma distância d de q0, q1 sofre uma força repulsiva informações que seguem.
de módulo F. Substituindo q1 por uma carga q2 na mesma posição, Três esferas de dimensões desprezíveis A, B e C estão eletrica-
q2 sofre uma força atrativa de módulo 2F. mente carregadas com cargas elétricas respectivamente iguais a 2q,
Se as cargas q1 e q2 são colocadas a uma distância 2d entre si, a q e q. Todas encontram-se fixas, apoiadas em suportes isolantes e
força entre elas é alinhadas horizontalmente, como mostra a figura abaixo:
a) repulsiva, de módulo F
b) repulsiva, de módulo 2F
c) atrativa, de módulo F
d) atrativa, de módulo 2F
e) atrativa, de módulo 4F
O módulo da força elétrica exercida por B na esfera C é F. O módulo
15. Duas pequenas esferas condutoras idênticas estão eletrizadas. da força elétrica exercida por A na esfera B é
A primeira esfera tem uma carga de 2Q e a segunda uma carga de
6Q. As duas esferas estão separadas por uma distância d e a força a) F/4 c) F e) 4F
eletrostática entre elas é F1. Em seguida, as esferas são colocadas b) F/2 d) 2F
em contato e depois separadas por uma distância 2d. Nessa nova
configuração, a força eletrostática entre as esferas é F2. 18. (ACAFE) Utilizado nos laboratórios didáticos de física, os
Pode-se afirmar sobre a relação entre as forças F1 e F2, que: eletroscópios são aparelhos geralmente usados para detectar se
um corpo possui carga elétrica ou não.
a) F1 = 3F2. d) F1 = 4F2.
F2 e) F1 = F2.
b) F1 = .
12
F2
c) F1 = .
3
16. (ENEM (LIBRAS)) Um pente plástico é atritado com papel toalha
seco. A seguir ele é aproximado de pedaços de papel que estavam
sobre a mesa. Observa-se que os pedaços de papel são atraídos e
acabam grudados ao pente, como mostra a figura.

Considerando o eletroscópio da figura anterior, carregado


positivamente, assinale a alternativa correta que completa a lacuna
da frase a seguir.
Tocando-se o dedo na esfera, verifica-se que as lâminas se fecham,
porque o eletroscópio _______.
a) perde elétrons c) ganha prótons
b) ganha elétrons d) perde prótons

19. (MACKENZIE) Uma esfera metálica A, eletrizada com carga


elétrica igual a -20,0 µC, é colocada em contato com outra esfera
idêntica B, eletricamente neutra. Em seguida, encosta-se a esfera B
em outra C, também idêntica eletrizada com carga elétrica igual a
50,0 µC. Após esse procedimento, as esferas B e C são separadas.
A carga elétrica armazenada na esfera B, no final desse processo,
é igual a
a) 20,0 µC d) 50,0 µC
Nessa situação, a movimentação dos pedaços de papel até o pente
b) 30,0 µC e) 60,0 µC
é explicada pelo fato de os papeizinhos
c) 40,0 µC
a) serem influenciados pela força de atrito que ficou retida no
pente.
20. (MACKENZIE) Duas pequenas esferas eletrizadas, com cargas
b) serem influenciados pela força de resistência do ar em Q1 e Q2, separadas pela distância d, se repelem com uma força de
movimento. intensidade 4·10-3N. Substituindo-se a carga Q1 por outra carga
c) experimentarem um campo elétrico capaz de exercer forças igual a 3·Q1 e aumentando-se a distância entre elas para 2·d, o valor
elétricas. da força de repulsão será
d) experimentarem um campo magnético capaz de exercer forças a) 3·10-3 N d) 5·10-4 N
magnéticas. b) 2·10 N-3
e) 8·10-4 N
e) possuírem carga elétrica que permite serem atraídos ou c) 1·10-3 N
repelidos pelo pente.

PRÉ-VESTIBULAR PROENEM.COM.BR 231


FÍSICA II 11 ELETRIZAÇÃO E FORÇA ELÉTRICA

03. (FUVEST) Um equipamento, como o esquematizado na figura


EXERCÍCIOS DE abaixo, foi utilizado por J.J.Thomson, no final do século XIX, para
APROFUNDAMENTO o estudo de raios catódicos em vácuo. Um feixe fino de elétrons
(cada elétron tem massa m e carga e) com velocidade de módulo
v0, na direção horizontal x, atravessa a região entre um par de
01. (UERJ) O esquema abaixo representa as esferas metálicas A e placas paralelas, horizontais, de comprimento L. Entre as placas,
B, ambas com massas de 10-3 kg e carga elétrica de módulo igual há um campo elétrico de módulo constante E na direção vertical y.
a 10-6 C. As esferas estão presas por fios isolantes a suportes, e a Após saírem da região entre as placas, os elétrons descrevem uma
distância entre elas é de 1 m. trajetória retilínea até a tela fluorescente T.

Admita que o fio que prende a esfera A foi cortado e que a força
resultante sobre essa esfera corresponde apenas à força de
interação elétrica.
Calcule a aceleração, em m/s2, adquirida pela esfera A imediata-
mente após o corte do fio.
Dado: constante eletrostática do meio, k = 9 x 109 N·m2·C-2.

02. (FUVEST) Um grupo de alunos, em uma aula de laboratório,


eletriza um canudo de refrigerante por atrito, com um lenço de papel.
Em seguida, com o canudo, eles eletrizam uma pequena esfera
condutora, de massa 9g, inicialmente neutra, pendurada em um fio de Determine
seda isolante, de comprimento L, preso em um ponto fixo P. No final
a) o módulo a da aceleração dos elétrons enquanto estão entre
do processo, a esfera e o canudo estão com cargas de sinais opostos.
as placas;
a) Descreva as etapas do processo de eletrização da esfera.
b) o intervalo de tempo ∆t que os elétrons permanecem entre as
Em seguida, os alunos colocam a esfera eletrizada (E1) em placas;
contato com outra esfera (E2), idêntica à primeira, eletricamente
c) o desvio ∆y na trajetória dos elétrons, na direção vertical, ao
neutra e presa na extremidade de outro fio de seda isolante,
final de seu movimento entre as placas;
também de comprimento L, fixo no ponto P. O sistema adquire
a configuração ilustrada na figura, sendo d = 8 cm. d) a componente vertical vy da velocidade dos elétrons ao saírem
da região entre as placas.
Note e adote: Ignore os efeitos de borda no campo elétrico; Ignore
efeitos gravitacionais.

04. (UERJ) Três pequenas esferas metálicas, E1, E2 e E3, eletricamente


carregadas e isoladas, estão alinhadas, em posições fixas, sendo E2
equidistante de E1 e E3. Seus raios possuem o mesmo valor, que é
muito menor que as distâncias entre elas, como mostra a figura:

• • •
E1 E2 E3

As cargas elétricas das esferas têm, respectivamente, os seguintes


valores:
Q1 = 20 µC Q2 = -4µC Q3 = 1µC
Admita que, em um determinado instante, E1 e E2 são conectadas
Para o sistema em equilíbrio nessa configuração final, determine por um fio metálico; após alguns segundos, a conexão é desfeita.
 Nessa nova configuração, determine as cargas elétricas de E1 e
b) o módulo da tensão T em um dos fios isolantes;
E2 e apresente um esquema com a direção e o sentido da força
c) o módulo da carga q2 da esfera E2; resultante sobre E3.
d) a diferença N entre o número de elétrons e de prótons na esfera
E2 após a eletrização. 05. (FUVEST) Quatro pequenas esferas de massa m, estão
Note e adote: carregadas com carga de mesmo valor absoluto q, sendo duas
Para a situação descrita, utilize: cos θ = 1 e sen θ = 0,1. negativas e duas positivas, como mostra a figura. As esferas estão
dispostas formando um quadrado de lado a e giram numa trajetória
Aceleração da gravidade: 10 m/s². circular de centro O, no plano do quadrado, com velocidade de
Força elétrica entre duas cargas puntiformes Q1 e Q2, distantes r módulo constante v. Suponha que as ÚNICAS forças atuantes
uma da outra: KQ1Q2/r2 sobre as esferas são devidas à interação eletrostática. A constante
K = 9 x 109 Nm²/C². de permissividade elétrica é ε0. Todas as grandezas (dadas e
solicitadas) estão em unidades SI.
Carga do elétron: 1,6 x 10-19C.
Ignore a massa dos fios.

232 PROENEM.COM.BR PRÉ-VESTIBULAR


11 ELETRIZAÇÃO E FORÇA ELÉTRICA FÍSICA II
Fres = Felet ⇒ m a = | q | E ⇒ m a = | e | E ⇒
| e |E
03. a) A força resultante sobre o elétron é a força elétrica: a = .
m
b) Como a força elétrica atua apenas no eixo y, no eixo x a componente da velocidade
L
∆x = constante,
permanece v0 ∆t ⇒ igual
L = av0v0∆. Então:
t ⇒ ∆t = .
0 v 2
1 2 1 | e |E  L  | e | E L2
c) No eixo y, o movimento
= é yuniformemente
∆ a t ⇒= y
∆variado. Sendo ∆y
 v0 =⇒0:= .
2 2 m  v0  y 2 m v20
| e |E L | e |E L
v y = a t ⇒noveixo
d) Aplicando a função horária da velocidade y = y, com ⇒0: v y =
⋅ v0 = .
m v0 y m v0
04. Conectando as esferas por fios condutores, haverá um rearranjo das cargas.
Considerando as esferas idênticas, a carga final de cada uma após a conexão é dada por:
Q A + QB 20 + ( −4)
=Q' = ∴ Q' = 8µC
2 2
Como a carga final de todas as esferas é positiva, a força entre elas será repulsiva. Assim
sendo, após a desconexão dos cabos condutores, a força resultante sobre a partícula 3
pode ser representada pela ilustração abaixo:
a) Determine a expressão do módulo da força eletrostática
resultante F que atua em cada esfera e indique sua direção.

b) Determine a expressão do módulo da velocidade tangencial v 1 q2  1
das esferas. R
05. a)= ⋅ ⋅ 2 − 
( 4 πε0 ) aa  2

GABARITO   2  
=b) v q 1/ (4πε0 ) . 1/ a.m . (1 − ) 
  4  
EXERCÍCIOS PROPOSTOS
01. B 05. C 09. C 13. E 17. B
ANOTAÇÕES
02. D 06. A 10. C 14. C 18. B
03. A 07. B 11. B 15. A 19. A
04. E 08. B 12. B 16. C 20. A

EXERCÍCIOS DE APROFUNDAMENTO
01. a = 9 m/s²
02. a) O canudo eletrizado é aproximado da esfera sem que haja contato, esta é então
atraída por ele por indução eletrostática, com as cargas de sinal contrário às do canudo
concentradas no lado próximo a ele. Em seguida, aterra-se o lado oposto da esfera,
fazendo com que ela perca ou receba elétrons. Após rompida a ligação, afasta-se o
canudo, ficando ele e a esfera eletrizados com cargas de sinais opostos.
b) Na situação de equilíbrio, para uma das esferas, temos:

T cos θ =P
T ⋅ 1 = 9 ⋅ 10−3 ⋅ 10
∴ T = 9 ⋅ 10−2 N
c) Teremos:
Fel T sen θ
=
k q1 q2
= T sen θ
d2
2
9 ⋅ 109 ⋅ q2
9 ⋅ 10−2 ⋅ 0,1
= ( q1 =q2 )
( 8 ⋅ 10 )
−2 2

∴ q2 =8 ⋅ 10−8 C

d) Teremos:
q2 = N ⋅ e
8 ⋅ 10−8 =N ⋅ 1,6 ⋅ 10−19
∴N = 5 ⋅ 1011

PRÉ-VESTIBULAR PROENEM.COM.BR 233


FÍSICA II 11 ELETRIZAÇÃO E FORÇA ELÉTRICA

234 PROENEM.COM.BR PRÉ-VESTIBULAR


12
FÍSICA II
CAMPO ELÉTRICO

CAMPO ELÉTRICO do campo elétrico em um ponto do espaço é de | E | = 10


 N
Considere uma região no espaço que está inicialmente livre de
C
e que nesse ponto seja colocada uma carga q = +4C, atuará
influência de qualquer corpo eletrizado. Imagine que nessa região nela uma força elétrica cuja intensidade é calculada por:
seja colocado um corpo eletrizado com carga Q. A presença desse  
corpo (Q) gera, na região ao seu redor, uma propriedade física Fe = q⋅ | E |
 
VETORIAL que chamamos de Campo Elétrico ( E ).
Fe = 4 ⋅ 10
É através do campo elétrico que as cargas, mesmo à distância, 
conseguem interagir e exercer força elétrica mutuamente. Fe = 40N
Observamos que quando uma carga de prova q é imersa em um
   N
campo elétrico E , ela sofre uma força elétrica Fe , a relação entre Perceba que | E | = 10 significa que, para cada +1C, a carga
essas grandezas é dada por: C
de prova q sofrerá uma força de 10N, como no exemplo tínhamos a
 
Fe= q ⋅ E carga q = +4C ela sofreu uma força total de 40N, se colocássemos
outra carga de +8C sob influência de mesma intensidade de campo
  N
Característica de E : elétrico | E | = 10 , ela sofreria uma força de 80N e assim em
 diante. C
 Fe
• Intensidade: E =
q
 CAMPO ELÉTRICO GERADO POR UMA
• Direção: Mesma de Fe CARGA PUNTIFORME
• Sentido: O vetor campo elétrico sempre tem sentido de Uma carga puntiforme Q, na figura abaixo, gera um campo
afastamento, em relação a carga Q, se esta for positiva. O elétrico, no ponto A, distante d, onde está colocada uma carga de
vetor campo elétrico sempre tem sentido de aproximação, prova q, que sofre uma força elétrica F.
em relação a carga Q, se esta for negativa. Essas
afirmações independem do sinal da carga de prova q.
Observações:
I. Estamos usando como notação para  carga fonte (Q), a
carga geradora do campo elétrico E na região e para carga Pela definição de campo elétrico, sabemos que:

de prova (q), a carga sob a influência do campo
 elétrico E 
 F
. Deve-se entender que o campo elétrico E independe da EA =
carga de prova (q). q

II. Quando a carga de prova q é positiva os vetores Fe e
Pela Lei de Coulomb, o módulo da força elétrica é dado por:
possuem mesma direção e sentido.
Se q é positiva (q > 0): |Q||q|

F k 2
d
Substituindo, teremos a expressão para o módulo do vetor
campo elétrico, gerado por uma carga puntiforme de valor Q a uma
distância d.
 
III. Quando a carga de prova q é negativa os vetores Fe e E |Q||q|

k 2
possuem mesma direção e sentidos opostos. F d |Q|
EA   EA = k ⋅
Se q é negativa (q < 0): |q |q| d2

 newton N
=
IV. Unidade no Sistema internacional E : coulomb C
Perceba que essa unidade mostra que a intensidade do
vetor campo elétrico representa o valor da força elétrica por
unidade de carga. Imagine, por exemplo, que a intensidade

PRÉ-VESTIBULAR PROENEM.COM.BR 235


FÍSICA II 12 CAMPO ELÉTRICO

CAMPO ELÉTRICO GERADO POR VÁRIAS


CARGAS PUNTIFORMES
Em um sistema em que haja várias cargas puntiformes, para
determinarmos o campo elétrico em um ponto, devemos calcular o
campo elétrico gerado por cada carga e depois somarmos. O aluno
deverá ter muito cuidado, pois será uma soma vetorial, já que a
grandeza é vetorial.
    
ER = E1 + E2 + E3 +  + En  
EB > E A

LINHAS DE FORÇA Na figura abaixo, as linhas de força demonstram o


comportamento do campo elétrico com duas cargas de sinais
As linhas de força são utilizadas para demonstrar o opostos.
comportamento do campo elétrico em determinada região do
espaço. A direção e o sentido do vetor Campo elétrico é tangente
às linhas de força (ou campo), por isso jamais teremos um
cruzamento das linhas, pois isso representaria duas direções
possíveis de campo para um mesmo ponto do espaço.

Para uma carga pontual, a direção do campo é radial e o sentido


dependente do sinal.
Na figura abaixo, temos as linhas de força para duas cargas de
mesmo sinal.

Sentido divergente

Linhas de força geradas por duas cargas positivas e de mesmo


valor.

Sentido convergente
Além disso, a densidade de linhas de força (número de linhas
que atravessam uma unidade de área) está associada à intensidade
do Campo Elétrico.

Linhas de força geradas por duas cargas negativas e de


mesmo valor.

236 PROENEM.COM.BR PRÉ-VESTIBULAR


12 CAMPO ELÉTRICO FÍSICA II

CAMPO ELÉTRICO NO INTERIOR PODER DAS PONTAS


DE UM CONDUTOR Em condutores esféricos eletricamente carregados as cargas
elétricas em excesso se distribuem na superfície externa de forma
Um condutor é caracterizado por ter uma mobilidade elétrica, homogênea, ou seja, com densidade superficial de cargas uniforme.
isso acontece porque os elétrons da última camada de valência
trocam de átomos com facilidade, são os chamados elétrons livres.
Quando um corpo condutor é carregado, essas cargas elétricas se
depositam na parte mais externa do corpo.

Fazendo uma análise mais didática, suponha uma esfera Entretanto para condutores que possuem regiões pontiagudas,
condutora e carregada, vamos representar o vetor Campo elétrico nas pontas a densidade de cargas é maior, o que gera um campo
no centro do corpo. elétrico na região externa mais intenso nessa região pontiaguda.
Esse fenômeno é denominado de poder das pontas.

Todo vetor será anulado pelo seu simétrico, portanto o campo


elétrico no centro do condutor será zero. Esse resultado será
ampliado por nós para qualquer ponto no interior do condutor, esse
fenômeno também é conhecido como blindagem eletrostática.

EInterior = 0
Detalhe:
É difícil manter eletrizado um condutor que possua regiões
BLINDAGEM ELETROSTÁTICA E pontiagudas, pois essas regiões perdem cargas com maior
facilidade do que outras regiões.
GAIOLA DE FARADAY
Como explicado acima, em um condutor eletricamente
carregado, as cargas em excesso se distribuem pela superfície PARA-RAIOS DE FRANKLIN
externa, gerando uma blindagem eletrostática, um famoso O objetivo para o uso de para-raios é a proteção de uma casa,
experimento que envolve esse fenômeno é o da Gaiola de Faraday. um prédio, uma região, de ação prejudicial de um raio. O para-raios
Michael Faraday, em 1836, para demonstrar que um condutor arquiteta um percurso seguro, da descarga elétrica principal, entre
eletrizado possui um campo elétrico nulo em seu interior construiu a Terra e a nuvem.
uma gaiola metálica, onde foi colocada um isolante e uma cadeira
de madeira onde Faraday sentou. Ao induzir uma descarga elétrica
na gaiola as cargas elétricas se distribuíram pela superfície externa
da gaiola e Michael Faraday não foi atingido.

https://pt.wikipedia.org/wiki/Ficheiro:Prof._Giovane_Irribarem_de_Mello_dentro_da_ https://pixabay.com/pt/photos/nova-iorque-tempestade-de-raios-938212/
Gaiola_de_Faraday.JPG

PRÉ-VESTIBULAR PROENEM.COM.BR 237


FÍSICA II 12 CAMPO ELÉTRICO

O para-raios é formado por uma haste condutora


com uma extremidade pontiaguda, quando uma nuvem 02.(UDESC) A carga elétrica de uma partícula com 2,0 g
eletrizada se aproxima da região, ela induz nas pontas uma de massa, para que ela permaneça em repouso, quando
colocada em um campo elétrico vertical, com sentido para
carga de sinal oposto. O campo elétrico, próximo as pontas,
baixo e intensidade igual a 500 N/C, é:
fica tão intenso que ioniza o ar e induz uma descarga elétrica
a) + 40 nC d) - 40 µC
através da haste até a Terra. Essa haste, cujas pontas ficam
no topo dos prédios, é aterrada para conduzir a eletricidade b) + 40 µC e) - 40 mC
para o solo e não danificar as construções ao redor. c) + 40 mC
Essa questão é a chave para que você consiga resolver as
EXERCÍCIO RESOLVIDO questões 1, 2 e 10 dos exercícios propostos, leia com atenção
a resolução.
01.(IFSUL) As cargas elétricas puntiformes q1 = 20µC
Resolução: D
e q2 = 64µC estão fixas no vácuo (k0 = 9 x 109 N·m2/C2)
respectivamente nos pontos A e B, conforme a figura a seguir. A figura mostra o campo elétrico e as forças que agem na
partícula. Observe que a carga deve ser negativa, pois, a Força
Elétrica deve ser para cima para que anule a força Peso, sendo
assim, a Força elétrica deve ter sentido oposto ao Campo
Elétrico (que na questão é para baixo) e isso ocorre apenas
para cargas negativas.

O campo elétrico resultante no ponto P tem intensidade de


a) 3,0 x 106 N/C d) 4,5 x 106 N/C
b) 3,6 x 10 N/C 6
e) 5,0 x 106 N/C
c) 4,0 x 106 N/C
Resolução: B

Cálculo do campo elétrico E1 no ponto P gerado pela carga q1:
Nm2
9 ⋅ 109 ⋅ 20 ⋅ 10−6 C
k 0 ⋅ q1 C2
E= ⇒ E= ⇒
( )
1 1
d12 2 Para haver equilíbrio é preciso que:
2 ⋅ 10−1 m2
mg 2 × 10−3 × 10
Fe =P → q E =mg → q = = =4 × 10−5 C =40µC
Nm 2 E 500
9 ⋅ 109 ⋅ 20 ⋅ 10−6 C
C2 N q=−40µC
E1 = ⇒ E1 = 45 ⋅ 105
4 ⋅ 10−2 m2 C
de intensidade e sentido para direita de q1.
 EXERCÍCIOS
Cálculo do campo elétrico E2 no ponto P gerado pela carga q2:
Nm2
PROTREINO
9 ⋅ 109 ⋅ 64 ⋅ 10−6 C
k 0 ⋅ q2 C2
E= ⇒ E= ⇒
( )
2 2 2
d22 8 ⋅ 10−1 m2 01. Determine a intensidade, a direção e o sentido do vetor campo
elétrico nos pontos A e B da figura, gerado pela carga puntiforme
Q = 4·10-9 C.
Nm2
9 ⋅ 109 ⋅ 64 ⋅ 10−6 C Adote: k = 9·109 N·m²/C²
C2 N
E2 = ⇒ E2 =9 ⋅ 105
64 ⋅ 10−2 m2 C

de intensidade e sentido para esquerda de q2.


Cálculo do campo elétrico resultante de acordo com o
esquema abaixo:

02. Uma carga Q = 2,5·10² µC está fixa no ponto médio do segmento


de reta contido entre os pontos A e B. Calcule a intensidade, a
direção e o sentido do campo elétrico no ponto A.
Determine, em seguida, a direção e o sentido da força elétrica que atua
em uma carga de prova q = -1µC quando colocada no sobre o ponto B.
Logo, o campo resultante tem direção horizontal, no sentido
de A para B, cuja intensidade é dada pela soma vetorial dos
campos de cada carga em P:
N N N N
Er = E1 + E2 = 45 ⋅ 105 − 9 ⋅ 105 = 36 ⋅ 105 ∴Er = 3,6 ⋅ 106
C C C C

238 PROENEM.COM.BR PRÉ-VESTIBULAR


12 CAMPO ELÉTRICO FÍSICA II

03. Nos pontos A e B, separados por uma distância de 12 metros, Sabendo que na região do campo elétrico a velocidade do elétron
fixam-se cargas elétricas puntiformes QA = +9mC e QB = +1mC aumentou com aceleração constante, o campo elétrico entre os
respectivamente. pontos X e Y tem sentido
Adote: K = 9·109 N·m²/C²
a) de Y para X, com intensidade maior em Y.
b) de Y para X, com intensidade maior em X.
c) de Y para X, com intensidade constante.
d) de X para Y, com intensidade constante.
e) de X para Y, com intensidade maior em X.

a) Calcule o campo elétrico resultante no ponto médio do


02. (ESPCEX (AMAN)) Uma partícula de carga q e massa 10–6 kg
segmento de reta AB;
foi colocada num ponto próximo à superfície da Terra onde existe
b) Calcule a distância do ponto A até o ponto P, sabendo que no um campo elétrico uniforme, vertical e ascendente de intensidade
ponto P o vetor campo elétrico resultante é nulo. E = 105 N/C.

04. Nos pontos A e B, separados por uma distância d = 0,3 metros,


fixam-se cargas elétricas puntiformes QA = +4 nC e QB = -4nC
respectivamente.
Calcule o módulo do campo elétrico resultante no ponto C.
Adote: K = 9·109 N·m²/C²

Sabendo que a partícula está em equilíbrio, considerando a


intensidade da aceleração da gravidade g = 10 m/s2, o valor da
carga q e o seu sinal são, respectivamente:
a) 10-3 µC, negativa.
b) 10-5 µC, positiva.
c) 10-5 µC, negativa.
d) 10-4 µC, positiva.
05. Nos vértices de um quadrado de lado L = 2· 2 m foram fixadas e) 10-4 µC, negativa.
cargas elétricas puntiformes, conforme a imagem.
03. (ACAFE) Em uma atividade de eletrostática, são dispostas
quatro cargas pontuais (de mesmo módulo) nos vértices de um
quadrado. As cargas estão dispostas em ordem cíclica seguindo o
perímetro a partir de qualquer vértice.
A situação em que o valor do campo elétrico no centro do quadrado
Determine, em função da não será nulo é:
constante eletrostática
do vácuo k0 e da carga Q, a) +|q|, –|q|, +|q|, –|q|
a intensidade do campo b) +|q|, +|q|, +|q|, +|q|
elétrico resultante no ponto
A. c) +|q|, +|q|, –|q|, –|q|
d) –|q|, –|q|, –|q|, –|q|

04. (UECE) Imediatamente antes de um relâmpago, uma nuvem


EXERCÍCIOS tem em seu topo predominância de moléculas com cargas
PROPOSTOS elétricas positivas, enquanto sua base é carregada negativamente.
Considere um modelo simplificado que trata cada uma dessas
distribuições como planos de carga paralelos e com distribuição
01. (FAMERP) A figura representa um elétron atravessando uma uniforme. Sobre o vetor campo elétrico gerado por essas cargas em
região onde existe um campo elétrico. O elétron entrou nessa um ponto entre o topo e a base, é correto afirmar que
região pelo ponto X e saiu pelo ponto Y, em trajetória retilínea.
a) é vertical e tem sentido de baixo para cima.
b) é vertical e tem sentido de cima para baixo.
c) é horizontal e tem mesmo sentido da corrente de ar predomi-
nante no interior da nuvem.
d) é horizontal e tem mesmo sentido no norte magnético da Terra.
e) é horizontal e tem mesmo sentido da corrente de ar na região
externa da nuvem.

PRÉ-VESTIBULAR PROENEM.COM.BR 239


FÍSICA II 12 CAMPO ELÉTRICO

05. (UERN) Os pontos P, Q, R e S são equidistantes das cargas


Figura 3
localizadas nos vértices de cada figura a seguir:

Sobre os campos elétricos resultantes, é correto afirmar que


Lâmpada fluorescente
a) é nulo apenas no ponto R.
Disponível em: http://naveastro.com. Acesso em: 15 ago. 2012.
b) são nulos nos pontos P, Q e S.
c) são nulos apenas nos pontos R e S. A grandeza física associada ao brilho instantâneo da lâmpada
fluorescente, por estar próxima a uma descarga elétrica, é o(a)
d) são nulos apenas nos pontos P e Q.
a) carga elétrica. d) capacitância elétrica.
e) é nulo apenas em S.
b) campo elétrico. e) condutividade elétrica.
06. (UEA) Duas cargas elétricas puntiformes, Q e q, sendo Q positiva c) corrente elétrica.
e q negativa, são mantidas a uma certa distância uma da outra,
conforme mostra a figura. 08. (UFRGS) As cargas elétricas +Q, -Q e +2Q estão dispostas num
círculo de raio R, conforme representado na figura abaixo.

A força elétrica F, que a carga negativa q sofre, e o campo elétrico


E, presente no ponto onde ela é fixada, estão corretamente
representados por

a)

b)

c)

d)

e) Com base nos dados da figura, é correto afirmar que o campo


elétrico resultante no ponto situado no centro do círculo está
representado pelo vetor
07. (ENEM PPL) Em museus de ciências, é comum encontrarem- a) E1. c) E3. e) E5.
se máquinas que eletrizam materiais e geram intensas descargas b) E2. d) E4.
elétricas. O gerador de Van de Graaff (Figura 1) é um exemplo,
como atestam as faíscas (Figura 2) que ele produz. O experimento
09. (UFSM) A luz é uma onda eletromagnética, isto é, a propagação
fica mais interessante quando se aproxima do gerador em
de uma perturbação dos campos elétrico e magnético locais.
funcionamento, com a mão, uma lâmpada fluorescente (Figura 3).
Quando a descarga atinge a lâmpada, mesmo desconectada da rede Analise as afirmações a seguir, que estão relacionadas com as
elétrica, ela brilha por breves instantes. Muitas pessoas pensam que propriedades do campo elétrico.
é o fato de a descarga atingir a lâmpada que a faz brilhar. Contudo, I. O vetor campo elétrico é tangente às linhas de força.
se a lâmpada for aproximada dos corpos da situação (Figura 2), no
II. Um campo elétrico uniforme se caracteriza por ter as linhas de
momento em que a descarga ocorrer entre eles, a lâmpada também
força paralelas e igualmente espaçadas.
brilhará, apesar de não receber nenhuma descarga elétrica.
III. O número de linhas de força por unidade de volume de um
Figura 1 Figura 2 campo elétrico é proporcional à quantidade de cargas do corpo.
Está(ão) correta(s)
a) apenas I. c) apenas I e II. e) I, II e III.
b) apenas II. d) apenas III.

10. (UFJF) Junto ao solo, a céu aberto, o campo elétrico da Terra é E


= 150 N/C e está dirigido para baixo como mostra a figura. Adotando
a aceleração da gravidade como sendo g = 10 m/s2 e desprezando
a resistência do ar, a massa m, em gramas, de uma esfera de carga
Descarga elétrica no gerador q = –4 µC, para que ela fique em equilíbrio no campo gravitacional
da Terra, é:
Gerador de Van de Graaff

240 PROENEM.COM.BR PRÉ-VESTIBULAR


12 CAMPO ELÉTRICO FÍSICA II

a) 0,06. c) 0,03. e) 0,4.


b) 0,5. d) 0,02.

11. (UEFS) Duas cargas elétricas puntiformes, Q1 e Q2, estão fixas


sobre uma circunferência de centro O, conforme a figura. Se E for o módulo do campo elétrico no ponto P, centro do quadrado,
devido à carga Q1, o campo elétrico resultante no ponto P, devido à
presença das quatro cargas, terá módulo
a) zero c) 2 ⋅E e) 4 ⋅ 2 ⋅ E
b) 4 · E d) 2 ⋅ 2 ⋅ E

14. (FUVEST) Os centros de quatro esferas idênticas, I, II, III e IV, com
distribuições uniformes de carga, formam um quadrado. Um feixe
de elétrons penetra na região delimitada por esse quadrado, pelo
ponto equidistante dos centros das esferas III e IV, com velocidade

inicial v na direção perpendicular à reta que une os centros de III e
IV, conforme representado na figura.


Considerando que E representa o vetor campo elétrico criado por
uma carga elétrica puntiforme em determinado ponto e que E
representa o módulo desse vetor, é correto afirmar que, no ponto O:
 
a) E2 =−2 ⋅ E1
 
b) E2= 2 ⋅ E1
 
c) E2 = E1

d) E2 = −E1 A trajetória dos elétrons será retilínea, na direção de v e eles serão
e) E2 =−2 ⋅ E1 acelerados com velocidade crescente dentro da região plana
delimitada pelo quadrado, se as esferas I, II, III e IV estiverem,
12. (IFSUL) As cargas elétricas puntiformes q1 = 20 µC e q2 = 64 respectivamente, eletrizadas com cargas
 Nm2  Note e adote: Q é um número positivo.
µC estão fixas no vácuo  k0 = 9 × 109 2  , respectivamente nos
 C  a) +Q, -Q, -Q, +Q d) -Q, -Q, +Q, +Q
pontos A e B, conforme a figura a seguir. b) +2Q, -Q, +Q, -2Q e) +Q, +2Q, -2Q, -Q
c) +Q, +Q, -Q, -Q

15. (UERJ) Na ilustração, estão representados os pontos I, II, III e IV


em um campo elétrico uniforme.

O campo elétrico resultante no ponto P tem intensidade de


N N
a) 3,0 × 106 c) 4,0 × 106
C C
N N
b) 3,6 × 106 d) 4,5 × 106
C C

13. Quatro cargas elétricas puntiformes, Q1, Q2, Q3 e Q4, estão fixas
nos vértices de um quadrado, de modo que Q = 1| Q=2 | Q=3 | Q4 |.
As posições das cargas e seus respectivos sinais estão indicados
na figura.

PRÉ-VESTIBULAR PROENEM.COM.BR 241


FÍSICA II 12 CAMPO ELÉTRICO

Uma partícula de massa desprezível e carga positiva adquire a 20. (PUCRJ) Duas cargas pontuais q1 = 3,0 µC e q2 = 6,0 µC são
maior energia potencial elétrica possível se for colocada no ponto: colocadas a uma distância de 1,0 m entre si.
a) I c) III Calcule a distância, em metros, entre a carga q1 e a posição, situada
b) II d) IV entre as cargas, onde o campo elétrico é nulo.
Considere kC = 9 x 109 Nm²/C²
16. (ENEM PPL) Em uma manhã ensolarada, uma jovem vai até um a) 0,3 c) 0,5 e) 2,4
parque para acampar e ler. Ela monta sua barraca próxima de seu b) 0,4 d) 0,6
carro, de uma árvore e de um quiosque de madeira. Durante sua
leitura, a jovem não percebe a aproximação de uma tempestade
com muitos relâmpagos.
EXERCÍCIOS DE
A melhor maneira de essa jovem se proteger dos relâmpagos é
a) entrar no carro. APROFUNDAMENTO
b) entrar na barraca.
01. (FAMERP) Duas esferas metálicas de dimensões diferentes,
c) entrar no quiosque. situadas no ar, são eletrizadas e colocadas sobre suportes
d) abrir um guarda-chuva. isolantes com seus centros distando 6,0 metros entre si. As esferas
e) ficar embaixo da árvore. são unidas com um fio condutor até que atinjam o equilíbrio
eletrostático, situação em que a esfera A fica eletrizada com carga
17. (UECE) Considere as seguintes grandezas físicas: tempo, positiva de valor 8,0 x 10-8 C e a esfera B com carga também positiva
massa, campo elétrico. Essas grandezas são, respectivamente, de valor 5,0 x 10-8 C.

a) escalar, vetorial e vetorial.


b) vetorial, vetorial e vetorial.
c) vetorial, escalar e escalar.
d) escalar, escalar e vetorial.

18. (ENEM PPL) Durante a formação de uma tempestade, são


observadas várias descargas elétricas, os raios, que podem ocorrer:
a) Considerando que, para atingir o equilíbrio, 2,0 x 1011 elétrons
das nuvens para o solo (descarga descendente), do solo para as
foram transferidos entre as esferas, que a carga de cada elétron
nuvens (descarga ascendente) ou entre uma nuvem e outra.
é, em módulo, 1,6 x 10-19 C e que o processo durou 4,0 x 10-6
As descargas ascendentes e descendentes podem ocorrer por
segundos, calcule a intensidade média da corrente elétrica, em
causa do acúmulo de cargas elétricas positivas ou negativas, que
ampères, que percorreu o condutor nesse intervalo de tempo.
induz uma polarização oposta no solo. Essas descargas elétricas
ocorrem devido ao aumento da intensidade do(a) b) Considerando a constante eletrostática do ar igual a 9,0 x 109
(N·m²)/C², calcule a intensidade do campo elétrico, em N/C,
a) campo magnético da Terra.
resultante da ação das cargas elétricas das duas esferas no
b) corrente elétrica gerada dentro das nuvens. ponto M.
c) resistividade elétrica do ar entre as nuvens e o solo.
d) campo elétrico entre as nuvens e a superfície da Terra. 02. (UFJF-PISM 3) Duas cargas elétricas, q1 = +1µC e q2 = -4
µC, estão no vácuo, fixas nos pontos 1 e 2, e separadas por uma
e) força eletromotriz induzida nas cargas acumuladas no solo.
distância d = 60 cm, como mostra a figura abaixo.

19. (PUCRS) Para responder à questão, considere a figura abaixo,


que representa as linhas de força do campo elétrico gerado por
duas cargas puntuais QA e QB.

Como base nas informações, determine:


a) A intensidade, a direção e o sentido do vetor campo elétrico
resultante no ponto médio da linha reta que une as duas cargas.
b) O ponto em que o campo elétrico resultante é nulo à esquerda
de q1.

03. (UFPE) Três cargas elétricas, q1 = -16 µC, q2 = +1,0 µC e q3 = -4,0


µC, são mantidas fixas no vácuo e alinhadas, como mostrado na
figura. A distância d = 1,0 cm. Calcule o módulo do campo elétrico
produzido na posição da carga q2, em V/m.
Dados:
Aceleração da gravidade: 10 m/s².
Densidade do mercúrio: 13,6 g/cm³.
A soma QA e QB é necessariamente um número Pressão atmosférica: 1,0 · 105 N/m².
Constante eletrostática: k0 = 1/4πε0 = 9,0 · 109 N·m²/C².
a) par. d) positivo.
b) ímpar. e) negativo.
c) inteiro.

242 PROENEM.COM.BR PRÉ-VESTIBULAR


12 CAMPO ELÉTRICO FÍSICA II

04. (UFPE) Uma carga elétrica puntiforme gera campo elétrico nos
pontos P1 e P2. A figura a seguir mostra setas que indicam a direção
e o sentido do vetor campo elétrico nestes pontos. Contudo, os
comprimentos das setas não indicam os módulos destes vetores.
O modulo do campo elétrico no ponto P1 e 32 V/m. Calcule o
modulo do campo elétrico no ponto P2, em V/m.

05. (UFPE) Nos vértices de um triângulo isósceles são fixadas três


cargas puntiformes iguais a Q1 = +1,0 × 10-6 C; Q2 = - 2,0 × 10-6 C; e
Q3 = +4,0 × 10-6 C. O triângulo tem altura h = 3,0 mm e base D = 6,0
mm. Determine o módulo do campo elétrico no ponto médio M, da
base, em unidades de 109 V/m.

GABARITO
EXERCÍCIOS PROPOSTOS
01. C 05. B 09. C 13. D 17. D
02. D 06. B 10. A 14. C 18. D
03. C 07. B 11. B 15. A 19. D
04. B 08. B 12. B 16. A 20. B

EXERCÍCIOS DE APROFUNDAMENTO
01. a) im = 8 x 10-3 A.
b) ER = 30 N/C.
02. a) E = 5 x 105 N/C.
b) x = 60 – 60 → x = 0.
03. E = 0.
04. E2 = 16 V/m
05. 5·109 V/m

ANOTAÇÕES

PRÉ-VESTIBULAR PROENEM.COM.BR 243


FÍSICA II 12 CAMPO ELÉTRICO

244 PROENEM.COM.BR PRÉ-VESTIBULAR


13 ENERGIA POTENCIAL FÍSICA II

E POTENCIAL ELÉTRICO

ENERGIA POTENCIAL ELÉTRICA A energia potencial elétrica antes de adicionar a nova


Suponha duas partículas com cargas positivas, afastadas de partícula é a energia potencial do sistema formado pelas
uma distância d, ambas em repouso. Se forem soltas, ocorrerá cargas +2Q e -2Q a uma distância R.
uma repulsão devida à força elétrica, fazendo com que adquiram
velocidade (energia cinética). Como sabemos que a energia não k ( 2 Q)( −2 Q) k Q2
U= ⇒ U=
−4 .
pode ser criada e nem destruída, para que as partículas obtenham R R
energia cinética foi realizada uma transformação de energia que ali
estava armazenada no sistema, a energia potencial elétrica. Para o novo sistema, após adicionar a nova partícula, a
Seja Q uma carga elétrica puntiforme e a uma distância d a um energia potencial elétrica é U’ e a energia potencial do sistema
ponto A, onde é colocada uma outra carga de módulo q, a energia formado pelas cargas 2Q, - 2Q e +Q. Devemos considerar as
potencial elétrica adquirida porque ao ser colocada no ponto A será energias potencias duas a duas, ou seja, entre +2Q e -2Q,
dada por: entre +2Q e – Q e entre - 2Q e + Q
k ( 2 Q)( −2 Q) k ( 2 Q)( Q) k ( −2 Q)( Q)
=U' + + ⇒
R R /2 R /2
k ( Q)( Q) k ( Q)( Q) k ( Q)( Q)
U' = −4 +4 −4 ⇒
R R R
k Q2
Q⋅q U' = −4 .
EP= K ⋅ R
d
Perceba que a Energia Potencial Elétrica total de um
sistema é a soma das energias de todos os pares de cargas
Caso Q e q possuam o mesmo sinal, a energia potencial elétrica elétricas que o compõem, duas a duas.
será positiva, e se forem sinais opostos, a energia será negativa. Portanto, nesse caso U’ = U, mas cuidado, nem sempre
Veja o exemplo abaixo esses valores serão iguais.

EXERCÍCIO RESOLVIDO

01.(UFRGS 2012) Considere que U é a energia potencial POTENCIAL E TENSÃO ELÉTRICA


elétrica de duas partículas com cargas +2Q e -2Q fixas a uma
distância R uma da outra. Uma nova partícula de carga +Q Assim como uma carga produz um campo elétrico no espaço,
é agregada a este sistema entre as duas partículas iniciais, ela produz também um potencial elétrico em todo esse espaço.
conforme representado na figura a seguir. O potencial elétrico é a medida da energia potencial elétrica
armazenada por unidade de carga em cada ponto, portanto temos:

 K·Q
V=
d

A energia potencial elétrica desta nova configuração do Uma carga elétrica Q gera, em um ponto à distância d desta carga,
sistema é um potencial elétrico dado pela expressão acima.
a) zero. d) U. A unidade no SI é o Volt(V), homenagem ao físico Italiano,
b) U/4. e) 3U. Alessandro Volta (1745 – 1827).
c) U/2. [ V ] → volt(v)
Gabarito: D J oule
Perceba que na questão o que chamamos de EPA foi 1volt = 1
coulomb
designado pelo símbolo U. O aluno deve se acostumar com as
duas notações, visto que, as duas podem aparecer em provas Quando se obtém o potencial que uma carga produz em um
de vestibulares, o que deve ser observado é que em ambos os ponto muito distante, seu valor é um número muito pequeno, se
casos o significado é energia potencial elétrica. essa distância tender ao infinito, o potencial tenderá à zero.
A questão tem como objetivo é descobrir a energia
Q ⋅q
potencial elétrica, que é dada pela relação Ep = k ⋅ , nesse
d
Q⋅q
caso U= k ⋅ , antes e depois de adicionarmos uma nova
d
partícula de carga +Q.

PRÉ-VESTIBULAR PROENEM.COM.BR 245


FÍSICA II 13 ENERGIA POTENCIAL E POTENCIAL ELÉTRICO

POTENCIAL ELÉTRICO DE UM
Gabarito: E
SISTEMA DE CARGAS O potencial elétrico de uma carga puntiforme é uma
Se tivermos mais de uma carga, o processo será parecido com grandeza escalar dado pela expressão:
o do campo elétrico, porém o potencial elétrico é uma grandeza
escalar e bastará somar os valores dos potenciais gerados, não k0 ⋅ Q
esquecendo de considerar que uma carga negativa gera um
V= .
r
potencial negativo e uma positiva, um potencial positivo.
Assim, como vimos na teoria acima potencial elétrico
resultante no centro C da circunferência, é dado pela soma
dos potenciais, gerados por cada carga.
k 0 ⋅ Q k 0 ⋅ Q k 0 ⋅ ( −2Q)
VC = + + ⇒ VC = 0
r r r
A figura mostra o vetor campo elétrico no centro C da
circunferência devido a cada uma das cargas.

VP  V1  V2  V3  ...  Vn

Q1 ( − Q2 ) ( −Q3 ) Q
VP= K +K +K + ... + K n
d1 d2 d3 dn

Definimos como tensão elétrica (U) entre dois pontos, A e B, a


diferença de potencial elétrico entre esses pontos. A intensidade do vetor campo elétrico resultante, é dado
A tensão elétrica também pode ser chamada de diferença de pela soma vetorial nesse ponto, que é:
potencial (ddp).
A unidade do potencial e da tensão elétrica é o volt (V). k ⋅ | q | k ⋅ | −2Q| 2 ⋅k ⋅Q
EC =
E 3 =0 2 3 =0 2 ⇒ E C = 20
UAB = VA – VB r r r

EXERCÍCIO RESOLVIDO

02. (UNESP 2017) Três esferas puntiformes, eletrizadas com SUPERFÍCIES EQUIPOTENCIAIS
cargas elétricas q1 = q2 = +Q e q3=-2Q, estão fixas e dispostas Dizemos que dois pontos estão em uma mesma superfície
sobre uma circunferência de raio r e centro C, em uma equipotencial quando seus potenciais elétricos são iguais.
região onde a constante eletrostática é igual a k0, conforme
representado na figura.

Considere VC o potencial eletrostático e EC o módulo do


campo elétrico no ponto C devido às três cargas. Os valores
de VC e EC são, respectivamente,
4 ⋅ k0 ⋅ Q
a) zero e
r2
4 ⋅ k0 ⋅ Q k ⋅Q Na figura anterior, as circunferências são superfícies
b) e 0 equipotenciais de potenciais V1, V2, V3 e V4.
r r2 Uma superfície equipotencial é perpendicular às linhas de força
c) zero e zero
do campo elétrico, portanto em uma região onde o campo elétrico
d) 2 ⋅ k 0 ⋅ Q e 2 ⋅ k 0 ⋅ Q é uniforme, as superfícies equipotenciais serão retas (no plano) ou
r2 r2 planos (no espaço). Além disso, como as linhas de força apontam
na direção das cargas negativas, o potencial elétrico diminui ao
e) zero e 2 ⋅ k 0 ⋅ Q
longo dessas linhas.
r2

246 PROENEM.COM.BR PRÉ-VESTIBULAR


13 ENERGIA POTENCIAL E POTENCIAL ELÉTRICO FÍSICA II

Exemplo:
(CESGRANRIO) O gráfico que melhor descreve a relação entre
potencial elétrico V, originado por uma carga elétrica Q < 0, e a
distância d de um ponto qualquer à carga, é:
a) d)

V1 > V 2 > V 3 > V 4 b) e)

03. (EPCAR (AFA) 2012) A figura abaixo representa as linhas


de força de um determinado campo elétrico.
c)

Gabarito: C
Sendo VA, VBe VC os potenciais eletrostáticos em três pontos
A, B e C, respectivamente, com VC<VA<VB pode-se afirmar que EXERCÍCIOS
a posição desses pontos é melhor representada na alternativa
PROTREINO
Para as questões abaixo, adote: k = 9·109 N·m²/C².

a) 01. Uma carga puntiforme Q = -80 µC é fixa no espaço conforme


imagem abaixo:

b)

c)

d)
Calcule a diferença de potencial entre os pontos A e B (VA – VB).

02. Nos pontos A e B, de uma circunferência, foram fixadas cargas


puntiformes QA = +2 µC e QB = -1 µC, respectivamente.
e)

Gabarito: C
Podemos escrever, VB > VA > VC.
Deslocando-se no sentido da linha de força, temos uma
diminuição do potencial.
Portanto a ordem correta é B → A → C.

PROEXPLICA

A construção de gráficos das grandezas físicas é uma


importante ferramenta para analisar determinados resultados.
O potencial elétrico é uma grandeza que se relaciona, de
maneira inversamente proporcional, a distância entre a carga
elétrica e o ponto analisado. A questão analisa como seria o
formato da curva do potencial elétrico criado por uma carga Calcule:
elétrica negativa. a) O módulo do campo elétrico no ponto O;
b) O potencial elétrico no ponto O;
c) A energia potencial eletrostática entre as cargas.

PRÉ-VESTIBULAR PROENEM.COM.BR 247


FÍSICA II 13 ENERGIA POTENCIAL E POTENCIAL ELÉTRICO

03. Nos pontos A, B e C três cargas puntiformes de cargas QA = 4


µC, QB = -2 µC e QC = 7 µC, respectivamente, foram fixadas. Calcule
a energia potencial eletrostática do sistema.
EXERCÍCIOS
PROPOSTOS
01. (ENEM CANCELADO) As células possuem potencial de mem-
brana, que pode ser classificado em repouso ou ação, e é uma es-
tratégia eletrofisiológica interessante e simples do ponto de vista
físico. Essa característica eletrofisiológica está presente na figura a
seguir, que mostra um potencial de ação disparado por uma célu-
la que compõe as fibras de Purkinje, responsáveis por conduzir os
impulsos elétricos para o tecido cardíaco, possibilitando assim a
contração cardíaca. Observa-se que existem quatro fases envolvi-
das nesse potencial de ação, sendo denominadas fases 0, 1, 2 e 3.

04. Quatro cargas puntiformes foram fixadas em uma circunferência


de raio 1,5 metro, conforme imagem abaixo:

O potencial de repouso dessa célula é -100 mV, e quando ocorre


influxo de íons Na+ e Ca2+, a polaridade celular pode atingir valores
de até +10 mV, o que se denomina despolarização celular. A
modificação no potencial de repouso pode disparar um potencial
de ação quando a voltagem da membrana atinge o limiar de disparo
que está representado na figura pela linha pontilhada. Contudo, a
célula não pode se manter despolarizada, pois isso acarretaria a
Determine o ângulo que o campo elétrico resultante, no centro C da morte celular. Assim, ocorre a repolarização celular, mecanismo
circunferência, faz com a horizontal. que reverte a despolarização e retorna a célula ao potencial de
Calcule o potencial elétrico no centro C da circunferência, quando repouso. Para tanto, há o efluxo celular de íons K+.
Q = 6 µC. Qual das fases, presentes na figura, indica o processo de
despolarização e repolarização celular, respectivamente?
05. Três cargas puntiformes foram fixadas nos vértices de um a) Fases 0 e 2. c) Fases 1 e 2. e) Fases 3 e 1.
quadrado de lado L = 2  m .
b) Fases 0 e 3. d) Fases 2 e 0.

02. (EEAR) São dadas duas cargas, conforme a figura:

Considerando E1 o módulo do campo elétrico devido à carga Q1,


E2 o módulo do campo elétrico devido à carga Q2, V1 o potencial
elétrico devido à carga Q1 e V2 o potencial elétrico devido à carga Q2.
Considere Ep o campo elétrico e Vp o potencial resultantes no ponto P.
Considerando Q = 1 nC, calcule: Julgue as expressões abaixo como verdadeiras (V) ou falsas (F).
a) O potencial elétrico resultante no centro do quadrado; ( ) Ep = E1 + E2
b) A carga elétrica que deve ser fixada no ponto A, de modo que se ( ) Vp = V1 + V2
torne nulo o potencial elétrico no centro do quadrado.   
( )  E=
p E1 + E2
( )  V=
p V1 + V2

248 PROENEM.COM.BR PRÉ-VESTIBULAR


13 ENERGIA POTENCIAL E POTENCIAL ELÉTRICO FÍSICA II

Assinale a alternativa que apresenta a sequência correta. a) zero. c) U/2. e) 3U.


a) V – V – F – F c) F – F – V – V b) U/4. d) U.
b) V – F – F – V d) F – V – V – F 
07. (MACKENZIE) A intensidade do campo elétrico (E ) e do
03. (UECE) Os aparelhos de televisão que antecederam a tecnologia potencial elétrico (V) em um ponto P gerado pela carga puntiforme
atual, de LED e LCD, utilizavam um tubo de raios catódicos para N
Q são, respectivamente, 50 e 100 V. A distância d que a carga
produção da imagem. De modo simplificado, esse dispositivo C
produz uma diferença de potencial da ordem de 25 kV entre pontos puntiforme se encontra do ponto P, imersa no ar, é
distantes de 50 cm um do outro. Essa diferença de potencial gera a) 1,0 m c) 3,0 m e) 5,0 m
um campo elétrico que acelera elétrons até que estes se choquem
com a frente do monitor, produzindo os pontos luminosos que b) 2,0 m d) 4,0 m
compõem a imagem.
08. (ITA) Considere as afirmações a seguir:
Com a simplificação acima, pode-se estimar corretamente que o
campo elétrico por onde passa esse feixe de elétrons é I. Em equilíbrio eletrostático, uma superfície metálica é
equipotencial.
a) 0,5 kV/m. c) 50.000 V/m.
II. Um objeto eletrostaticamente carregado induz uma carga
b) 25 kV. d) 1,250 kV · cm..
uniformemente distribuída numa superfície metálica próxima
quando em equilíbrio eletrostático.
04. (UDESC) Ao longo de um processo de aproximação de duas
partículas de mesma carga elétrica, a energia potencial elétrica do III. Uma carga negativa desloca-se da região de maior para a de
sistema: menor potencial elétrico.

a) diminui. IV. É nulo o trabalho para se deslocar uma carga teste do infinito
até o ponto médio entre duas cargas pontuais de mesmo
b) aumenta. módulo e sinais opostos.
c) aumenta inicialmente e, em seguida, diminui. Destas afirmações, é (são) correta(s) somente
d) permanece constante. a) I e II. c) I, II e IV. e) III.
e) diminui inicialmente e, em seguida, aumenta. b) I, II e III. d) I e IV.

05. (UFSM) A tecnologia dos aparelhos eletroeletrônicos está 09. (IFSC) Atingido por um raio na noite da última quinta-
baseada nos fenômenos de interação das partículas carregadas feira, o dedo médio da mão direita do Cristo Redentor (aquele
com campos elétricos e magnéticos. A figura representa as linhas popularmente conhecido como “pai de todos”) será restaurado
de campo de um campo elétrico. [...]. A restauração será feita com incentivos da Lei Rouanet e pelo
Instituto do Patrimônio Histórico e Artístico Nacional (Iphan).
Disponível em: http://veja.abril.com.br/noticia/brasil/dedo-de-cristo-redentor-
serarestaurado. Acesso: 20 mar. 2014. [Adaptado]

Assim, analise as afirmativas:


I. O campo é mais intenso na região A.
II. O potencial elétrico é maior na região B.
III. Uma partícula com carga negativa pode ser a fonte desse
campo.
Está(ão) correta(s)
a) apenas I. c) apenas III. e) I, II e III.
b) apenas II. d) apenas II e III. A descarga elétrica a que o texto se refere aconteceu no dia
16/01/2014. Assinale a alternativa que explica CORRETAMENTE o
06. (UFRGS) Considere que U é a energia potencial elétrica de duas fenômeno ao qual o Cristo Redentor foi vítima.
partículas com cargas +2Q e -2Q fixas a uma distância R uma da
outra. Uma nova partícula de carga +Q é agregada a este sistema a) O ar é bom condutor de eletricidade.
entre as duas partículas iniciais, conforme representado na figura b) Entre o Cristo Redentor e a nuvem havia uma diferença de
a seguir. potencial que permitiu a descarga elétrica.
c) O Cristo Redentor foi construído de material condutor.
d) Existe um excesso de carga elétrica na Terra.
e) A descarga elétrica foi um aviso para que o ser humano trate
melhor o planeta em que vive.
A energia potencial elétrica desta nova configuração do sistema é

PRÉ-VESTIBULAR PROENEM.COM.BR 249


FÍSICA II 13 ENERGIA POTENCIAL E POTENCIAL ELÉTRICO

10. (PUC-RJ) Ao colocarmos duas cargas pontuais q1 = 5,0 µC a) I e III. d) I e IV.


e q2 = 2,0 µC a uma distância d = 30,0 cm, realizamos trabalho. b) II e IV. e) Todas estão corretas.
Determine a energia potencial eletrostática, em joules, deste
sistema de cargas pontuais. c) II e III.

Dado: k0 = 9 x 109 Nm²/C².


15. (IFSP) Na figura a seguir, são representadas as linhas de força
a) 1 c) 3,0 x 10-1 e) 5,0 x 10-5 em uma região de um campo elétrico. A partir dos pontos A, B, C, e
b) 10 d) 2,0 x 10-5 D situados nesse campo, são feitas as seguintes afirmações:

11. (EEAR) Considere as seguintes afirmações a respeito de uma


esfera homogênea carregada em equilíbrio eletrostático:
I. As cargas elétricas se distribuem pela superfície da esfera,
independentemente de seu sinal.
II. Na superfície dessa esfera o campo elétrico é nulo.
III. Na superfície dessa esfera o campo elétrico é normal à
superfície e no seu interior ele é nulo.
I. A intensidade do vetor campo elétrico no ponto B é maior que
IV. A diferença de potencial elétrico entre dois pontos quaisquer da no ponto C.
sua superfície é nula.
II. O potencial elétrico no ponto D é menor que no ponto C.
A respeito dessas afirmações, pode-se dizer que:
III. Uma partícula carregada negativamente, abandonada no ponto
a) Todas estão corretas. d) II, III e IV estão corretas. B, se movimenta espontaneamente para regiões de menor
b) Apenas I está correta. e) Todas estão incorretas. potencial elétrico.
c) I, III e IV estão corretas. IV. A energia potencial elétrica de uma partícula positiva diminui
quando se movimenta de B para A.
12. (EFOMM) Um condutor P, de raio 4,0 cm e carregado com carga É correto o que se afirma apenas em
8,0 nC, está inicialmente muito distante de outros condutores e no a) I.
vácuo. Esse condutor é a seguir colocado concentricamente com
um outro condutor T, que é esférico, oco e neutro. As superfícies b) I e IV.
internas e externa de T têm raios 8,0 cm e 10,0 cm respectivamente. c) II e III.
Determine a diferença de potencial entre P e T quando P estiver no d) II e IV.
interior de T. e) I, II e III.
a) 154,8 · 102 V d) 9,8 · 101 V
b) 16 · 101 V e) 180,0 · 102 V 16. (IFSUL) A figura a seguir ilustra, graficamente, o comportamento
do Potencial Elétrico V, em função da Distância até o centro, de
c) 9,0 · 10 V
2
uma esfera condutora de raio R, eletrizada com carga positiva Q
e em equilíbrio eletrostático. Considere a origem do sistema de
13. (UECE ADAPTADA) Considere a energia potencial elétrica coordenadas localizado no centro da esfera.
armazenada em dois sistemas compostos por: (i) duas cargas
elétricas de mesmo sinal; (ii) duas cargas de sinais opostos. A energia
potencial no primeiro e no segundo sistema, respectivamente,
a) aumenta com a distância crescente entre as cargas e diminui
com a redução da separação.
b) diminui com a distância decrescente entre as cargas e não
depende da separação.
c) aumenta com a distância crescente entre as cargas e não
depende da separação.
d) diminui com o aumento da distância entre as cargas e aumenta
se a separação cresce.

14. Analise as seguintes afirmativas, relacionadas aos conceitos e


aos fenômenos estudados em Eletrostática.
Com base no gráfico e em seus conhecimentos de eletrostática,
I. O potencial elétrico aumenta, ao longo de uma linha de força e analise as seguintes afirmativas:
no sentido dela.
I. O potencial elétrico no interior da esfera é nulo.
II. Uma partícula eletrizada gera um campo elétrico na região do
II. O potencial elétrico no interior da esfera é igual em todos os
espaço que a circunda. Porém, no ponto onde ela foi colocada,
pontos.
o vetor campo elétrico, devido à própria partícula, é nulo.
III. O campo elétrico no interior da esfera é nulo.
III. Uma partícula eletrizada com carga positiva quando
abandonada sob a ação exclusiva de um campo elétrico, Estão corretas as afirmativas
movimenta-se no sentido da linha de força, dirigindo-se para a) I e II, apenas.
pontos de menor potencial. b) II e III, apenas.
IV. A diferença de potencial elétrico (ddp) entre dois pontos c) I e III, apenas.
quaisquer de um condutor em equilíbrio eletrostático é sempre
diferente de zero. d) I, II e III.
Estão corretas apenas as afirmativas

250 PROENEM.COM.BR PRÉ-VESTIBULAR


13 ENERGIA POTENCIAL E POTENCIAL ELÉTRICO FÍSICA II

17. (UECE) Seja o sistema composto por duas cargas elétricas


mantidas fixas a uma distância d e cujas massas são desprezíveis. EXERCÍCIOS DE
A energia potencial do sistema é APROFUNDAMENTO
a) inversamente proporcional a 1/d².
b) proporcional a d². 01. (UECE–ADAPTADA) “n” prótons, cada um de carga q, foram
c) proporcional a 1/d. distribuídos aleatoriamente ao longo de um arco de círculo de 60° e
raio r, conforme ilustra a figura.
d) proporcional a d.

18. (PUCPR) Um sistema de cargas pontuais é formado por


duas cargas positivas +q e uma negativa -q, todas de mesma
intensidade, cada qual fixa em um dos vértices de um triângulo
equilátero de lado r. Se substituirmos a carga negativa por uma
positiva de mesma intensidade, qual será a variação da energia
potencial elétrica do sistema? A constante de Coulomb é denotada
por k.
a) 2kq²/r d) 4kq²/r
b) -2kq²/r e) kq²/r
c) -4kq²/r
1
Considerando k = e o potencial de referência no infinito
(4πε0 )
19. (UFRGS) Uma esfera condutora e isolada, de raio R, foi
carregada com uma carga elétrica Q. Considerando o regime igual a zero, calcule o potencial elétrico no ponto O devido a esses
estacionário, assinale o gráfico abaixo que melhor representa prótons, em função dos dados apresentados.
o valor do potencial elétrico dentro da esfera, como função da
distância r < R até o centro da esfera. 02. (PUCRJ) Duas partículas com cargas Q e -Q têm posições
a) d) iniciais (x,y,z) = (0,0,R) e (0,0,0), respectivamente. A carga -Q está
fixa enquanto uma força (variável) leva a carga Q, em velocidade
muito baixa e constante, até a nova posição (0,0,2R) Considere a
constante eletrostática k conhecida.
a) Calcule a diferença de energia potencial do sistema entre a
posição final e a posição inicial.
b) O trabalho total realizado pelas forças eletrostáticas nas
cargas Q e -Q, ao longo do processo descrito no item anterior, é
positivo, nulo ou negativo? Justifique.

b) e) 03. (UERJ) Em um laboratório, um pesquisador colocou uma esfera


eletricamente carregada em uma câmara na qual foi feito vácuo.
O potencial e o módulo do campo elétrico medidos a certa distância
dessa esfera valem, respectivamente, 600 V e 200 V/m.
Determine o valor da carga elétrica da esfera.

04. (UNESP) Uma esfera condutora descarregada (potencial


elétrico nulo), de raio R1 = 5,0 cm, isolada, encontra-se distante de
outra esfera condutora, de raio R2 = 10,0 cm, carregada com carga
c) elétrica Q = 3,0 µC (potencial elétrico não nulo), também isolada.

Em seguida, liga-se uma esfera à outra, por meio de um fio condutor


longo, até que se estabeleça o equilíbrio eletrostático entre elas.
20. (UEG) Uma carga Q está fixa no espaço, a uma distância d Nesse processo, a carga elétrica total é conservada e o potencial
dela existe um ponto P, no qual é colocada uma carga de prova q0. elétrico em cada condutor esférico isolado descrito pela equação
Considerando-se esses dados, verifica-se que no ponto P q
V = k , onde k é a constante de Coulomb, q é a sua carga elétrica
a) o potencial elétrico devido a Q diminui com inverso de d.
r
e r o seu raio.
b) a força elétrica tem direção radial e aproximando de Q.
c) o campo elétrico depende apenas do módulo da carga Q.
d) a energia potencial elétrica das cargas depende com o inverso
de d².

PRÉ-VESTIBULAR PROENEM.COM.BR 251


FÍSICA II 13 ENERGIA POTENCIAL E POTENCIAL ELÉTRICO

GABARITO
EXERCÍCIOS PROPOSTOS
01. B 05. C 09. B 13. D 17. C
02. D 06. D 10. C 14. C 18. D
03. C 07. B 11. C 15. B 19. A
04. B 08. D 12. C 16. B 20. A

EXERCÍCIOS DE APROFUNDAMENTO
01. C
k ⋅Q ⋅q k ⋅Q ⋅q k ⋅Q ⋅q
∆E
=p − ∴ ∆E
02. a) =p
2R R 2R
b) O trabalho total realizado pelas forças eletrostáticas é zero, pois as cargas se afastam
pela aplicação de uma força variável externa que equilibra as forças eletrostáticas sendo a
Supondo que nenhuma carga elétrica se acumule no fio condutor, força resultante nula, pois o deslocamento se dá em velocidade constante.
determine a carga elétrica final em cada uma das esferas. 03. Q = 2 x 10-7 C.
04. Q1 = 1 µC; Q2 = 2 µC
05. (FUVEST) Duas pequenas esferas iguais, A e B, carregadas, 05. a)
cada uma, com uma carga elétrica Q igual a -4,8 x 10-9 C, estão
fixas e com seus centros separados por uma distância de 12 cm.
Deseja-se fornecer energia cinética a um elétron, inicialmente
muito distante das esferas, de tal maneira que ele possa atravessar
a região onde se situam essas esferas, ao longo da direção x,
indicada na figura, mantendo-se equidistante das cargas.

b) V = -1,44 · 103 V.
c) E = 1,44 · 103 eV.

ANOTAÇÕES

a) Esquematize, na figura a seguir, a direção e o sentido das


forças resultantes F1 e F2, que agem sobre o elétron quando ele
está nas posições indicadas por P1 e P2.

b) Calcule o potencial elétrico V, em volts, criado pelas duas


esferas no ponto P0.
c) Estime a menor energia cinética E, em eV, que deve ser fornecida
ao elétron, para que ele ultrapasse o ponto P0 e atinja a região à
direita de P0 na figura.
NOTE E ADOTE:
Considere V = 0 no infinito.
Num ponto P, V = KQ/r, onde r é a distância da carga Q ao ponto P.
K = 9 x 109 (N·m²/C²).
qe = carga do elétron = -1,6 x 10-19 C.
1eV = 1,6 x 10-19 J.

252 PROENEM.COM.BR PRÉ-VESTIBULAR


14 CAMPO UNIFORME FÍSICA II

E TRABALHO ELÉTRICO

Já foi visto que o trabalho de uma força conservativa (como com uma diferença de potencial entre as placas igual a U, o módulo
exemplo a força peso) não depende da trajetória escolhida e, além do vetor campo elétrico E, dentro da placa, será dado por:
disso, existe uma energia potencial associada à força.
τ = F ⋅ d ⋅ cos θ τ = q(VA − VB )
Vejamos esse raciocínio na mecânica:  
UAB
τ = F ⋅ ∆S ⋅ cos θ
=τ mg(hA − hB ) Igualando...
=τ mghA − mghB
F ⋅d =qUAB
=
τ E PA − E PB

mas como E P= V ⋅ q q ⋅ E ⋅ d =q ⋅ UAB
τ= VA ⋅ q − VB ⋅ q
τ = q ⋅ (VA − VB ) UAB = E ⋅ d

O aluno deverá ter atenção, pois será a diferença entre o potencial Temos duas maneiras de interpretar a relação: primeiro podemos
inicial e o final, e não o potencial final menos o inicial, como aparece definir que uma diferença de potencial entre dois pontos gera um
mais frequentemente, quando, na Física, falamos de variação. campo elétrico, responsável inclusive por mover as cargas. Ou
Se levarmos uma carga negativa de um ponto de menor simplesmente estabelecer que no interior de uma região de campo
potencial para outro de maior, teremos um trabalho positivo; do elétrico, dois pontos não pertencentes a uma mesma superfície
contrário, o trabalho será negativo. equipotencial possuem potenciais diferentes, sendo que ao longo das
Tratando-se de uma carga positiva, acontece o contrário. O linhas de força, no sentido do campo elétrico, o potencial diminui.
trabalho será positivo, levando-se a carga de um potencial maior
para outro menor. EXERCÍCIO RESOLVIDO
É importante lembrar que, assim como acontece com o
trabalho da força peso, o trabalho da força elétrica independe da 01. (UNIRIO-RJ) Com base no esquema, que representa a
trajetória, pois a força elétrica é conservativa. configuração das linhas de força e das superfícies equipoten-
ciais de um campo elétricouniforme de intensidade E = 5 .102
V/m, determine:
a) A distância entre as superfícies equipotenciais S1 e S2;
b) O trabalho da força elétrica que age em q = 2·10-6 ºC para
esta ser deslocada de A para B.
S1 S2

A

E
τ1 =τ2 =τ3

A unidade no SI de Trabalho é o Joule (J).

CAMPO ELÉTRICO UNIFORME B


Dizemos que um campo elétrico é uniforme quando o vetor
campo elétrico é o mesmo (em módulo, direção e sentido) em todo
o espaço. Isto pode ser obtido através da eletrização homogênea 100 V 50 V
de uma placa superficial plana, ou seja, as cargas serão distribuídas
uniformemente. Resolução:

a) U= E ⋅ d b) τ =U.q
100 − 50 = 5.102.d =τ (100 − 50) ⋅ 2 ⋅ 10 −6
50 = 5.102.d =τ 50.2 ⋅ 10 −6
50 τ =100.10 −6
d=
5.102
τ =1.10 −4 J
A direção do campo elétrico é perpendicular às placas e o d = 0,1m
sentido da placa positiva para a negativa. Fora das placas, o campo
elétrico é nulo. Sendo duas placas distantes de um certo valor d e

PRÉ-VESTIBULAR PROENEM.COM.BR 253


FÍSICA II 14 CAMPO UNIFORME E TRABALHO ELÉTRICO

03. Uma carga de prova q = -2,5 µC é movida do ponto A até o


02. (PUC-MG) A figura ao lado mostra as linhas de força de ponto B por dois caminhos diferentes I e II, conforme representado
um campo elétrico uniforme, duas superfícies equipotenciais na imagem abaixo. As superfícies equipotenciais são verticais e
desse campo quatro pontos, A, B, C e D, dessa região. igualmente espaçadas.
Considere o trabalho (W) realizado para levar uma partícula,
carregada positivamente, do ponto A até o ponto B,
percorrendo as trajetórias: 1-ADB; 2-AB; 3-ACB. A relação
entre os trabalhos realizados ao longo desses percursos está
indicada corretamente em:
v1 v2

D B E

30 cm

A C
a) Calcule o trabalho da força elétrica nesse deslocamento por
40 cm cada trajeto.
b) Indique a direção, o sentido e o módulo do campo elétrico
nessa região do espaço.
a) W2 = 0, W1 = W3 d) W1 = W2 = W3
W W1 W3 04. Uma gota de óleo de 10 gramas eletrizada com carga q = 1,0 µC
b) W1 = W3 = 2 e) = W=
2
5 7 5 é abandonada no ponto A e acelerada sob ação exclusiva da força
W2 W3 elétrica em região do espaço de campo elétrico uniforme, conforme
c) W
=1
=
7 3 imagem a seguir.

Resolução: D
Perceba que o trabalho da força elétrica independe da
trajetória, sendo os potenciais inicial VA, no ponto A, e final VB,
no ponto B iguais em todos os percursos, todos os trabalhos
W1, W2 e W3 serão idênticos.

EXERCÍCIOS
PROTREINO
01. Uma carga puntiforme Q = 2,0 µC é fixa no espaço conforme Calcule o módulo da velocidade com que a gota passará pelo ponto
imagem abaixo: B.

05. Uma gotícula eletrizada, com excesso de elétrons, se encontra


em equilíbrio em uma região do espaço de campo elétrico uniforme
gerado por duas placas, planas e paralelas, conforme imagem
abaixo:

Calcule o trabalho da força elétrica quando uma carga puntiforme


de prova q = 1 µC é levada do ponto B para o ponto A.

02. A figura representa as superfícies equipotenciais planas,


paralelas e igualmente espaçadas.

Considere que a gotícula eletrizada fique em equilíbrio quando a


distância entre as placas d = 0,1 metro e a diferença de potencial
é U = 4·104 V.
Nessas condições, determine:
a) a intensidade, direção e sentido do campo elétrico na região
entre as placas.
Indique a direção, o sentido e o módulo do campo elétrico na região
b) a razão q/m da gotícula em equilíbrio?
representada na imagem acima.

254 PROENEM.COM.BR PRÉ-VESTIBULAR


14 CAMPO UNIFORME E TRABALHO ELÉTRICO FÍSICA II

04. (UNIFESP) Na figura, as linhas tracejadas representam


superfícies equipotenciais de um campo elétrico; as linhas cheias I,
II, III, IV e V representam cinco possíveis trajetórias de uma partícula
EXERCÍCIOS de carga q, positiva, realizadas entre dois pontos dessas superfícies,
PROPOSTOS por um agente externo mínimo. A trajetória em que esse trabalho é
maior, em módulo, é:
01. (MACKENZIE)

a) I
b) II
c) III
d) IV
e) V

Uma carga elétrica de intensidade Q = 10,0 µC, no vácuo, gera 05. (UERJ) A aplicação de campo elétrico entre dois eletrodos é um
um campo elétrico em dois pontos A e B, conforme figura acima. recurso eficaz para separação de compostos iônicos. Sob o efeito
Sabendo-se que a constante eletrostática do vácuo é k0 = 9·109 do campo elétrico, os íons são atraídos para os eletrodos de carga
Nm²/C² o trabalho realizado pela força elétrica para transferir uma oposta.
carga q = 2,00 µC do ponto B até o ponto A é, em mJ, igual a Admita que a distância entre os eletrodos de um campo elétrico é
a) 90,0 c) 270 e) 200 de 20 cm e que a diferença de potencial efetiva aplicada ao circuito
é de 6 V.
b) 180 d) 100
Nesse caso, a intensidade do campo elétrico, em V/m, equivale a:
02. (MACKENZIE) Duas cargas elétricas puntiformes, q1 = 3,00 µC a) 40 c) 20
e q2 = 4,00 µC, encontram-se num local onde k = 9·109 N·m²/C². b) 30 d) 10
Suas respectivas posições são os vértices dos ângulos agudos de
um triângulo retângulo isósceles, cujos catetos medem 3,00 mm
cada um. Ao colocar-se outra carga puntiforme, q3 = 1,00 µC, no 06. (PUC-SP ADAPTADA) A figura abaixo, ilustra o experimento
vértice do ângulo reto, esta adquire uma energia potencial elétrica, de Robert Millikan para a obtenção do valor da carga do elétron.
devido à presença de q1 e q2, igual a O vaporizador borrifa gotas de óleo extremamente pequenas que,
no seu processo de formação, são eletrizadas e, ao passar por
a) 9,0 J c) 21,0 J e) 50,0 J um pequeno orifício, ficam sujeitas a um campo elétrico uniforme,
b) 12,0 J d) 25,0 J estabelecido entre as duas placas A e B, mostradas na figura.
Variando adequadamente a tensão entre as placas, Millikan
03. (UFRGS) Assinale a alternativa que preenche corretamente as conseguiu estabelecer uma situação na qual a gotícula mantinha-
lacunas do texto a seguir, na ordem em que aparecem. se em equilíbrio. Conseguiu medir cargas de milhares de gotículas
Na figura que segue, um próton (carga +e) encontra-se inicialmente e concluiu que os valores eram sempre múltiplos inteiros de
fixo na posição A em uma região onde existe um campo elétrico 1,6.10-19C(carga do elétron).
uniforme. As superfícies equipotenciais associadas a esse campo Em uma aproximação da investigação descrita, pode-se
estão representadas pelas linhas tracejadas. considerar que uma gotícula de massa 1,2.10-12kg atingiu o equilíbrio
entre placas separadas de 1,6 cm, estando sujeita apenas às ações
dos campos elétrico e gravitacional.

Na situação representada na figura, o campo elétrico tem


módulo................. e aponta para .............., e o mínimo trabalho a ser
Supondo que entre as placas estabeleça-se uma tensão de
realizado por um agente externo para levar o próton até a posição
6,0.102V, determine o número de elétrons em excesso na gotícula.
B é de............... .
a) 1,0.103 elétrons
a) 1000 V/m direita -300 eV
b) 2,0.103 elétrons
b) 100 V/m direita -300 eV
c) 1,0.106 elétrons
c) 1000 V/m direita +300 eV
d) 2,0.106 elétrons
d) 100 V/m esquerda -300 eV
e) 1,0.106 elétrons
e) 1000 V/m esquerda +300 eV

PRÉ-VESTIBULAR PROENEM.COM.BR 255


FÍSICA II 14 CAMPO UNIFORME E TRABALHO ELÉTRICO

07. (ACAFE) Na figura abaixo temos o esquema de uma impressora 10. (UNICAMP) Quando um rolo de fita adesiva é desenrolado,
jato de tinta que mostra o caminho percorrido por uma gota de tinta ocorre uma transferência de cargas negativas da fita para o rolo,
eletrizada negativamente, numa região onde há um campo elétrico conforme ilustrado na figura a seguir.
uniforme. A gota é desviada para baixo e atinge o papel numa
posição P.

Quando o campo elétrico criado pela distribuição de cargas


é maior que o campo elétrico de ruptura do meio, ocorre uma
O vetor campo elétrico responsável pela deflexão nessa região é: descarga elétrica. Foi demonstrado recentemente que essa
descarga pode ser utilizada como uma fonte econômica de raios-X.
a) ↑ c) →
No ar, a ruptura dielétrica ocorre para campos elétricos a partir
b) ↓ d) ←
de E = 3,0 x 106 V/m. Suponha que ocorra uma descarga elétrica
entre a fita e o rolo para uma diferença de potencial V = 9 kV. Nessa
08. (FUVEST) Em uma aula de laboratório de Física, para estudar situação, pode-se afirmar que a distância máxima entre a fita e o
propriedades de cargas elétricas, foi realizado um experimento em rolo vale
que pequenas esferas eletrizadas são injetadas na parte superior
de uma câmara, em vácuo, onde há um campo elétrico uniforme a) 3 mm. c) 2 mm.
na mesma direção e sentido da aceleração local da gravidade. b) 27 mm. d) 37 nm.
Observou-se que, com campo elétrico de módulo igual a 2 x 103
V/m, uma das esferas, de massa 3,2 x 10-15 kg, permanecia com 11. (PUCCAMP) No interior das válvulas que comandavam os
velocidade constante no interior da câmara. Essa esfera tem tubos dos antigos televisores, os elétrons eram acelerados por
Note e adote: um campo elétrico. Suponha que um desses campos, uniforme e
de intensidade 4,0 x 10² N/C, acelerasse um elétron durante um
- carga do elétron = -1,6 x 10-19C
percurso de 5,0 x 10-4 m. Sabendo que o módulo da carga elétrica
- carga do próton = +1,6 x 10-19C do elétron é 1,6 x 10-19 a energia adquirida pelo elétron nesse
- aceleração local da gravidade = 10 m/s² deslocamento era de
a) 2,0 x 10-25 J c) 8,0 x 10-19 J e) 1,3 x 10-13 J
a) o mesmo número de elétrons e de prótons.
b) 3,2 x 10 -20
J d) 1,6 x 10 -17
J
b) 100 elétrons a mais que prótons.
c) 100 elétrons a menos que prótons. 12. (UFPR) Um próton movimenta-se em linha reta paralelamente
d) 2000 elétrons a mais que prótons. às linhas de força de um campo elétrico uniforme, conforme
e) 2000 elétrons a menos que prótons. mostrado na figura. Partindo do repouso no ponto 1 e somente sob
ação da força elétrica, ele percorre uma distância de 0,6 m e passa
pelo ponto 2. Entre os pontos 1 e 2 há uma diferença de potencial
09. (UNESP) Uma carga elétrica q > 0 de massa m penetra em uma
∆V igual a 32 V. Considerando a massa do próton igual a 1,6·10–27 kg
região entre duas grandes placas planas, paralelas e horizontais,
e sua carga igual a 1,6·10–19 C, assinale a alternativa que apresenta
eletrizadas com cargas de sinais opostos. Nessa região, a carga
corretamente a velocidade do próton ao passar pelo ponto 2.
percorre a trajetória representada
 na figura, sujeita apenas ao
campo elétrico uniforme E , representado
 por suas linhas de campo,
e ao campo gravitacional terrestre g.

a) 2,0·104 m/s d) 1,6·105 m/s


b) 4,0·10 m/s
4
e) 3,2·105 m/s
c) 8,0·10 m/s
4

13. (FGV) Muitos experimentos importantes para o desenvolvimento


científico ocorreram durante o século XIX. Entre eles, destaca-se a
É correto afirmar que, enquanto se move na região indicada entre
experiência de Millikan, que determinou a relação entre a carga q
as placas, a carga fica sujeita a uma força resultante de módulo
e a massa m de uma partícula eletrizada e que, posteriormente,
a) q · E + m · g. c) q · E – m · g. e) m · (E – g). levaria à determinação da carga e da massa das partículas
b) q · (E – g). d) m · q · (E – g). elementares. No interior de um recipiente cilíndrico, em que será

256 PROENEM.COM.BR PRÉ-VESTIBULAR


14 CAMPO UNIFORME E TRABALHO ELÉTRICO FÍSICA II

produzido alto vácuo, duas placas planas e paralelas, ocupando a a) e ⋅ V ⋅ d


maior área possível, são mantidas a uma curta distância d, e entre
elas é estabelecida uma diferença de potencial elétrico constante e⋅d
b)
U. Variando-se d e U, é possível fazer com que uma partícula de V
massa m eletrizada com carga q fique equilibrada, mantida em V ⋅d
c)
repouso entre as placas. No local da experiência, a aceleração da e
gravidade é constante de intensidade g.
d) e
Nessas condições, a relação g/m será dada por V ⋅d
d ⋅ U² d⋅g d⋅g e⋅V
a) . c) . e) . e)
g U² U d
b) g ⋅ U² . d⋅U 16. (UNICAMP) Existem na natureza forças que podemos observar
d) .
d g em nosso cotidiano. Dentre elas, a força gravitacional da Terra e
a força elétrica. Num experimento, solta-se uma bola com carga
14. (UPF) Uma pequena esfera de 1,6 g de massa é eletrizada elétrica positiva, a partir do repouso, de uma determinada altura,
retirando-se um número n de elétrons. Dessa forma, quando a numa região em que há um campo elétrico dirigido verticalmente
esfera é colocada em um campo elétrico uniforme de 1·109 N/C, para baixo, e mede-se a velocidade com que ela atinge o chão. O
na direção vertical para cima, a esfera fica flutuando no ar em experimento é realizado primeiramente com uma bola de massa
equilíbrio. Considerando que a aceleração gravitacional local g é 10 m e carga q, e em seguida com uma bola de massa 2m e mesma
m/s2 e a carga de um elétron é 1,6·10–19C, pode-se afirmar que o carga q.
número de elétrons retirados da esfera é:
a) 1·1019 b) 1·1010 c) 1·109 d) 1·108 e) 1·107

15. Modelos elétricos são frequentemente utilizados para


explicar a transmissão de informações em diversos sistemas do
corpo humano. O sistema nervoso, por exemplo, é composto por
neurônios (figura 1), células delimitadas por uma fina membrana
lipoproteica que separa o meio intracelular do meio extracelular. A
parte interna da membrana é negativamente carregada e a parte
externa possui carga positiva (figura 2), de maneira análoga ao que Desprezando a resistência do ar, é correto afirmar que, ao atingir
ocorre nas placas de um capacitor. o chão,
a) as duas bolas terão a mesma velocidade.
b) a velocidade de cada bola não depende do campo elétrico.
c) a velocidade da bola de massa m é maior que a velocidade da
bola de massa 2m.
d) a velocidade da bola de massa m é menor que a velocidade da
bola de massa 2m.

17. (UECE) Considere um capacitor ideal, composto por um par


de placas metálicas paralelas, bem próximas uma da outra, e
carregadas eletricamente com cargas opostas. Na região entre as
placas, distante das bordas, o vetor campo elétrico
a) tem direção tangente às placas.
b) tem direção normal às placas.
c) é nulo, pois as placas são condutoras.
d) é perpendicular ao vetor campo magnético gerado pela
distribuição estática de cargas nas placas.

18. (FUVEST) A energia potencial elétrica U de duas partículas em


função da distância r que as separa está representada no gráfico
da figura abaixo.

A figura 3 representa um fragmento ampliado dessa membrana,


de espessura d que está sob ação de um campo elétrico uniforme,
representado na figura por suas linhas de força paralelas entre
si e orientadas para cima. A diferença de potencial entre o meio
intracelular e o extracelular é V. Considerando a carga elétrica
elementar como e, o íon de potássio K+, indicado na figura 3, sob
ação desse campo elétrico, ficaria sujeito a uma força elétrica cujo
módulo pode ser escrito por

PRÉ-VESTIBULAR PROENEM.COM.BR 257


FÍSICA II 14 CAMPO UNIFORME E TRABALHO ELÉTRICO

Uma das partículas está fixa em uma posição, enquanto a outra 20. (UEG) Embora as experiências realizadas por Millikan tenham
se move apenas devido à força elétrica de interação entre elas. sido muito trabalhosas, as ideias básicas nas quais elas se
Quando a distância entre as partículas varia de ri = 3 ·10–10 m a apoiam são relativamente simples. Simplificadamente, em suas
rf = 9 ·10–10 m, a energia cinética da partícula em movimento experiências, R. Millikan conseguiu determinar o valor da carga
a) diminui 1·10–18 J do elétron equilibrando o peso de gotículas de óleo eletrizadas,
colocadas em um campo elétrico vertical e uniforme, produzido
b) aumenta 1·10–18 J por duas placas planas ligadas a uma fonte de voltagem, conforme
c) diminui 2·10–18 J ilustrado na figura abaixo.
d) aumenta 2·10–18 J
e) não se altera.

19. (FUVEST) Na figura, A e B representam duas placas metálicas;


a diferença de potencial entre elas é VB – VA = 2,0 x 104 V. As linhas
tracejadas 1 e 2 representam duas possíveis trajetórias de um
elétron, no plano da figura.
Supondo que cada gotícula contenha cinco elétrons em excesso,
ficando em equilíbrio entre as placas separadas por d = 1,50 cm e
submetendo-se a uma diferença de potencial VAB= 600 V, a massa
de cada gota vale, em kg:
a) 1,6·10–15 c) 6,4·10–15
b) 3,2·10–15 d) 9,6·10–15

EXERCÍCIOS DE
APROFUNDAMENTO
Considere a carga do elétron igual a -1,6 x 10 C e as seguintes
-19

afirmações com relação à energia cinética de um elétron que sai do 01. (FUVEST) Duas placas metálicas planas e circulares, de raio
ponto X na placa A e atinge a placa B: R, separadas por uma distância d<<R, estão dispostas na direção
horizontal. Entre elas, é aplicada uma diferença de potencial V, de
I. Se o elétron tiver velocidade inicial nula, sua energia cinética, ao
modo que a placa de cima fica com carga negativa e a de baixo,
atingir a placa B, será 3,2 x 10-15 J.
positiva. No centro da placa superior, está afixado um fio isolante de
II. A variação da energia cinética do elétron é a mesma, comprimento L < d com uma pequena esfera metálica presa em sua
independentemente de ele ter percorrido as trajetórias 1 ou 2. extremidade, como mostra a figura. Essa esfera tem massa m e está
III. O trabalho realizado pela força elétrica sobre o elétron na traje- carregada com carga negativa -q. O fio é afastado da posição de
tória 2 é maior do que o realizado sobre o elétron na trajetória 1. equilíbrio de um ângulo θ, e a esfera é posta em movimento circular
uniforme com o fio mantendo o ângulo θ com a vertical.
Apenas é correto o que se afirma em
a) I. b) II. c) III. d) I e II. e) I e III.

TEXTO PARA A PRÓXIMA QUESTÃO:

Os Dez Mais Belos Experimentos da Física


A edição de setembro de 2002 da revista Physics World
apresentou o resultado de uma enquete realizada entre seus
leitores sobre o mais belo experimento da Física. Na tabela abaixo
são listados os dez experimentos mais votados.

1) Experimento da dupla fenda 6) Experimento com a Determine


de Young, realizado com balança de torsão, realizada
elétrons. por Cavendish. a) o módulo E do campo elétrico entre as placas;
b) os módulos T e F, respectivamente, da tração no fio e da força
7) Medida da circunferência resultante na esfera;
2) Experimento da queda dos da Terra, realizada por
corpos, realizada por Galileu. c) a velocidade angular ω da esfera.
Erastóstenes.
Note e adote:
8) Experimento sobre o
A aceleração da gravidade é g.
movimento de corpos num
3) Experimento da gota de óleo. Forças dissipativas devem ser ignoradas.
plano inclinado, realizado por
Galileu.
02. (UERJ)
 O esquema abaixo representa um campo elétrico
4) Decomposição da luz solar
uniforme E, no qual as linhas verticais correspondem às superfícies
com um prisma, realizada por 9) Experimento de Rutherford.
equipotenciais. Uma carga elétrica puntiforme, de intensidade 400
Newton.
µC, colocada no ponto A, passa pelo ponto B após algum tempo.
5) Experimento da interferência
da luz, 10) Experiência do pêndulo
de Foucault.
realizada por Young.

258 PROENEM.COM.BR PRÉ-VESTIBULAR


14 CAMPO UNIFORME E TRABALHO ELÉTRICO FÍSICA II

à ação simultânea do campo gravitacional e do campo elétrico e


deixa o capacitor em um ponto Q, como registrado na figura.

Determine, em joules, o trabalho realizado pela força elétrica para


deslocar essa carga entre os pontos A e B.

03. (FUVEST) A região entre duas placas metálicas, planas e


paralelas está esquematizada na figura abaixo. As linhas tracejadas
representam o campo elétrico uniforme existente entre as placas. Deduza a razão q/m, em termos do campo E e das distâncias d e h.
A distância entre as placas é 5 mm e a diferença de potencial entre
elas é 300 V. As coordenadas dos pontos A, B e C são mostradas
na figura. Determine GABARITO
EXERCÍCIOS PROPOSTOS
01. A 05. B 09. C 13. E 17. B
02. C 06. B 10. A 14. D 18. D
03. A 07. A 11. B 15. E 19. D
04. E 08. B 12. C 16. C 20. B

V= E ⋅ d
EXERCÍCIOS DE APROFUNDAMENTO
mgd + qV
=F a)
01.
F Tsen
∴ E== θV ⋅ sen θ
cp = F dcos θ
d
m
(
2 ( mgd + qV )
mgd + qV ) sen θ
= =
b)
∴ Fω Lsen θ tgθ ⋅
d d cos θ
mgd + qV
∴ω =
c)
mdLcos θ
02. τA → B = 32 · 10-3 J
03. a) E = 6 x 104 V/m.
b) VBC = 0 V.
c) τ = 2,88 x 10-17 J.
04. U = 400 V.
(g ⋅ d)
05. q m =
(E ⋅ h)

a) os módulos EA, EB e EC do campo elétrico nos pontos A, B e C, ANOTAÇÕES


respectivamente;
b) as diferenças de potencial VAB e VBC entre os pontos A e B e
entre os pontos B e C, respectivamente;
c) o trabalho τ realizado pela força elétrica sobre um elétron que
se desloca do ponto C ao ponto A.
Note e adote:
O sistema está em vácuo.
Carga do elétron = -1,6 x 10-19 C.

04. (UERJ) No experimento de Millikan, que determinou a carga


do elétron, pequenas gotas de óleo eletricamente carregadas são
borrifadas entre duas placas metálicas paralelas. Ao aplicar um
campo elétrico uniforme entre as placas, da ordem de 2 x 104 V/m,
é possível manter as gotas em equilíbrio, evitando que caiam sob a
ação da gravidade.
Considerando que as placas estão separadas por uma distância
igual a 2 cm, determine a diferença de potencial necessária para
estabelecer esse campo elétrico entre elas.

05. (UNESP) Em um seletor de cargas, uma partícula de massa m


e eletrizada com carga q é abandonada em repouso em um ponto
P, entre as placas paralelas de um capacitor polarizado com um
campo elétrico E. A partícula sofre deflexão em sua trajetória devido

PRÉ-VESTIBULAR PROENEM.COM.BR 259


FÍSICA II 14 CAMPO UNIFORME E TRABALHO ELÉTRICO

260 PROENEM.COM.BR PRÉ-VESTIBULAR


15
FÍSICA II
ELETROSTÁTICA: REVISÃO

CARGA ELÉTRICA TRABALHO ELÉTRICO


• São de duas naturezas: positivas e negativas, cargas O trabalho da força elétrica para levar uma carga de um ponto
opostas se atraem e cargas iguais se repelem. A ao ponto B não depende do caminho, apenas dos dois pontos
• É uma grandeza conservada. inicial e final.
• É uma grandeza quantizada: Q = n . e

FORÇA ELÉTRICA
É uma interação entre duas cargas, pode ser de atração e repulsão:

τI =τII =τIII
K ⋅ Q1 ⋅ Q2
F=
d2   q(VA  VB )

K0 = 9 . 10 Nm /c
9 2 2

CONDUTOR ELÉTRICO
O campo elétrico no interior de um condutor é nulo e o potencial

CAMPO ELÉTRICO elétrico é constante, com o mesmo valor do potencial em sua


superfície (d = R)
Responsável pela ação à distância entre as duas cargas, pela
interação entre elas. Uma carga gera um campo elétrico à sua
volta, que é uma grandeza vetorial e seu sentido depende do sinal 
da carga geradora. E int = 0
 V = cons tante
 F KQ
E= E=
q d2

• SUPERFÍCIE EQUIPOTENCIAL - Todos os pontos possuem

o mesmo potencial elétrico.


• CAMPO ELÉTRICO UNIFORME – Região onde todos os
pontos possuem o mesmo vetor campo elétrico. A ddp entre
dois pontos no interior dessa região pode ser encontrada pelo
produto do campo elétrico constante e a distância entre as
superfícies equipotenciais que os pontos pertencem.
POTENCIAL ELÉTRICO 1 2 3
Medida da energia potencial em um ponto pela carga existente +  –
nele. A diferença de potencial (ddp) entre dois pontos é responsável + E –
pelo movimento de cargas (corrente elétrica). O potencial elétrico é + – UAB = E . d
+ B –
uma grandeza escalar, gerado no espaço por uma carga. A
+ –
KQ1Q2 + – Distâncias entre
EP EP = + –
V= d as equipotenciais
q + –
Energia Potencial Elétrica + –
KQ V1 > V 2 > V 3
V=


d
Superfícies Equipotenciais

UNIDADES DO SI:
[ F ] – Newton (N) [ τ ] – Joule (J)
[ Ep ] – Joule (J) [ E ] – Newton/Coulomb (N/C) ou
Volt/Metro (V/m)
[ V ] – Volt (V)

PRÉ-VESTIBULAR PROENEM.COM.BR 261


FÍSICA II 15 ELETROSTÁTICA: REVISÃO

PROEXPLICA

As ondas eletromagnéticas são essenciais para o nosso


mundo atual, principalmente quando pensamos nas
tecnologias de comunicação. Elas são as responsáveis por
tornar possível a existência de controles remotos, celulares
etc. Na questão, deve ser analisada uma situação onde as
ondas eletromagnéticas não interagem com um condutor.

EXERCÍCIO RESOLVIDO

01. (ENEM) Duas irmãs, que dividem o mesmo quarto de Calcule o módulo e indique a direção e sentido do campo elétrico
estudos, combinaram de comprar duas caixas com tampas no ponto A.
para guardarem seus pertences dentro de suas caixas, Adote: K = 9·109 N·m²/C²
evitando, assim, a bagunça sobre a mesa de estudos.
Uma delas comprou uma metálica, e a outra, uma caixa de
03. Nos vértices de um triângulo equilátero de lado L = 27 cm foram
madeira de área e espessura lateral diferentes, para facilitar
fixadas cargas elétricas puntiformes idênticas Q = +8 µC, conforme
a identificação. Um  dia as meninas foram estudar para a
a imagem.
prova de Física e, ao se acomodarem na mesa de estudos,
guardaram seus celulares ligados dentro de suas caixas. Ao
longo desse dia, uma delas recebeu ligações telefônicas,
enquanto os amigos da outra tentavam ligar e recebiam a
mensagem de que o celular estava fora da área de cobertura
ou desligado.
Para explicar essa situação, um físico deveria afirmar que
o material da caixa cujo telefone celular não recebeu as
ligações é de:
a) madeira, e o telefone não funcionava porque a madeira
não é um bom condutor de eletricidade.
b) metal, e o telefone não funcionava devido à blindagem
eletrostática que o metal proporcionava.
c) metal, e o telefone não funcionava porque o metal refletia Calcule a energia potencial eletrostática do sistema.
todo tipo de radiação que nele incidia. Adote: K = 9·109 N·m²/C²
d) metal, e o telefone não funcionava porque a área lateral da
caixa de metal era maior. 04. Uma carga de prova q = +5 µC é movida do ponto A até o
ponto B, pela trajetória circular, conforme representado na imagem
e) madeira, e o telefone não funcionava porque a espessura
abaixo. As superfícies equipotenciais são verticais e igualmente
desta caixa era maior que a espessura da caixa de metal
espaçadas geradas por placas paralelas eletrizadas.
Resolução: B
O material que envolve a caixa tem que ser de metal, formando
uma espece de casca oca por dentro. Nessa situação,
dizemos que ocorreu uma blindagem eletrostática, chamada
de Gaiola de Faraday. A Gaiola de Faraday faz com que toda
carga que entre em contato com o material mantenha-se na
superfície, mantendo o campo elétrico no interior nulo.

EXERCÍCIOS
PROTREINO
Calcule:
01. Duas pequenas esferas condutoras idênticas estão eletrizadas. a) a direção, sentido e módulo do campo elétrico entre as placas;
A primeira esfera tem uma carga de 4Q e a segunda uma carga b) o trabalho da força elétrica no deslocamento da partícula de
de 8Q. Inicialmente, as duas esferas estão separadas por uma A para B.
distância d e a força eletrostática entre elas é F1. Em seguida, as
esferas são colocadas em contato e depois separadas por uma
05. Uma esfera eletrizada com carga de módulo |q| = 2,5 mC, foi
distância 3d. Nessa nova configuração, a força eletrostática entre
presa a um fio ideal sobre influência de um campo gravitacional e de
as esferas é F2. Determine a relação entre as forças F1 e F2.
um campo elétrico gerado por duas placas paralelas e eletrizadas.
A esfera se encontra em equilíbrio formando um ângulo w = 45º
02. Nos vértices de um triângulo equilátero de lado L = 3 m foram com a vertical, conforme imagem abaixo:
fixadas duas cargas elétricas puntiformes idênticas Q = + 3  µC,
conforme a imagem.

262 PROENEM.COM.BR PRÉ-VESTIBULAR


15 ELETROSTÁTICA: REVISÃO FÍSICA II

a) 4,0 × 10-5 N. c) 1,2 × 10-4 N. e) 2,0 × 10-4 N.


b) 8,0 × 10 N. -5
d) 1,6 × 10 N. -4

04. (FATEC) Três esferas condutoras idênticas A, B e C estão sobre


tripés isolantes. A esfera A tem inicialmente carga elétrica de 6,4μC,
enquanto B e C estão neutras.

Encostam-se as esferas A e B até o equilíbrio eletrostático e


a) Indique a direção e o sentido do campo elétrico entre as placas separam-se as esferas. Após isso, o procedimento é repetido, desta
e calcule seu módulo. vez feita com as esferas B e C. Sendo a carga elementar 1,6 . 10-19
b) Complete o desenho representando a polaridade de cada placa. C o número total de elétrons que, nessas duas operações, passam
c) Determine se a esfera possui excesso de prótons ou de elétrons. de uma esfera a outra é
d) Calcule a massa da esfera. a) 1,0 . 1013 c) 3,0 . 1013 e) 8,0 . 1013
b) 2,0 . 1013 d) 4,0 . 1013

05. (FATEC) Analise as afirmações a seguir:


EXERCÍCIOS
I. Todo objeto que tem grande quantidade de elétrons está
PROPOSTOS eletrizado negativamente.
II. Eletrizando-se por atrito dois objetos neutros obtêm-se, ao final
deste processo de eletrização, dois objetos eletrizados com
01. (PUC-RJ) Dois objetos metálicos esféricos idênticos, contendo carga de mesmo sinal.
cargas elétricas de 1 C e de 5 C, são colocados em contato e depois III. Encostando-se um objeto A, eletrizado negativamente, em um
afastados a uma distância de 3 m. Considerando a Constante de pequeno objeto B, neutro, após algum tempo o objeto A ficará
Coulomb k = 9 × 109 N m2/C2, podemos dizer que a força que atua neutro.
entre as cargas após o contato é:
Deve-se concluir, da análise dessas afirmações, que
a) atrativa e tem módulo 3 × 109 N.
a) apenas I é correta.
b) atrativa e tem módulo 9 × 109 N.
b) apenas II é correta.
c) repulsiva e tem módulo 3 × 109 N.
c) apenas II e III são corretas.
d) repulsiva e tem módulo 9 × 109 N.
d) I, II e III são corretas.
e) zero.
e) não há nenhuma correta.

02. (UNIFESP) Considere a seguinte “unidade” de medida: a


06. (UFRGS) Duas pequenas esferas metálicas idênticas e
intensidade da força elétrica entre duas cargas q, quando separadas
eletricamente isoladas, X e Y, estão carregadas com cargas
por uma distância d, é F. Suponha em seguida que uma carga q1
elétricas + 4 C e –8 C, respectivamente. As esferas X e Y estão
= q seja colocada frente a duas outras cargas, q2 = 3q e q3 = 4q,
separadas por uma distância que é grande em comparação com
segundo a disposição mostrada na figura.
seus diâmetros. Uma terceira esfera Z, idêntica às duas primeiras,
isolada e inicialmente descarregada, é posta em contato, primeiro,
com a esfera X e, depois, com a esfera Y.
A intensidade da força elétrica
As cargas elétricas finais nas esferas X, Y e Z são, respectivamente,
resultante sobre a carga q1,
devido às cargas q2 e q3, será a) + 2 C, –3 C e –3 C. d) 0, –2 C e –2 C.
a) 2F. b) + 2 C, + 4 C e –4 C. e) 0, 0 e –4 C.
b) 3F. c) + 4 C, 0 e –8 C.
c) 4F.
07. (PUC-MG) Dispõe-se de duas esferas metálicas, iguais e
d) 5F.
inicialmente descarregadas, montadas sobre pés isolantes e de
e) 9F. um bastão de ebonite, carregado negativamente. Os itens de I a IV
podem ser colocados numa ordem que descreva uma experiência
03. (UFRGS) Três cargas elétricas puntiformes idênticas, Q1, Q2 em que as esferas sejam carregadas por indução.
e Q3, são mantidas fixas em suas posições sobre uma linha reta, I. Aproximar o bastão de uma das esferas.
conforme indica a figura a seguir.
II. Colocar as esferas em contato.
III. Separar as esferas.
IV. Afastar o bastão.
Sabendo-se que o módulo da força elétrica exercida por Q1 sobre Qual é a opção que ordena de maneira ADEQUADA as operações?
Q2 é de 4,0 × 10-5 N, qual é o módulo da força elétrica resultante a) II, I, III, IV. c) I, III, IV, II.
sobre Q2? b) II, I, IV, III. d) IV, II, III, I.

PRÉ-VESTIBULAR PROENEM.COM.BR 263


FÍSICA II 15 ELETROSTÁTICA: REVISÃO

08. (UNIFESP) A figura representa a configuração de um campo 10. (UFU) Na figura a seguir, são apresentadas cinco linhas
elétrico gerado por duas partículas carregadas, A e B. equipotenciais, A-E, com os respectivos valores do potencial
elétrico.

Inicialmente, um aglomerado de partículas com carga total igual a


2,0 C está sobre a equipotencial A. Esse aglomerado é deslocado
para a equipotencial B. Em B o aglomerado sofre uma mudança
estrutural e sua carga passa de 2,0 C para 1,5 C. Esse novo
aglomerado de 1,5 C é deslocado para a equipotencial C e, em
seguida, para D, conservando-se a carga de 1,5 C. Em D ocorre uma
Assinale a alternativa que apresenta as indicações corretas para nova mudança estrutural e sua carga passa para 1,0 C. Por último,
as convenções gráficas que ainda não estão apresentadas nessa esse aglomerado de 1,0 C é deslocado para a equipotencial E.
figura (círculos A e B) e para explicar as que já estão apresentadas Considerando as afirmações apresentadas no enunciado anterior,
(linhas cheias e tracejadas). assinale a alternativa que corresponde ao trabalho realizado sobre
a) carga da partícula A: (+) o aglomerado para deslocá-lo de A para E.
carga da partícula B: (+) a) 12 J c) 8 J
linhas cheias com setas: linha de força b) 16 J d) 10 J
linhas tracejadas: superfície equipotencial
b) carga da partícula A: (+) 11. Como funciona a Máquina de Xerox
carga da partícula B: (–) Quando se inicia a operação em uma máquina de Xerox,
linhas cheias com setas: superfície equipotencial acende-se uma lâmpada, que varre todo o documento a ser
linhas tracejadas: linha de força copiado. A imagem é projetada por meio de espelhos e lentes
c) carga da partícula A: (–) sobre a superfície de um tambor fotossensível, que é um cilindro de
carga da partícula B: (–) alumínio revestido de um material fotocondutor.
linhas cheias com setas: linha de força Os fotocondutores são materiais com propriedade isolante
linhas tracejadas: superfície equipotencial no escuro. Mas, quando expostos à luz, são condutores. Assim,
d) carga da partícula A: (–) quando a imagem refletida nos espelhos chega ao tambor, as
carga da partícula B: (+) cargas superficiais do cilindro se alteram: as áreas claras do
linhas cheias com setas: superfície equipotencial documento eliminam as cargas elétricas que estão sobre a
linhas tracejadas: linha de força superfície do cilindro e as áreas escuras as preservam. Forma-se,
e) carga da partícula A: (+) então, uma imagem latente, que ainda precisa ser revelada. Para
carga da partícula B: (–) isso, o cilindro é revestido por uma fina tinta de pó, o tonalizador,
linhas cheias com setas: linha de força ou toner, que adere à imagem latente formada sobre o tambor. Em
linhas tracejadas: superfície equipotencial seguida, toda a imagem passa para as fibras do papel, através de
pressão e calor. E, assim, chega-se à cópia final.
Fonte: Revista Globo Ciência, dez. 1996, p. 18.
09. (UFMG) Duas pequenas esferas isolantes - I e II -, eletricamente
carregadas com cargas de sinais contrários, estão fixas nas
O texto acima se refere a uma aplicação do fenômeno de
posições representadas nesta figura:
eletrização, pois é graças a ele que o toner adere ao cilindro
metálico mencionado. O processo de eletrização pode ocorrer
de três formas distintas: atrito, indução e contato, mas todos os
processos têm algo em comum. É CORRETO afirmar que o comum
destes processos é:
A carga da esfera I é positiva e seu módulo é maior que o da
esfera II. a) Deixar o corpo eletrizado, com um desequilíbrio entre o número
de cargas elétricas positivas e negativas.
Guilherme posiciona uma carga pontual positiva, de peso
desprezível, ao longo da linha que une essas duas esferas, de forma b) Deixar o corpo eletrizado, com um equilíbrio entre o número de
que ela fique em equilíbrio. cargas elétricas positivas e negativas.
Considerando-se essas informações, é CORRETO afirmar que c) Arrancar as cargas positivas do corpo eletrizado.
o ponto que melhor representa a posição de equilíbrio da carga d) Deixar o corpo eletrizado com uma corrente elétrica negativa.
pontual, na situação descrita, é o e) Deixar o corpo eletrizado com um campo magnético.
a) R. c) S.
b) P. d) Q. 12. Considere quatro esferas metálicas idênticas, A, B, C e D,
inicialmente separadas entre si. Duas delas, B e D estão inicialmente
neutras, enquanto as esferas A e C possuem cargas elétricas

264 PROENEM.COM.BR PRÉ-VESTIBULAR


15 ELETROSTÁTICA: REVISÃO FÍSICA II

iniciais, respectivamente, iguais a 3Q e –Q. Determine a carga 16. (PUCPR) Uma indústria automotiva faz a pintura de peças de um
elétrica final da esfera C após contatos sucessivos com as esferas veículo usando a pintura eletrostática, processo também conhecido
A, B e D, nessa ordem, considerando que após cada contato, as como pintura a pó. Nele, a pinça de um braço robótico condutor que
esferas são novamente separadas. segura a peça é ligada a um potencial de 1kV. A pinça junto com a
Q c) 2Q peça é imersa em um tanque de tinta em pó à 0V. A diferença de
a) potencial promove a adesão da tinta à peça, que depois é conduzida
4 d) 4Q pelo mesmo braço robótico a um forno para secagem. Após essa
Q e) 0 etapa, o robô libera a peça pintada e o processo é reiniciado. A
b)
2 ilustração a seguir mostra parte desse processo.
13. Na resolução, use quando necessário: g = 10 m/s², π = 3,14, c
= 3,0 x 108 m/s.
Para uma feira de ciências, os alunos pretendem fazer uma
câmara “antigravidade”. Para isso, os estudantes colocaram duas
placas metálicas paralelas entre si, paralelas à superfície da Terra,
com uma distância de 10,0 cm entre elas. Ligando essas placas a
uma bateria, eles conseguiram criar um campo elétrico uniforme
de 2,0 N/C. Para demonstrar o efeito “antigravidade”, eles devem
carregar eletricamente uma bolinha de isopor e inseri-la entre as
placas.
Sabendo que a massa da bolinha é igual a 0,50 g e que a placa
carregada negativamente está localizada no fundo da caixa,
escolha a opção que apresenta a carga com que se deve carregar a A indústria tem enfrentado um problema com a produção em série:
bolinha para que ela flutue. após duas ou três peças pintadas, a tinta deixa de ter adesão nas
Considere que apenas a força elétrica e a força peso atuam sobre peças. Uma possível causa para tal problema é:
a bolinha. a) o movimento do braço robótico carregando a peça no interior
a) 3,5 x 10-2 C c) -2,5 x 10-3 C e) -3,5 x 10-3 C da tinta gera atrito e aquece o sistema, anulando a diferença de
potencial e impedindo a adesão eletrostática.
b) -3,5 x 10-2 C d) 2,5 x 10-3 C
b) a ausência de materiais condutores faz com que não exista
14. (EEAR) Duas esferas idênticas e eletrizadas com cargas elétricas diferença de potencial entre a peça e a tinta.
q1 e q2 se atraem com uma força de 9N. Se a carga da primeira esfera c) cada peça pintada diminui a diferença de potencial até que,
aumentar cinco vezes e a carga da segunda esfera for aumentada após duas ou três peças pintadas, ela torne-se nula.
oito vezes, qual será o valor da força, em newtons, entre elas?
d) quando a pinça e a peça são imersas na tinta, ambos entram
a) 40 b) 49 c) 117 d) 360 e) 420 em equilíbrio eletrostático, o que impede que a tinta tenha
aderência sobre a superfície da peça.
15. e) com o tempo, a pinça acaba ficando recoberta por uma camada
de tinta que atua como isolante elétrico anulando a diferença
de potencial entre a peça e a tinta.

17. (UNIFESP) A presença de íons na atmosfera é responsável


pela existência de um campo elétrico dirigido e apontado para a
Terra. Próximo ao solo, longe de concentrações urbanas, num dia
claro e limpo, o campo elétrico é uniforme e perpendicular ao solo
horizontal e sua intensidade é de 120 V/m. A figura mostra as
linhas de campo e dois pontos dessa região, M e N.

Verificou-se que, numa dada região, o potencial elétrico V segue o


comportamento descrito pelo gráfico V x r acima.
(Considere que a carga elétrica do elétron é -1,6 · 10-19 C)
Baseado nesse gráfico, considere as seguintes afirmativas:
1. A força elétrica que age sobre uma carga q = 4 µC colocada na
posição r = 8 cm vale 2,5 · 10-7 N.
2. O campo elétrico, para r = 2,5 cm possui módulo E = 0,1 N/C.
3. Entre 10 cm e 20 cm o campo elétrico é uniforme.
O ponto M está a 1,20 m do solo, e N está no solo. A diferença de
4. Ao se transferir um elétron de r = 10 cm para r = 20 cm a energia
potencial elétrica aumenta de 8,0 · 10-22 J. potencial entre os pontos M e N é:

Assinale a alternativa correta. a) 100 V


a) Somente as afirmativas 1 e 3 são verdadeiras. b) 120 V
b) Somente as afirmativas 2 e 4 são verdadeiras. c) 125 V
c) Somente as afirmativas 1, 3 e 4 são verdadeiras. d) 134 V
d) Somente as afirmativas 2, 3 e 4 são verdadeiras. e) 144 V
e) As afirmativas 1, 2, 3 e 4 são verdadeiras.

PRÉ-VESTIBULAR PROENEM.COM.BR 265


FÍSICA II 15 ELETROSTÁTICA: REVISÃO

18. (PUCRJ) Um sistema A é formado por cargas elétricas positivas


e negativas situadas em posições fixas. A energia eletrostática total
do sistema é 54 µJ. Seja um outro sistema B similar ao sistema A,
exceto por duas diferenças: as cargas em B têm o dobro do valor
das cargas em A; as distâncias entre as cargas em B são o triplo
das distâncias em A.
Calcule em µJ a energia eletrostática do sistema B.
a) 18 d) 108
b) 54 e) 162
a) Qual a massa das maiores gotículas de óleo? Considere a gota
c) 72 esférica, a densidade do óleo η(óleo) = 9,0 x 102 kg/m3 e δ = 3.
b) Quanto tempo a gotícula leva para atravessar o coletor?
19. (UFLA) Duas esferas condutoras descarregadas e iguais 1 e
Considere a velocidade do ar arrastado pela ventoinha como
2 estão em contato entre si e apoiadas numa superfície isolante.
sendo 0,6 m/s e o comprimento do coletor igual a 0,30 m.
Aproxima‐se de uma delas um bastão eletrizado positivamente,
sem tocá‐la, conforme figura a seguir. c) Uma das gotículas de maior diâmetro tem uma carga de
8 x 10-19 C (equivalente à carga de apenas 5 elétrons!). Essa
Em seguida as esferas são afastadas e o bastão eletrizado é
gotícula fica retida no coletor para o caso ilustrado na figura?
removido.
A diferença de potencial entre as placas é de 50 V, e a distância
entre as placas do coletor é de 1 cm. Despreze os efeitos do
atrito e da gravidade.

03. (PUCRJ) Duas partículas de cargas q1 = 4·10-5 C e q2 = 1·10-5


C estão alinhadas no eixo x sendo a separação entre elas de 6 m.
Sabendo que q1 encontra-se na origem do sistema de coordenadas
e considerando k = 9·109 Nm²/C², determine:
É correto afirmar que a) a posição x, entre as cargas, onde o campo elétrico é nulo;
a) as esferas permanecem descarregadas, pois não há b) o potencial eletrostático no ponto x = 3 m;
transferência de cargas entre bastão e esferas. c) o módulo, a direção e o sentido da aceleração, no caso de ser
b) a esfera 1, mais próxima do bastão, fica carregada colocada uma partícula de carga q3 = -1·10-5 C e massa m3 =
positivamente e a esfera 2 carregada negativamente. 1,0 kg, no ponto do meio da distância entre q1 e q2.
c) as esferas ficam eletrizadas com cargas iguais e de sinais
opostos. 04. (PUCRJ) Quatro cargas (Q, 2Q, -Q e -2Q) estão colocadas nos
vértices de um quadrado de lado L.
d) as esferas ficam carregadas com cargas de sinais iguais e
ambas de sinal negativo, pois o bastão atrai cargas opostas. a) Faça um desenho da configuração das 4 cargas de modo que
o sistema possua a mais baixa energia eletrostática. Calcule
essa energia.
20. (FUVEST) Um sistema formado por três cargas puntiformes
iguais, colocadas em repouso nos vértices de um triângulo b) Na situação do item anterior, calcule o módulo da resultante
equilátero, tem energia potencial eletrostática igual a U. Substitui- das forças eletrostáticas agindo sobre a carga +Q.
se uma das cargas por outra, na mesma posição, mas com o dobro
do valor. A energia potencial eletrostática do novo sistema será 05. (UNIFESP) Uma carga elétrica puntiforme Q > 0 está fixa em
igual a: uma região do espaço e cria um campo elétrico ao seu redor.
a) 4U/3 d) 2U Outra carga elétrica puntiforme q, também positiva, é colocada
em determinada posição desse campo elétrico, podendo mover-
b) 3U/2 e) 3U se dentro dele. A malha quadriculada representada na figura está
c) 5U/3 contida em um plano xy, que também contém as cargas.

EXERCÍCIOS DE
APROFUNDAMENTO
01. (UDESC–ADAPTADA) Duas pequenas esferas estão separadas
por uma distância de 30 cm. As duas esferas repelem-se com uma
força de 7,5 x 10-6 N. Considerando que a soma da carga elétrica
das duas esferas é 20 nC, calcule a carga elétrica de cada esfera.

02. (UNICAMP) A fumaça liberada no fogão durante a preparação


de alimentos apresenta gotículas de óleo com diâmetros entre 0,05
µm e 1 µm. Uma das técnicas possíveis para reter estas gotículas
de óleo é utilizar uma coifa eletrostática, cujo funcionamento
é apresentado no esquema a seguir: a fumaça é aspirada por
uma ventoinha, forçando sua passagem através de um estágio
de ionização, onde as gotículas de óleo adquirem carga elétrica.
Estas gotículas carregadas são conduzidas para um conjunto de Quando na posição A, q fica sujeita a uma força eletrostática de
coletores formados por placas paralelas, com um campo elétrico módulo F exercida por Q.
entre elas, e precipitam-se nos coletores.

266 PROENEM.COM.BR PRÉ-VESTIBULAR


15 ELETROSTÁTICA: REVISÃO FÍSICA II

a) Calcule o módulo da força eletrostática entre Q e q, em função


apenas de F, quando q estiver na posição B.
b) Adotando 2 = 1,4 e sendo K a constante eletrostática do meio
onde se encontram as cargas, calcule o trabalho realizado pela
força elétrica quando a carga q é transportada de A para B.

GABARITO
EXERCÍCIOS PROPOSTOS
01. D 05. E 09. C 13. C 17. E
02. D 06. A 10. A 14. D 18. C
03. C 07. A 11. A 15. D 19. C
04. C 08. E 12. A 16. E 20. C

EXERCÍCIOS DE APROFUNDAMENTO
01. q2 = 15nC ou q2 = 5nC
02. a) m = 4,5·1016 kg
b) ∆t = 0,5s
c) A gotícula fica retida no coletor, pois t < ∆t.
03. a) x = 4 m
b) 15 x 104 V
c) i) módulo de a = 0,3 m/s2; ii) direção: eixo X; iii) sentido negativo.
5 Q2
04. a) Ep = 2 k0
2 L
22
 33   Q  33 Q2
Fr
b)=   ⋅  k0 2  ∴
= Fr k0 2
 4   L  2 L

05. a) F' = F
2
3kQq
b) WAB =
40d

ANOTAÇÕES

PRÉ-VESTIBULAR PROENEM.COM.BR 267


FÍSICA II 15 ELETROSTÁTICA: REVISÃO

268 PROENEM.COM.BR PRÉ-VESTIBULAR


16 CORRENTE ELÉTRICA E FÍSICA II

RESISTORES

CORRENTE ELÉTRICA: DEFINIÇÃO


Nos condutores, as cargas elétricas que podem se mover
estão num movimento desordenado, caótico. Quando se aplica no
condutor uma diferença de potencial (ddp), o movimento das cargas
livres passa a ser orientado, ordenado. Ao movimento ordenado de
cargas elétricas é o que denominamos Corrente Elétrica.

CORRENTE ALTERNADA (AC) E


CORRENTE CONTÍNUA (DC)
SENTIDO REAL X SENTIDO CONVENCIONAL Os elétrons que se deslocam por um condutor podem vir
No início do desenvolvimento da eletricidade, o homem achou sempre de um mesmo sentido ou alterná-los. A corrente fornecida
que eram as cargas positivas, prótons, que se deslocavam no por uma bateria ou por uma pilha, por exemplo, possui um sentido
sentido do campo elétrico gerado no interior do condutor. Somente único se movendo do polo positivo para o polo negativo (sentido
muito tempo depois foi percebido o equívoco: são os elétrons que convencional), essa é a chamada corrente contínua. Já a corrente
se deslocam em sentido contrário ao do campo elétrico gerado que chega às nossas casas, gerada nas usinas, possui um
no interior do condutor. O sentido real da corrente elétrica num sentido que se alterna periodicamente, aqui no Brasil a frequência
condutor metálico é o do movimento dos elétrons. da corrente alternada distribuída é de 60Hz. Essa corrente que
muda de sentido é gerada por uma variação do fluxo de campo
magnético, a chamada Lei de Faraday-Lenz, que estudaremos com
mais detalhes nos próximos módulos.

Mas, por tradição, adota-se para sentido da corrente elétrica,


aquele que o homem primeiro pensou, o dos movimentos dos
prótons no sentido do campo elétrico. Este é o denominado sentido
convencional da corrente elétrica. Quando simplesmente se fala
em sentido da corrente elétrica, estamos falando no convencional.

INTENSIDADE DE CORRENTE ELÉTRICA RESISTORES


Denomina-se intensidade média da corrente elétrica a razão O resistor é aquele dispositivo do circuito elétrico que possui
entre a quantidade de carga elétrica (Δq) que passa por uma como principal característica transformar energia elétrica em
seção do condutor e o intervalo de tempo (Δt) que ela gasta para energia térmica, fenômeno conhecido como Efeito Joule. Essa
passar. Quando o fluxo de carga se mantém constante, não há dissipação se deve ao atrito entre os elétrons da corrente elétrica
necessidade de utilizar a expressão intensidade de corrente média, e os átomos do material e existem várias aplicações no dia a
basta usar intensidade de corrente. dia, como um chuveiro elétrico, um ferro de passar roupas ou até
mesmo em uma lâmpada incandescente.
Q
i= Em um circuito elétrico, o resistor é normalmente representado
∆t
pelos seguintes símbolos abaixo:
A unidade SI de intensidade de corrente elétrica é o ampère (A),
razão entre o coulomb (C) e o tempo (s). Quando se diz que uma
corrente é de 1 A, queremos dizer que passa por uma seção do
condutor uma carga de 1C num intervalo de 1s.

PROPRIEDADE DO GRÁFICO I X T A dificuldade que a corrente elétrica tem em atravessar


Em qualquer gráfico de intensidade de corrente versus tempo, um resistor é denominada Resistência Elétrica (R). É comum na
i x t, a área entre a função e o eixo dos tempos fornece o valor linguagem diária dizer que queimou a resistência do chuveiro,
numérico da carga elétrica que atravessa o condutor no intervalo quando na realidade o que queimou foi o resistor.
de tempo considerado.

PRÉ-VESTIBULAR PROENEM.COM.BR 269


FÍSICA II 16 CORRENTE ELÉTRICA E RESISTORES

RESISTORES ÔHMICOS E NÃO ÔHMICOS R=


ρ ⋅L
Existem basicamente dois tipos de resistores: o ôhmico, que A
é aquele que obedece às Leis de Ohm (por isso o nome), quando
mantidos à temperatura constante apresentam resistência O (ρ) representa a constante de proporcionalidade denomi-
constante, e os não ôhmicos são aqueles que possuem sua nada resistividade, e que caracteriza o material de que é feito
resistência variável e por isso não obedecem as leis de Ohm. o condutor. Cada material possui um valor de resistividade, que
quando necessário é fornecido nos problemas.
Obviamente quanto maior a resistividade do material, maior a
PRIMEIRA LEI DE OHM resistência elétrica do resistor. Portanto, substâncias de resistivi-
Se experimentalmente impusermos a um resistor diferentes dade alta possuem baixa condutividade e vice-versa.
diferenças de potencial (d.d.p) ele será percorrido por diferentes
correntes elétricas. Definimos como resistores ôhmicos quando,
MATERIAL RESISTIVIDADE (ω⋅m)
para uma dada temperatura:
Prata 1,6 x 10-8
U1 U2 U3 Un
= = = = ... = R (resistência constante)
i1 i2 i3 in Cobre 1,7 x 10-8
Ouro 2,4 x 10-8
Ou seja, U = R.i
Alumínio 2,8 x 10-8
Gráfico: Resistor Ôhmico
Chumbo 2,2 x 10-7
Vidro 1 x 1010 a 1 x 1014
Borracha ≈ 1013

PROEXPLICA

A resistência elétrica dos fios é uma característica de cada fio.


Tanto o material que o constitui, quanto o seu comprimento
e diâmetro devem ser levados em consideração na hora dos
cálculos. A questão abaixo compara fios feitos de componen-
tes diferentes.
Ohm percebeu que a corrente em um circuito é diretamente
proporcional à d.d.p estabelecida no circuito, e inversamente
proporcional à resistência do circuito. Analisando as unidades
EXERCÍCIO RESOLVIDO
Volts
Amperes =
Ohms 01. (ENEM) A resistência de um fio é determinada pelas
dimensões e pelas propriedades estruturais do material. A
A Lei de Ohm afirma que quando uma diferença de potencial de condutividade (σ) caracteriza a estrutura do material, de tal
1V é aplicada através de um circuito de resistência 1Ω , produzirá forma que a resistência de um fio, pode ser determinado
uma corrente de 1A. conhecendo-se L, o comprimento do fio, e A, a área de
Para um circuito com resistores ôhmicos, ou seja, resistores seção reta. A tabela relaciona o material à sua respectiva
de resistência constante, a corrente e a voltagem são proporcionais resistividade em temperatura ambiente.
entre si, na prática isso significa que a corrente dobrará sua
intensidade quando a voltagem também dobrar a sua. Mas se a MATERIAL CONDUTIVIDADE (S · M/MM2)
resistência do circuito for dobrada, mantendo a d.d.p constante,
Alumínio 34,2
a corrente terá sua intensidade caindo a metade do valor inicial.
Quanto maior a resistência do circuito, menor será a intensidade da Cobre 61,7
corrente elétrica a uma certa d.d.p.
Ferro 10,2
Prata 65,2
SEGUNDA LEI DE OHM
A Segunda Lei de Ohm mostra como calcular a resistência Tungstênio 18,8
elétrica de um resistor, baseando-se nos fatores que a influenciam: Mantendo-se as mesmas dimensões geométricas, o fio que
as dimensões e o material de que é feito o resistor. apresenta menor resistência elétrica é aquele feito de:
Através de experiências, pode-se concluir que quanto maior for a) tungstênio. d) cobre.
o comprimento do fio, maior o valor de sua resistência elétrica, isto
b) alumínio. e) prata.
é, uma relação diretamente proporcional. E quanto maior for a sua
área de seção reta, menor o valor de sua resistência, uma relação c) ferro.
inversamente proporcional. Resolução: E
Na condição de todos os fios terem mesmas dimensões
geométricas, terá menor resistência aquele que tiver
maior condutividade. Pelos dados apresentados na tabela,
observamos que a prata é o material de maior condutividade,
portanto, menor resistividade e menor resistência elétrica.
Considerando o resistor como um fio condutor (como na figura
anterior) de área de seção A e comprimento L, chegamos na relação:

270 PROENEM.COM.BR PRÉ-VESTIBULAR


16 CORRENTE ELÉTRICA E RESISTORES FÍSICA II

EXERCÍCIOS
PROTREINO EXERCÍCIOS

01. Considere que uma bateria, inicialmente descarregada, é PROPOSTOS


carrega com uma corrente média im = 2,4 A até atingir sua carga
máxima de Q = 0,8 Ah. Calcule o tempo gasto, em minutos, para 01. (UERJ) Pela seção de um condutor metálico submetido a uma
carregar a bateria. tensão elétrica, atravessam 4,0 × 1018 elétrons em 20 segundos.
A intensidade média da corrente elétrica, em ampère, que se
02. O carro elétrico é uma alternativa aos veículos que consomem estabelece no condutor corresponde a:
energia não renovável e poluente. Determine a autonomia, Dado: carga elementar = 1,6 × 10-19 C.
quantos quilômetros o carro consegue se deslocar sem precisar a) 1,0 × 10-2 c) 2,4 × 10-3
recarregar, de um carro elétrico que se desloca a 50 km/h, se a
corrente empregada nessa velocidade é de 25 A e a carga máxima b) 3,2 × 10 -2
d) 4,1 × 10-3
armazenada em suas baterias é de Q = 100 Ah.
02. (EFOMM) Por uma seção transversal de um fio cilíndrico de
03. O comportamento da intensidade da corrente elétrica em um cobre passam, a cada hora, 9,00 × 1022 elétrons. O valor aproximado
circuito em relação ao tempo é mostrado no gráfico a seguir: da corrente elétrica média no fio, em ampères, é
Dado: carga elementar e = 1,60 × 10-19 C.
a) 14,4 d) 4,00
b) 12,0 e) 1,20
c) 9,00

03. (UTFPR) Assinale a alternativa correta.


A grandeza intensidade de corrente elétrica tem como unidade
de medida ampère e essa unidade é definida pela razão (divisão)
entre duas outras unidades, que são, respectivamente,
a) coulomb e segundo.
b) volt e segundo.
c) coulomb e volt.
d) joule e volt.
e) volt e ohm.

04. (UNICAMP—ADAPTADA) Quando as fontes de tensão contínua


que alimentam os aparelhos elétricos e eletrônicos são desligadas,
Calcule a carga elétrica que atravessa uma área da seção do elas levam normalmente certo tempo para atingir a tensão de U
circuito no intervalo de tempo de 0 a 30 segundos. = 0V. Um estudante interessado em estudar tal fenômeno usa
um amperímetro e um relógio para acompanhar o decréscimo da
04. O comportamento dos resistores R e r não se altera para corrente que circula pelo circuito a seguir em função do tempo, após
valores de ddp até 100 volts. Analise o gráfico e determine se a fonte ser desligada em t = 0s. Usando os valores de corrente e
os resistores são ôhmicos ou não-ôhmicos e se R possui maior tempo medidos pelo estudante, pode-se dizer que a diferença de
resistência elétrica que r. potencial sobre o resistor R = 0,5 kW para t = 400ms é igual a

05. Durante um experimento realizado com um condutor que a) 6V d) 40V


obedece à Lei de Ohm, observou-se que seu comprimento triplicou,
enquanto a área da sua secção transversal foi reduzida à metade. b) 12V e) 60V
Neste caso, se as demais condições experimentais permanecerem c) 20V
inalteradas, determine a resistência final do condutor, em relação
ao original.

PRÉ-VESTIBULAR PROENEM.COM.BR 271


FÍSICA II 16 CORRENTE ELÉTRICA E RESISTORES

05. (UERJ SIMULADO) O gráfico abaixo indica o comportamento em minuto, que esse notebook pode ser usado antes que ela
da corrente elétrica em função do tempo em um condutor. “descarregue” completamente é
a) 24,0 c) 132 e) 528
A carga elétrica, b) 36,7 d) 333
em coulombs, que
passa por uma seção
08. (IFSP) Dois dos principais efeitos causados pela passagem de
transversal desse
uma corrente elétrica são:
condutor em 15s é
igual a: I. Efeito térmico ou Joule – uma corrente elétrica, pela sua
passagem, provoca uma variação de temperatura num condutor.
a) 450
II. Efeito magnético – toda corrente elétrica cria ao seu redor um
b) 600
campo magnético no espaço em torno de si. Isto pode ser
c) 750 verificado experimentalmente aproximando-se uma bússola
d) 900 de um condutor sendo percorrido por corrente elétrica: a
agulha sobre deflexão.
06. (PUC-PR) Leia as informações a seguir. Considerando um condutor metálico percorrido por uma corrente
de intensidade de 1A, assinale a alternativa correta.
a) A corrente de intensidade de 1A corresponde a 2 Coulomb (C)
por segundo.
b) Em 12s, a quantidade de carga que atravessará uma região do
condutor será de 10C.
c) Esta corrente elétrica corresponde a um fluxo de 6,25 × 1018
elétrons por segundo, considerando a carga elementar igual
a 1,6 × 10-19C.
d) A velocidade média dos elétrons que constituem a corrente é
igual à velocidade da luz no vácuo.
e) O sentido convencional da corrente é igual ao movimento dos
elétrons livres dentro do condutor.

09. (UECE) Considere um resistor ligado a uma bateria e dissipando


calor por efeito Joule. Pelo resistor, são medidos 3μC/s de carga
elétrica. Assim, a corrente elétrica pelo resistor é
a) 3 · 106A c) 3 · 106 μA
ARTUSO, Alysson R., SOARES, Marlon V. Vivá Física. Curitiba: Ed. Positivo, Vol. 3, 2016, p. 210.
b) 3 · 10 μA
-6
d) 3 · 10-6 A
A grande diversidade nos regimes de oferta de energia em cada
região confere ao sistema elétrico brasileiro uma característica 10. (UNIGRANRIO - MEDICINA) Dependendo da intensidade
muito peculiar: a demanda de energia pode ser atendida por uma da corrente elétrica que atravesse o corpo humano, é possível
grande variedade de gerações ao longo do território nacional. [...] sentir vários efeitos, como dores, contrações musculares, parada
O esquema a seguir mostra as etapas da transmissão da energia respiratória, entre outros, que podem ser fatais. Suponha que
elétrica. uma corrente de 0,1A atravesse o corpo de uma pessoa durante
2,0 minutos. Qual o número de elétrons que atravessa esse corpo,
A tensão elétrica produzida pela usina é elevada antes da
sabendo que o valor da carga elementar do elétron é 1,6 · 10-19C.
transmissão e depois rebaixada antes de ser distribuída para a área
residencial. A razão para que seja adotado tal procedimento é a) 1,2 · 1018 c) 7,5 · 1019 e) 3,2 · 1019
a) a economia gerada pela possibilidade de usar fios mais finos b) 1,9 · 10 20
d) 3,7 · 10 19

nas linhas de transmissão.


b) o aumento da potência elétrica transmitida para as residências 11. (UNICAMP) Drones vêm sendo utilizados por empresas
ao final do processo. americanas para monitorar o ambiente subaquático. Esses drones
podem substituir mergulhadores, sendo capazes de realizar
c) a redução dos efeitos gravitacionais sobre a corrente elétrica mergulhos de até cinquenta metros de profundidade e operar por
transmitida. até duas horas e meia.
d) o aumento da velocidade de transmissão da corrente elétrica. Considere um drone que utiliza uma bateria com carga total q =
e) a criação de uma corrente elétrica variável na rede. 900 mAh. Se o drone operar por um intervalo de tempo igual a ∆t =
90 min a corrente média fornecida pela bateria nesse intervalo de
07. (ENEM PPL) A figura mostra a bateria de um computador tempo será igual a
portátil, a qual necessita de uma corrente elétrica de 2 A para Dados: Se necessário, use aceleração da gravidade g = 10 m/s²
funcionar corretamente. aproxime π = 3,0 e 1 atm = 105 Pa.
a) 10 mA c) 1.350 mA
b) 600 mA d) 81.000 mA

12. (EEAR) Uma barra homogênea de grafite no formato de um


paralelepípedo, com as dimensões indicadas na figura, é ligada a
um circuito elétrico pelos condutores ideais A e B. Neste caso, a
resistência elétrica entre os terminais A e B é de ____ ohms.
Quando a bateria está completamente carregada, o tempo máximo,

272 PROENEM.COM.BR PRÉ-VESTIBULAR


16 CORRENTE ELÉTRICA E RESISTORES FÍSICA II

A seção reta de um átomo de cobre é 0,05 nm² e a resistividade do


cobre é 17 Ω · nm. Um engenheiro precisa estimar se seria possível
introduzir esses nanofios nos microprocessadores atuais.

Considere:
mm2
1. a resistividade do grafite: ρ= 75Ω
m
2. a barra como um resistor ôhmico.
a) 0,5 c) 1,5
b) 1,0 d) 2,0

13. (UEFS) A figura representa a intensidade da corrente elétrica I,


que percorre um fio condutor, em função do tempo t.
Um nanofio utilizando as aproximações propostas possui resistên-
cia elétrica de
Nessas condições,
é correto afirmar a) 170 nΩ
que a corrente b) 0,17 nΩ
média circulando no c) 1,7 nΩ
condutor no intervalo
d) 17 nΩ
de tempo entre t = 0
e t = 6,0 ms, em mA é e) 170 Ω
igual a
a) 6,0 16. (ENEM) As redes de alta tensão para transmissão de energia
elétrica geram campo magnético variável o suficiente para induzir
b) 7,0 corrente elétrica no arame das cercas. Tanto os animais quanto
c) 8,0 os funcionários das propriedades rurais ou das concessionárias de
d) 9,0 energia devem ter muito cuidado ao se aproximarem de uma cerca
quando esta estiver próxima a uma rede de alta tensão, pois, se
e) 10,0
tocarem no arame da cerca, poderão sofrer choque elétrico.
14. (UNICAMP) Tecnologias móveis como celulares e tablets Para minimizar este tipo de problema, deve-se:
têm tempo de autonomia limitado pela carga armazenada em a) Fazer o aterramento dos arames da cerca.
suas baterias. O gráfico abaixo apresenta, de forma simplificada, b) Acrescentar fusível de segurança na cerca.
a corrente de recarga de uma célula de bateria de íon de lítio, em
c) Realizar o aterramento da rede de alta tensão.
função do tempo.
d) Instalar fusível de segurança na rede de alta tensão.
e) Utilizar fios encapados com isolante na rede de alta tensão.

17. (UNIFESP) Um condutor é percorrido por uma corrente elétrica


de intensidade i = 800 mA. Conhecida a carga elétrica elementar, e =
1,6 × 10-19C, o número de elétrons que atravessa uma seção normal
desse condutor, por segundo, é
a) 8,0 × 1019
b) 5,0 × 1020
c) 5,0 × 1018
d) 1,6 × 1020
e) 1,6 × 1022

18. (ENEM) Ao pesquisar um resistor feito de um novo tipo de


material, um cientista observou o comportamento mostrado no
gráfico tensão versus corrente.
Considere uma célula de bateria inicialmente descarregada e que
é carregada seguindo essa curva de corrente. A sua carga no final
da recarga é de
a) 3,3 C.
b) 11.880 C.
c) 1.200 C.
d) 3.300 C.

15. (ENEM—PPL)Recentemente foram obtidos os fios de cobre


mais finos possíveis, contendo apenas um átomo de espessura,
que podem, futuramente, ser utilizados em microprocessadores.
O chamado nanofio, representado na figura, pode ser aproximado Após a análise do gráfico, ele concluiu que a tensão em função da
por um pequeno cilindro de comprimento 0,5 nm (1 mm = 10-9 m). corrente é dada pela equação V = 10i + i².

PRÉ-VESTIBULAR PROENEM.COM.BR 273


FÍSICA II 16 CORRENTE ELÉTRICA E RESISTORES

O gráfico da resistência elétrica (R) do resistor em função da


corrente (i) é
a)

b)
A intensidade de corrente elétrica em ampères, fornecida pelo
gerador ao circuito, é:
a) 16 c) 8
b) 0,8 d) 1,6

20. (UERJ) Em um experimento, quatro condutores, I, II, III e IV,


constituídos por metais diferentes e com mesmo comprimento e
espessura, estão submetidos à tensão elétrica. O gráfico abaixo
apresenta a variação da tensão u em cada resistor em função da
corrente elétrica i.

c)

d)

O condutor que apresenta a maior resistividade elétrica é:


a) I c) III
b) II d) IV

EXERCÍCIOS DE
APROFUNDAMENTO
e)
01. (UEMA) Poraquê é o nome popular do peixe elétrico da
Amazônia. Seu nome vem da língua tupi e significa “o que coloca
pra dormir”. Ele é comparado com uma pilha e pode produzir
descarga de até 1500 volts, a denominação não podia ser mais
apropriada.

19. (ACAFE) Sejam dois resistores ôhmicos Rx e Ry associados


em paralelo e ligados a uma bateria ideal de 12 V. A figura abaixo
mostra as curvas que caracterizam esses resistores.

274 PROENEM.COM.BR PRÉ-VESTIBULAR


16 CORRENTE ELÉTRICA E RESISTORES FÍSICA II

Caso uma pessoa mergulhe a uma distância de 20 cm desse 05. (UNICAMP) A experimentação é parte essencial do método
peixe, considerando uma descarga elétrica de 1500 V, calcule a científico, e muitas vezes podemos fazer medidas de grandezas
intensidade físicas usando instrumentos extremamente simples.
a) do campo elétrico produzido por esse peixe para essa distância. a) Usando o relógio e a régua graduada em centímetros da figura a
b) da corrente elétrica, considerando a resistência elétrica da seguir, determine o módulo da velocidade que a extremidade do
água desse local de 120 Ω. ponteiro dos segundos (o mais fino) possui no seu movimento
circular uniforme.
02. (UFPE) Um fio metálico e cilíndrico é percorrido por uma b) Para o seu funcionamento, o relógio usa uma pilha que, quando
corrente elétrica constante de 0,4 A. Considere o módulo da carga nova, tem a capacidade de fornecer uma carga q = 2,4 Ah = 8,64
do elétron igual a 1,6 x 10-19 C. Expressando a ordem de grandeza × 103 C. Observa-se que o relógio funciona durante 400 dias
do número de elétrons de condução que atravessam uma seção até que a pilha fique completamente descarregada. Qual é a
transversal do fio em 60 segundos na forma 10N, qual o valor de N? corrente elétrica média fornecida pela pilha?

03. (UFPE) O gráfico mostra a variação da corrente elétrica I, em


ampère, num fio em função do tempo t, em segundos. Qual a carga
elétrica, em coulomb, que passa por uma seção transversal do
condutor nos primeiros 4,0 segundos?

GABARITO
EXERCÍCIOS PROPOSTOS
01. B 05. A 09. D 13. C 17. C
04. (UNICAMP) Um desafio tecnológico atual é a produção de
02. D 06. A 10. C 14. B 18. D
baterias biocompatíveis e biodegradáveis que possam ser usadas
para alimentar dispositivos inteligentes com funções médicas. 03. A 07. C 11. B 15. E 19. D

Um parâmetro importante de uma bateria biocompatível é sua 04. A 08. C 12. C 16. A 20. A

capacidade específica (C), definida como a sua carga por unidade EXERCÍCIOS DE APROFUNDAMENTO
massa, geralmente dada em mAh/g. O gráfico abaixo mostra de 01. a) E = 7500 V/m 04. a) Q = 25 mA
maneira simplificada a diferença de potencial de uma bateria à b) i = 12,5 A b) P = 0,4 mW.
base de melanina em função de C. 02. 20 05. a) v = 3,0 mm/s
a) Para uma diferença de potencial de 0,4 V, que corrente média 03. 10 C b) i = 0,25 mA.
a bateria de massa m = 5,0 g fornece, supondo que ela se
descarregue completamente em um tempo t = 4h?
ANOTAÇÕES
b) Suponha que uma bateria preparada com C = 10 mAh/g
esteja fornecendo uma corrente constante total i = 2mA a um
dispositivo. Qual é a potência elétrica fornecida ao dispositivo
nessa situação?

PRÉ-VESTIBULAR PROENEM.COM.BR 275


FÍSICA II 16 CORRENTE ELÉTRICA E RESISTORES

ANOTAÇÕES

276 PROENEM.COM.BR PRÉ-VESTIBULAR


17 LEI DE OHM, POTÊNCIA E FÍSICA II

ENERGIA ELÉTRICA

Retomaremos o conteúdo de resistores apresentado no


capítulo passado, mais especificamente para resistores ôhmicos. PROEXPLICA

PRIMEIRA LEI DE OHM Observe o gráfico abaixo:

O físico George S. Ohm verificou, experimentalmente, no século


XIX, que se mantida a temperatura constante de certos resistores,
a diferença de potencial (U) em seus terminais era diretamente
proporcional à corrente elétrica (i) que o atravessava, a constante
de proporcionalidade entre essas grandezas foi chamada de
resistência elétrica do resistor (R), chegando à relação:
U=R.i
Vale lembrar que a resistência elétrica não depende da ddp,
mas apenas do condutor e de sua temperatura.
No Sistema Internacional (SI), a unidade de resistência elétrica
é o ohm (Ω), sendo:

1V
1Ω =
1A
Pela análise do gráfico podemos afirmar que o resistor não é
Graficamente, o resultado da Lei de Ohm pode ser expresso ôhmico pois a função não é uma reta. Em resistores ôhmicos
como a seguir: o valor da resistência R é a mesma em qualquer ponto
estudado, no gráfico acima vemos que:
U 4 16 25
R= ; R1 = = 1Ω ; R2 = = 2Ω ; R= = 2,5Ω
i 4 8 3
10
Veja que R1< R2 < R3, ou seja, elas não apresentam mesmo
valor, a resistência R varia em cada ponto.
Observe o gráfico abaixo:

Podemos perceber que o gráfico U × i de um resistor ôhmico é


uma reta inclinada que passa pela origem.
Para um resistor não ôhmico a relação U = R ⋅ i também é
válida, porém, como a resistência é variável, o gráfico não se trata
de uma reta.

U U
R=
i

Pela análise do gráfico podemos afirmar que o resistor é


ôhmico pois a função é uma reta crescente que parte do zero.
O I Em qualquer ponto estudado no gráfico acima, vemos que:
U 10 20 30
R= ; R1 = = 2Ω ; R2 = = 2Ω ; R3 = = 2Ω
i 5 10 15
Veja que R1 = R2 = R3, ou seja, independente do ponto (1, 2 ou
3) usado para análise o valor da resistência é a mesma.

PRÉ-VESTIBULAR PROENEM.COM.BR 1
FÍSICA II 17 LEI DE OHM, POTÊNCIA E ENERGIA ELÉTRICA

EXERCÍCIO RESOLVIDO POTÊNCIA ELÉTRICA


Para se medir a quantidade de energia dissipada por um
01. (PUC-RJ) O gráfico abaixo apresenta a medida da resistor, é possível encontrar a expressão da potência através de
variação de potencial em função da corrente que passa em resultados obtidos na eletrostática, além da sua definição:
um circuito elétrico.

i =
Pot = U . i
Podemos dizer que a resistência elétrica deste circuito é de: Como sabemos, pela Lei de Ohm, que U = R · i, podemos achar
outras duas maneiras de representar esta expressão:
a) 2,0 mΩ c) 0,5 Ω e) 0,5 kΩ
b) 0,2 Ω d) 2,0 kΩ U²
Pot =
R ⋅ i² Pot =
R
Resolução: D É bom que você saiba que a primeira equação se aplica a
Fique atento à unidade do eixo horizontal, 1 mA = 1. 10-3A qualquer dispositivo elétrico, enquanto, as duas últimas, como são
combinações da primeira com a U = Ri, portanto, só se aplicam a
Primeira Lei de Ohm
dispositivos ôhmicos.
U= R ⋅ i O mais importante é que o aluno não esqueça de, ao calcular
12 = R ⋅ 6 ⋅ 10−3 a potência de um determinado resistor, utilizar a ddp, a corrente e
12 a resistência elétrica deste resistor e não de uma parte ou de todo
R= circuito.
6 ⋅ 10−3
R= 2 ⋅ 103 Ω A unidade no SI para potência é o watt (W), que é equivalente a
V ⋅ A (volt-ampère).
R= 2kΩ

PROEXPLICA
02. O gráfico abaixo apresenta a medida da variação de A potência dissipada por um aparelho não é uma característica
potencial em função da corrente que passa em três resistores imutável desse aparelho. Ela depende da ddp aplicada sobre
diferentes. ele. Por exemplo, uma lâmpada que possui especificações
120V – 60W, significa que sua potência dissipada vale 60W
apenas quando a ddp em seus terminais for de 120V. Se
alterarmos a ddp, a potência dissipada também se altera,
variando com o quadrado de U. Assim é fácil concluir que
a potência dissipada por um dispositivo é diretamente
proporcional ao quadrado da ddp aplicada em seus terminais.

Para esses resistores podemos afirmar que ENERGIA ELÉTRICA


a) São ôhmicos e R1 > R2 > R3 Alguns problemas citam uma grandeza chamada de energia
b) São ôhmicos e R1 < R2 < R3 elétrica consumida.
c) São ôhmicos e R1 = R2 = R3 No caso do resistor, a energia elétrica consumida é a própria
energia dissipada sob a forma de calor, já que a função do resistor
d) Não são ôhmicos e R1 < R2 < R3 é unicamente dissipar energia elétrica. Mas, no caso de outros
e) Não são ôhmicos e R1 > R2 > R3 aparelhos como lâmpadas fluorescentes e aparelhos domésticos
em geral, a energia elétrica é utilizada para vários fins. Para nós, o
Resolução: A
objetivo sempre será o cálculo desta energia, o que é feito sempre
Para resistores ôhmicos o gráfico U × i é uma reta crescente da mesma maneira, multiplicando-se a potência elétrica dissipada
que começa da origem dos eixos. pelo aparelho e o tempo em que este é utilizado.
Eel = Pot · ∆t
Existem várias unidades de energia, no SI é o joule (J), sendo
muito comum o uso do quilowatt-hora (kWh).
1J=1W⋅s
1 kWh = 3,6 ⋅ 106 J
Logo quanto maior maior será a resistência elétrica (R).
1 kWh = 1 kW ⋅ 1h

2 PROENEM.COM.BR PRÉ-VESTIBULAR
17 LEI DE OHM, POTÊNCIA E ENERGIA ELÉTRICA FÍSICA II

EXERCÍCIO RESOLVIDO Resolução: A


Cálculo da potência elétrica:
03. (ENEM) Podemos estimar o consumo de energia elétrica
U2 122
de uma casa considerando as principais fontes desse P= ⇒ P= ∴P= 72 W
consumo. Pense na situação em que apenas os aparelhos R 2
que constam da tabela a seguir fossem utilizados diariamente Cálculo do calor sensível necessário para o aquecimento da
da mesma forma. água:
Tabela: a tabela fornece a potência e o tempo efetivo cal
de uso diário de cada aparelho doméstico. Q= m ⋅ c ⋅ ∆T ⇒ Q= 100 g ⋅ 1 ⋅ ( 30 − 20 ) °C ∴ Q= 1000 cal
g ⋅ °C
Potência Tempo de uso 4,2 J
Aparelho
(kW) diário (horas) Q ( SI=
) 1000 cal ⋅ ∴ Q ( SI=) 4200 J
1cal
Ar condicionado 1,5 8
Cálculo do tempo necessário para o aquecimento através da
Chuveiro elétrico 3,3 1/3 potência:
Q Q 4200 J
Freezer 0,2 10 P= ⇒ ∆t = ⇒ ∆t = ∴ ∆t = 58,3 s
∆t P 72 W
Geladeira 0,35 10
Lâmpadas 0,1 6
Supondo que o mês tenha 30 dias e que o custo de 1kWh é EXERCÍCIOS
R$ 0,40, o consumo de energia elétrica mensal dessa casa, é
de, aproximadamente PROTREINO
a) R$ 135. c) R$ 190. e) R$ 230.
b) R$ 165. d) R$ 210.
01. O gráfico abaixo apresenta a medida da variação de potencial
em função da corrente que passa em um circuito elétrico.
Resolução: E

Tempo de
Potência Consumo Mensal
Aparelho uso diário
(kW) (kWh)
(horas)

Ar condicionado 1,5 8 30 × 8 x 1,5 = 360

Chuveiro elétrico 3,3 1/3 30 × 3,3 × 1/3 = 33


Freezer 0,2 10 30 × 10 × 0,2 = 60
Geladeira 0,35 10 30 × 10 × 0,35 = 105
Lâmpadas 0,1 6 30 × 6 × 0,1 = 18
Total 576

Custo 576 × 0,4 = R$230,40


Determine a resistência elétrica do circuito.

02. O gráfico abaixo apresenta a medida da variação de potencial


EFEITO JOULE em função da corrente que passa em um circuito elétrico.
É a transformação de energia elétrica em energia térmica.
A maioria dos problemas envolvendo o Efeito Joule relacionará
conceitos de Potência Elétrica e calorimetria.

EXERCÍCIO RESOLVIDO

04. (UDESC) Um recipiente com paredes adiabáticas contém


100 g de água a 20 °C. Um resistor com resistência elétrica
de 2,0 Ω é ligado a uma fonte de tensão de 12V e é imerso
na água.
Desconsidere a capacidade térmica do recipiente, e assinale
a alternativa que corresponde, aproximadamente, ao tempo
necessário para a água atingir 30°C.
a) 58 s d) 29 s
b) 14 s e) 87 s
Determine x e y.
c) 44 s

PRÉ-VESTIBULAR PROENEM.COM.BR 3
FÍSICA II 17 LEI DE OHM, POTÊNCIA E ENERGIA ELÉTRICA

03. O gráfico abaixo apresenta a medida da variação de potencial


em função da corrente que passa em um resistor.
EXERCÍCIOS
PROPOSTOS
01. (UERJ) A produção e a transmissão do impulso nervoso nos
neurônios têm origem no mecanismo da bomba de sódio-potássio.
Esse mecanismo é responsável pelo transporte de íons Na+ para
o meio extracelular e K+ para o interior da célula, gerando o sinal
elétrico. A ilustração abaixo representa esse processo.

Calcule a potência dissipada pelo resistor quando a d.d.p. é de 20


volts.

04. A potência e tempo de uso de alguns aparelhos está listado na


tabela a seguir:

Potência Tempo de uso


Aparelho
(watts) diário (horas)
T.V de Plasma 300 10
O impulso nervoso, ou potencial de ação, é uma consequência da
Geladeira duplex 500 24 alteração brusca e rápida da diferença de potencial transmembrana
dos neurônios. Admita que a diferença de potencial corresponde a
Chuveiro 4000 1 0,07 V e a intensidade da corrente estabelecida, a 7,0 x 10-6 A.
Supondo que o mês tenha 30 dias e que o custo de 1kWh é R$0,50, A ordem de grandeza da resistência elétrica dos neurônios, em
calcule o consumo de energia elétrica mensal, em kWh, e custo ohms, equivale a:
desse consumo. a) 102 c) 104
b) 10 3
d) 105
05. Num balde contendo 2 litros de água, a uma temperatura
inicial de 10°C existe um resistor ôhmico, imerso na água, de 02. (UECE) USB é a sigla para Universal Serial Bus. Esta sigla se
resistência elétrica R = 10 Ω alimentado por um gerador ideal de tornou bastante conhecida com a popularização de telefones
força eletromotriz 200V, conforme o desenho abaixo: celulares. Trata-se de uma tecnologia para conexão de dispositivos
como teclados, impressoras, carregadores de celular, dentre outros.
Pode-se usar a porta USB de um computador também como uma
fonte de energia para ligar componentes eletrônicos como, por
exemplo, um resistor. O padrão USB 2.0 fornece 5 V de tensão e até
500 mA de corrente. O menor valor de uma resistência, em Ohms,
que pode ser ligada de modo seguro em uma porta USB 2.0 é
a) 0,01
b) 2.500
c) 10
d) 100

03. (MACKENZIE) Um chuveiro elétrico apresenta as posições


inverno e verão. Para a posição verão, a água sai com temperaturas
mais amenas e, para a posição inverno, a água sai com temperaturas
mais elevadas.
Em um dia frio, para aumentar a temperatura da água, ao mudar da
posição verão para inverno, o circuito elétrico no qual o chuveiro é
ligado tem
O experimento foi feito ao nível do mar. Considerando que a energia
dissipada pelo resistor foi totalmente absorvida pela água e que o a) sua voltagem aumentada.
resistor permaneceu ligado por 60 segundos. Calcule a temperatura b) sua voltagem diminuída.
final da água, após receber a energia dissipada pelo resistor. c) sua resistência elétrica aumentada.
Dados: calor específico da água: 1 cal/g°C d) sua resistência elétrica diminuída.
calor latente de vaporização da água: 540 cal/g
densidade da água = 1 kg/L e) sua corrente elétrica diminuída.
1 cal = 4 J

4 PROENEM.COM.BR PRÉ-VESTIBULAR
17 LEI DE OHM, POTÊNCIA E ENERGIA ELÉTRICA FÍSICA II

04. (MACKENZIE) Um chuveiro domiciliar, que desenvolve potência 08. (PUCRJ) Um circuito elétrico, formado por um resistor e uma
de 5.200 W, quando instalado em uma diferença de potencial de 220 bateria, dissipa uma potência de 80 mW.
V, tem toda a energia dissipada por seu resistor transferida para a Se duplicarmos os valores da resistência do resistor e da voltagem
porção de água que por ele passa. Em um dia em que a temperatura da bateria, a nova potência dissipada, em mW, será
ambiente vale 20°C e, supondo-se que pelo chuveiro passe 52
gramas de água por segundo, pode-se afirmar corretamente que a a) 0
temperatura com que a água sai do chuveiro vale em °C. b) 40
(Considere o calor específico sensível da água 4,0 J/g°C) c) 80
a) 52 d) 160
b) 50 e) 640
c) 45
09. (CFTMG) No circuito elétrico das residências, há algumas
d) 40
chaves disjuntoras de segurança que se desligam automaticamente
e) 30 em caso de sobrecarga. Na cozinha de uma casa pode ocorrer de
funcionarem, ao mesmo tempo, uma geladeira de 1.000 W, um
05. (ACAFE) O quadro abaixo apresenta algumas informações de forno de 2.100 W, uma lâmpada de 50 W e um liquidificador de 150
uma fatura da conta de energia elétrica de uma residência por um W. Se essa casa possui uma rede elétrica de 110 V, o disjuntor da
período de 30 dias. cozinha deve ser capaz de suportar uma corrente, em amperes, de,
no mínimo,
a) 15.
b) 30.
c) 45.
d) 60.

10. (PUCCAMP) Há alguns anos a iluminação residencial era


predominantemente feita por meio de lâmpadas incandescentes.
Atualmente, dando-se atenção à política de preservação de bens
Sabe-se que uma chaleira elétrica é utilizada todos os dias por
naturais, estas lâmpadas estão sendo trocadas por outros tipos
quinze minutos e que a energia gasta por ela, em 30 dias, representa
de lâmpadas muito mais econômicas, como as fluorescentes
2% do consumo de energia da casa.
compactas e de LED.
A alternativa correta que apresenta o valor da potência dessa
Numa residência usavam-se 10 lâmpadas incandescentes de 100
chaleira, em watt, é:
W que ficavam ligadas em média 5 horas por dia. Estas lâmpadas
a) 1500 foram substituídas por 10 lâmpadas fluorescentes compactas que
b) 1200 consomem 20 W cada uma e também ficam ligadas em média 5
horas por dia.
c) 1000
Adotando o valor R$ 0,40 para o preço do quilowatt-hora, a economia
d) 800
que esta troca proporciona em um mês de trinta dias é de
06. (PUCRJ) Para fazer seu chimarrão, uma pessoa esquenta 1 a) R$ 18,00.
litro de água à temperatura inicial de 25°C utilizando um aquecedor b) R$ 48,00.
elétrico. A água alcança a temperatura ideal de 85°C após 6 c) R$ 60,00.
minutos.
d) R$ 120,00.
Qual é a potência desse aquecedor, em Watts? Despreze perdas de
e) R$ 248,00.
calor ao ambiente.
Dados: Densidade da água: 1,0 g/mL
11. (PUCCAMP) O mostrador digital de um amperímetro fornece
Calor específico da água: 1,0 cal/g°C ≈ 4,2 J/g°C indicação de 0,40 A em um circuito elétrico simples contendo uma
a) 167 fonte de força eletromotriz ideal e um resistor ôhmico de resistência
elétrica 10 Ω.
b) 252
Se for colocado no circuito um outro resistor, de mesmas caracte-
c) 700
rísticas, em série com o primeiro, a nova potência elétrica dissipada
d) 992 no circuito será, em watts,
e) 4.200 a) 0,64.
b) 0,32.
07. (UECE) A Agência Nacional de Energia Elétrica anunciou
bandeira vermelha 2 para as contas de luz de junho deste ano, o que c) 0,50.
significa um adicional de R$ 5,00 para cada 100 kWh consumido. d) 0,20.
Considerando que uma certa indústria utilizou um resistor para e) 0,80.
aquecimento, cuja potência é 50 kW, por 4 horas durante esse mês,
o adicional na conta associado a este consumo foi, em R$,
a) 10.
b) 200.
c) 50.
d) 4.

PRÉ-VESTIBULAR PROENEM.COM.BR 5
FÍSICA II 17 LEI DE OHM, POTÊNCIA E ENERGIA ELÉTRICA

12. (CPS) Para que a corrente não seja letal a uma pessoa que toca a cerca
eletrificada, o gerador de tensão deve possuir uma resistência
Morador Tempo diário em minutos
interna que, em relação à do corpo humano, é
Mãe 20 a) praticamente nula.
Pai 15 b) aproximadamente igual.
Irmã 20 c) milhares de vezes maior.

Irmão 5 d) da ordem de 10 vezes maior.


e) da ordem de 10 vezes menor.
Ele próprio 30
Um estudante avaliou o tempo diário do uso do chuveiro em sua 15. (ENEM PPL) As especificações de um chuveiro elétrico são:
casa no decorrer de trinta dias consecutivos, o que permitiu a potência de 4.000 W, consumo máximo mensal de 21,6 kWh
construção do quadro. e vazão máxima de 3 L/min. Em um mês, durante os banhos,
Sabendo que o chuveiro de sua casa tem potência de 2800 W, o esse chuveiro foi usado com vazão máxima, consumindo o valor
estudante calculou que, no período avaliado, o consumo de energia máximo de energia especificado. O calor específico da água é de
em sua casa, devido ao uso do chuveiro, foi, aproximadamente, de 4.200 J/(kg°C) e sua densidade é igual a 1 kg/L.
a) 90 kWh. A variação da temperatura da água usada nesses banhos foi mais
próxima de
b) 150 kWh.
a) 16°C. c) 37°C. e) 60°C.
c) 125 kWh.
b) 19°C. d) 57°C.
d) 140 kWh.
e) 155 kWh. 16. (PUCRJ) Quando duas resistências R idênticas são colocadas
em paralelo e ligadas a uma bateria V, a corrente que flui pelo
13. (ENEM) Dispositivos eletrônicos que utilizam materiais de baixo circuito é I0.
custo, como polímeros semicondutores, têm sido desenvolvidos Se o valor das resistências dobrar, qual será a corrente no circuito?
para monitorar a concentração de amônia (gás tóxico e incolor)
em granjas avícolas. A polianilina é um polímero semicondutor a) I0/4 c) I0 e) 4I0
que tem o valor de sua resistência elétrica nominal quadruplicado b) I0/2 d) 2I0
quando exposta a altas concentrações de amônia. Na ausência de
amônia, a polianilina se comporta como um resistor ôhmico e a sua 17. (PUCCAMP) A distribuição de energia elétrica para residências
resposta elétrica é mostrada no gráfico. no Brasil é feita basicamente por redes que utilizam as tensões de
127 V e de 220 V, de modo que os aparelhos eletrodomésticos são
projetados para funcionarem sob essas tensões. A tabela mostra
a tensão e a intensidade da corrente elétrica que percorre alguns
aparelhos elétricos resistivos quando em suas condições normais
de funcionamento.

Aparelho Tensão (V) Corrente (A)


Chuveiro 220 20
Lâmpada incandescente 127 1,5
Ferro de passar 127 8

Sendo RC, RL e RF, respectivamente, as resistências elétricas


do chuveiro, da lâmpada e do ferro de passar, quando em suas
condições normais de funcionamento, é correto afirmar que
a) RF > RL > RC d) RC > RF > RL
b) RL > RC > RF e) RL > RF > RC
O valor da resistência elétrica da polianilina na presença de altas c) RC > RL > RF
concentrações de amônia, em ohm, é igual a
18. (PUCCAMP) Um chefe de cozinha precisa transformar 10
a) 0,5 x 100.
g de gelo a 0°C em água a 40°C em 10 minutos. Para isto utiliza
b) 0,2 x 10 .
0
uma resistência elétrica percorrida por uma corrente elétrica que
c) 2,5 x 105. fornecerá calor para o gelo. Supondo-se que todo calor fornecido
d) 5,0 x 105. pela resistência seja absorvido pelo gelo e desprezando-se perdas
de calor para o meio ambiente e para o frasco que contém o gelo,
e) 2,0 x 106. a potência desta resistência deve ser, em watts, no mínimo, igual a:
Dados da água:
14. (ENEM) Em algumas residências, cercas eletrificadas são
Calor específico no estado sólido: 0,50 cal/g°C
utilizadas com o objetivo de afastar possíveis invasores. Uma cerca
Calor específico no estado líquido: 1,0 cal/g°C
eletrificada funciona com uma diferença de potencial elétrico de
Calor latente de fusão do gelo: 80 cal/g
aproximadamente 10.000 V. Para que não seja letal, a corrente que
Adote 1 cal = 4 J
pode ser transmitida através de uma pessoa não deve ser maior do
que 0,01 A. Já a resistência elétrica corporal entre as mãos e os pés a) 4. c) 10. e) 120.
de uma pessoa é da ordem de 1.000 Ω.
b) 8. d) 80.

6 PROENEM.COM.BR PRÉ-VESTIBULAR
17 LEI DE OHM, POTÊNCIA E ENERGIA ELÉTRICA FÍSICA II

19. (EEAR) Sabendo que a diferença de potencial entre uma nuvem comprimento médio 22,0 m
e a Terra, para que aconteça a descarga elétrica de um raio, é em
torno de 3 · 108 V e que a corrente elétrica produzida neste caso é largura 3,00 m
aproximadamente de 1 · 105 A, qual a resistência média do ar, em altura 3,60 m
ohms (Ω)?
tipo I 38.000 kg
a) 1.000 Por vagão massa
tipo II 35.000 kg
b) 2.000
quantidade 4
c) 3.000 motores
potência por motor 140 kW
d) 4.000
capacidade máxima 8 passageiros/m²
20. (EEAR) Considere um cubo de gelo de massa 1 kg que se
encontra à temperatura de -2°C. Colocado ao sol, recebe 14 J de 02. (UERJ) O vagão de uma composição do metrô possui 12
calor a cada segundo. Dados o calor específico do gelo igual a 0,5 lâmpadas eletrônicas idênticas, de 25 W cada uma. Essas
cal/g·°C e 1 cal igual a 4,2 J. lâmpadas ficam acesas durante 15 horas em cada dia. Admita que
Quantos minutos, aproximadamente, o gelo deverá ficar ao sol para a tarifa praticada pela concessionária de energia elétrica seja igual
começar a se fundir? a R$ 0,80 por kWh consumido.
a) 0,005 Calcule, em reais, o valor do consumo de energia elétrica das
lâmpadas do vagão em um período de 30 dias.
b) 0,5
c) 5
03. (UERJ) Considere que a energia elétrica produzida por todos
d) 50 os motores dos vagões seja integralmente transferida para
movimentar a composição do metrô.
EXERCÍCIOS DE Calcule, em quilojoules, o trabalho realizado pela força elétrica no
APROFUNDAMENTO intervalo de tempo de 100 segundos.

04. (UNESP) Uma bateria de smartphone de 4.000 mA · h e 5,0 V


pode fornecer uma corrente elétrica média de 4.000 mA durante
01. (UFPR) Um aquecedor elétrico de potência constante P = 2100 W
uma hora até que se descarregue.
foi utilizado para transferir energia para uma massa de água na
forma de gelo de valor m = 200 g, cuja temperatura inicial era a) Calcule a quantidade de carga elétrica, em coulombs, que essa
T0 = 0°C. Essa massa de gelo está colocada num recipiente de bateria pode fornecer ao circuito.
capacidade térmica desprezível e, por hipótese, toda a energia b) Considerando que, em funcionamento contínuo, a bateria
fornecida pelo aquecedor foi transferida sem perdas para o gelo. Os desse smartphone se descarregue em 8,0 horas, calcule a
calores específicos de gelo e água líquida são cgelo = 0,5 cal/g°C e potência média do aparelho, em watts.
cágua = 1,0 cal/g°C, e podem ser supostos constantes na faixa de
temperatura considerada. Além disso, os calores de fusão do gelo 05. (UNIFESP) Uma espira metálica circular homogênea e de
e ebulição da água são Lfusão = 80 cal/g e Lebulição = 540 cal/g. espessura constante é ligada com fios ideais, pelos pontos A e B,
Sabe-se que o aquecedor forneceu uma energia total de valor Q = a um gerador ideal que mantém uma ddp constante de 12 V entre
84 kJ. Se necessário, use a conversão 1 cal = 4,2 J. O sistema está esses pontos. Nessas condições, o trecho AB da espira é percorrido
ao nível do mar, sujeito à pressão atmosférica usual de 1 atm, e por uma corrente elétrica de intensidade iAB = 6 A e o trecho ACB é
onde a água evapora a 100 °C e solidifica a 0°C. percorrido por uma corrente elétrica de intensidade iACB, conforme
a) Determine a temperatura final Tf da massa de água após a a figura.
transferência de energia.
b) Determine o intervalo de tempo ∆t em que o aquecedor ficou
ligado.

TEXTO PARA AS PRÓXIMAS 2 QUESTÕES:


As quesões a seguir abordam situações relacionadas ao
ambiente do metrô, referindo-se a uma mesma composição,
formada por oito vagões de dois tipos e movida por tração elétrica.
Para seus cálculos, sempre que necessário, utilize os dados e as
fórmulas abaixo.

Características da composição
velocidade máxima 100 km/h
aceleração constante 1,10 m/s²
desaceleração constante 1,25 m/s²
Gerais
quantidade de tipo I 2 Calcule:
vagões tipo II 6 a) as resistências elétricas RAB e RACB, em ohms, dos trechos AB e
ACB da espira.
massa média por passageiro 60 kg
b) a potência elétrica, em W, dissipada pela espira.

PRÉ-VESTIBULAR PROENEM.COM.BR 7
FÍSICA II 17 LEI DE OHM, POTÊNCIA E ENERGIA ELÉTRICA

GABARITO

EXERCÍCIOS PROPOSTOS
01. C 05. C 09. B 13. E 17. E
02. C 06. C 10. B 14. C 18. B
03. D 07. A 11. E 15. B 19. C
04. C 08. D 12. C 16. B 20. C

EXERCÍCIOS DE APROFUNDAMENTO
01. a) 20°C 02. R$ 108,00 04. a) 14400 C 05. a) 2Ω e 10 Ω
b) 40 s 03. 448000 kJ b) 2,5 W b) 86,4 W

ANOTAÇÕES

8 PROENEM.COM.BR PRÉ-VESTIBULAR
18
FÍSICA II
ASSOCIAÇÃO DE RESISTORES

INTRODUÇÃO DIVISÃO DE DDP


Nos circuitos elétricos, de forma geral, o aluno não encontrará Na associação em série a ddp total será dividida entre os
apenas um resistor em toda sua extensão, mas sim vários resistores. resistores, portanto:
Nesse caso, dizemos que os resistores estão associados. i1 = i2 = i3 = ...=in
Existem dois tipos de associação de resistores: paralelo
e série. Quando em um circuito encontramos os dois tipos de U
com U = R ⋅ i, i = , substituindo...
associação, dizemos, então, que há uma associação mista. R
Quando existe uma associação de resistores, o processo U1 U2
normal de resolução é calcular o resistor equivalente (Req), que é =
R1 R2
um resistor que sozinho funcionaria como todos os outros juntos,
isto é, quando sob a mesma ddp da associação geraria a mesma
DDP e R são diretamento proporcionais.
corrente.
Dois ou mais resistores estão ligados em série quando o
terminal de saída de um está ligado ao terminal de entrada do EXERCÍCIO RESOLVIDO
resistor seguinte e assim por diante, e dessa forma a corrente
elétrica que atravessa os resistores é a mesma.
01. Um circuito elétrico é composto de quatro resistores
associados de acordo com a imagem. Determine a tensão
que cada um dos resistores consome.

Consideremos, como exemplo, a figura acima, em que três


resistores R1, R2 e R3 estão em série. Eles são percorridos pela
mesma corrente elétrica ( i ), porém ddps diferentes U1, U2 e U3,
sendo a ddp total U. Para determinar o resistor equivalente, vamos
imaginar um resistor que com a mesma ddp U gera a mesma a) U1 4,8V;
= = U2 2,4=
V; U3 9,6
= V;U4 7,2V
corrente i.
b) U1 2,4
= = V; U2 4,8V;
= U3 7,2V;U
= 4 9,6 V
c) U1 9,6
= = V; U2 7,2V;
= U3 4,8V;U
= 4 2,4 V
d) U1 4,2V;
= = U2 2,6=
V; U3 4,8V;U
= 4 7,6 V
e) U1 9,6
= = V; U2 7,2V;
= U3 4,4
= V;U4 2,8V

Resolução:
• Resistência Equivalente
Como: U = U1 + U2 + U3 (1) e U = Req ⋅ i (2) Req=R1+R2+R3+R4
Podemos substituir (2) em (1) e teremos: Req= 20 + 10 + 40 + 30
Req=100 Ω
Re q ⋅ =
i R1 ⋅ i + R2 ⋅ i + R3 ⋅ i
• Intensidade da corrente elétrica
Como as correntes são iguais: U=R .ieq
24 = 100 i
Req = R1 + R2 + R3
i = 0,24A
Relação geral para resistência equivalente de resistores em
série: • Tensões
Req = R1 + R2 + R3 + ...+ Rn U1 = R1 . i U3 = R3 . i
U = 20 . 0,24
1
U3 = 40 . 0,24
Na associação em série quando um dos aparelhos for U1= 4,8 V U3 = 9,6 V
desligado ou simplesmente “queimar”, todos os outros aparelhos
param de funcionar. U2 = R2 . i U4 = R4 . i
U2 = 10 . 0,24 U4 = 30 . 0,24
U2 = 2,4 V U4 = 7,2 V

PRÉ-VESTIBULAR PROENEM.COM.BR 1
FÍSICA II 18 ASSOCIAÇÃO DE RESISTORES

ASSOCIAÇÃO EM PARALELO EXERCÍCIO RESOLVIDO


Na associação em paralelo, os terminais de entrada dos
resistores são ligados entre si e os terminais de saída também.
02. Dado o circuito abaixo, determine a corrente total do
Dessa maneira os elétrons passam a ter, quando ocorre esse tipo
circuito.
de associação, vários caminhos diferentes e a corrente elétrica vai,
então, se dividir.

a) 3,6 A d) 5,2 A
b) 4,2 A e) 6,0 A
c) 4,8 A
Resolução: B
itotal 24 V 24 V 24 V
Utilizando a figura anterior como exemplo, percebemos que a
corrente elétrica se dividiu em três partes: i1, i2 e i3, sendo a corrente i1 i2 i3
total igual a soma dessas três.
U = 24 V R1=20  R2=10  R3=40 
i = i1 + i 2 + i 3
Pela primeira Lei de Ohm, podemos tirar a relação: i1 i2 i3
0V 0V 0V
U U U U U
= 1+ 2 + 3 ⇒ i= Intensidade das correntes
Re q R1 R2 R3 R
U = R1 . i1 U = R2 . i2 U = R3 . i3
Substituindo na equação da soma das correntes:
24 = 20 ⋅ i1 24 = 10 ⋅ i2 24 = 40 ⋅ i3
Como as ddps são iguais: i1= 1,2 A i2= 2,4 A i3= 0,6 A
1 1 1 1 1 itota l= i1+ i2+ i3
= + + + ... +
Req R1 R2 R3 Rn itotal = 1,2 + 2,4 + 0,6
itotal = 4,2 A
Relação geral para resitência equivalente de resistores em
paralelo:
1 1 1 1 1
= + + + ... +
Req R1 R2 R3 Rn ASSOCIAÇÃO MISTA DE RESISTORES
Algumas vezes o aluno poderá encontrar um circuito em
Em uma associação em paralelo para se obter o resistor que ocorram associações em paralelo e em série. Nesse caso,
equivalente, deve-se somar os inversos dos resistores e fazer o que engloba a maioria absoluta dos problemas, teremos uma
inverso do resultado obtido. associação mista. O aluno, então, deverá aplicar as relações citadas
anteriormente, verificando quando se trata de cada associação.
PROEXPLICA
EXERCÍCIO RESOLVIDO
I) Quando em uma associação em paralelo todos os (n)
resistores forem iguais, a R, por exemplo, basta dividir este 03. (IFSUL) A imagem abaixo ilustra a associação de resistores
valor, R, pelo número de resistores(n): em um circuito misto.
R
Re q =
n

II) Quando a associação for de apenas dois resistores, a


resistência equivalente será dada pelo produto sobre a soma:
R1 × R2
Re q =
R1 + R2

DIVISÃO DA CORRENTE Considerando que todos os resistores possuem a mesma


resistência elétrica R, a resistência equivalente da associação
U1 = U2 = U3 = ...=Un é igual a
a) R c) 3R/5 e) 5R/2
Como U = R ⋅ i, substituindo:
b) 4R d) 4R/3
↑ R1 . i1 ↓= R2 . i2
Resolução: C
R e i são inversamente proporcionais.

2 PROENEM.COM.BR PRÉ-VESTIBULAR
18 ASSOCIAÇÃO DE RESISTORES FÍSICA II

Quando o circuito está em série utilizamos a fórmula:


Req = R1 + R2 + R3 +  + Rn

E quando o circuito está em paralelo usamos a fórmula:


1 1 1 1 1
= + + + +
Req R1 R2 R3 Rn

Quando o circuito está em paralelo e todas as resistências


R
são iguais, usamos essa fórmula: Req = , onde n é o número
de resistores. n

Figura 2:
R R
Req = ⇒ Req =
n 2
Figura 3:
Req= R1 + R2
R
Req= R +
2
3
=Req 1,5
= R ou Req R
2
Figura 4:
1 1 1 1 1 1 3R
= + ⇒ = + ⇒ Req =
Req R 1,5R Req R 3 2 ⋅ R 5

EXERCÍCIOS
PROTREINO
01. O circuito abaixo possui dois resistores ôhmicos R1 e R2 de
resistências 4Ω e 16Ω, respectivamente, e um gerador ideal de
U = 80 V e fios ideais.

Calcule a resistência equivalente, a intensidade da corrente elétrica


e a voltagem que está submetido a cada resistor.

PRÉ-VESTIBULAR PROENEM.COM.BR 3
FÍSICA II 18 ASSOCIAÇÃO DE RESISTORES

02. O circuito abaixo possui 3 resistores ôhmicos R1, e R2 e R3, de


resistências 3Ω, 6 Ω e 2 Ω respectivamente. Os fios e o gerador de
força eletromotriz 180 V são ideais.

Aparelho Representação Dado

Determine, nas unidades do S.I.: R1 8,0 Ω


a) a voltagem e a intensidade da corrente elétrica que está R2 4,0 Ω
submetido cada resistor;
b) a intensidade da corrente total do circuito (i total) e i’; R3 2,0 Ω

c) a resistência equivalente. Resistor R4 2,0 Ω


d) a potência dissipada pelo resistor R3 R5 12,0 Ω

03. O esquema abaixo representa um circuito composto por R6 6,0 Ω


resistores ôhmicos, um gerador e fios ideais. R7 6,0 Ω
Gerador U 114 V

Considerando o gerador e os fios ideais, determine a resistência


equivalente, a intensidade da corrente total e a potência dissipada
pelo circuito.

05. O circuito abaixo é composto por resistores ôhmicos, fios ideais


e um gerador ideal.

Aparelho Representação Dado

R1 5,0 Ω
Há dois tipos de resistores R e R’, de 4Ω e 8Ω, respectivamente.
R2 4,0 Ω Considerando U = 48V, determine a resistência equivalente, a
intensidade da corrente elétrica e a potência dissipada pelo circuito.
Resistor R3 12,0 Ω
R4 2,0 Ω
R5 7,5 Ω EXERCÍCIOS
Gerador U 320 V PROPOSTOS
A partir dos dados apresentados, determine:
a) A resistência equivalente do circuito; 01. (FUVEST) Um fabricante projetou resistores para utilizar
b) As intensidades das correntes indicadas no esquema; em uma lâmpada de resistência L. Cada um deles deveria ter
c) A d.d.p. entre os pontos A e B; resistência R. Após a fabricação, ele notou que alguns deles foram
projetados erroneamente, de forma que cada um deles possui uma
d) A potência dissipada pelo circuito.
resistência RD = R/2. Tendo em vista que a lâmpada queimará se
for percorrida por uma corrente elétrica superior a V/(R + L), em
04. Em um projeto de maquete as lâmpadas são representadas por qual(is) dos circuitos a lâmpada queimará?
resistores ôhmicos, observe o esquema e a tabela informativa a
seguir:

4 PROENEM.COM.BR PRÉ-VESTIBULAR
18 ASSOCIAÇÃO DE RESISTORES FÍSICA II

04. (FUVEST) Atualmente são usados LEDs (Light Emitting Diode)


na iluminação doméstica. LEDs são dispositivos semicondutores
que conduzem a corrente elétrica apenas em um sentido. Na figura,
há um circuito de alimentação de um LED (L) de 8 W, que opera
com 4 V, sendo alimentado por uma fonte (F) de 6 V.

a) 1, apenas.
b) 2, apenas.
c) 1 e 3, apenas.
d) 2 e 3, apenas.
e) 1, 2 e 3.

02. (ENEM) Uma casa tem um cabo elétrico mal dimensionado, de


resistência igual a 10 Ω, que a conecta à rede elétrica de 120 V.
Nessa casa, cinco lâmpadas, de resistência igual a 200 Ω, estão O valor da resistência do resistor (R), em Ω, necessário para que o
conectadas ao mesmo circuito que uma televisão de resistência LED opere com seus valores nominais é, aproximadamente,
igual a 50 Ω, conforme ilustrado no esquema. A televisão funciona a) 1,0. c) 3,0. e) 5,0.
apenas com tensão entre 90 V e 130 V. b) 2,0. d) 4,0.

05. (PUCRJ) Um circuito tem 3 resistores idênticos, dois deles


colocados em paralelo entre si, e ligados em série com o terceiro
resistor e com uma fonte de 12 V. A corrente que passa pela fonte é
de 5,0 mA. Qual é a resistência de cada resistor, em kΩ?
a) 0,60 c) 1,2 e) 2,4
b) 0,80 d) 1,6

06. (UNICAMP) Nos últimos anos, materiais exóticos conhecidos


como isolantes topológicos se tornaram objeto de intensa
O número máximo de lâmpadas que podem ser ligadas sem que a investigação científica em todo o mundo. De forma simplificada,
televisão pare de funcionar é: esses materiais se caracterizam por serem isolantes elétricos no
a) 1. seu interior, mas condutores na sua superfície. Desta forma, se um
b) 2. isolante topológico for submetido a uma diferença de potencial
U, teremos uma resistência efetiva na superfície diferente da
c) 3. resistência do seu volume, como mostra o circuito equivalente da
d) 4. figura abaixo.
e) 5.

03. (UERJ) Resistores ôhmicos idênticos foram associados em


quatro circuitos distintos e submetidos à mesma tensão UA,B.
Observe os esquemas:

is
Nessas condições, a corrente elétrica de menor intensidade se Nessa situação, a razão F = entre a corrente iS que atravessa
estabelece no seguinte circuito: iv
a porção condutora na superfície e a corrente iv que atravessa a
a) I
b) II porção isolante no interior do material vale
c) III a) 0,002. c) 100,2.
d) IV b) 0,2. d) 500.

PRÉ-VESTIBULAR PROENEM.COM.BR 5
FÍSICA II 18 ASSOCIAÇÃO DE RESISTORES

07. (FMP) Suponha uma bateria ideal que é capaz de manter 08. (ENEM) Muitos smartphones e tablets não precisam mais
uma diferença de potencial constante entre seus terminais de teclas, uma vez que todos os comandos podem ser dados
independentemente das resistências conectadas a ela, e considere ao se pressionar a própria tela. Inicialmente essa tecnologia foi
três resistores idênticos, cada um com uma resistência R. Podem proporcionada por meio das telas resistivas, formadas basicamente
ser feitas as diferentes montagens mostradas na figura abaixo, por duas camadas de material condutor transparente que não se
usando um, dois ou três desses resistores. encostam até que alguém as pressione, modificando a resistência
total do circuito de acordo com o ponto onde ocorre o toque. A
imagem é uma simplificação do circuito formado pelas placas, em
que A e B representam pontos onde o circuito pode ser fechado por
meio do toque.

Qual é a resistência equivalente no circuito provocada por um toque


que fecha o circuito no ponto A?
a) 1,3 kΩ d) 6,7 kΩ
b) 4,0 kΩ e) 12,0 kΩ
c) 6,0 kΩ

09. (PUCRJ) Quatro resistores idênticos, de resistência R, estão


ligados a uma bateria de 12 V. Pela bateria, flui uma corrente I = 12 mA.
A resistência R de cada resistor, em kΩ, é

Uma dessas montagens será posta no lugar em que se encontra a) 4 d) 5/3


o símbolo “?” da figura abaixo para aquecer a água do recipiente. b) 1 e) 1/4
c) 3/4

10. (FMP) Numa instalação elétrica de um escritório, são colocadas


3 lâmpadas idênticas em paralelo conectadas a uma fonte de
tensão.

Qual das montagens produzirá o aquecimento mais rápido da


água?
a) V
b) IV
c) I
d) II
e) III

6 PROENEM.COM.BR PRÉ-VESTIBULAR
18 ASSOCIAÇÃO DE RESISTORES FÍSICA II

Se uma das lâmpadas queimar, o que acontecerá com a corrente Dessa forma, considerando desprezíveis as resistências dos fios de
nas outras lâmpadas? ligação entre eles, a máxima diferença de potencial, em volts, que
a) Aumentará por um fator 1,5. pode ser estabelecida entre os pontos A e B do circuito, sem que
haja riscos, é igual a
b) Aumentará por um fator 2.
a) 30.
c) Diminuirá por um fator 1,5.
b) 50.
d) Diminuirá por um fator 2.
c) 20.
e) Permanecerá a mesma.
d) 40.
11. (MACKENZIE) e) 60.

14. (PUCRJ) Um sistema de quatro resistências idênticas R = 1,00


kΩ e uma bateria V = 12,0 V estão acoplados, como na figura, por
fios perfeitos.

A figura acima representa um circuito elétrico constituído de uma


fonte de tensão contínua de 100 V alimentando quatro resistores.
Pode-se afirmar que a tensão elétrica nas extremidades do resistor
de resistência elétrica 30 Ω vale
a) 20 V c) 40 V e) 100 V
b) 30 V d) 50 V
Calcule, em W, a potência elétrica total consumida no sistema.
12. (IMED) O circuito elétrico representado abaixo é composto por a) 0,000
fios e bateria ideais:
b) 0,120
c) 0,144
d) 12,0
e) 144

15. (ENEM) Três lâmpadas idênticas foram ligadas no circuito


esquematizado. A bateria apresenta resistência interna desprezível,
e os fios possuem resistência nula. Um técnico fez uma análise do
circuito para prever a corrente elétrica nos pontos: A, B, C, D e E; e
rotulou essas correntes de IA, IB, IC, ID e IE, respectivamente.

Com base nas informações, qual o valor da resistência R indicada?


a) 5 Ω. c) 7 Ω. e) 9 Ω.
b) 6 Ω. d) 8 Ω.

13. (UNESP) Em um trecho de uma instalação elétrica, três


resistores Ôhmicos idênticos e de resistência 80 Ω cada um
são ligados como representado na figura. Por uma questão de
segurança, a maior potência que cada um deles pode dissipar,
separadamente, é de 20 W.

O técnico concluiu que as correntes que apresentam o mesmo


valor são
a) IA = IE e IC = ID.
b) IA = IB = IE e IC = ID.
c) IA = IB, apenas.
d) IA = IB = IE, apenas.
e) IC = IB, apenas.

PRÉ-VESTIBULAR PROENEM.COM.BR 7
FÍSICA II 18 ASSOCIAÇÃO DE RESISTORES

16. (ENEM) Por apresentar significativa resistividade elétrica, o 18. (PUCRJ) No circuito abaixo, a corrente que passa pelo trecho
grafite pode ser utilizado para simular resistores elétricos em AB vale 1,0 A.
circuitos desenhados no papel, com o uso de lápis e lapiseiras.
Dependendo da espessura e do comprimento das linhas
desenhadas, é possível determinar a resistência elétrica de cada
traçado produzido. No esquema foram utilizados três tipos de lápis
diferentes (2H, HB e 6B) para efetuar três traçados distintos.

O valor da resistência R é, em ohms:


a) 30 c) 20 e) 50
Munida dessas informações, um estudante pegou uma folha de
papel e fez o desenho de um sorvete de casquinha utilizando-se b) 10 d) 12
desses traçados. Os valores encontrados nesse experimento,
para as resistências elétricas (R), medidas com o auxílio de 19. (ENEM PPL) Os manuais dos fornos micro-ondas
um ohmímetro ligado nas extremidades das resistências, são desaconselham, sob pena de perda da garantia, que eles sejam
mostrados na figura. Verificou-se que os resistores obedeciam a ligados em paralelo juntamente a outros aparelhos eletrodomésticos
Lei de Ohm. por meio de tomadas múltiplas, popularmente conhecidas como
“benjamins” ou “tês”, devido ao alto risco de incêndio e derretimento
dessas tomadas, bem como daquelas dos próprios aparelhos.
Os riscos citados são decorrentes da
a) resistividade da conexão, que diminui devido à variação de
temperatura do circuito.
b) corrente elétrica superior ao máximo que a tomada múltipla
pode suportar.
c) resistência elétrica elevada na conexão simultânea de aparelhos
eletrodomésticos.
d) tensão insuficiente para manter todos os aparelhos eletrodo-
mésticos em funcionamento.
e) intensidade do campo elétrico elevada, que causa o rompimento
da rigidez dielétrica da tomada múltipla.

Na sequência, conectou o ohmímetro nos terminais A e B do 20. (PUCRJ)


desenho e, em seguida, conectou-o nos terminais B e C, anotando
as leituras RAB e RBC, respectivamente. Ao estabelecer a razão
RAB
qual resultado o estudante obteve?
RBC
a) 1 14
d)
4 81
b)
7 4
e)
81
c) 10
27
No circuito mostrado na figura, a diferença de potencial entre os
pontos B e A vale, em volts:
17. (UERN) A resistência R na associação de resistores a seguir é
a) 3,0 c) 2,0 e) 0,75
igual a
b) 1,0 d) 4,5

EXERCÍCIOS DE
APROFUNDAMENTO
01. (UERJ) Observe na tabela as características de fábrica das
lâmpadas L1 e L2:

LÂMPADA TENSÃO (V) POTÊNCIA (W)


a) 10 Ω. L1 120 120
b) 20 Ω. L2 240 144
c) 30 Ω.
d) 40 Ω.

8 PROENEM.COM.BR PRÉ-VESTIBULAR
18 ASSOCIAÇÃO DE RESISTORES FÍSICA II

L1 e L2 foram associadas em paralelo a uma tensão U = 120 V,


Dispositivo Potência (W) DDP (V)
conforme representado na figura:
Ar-condicionado 1.100 110
Televisor 44 110
Lâmpada 22 110
a) Considerando o custo de 1 kWh igual a R$ 0,30 e os dados
da tabela, calcule, em reais, o custo total da energia elétrica
consumida pelos quatro dispositivos em um período de 5,0
horas.
b) Considerando que os dispositivos estejam associados em
paralelo e funcionando conforme as especificações da tabela,
calcule a intensidade da corrente elétrica total para esse
conjunto, em ampères.

04. (UERJ) Durante uma aula de eletricidade, um professor


analisou um circuito elétrico composto por uma bateria, de tensão
Nessas condições, calcule a resistência equivalente do circuito, em
constante U igual a 12 V, e quatro resistores idênticos R de 10 Ω,
ohms, e o valor da potência dissipada pela lâmpada L2, em watts.
conforme indicado no esquema.
02. (FUVEST) Considere o circuito mostrado na figura, onde todos
os resistores têm resistência R = 200 Ω. A diferença de potencial
VAB, entre os pontos A e B, é 120 V.

Determine
Determine, em ampères, a corrente elétrica que se estabelece na
a) a resistência Req equivalente deste circuito;
bateria.
b) a corrente total i no circuito e a corrente i4 no resistor R4;
c) a potência total P dissipada no circuito e a potência P3 dissipada 05. (UERJ) No esquema abaixo, está representada a instalação de
no resistor R3. uma torneira elétrica.

03. (UNESP) Em uma sala estão ligados um aparelho de ar-


condicionado, um televisor e duas lâmpadas idênticas, como
mostra a figura. A tabela informa a potência e a diferença de
potencial de funcionamento desses dispositivos.

De acordo com as informações do fabricante, a resistência interna


r da torneira corresponde a 200 Ω. A corrente que deve percorrer o
circuito da torneira é de 127 mA.
Determine o valor da resistência R que deve ser ligada em série
à torneira para que esta possa funcionar de acordo com a
especificação do fabricante, quando ligada a uma tomada de 127 V.
Calcule, em watts, a potência dissipada por essa torneira.

(http://t3.static.com)

PRÉ-VESTIBULAR PROENEM.COM.BR 9
FÍSICA II 18 ASSOCIAÇÃO DE RESISTORES

GABARITO

EXERCÍCIOS PROPOSTOS
01. D 05. D 09. A 13. E 17. C
02. B 06. D 10. E 14. C 18. A
03. C 07. A 11. B 15. A 19. B
04. A 08. C 12. C 16. B 20. C

EXERCÍCIOS DE APROFUNDAMENTO
01. 36 W c) Ptotal =144W e P3 = 72 W 04. 3 A
02. a) Req = 100 Ω 03. a) C ≅ R$ 1,78 05. P ≅ 3,23 W e R = 800 ohms
b) i = 1,2A e i4 = 0,3A b) i = 10,8 A

ANOTAÇÕES

10 PROENEM.COM.BR PRÉ-VESTIBULAR
19
FÍSICA II
ELEMENTOS DE UM CIRCUITO

AMPERÍMETRO CHAVES
É o elemento responsável pela mediação da corrente elétrica. Possuem a função de controlar a passagem de corrente elétrica.
Para que um amperímetro funcione perfeitamente deve-se As chaves podem se encontrar em duas posições: aberta ou fechada.
respeitar duas normas:
• o amperímetro deve ser colocado em série no circuito;
• a resistência interna do amperímetro ideal deve tender a
zero, pois assim o aparelho mede a corrente sem alterá-la.
Símbolo:

EXERCÍCIO RESOLVIDO
VOLTÍMETRO 01. No circuito abaixo é mostrado a colocação de chave
São os aparelhos que medem a diferença de potencial. Um itotal
voltímetro ideal deve:
i1 i2 i3
• ser colocado em paralelo com o elemento que se pretende
U R1 R2 R3
medir a ddp;
• possuir resistência interna MUITO ALTA (infinita) em i1 i2 i3
comparação com os demais elementos.
Símbolo:
ch

U R1 R2 R3

REOSTATO Quando chave está aberta observa-se que


• O reostato é um resistor que possui a sua resistência
variável, normalmente, em função de um cursor. a) i1 e i2 aumentam e a corrente total permanece constante.
b) i1 e i2 diminuem e a corrente total aumenta.
c) i1 e i2 permanecem constante e a corrente total diminui.
d) todas as correntes ficam nulas.
e) i1 aumenta e i2 diminui, mas a corrente total permanece
constante.
Resolução: C
Quando a chave é aberta não passará corrente pelo resistor
3, entretanto os resistores 1 e 2 permanecem com mesma
imagem de um reostato resistência e sob mesma diferença de potencial que antes,
por isso as correntes i1 e i2 permanecem constantes.
itotal = i1 + i2 = i3

i1 i2 i3


U R1 R2 R3
i1 i2 i3

i’total = i1 + i2

i1 i2 ch
U R1 R2 R3
i1 i2

Com a retirada do resistor 3 que estava em paralelo com


os outros dois resistores, a resistência total do circuito
aumentou e por isso a corrente elétrica total diminuiu. Outra
forma de perceber que a corrente total diminui é lembrar que
a corrente total no início é dada por itotal=i1 + i2 + i3 e após a
mudança itotal=i1 + i2

PRÉ-VESTIBULAR PROENEM.COM.BR 1
FÍSICA II 19 ELEMENTOS DE UM CIRCUITO

FUSÍVEIS OU DISJUNTORES Determine a razão entre as resistências equivalentes antes e


São os protetores do circuito. Todos os circuitos têm uma depois.
corrente limite e, no caso da corrente ultrapassar aquele valor,
a) 3/5
um aparelho pode “queimar”. No circuito protegido por fusível ou
disjuntor, havendo uma corrente maior que a corrente limite, o b) 6/5
fusível “queima” (impedindo a passagem da corrente). O disjuntor c) 8/5
“desarma” como se fosse uma chave. d) 11/5
e) 15/7
LÂMPADAS Resolução: B
As lâmpadas são elementos que aparecem constantemente Antes:
em provas e têm como característica máxima transformar energia 4 Paralelo
elétrica em energia luminosa, se for do tipo fluorescente; se for R’eq = 6 . 3
do tipo incandescente, ela transforma energia elétrica em energia 6+3
térmica. A luz é consequência da elevada temperatura do filamento.
R’eq=2
Toda lâmpada possui uma inscrição de ddp e potência, que 36V 6 3
significam sua tensão ideal e a potência por ela dissipada.
Exemplo:
120 V – 60 W – se esta lâmpada estiver a uma ddp de 120 V,
6
ela dissipará uma potência de 60 W; se estiver a uma ddp menor,
produzirá uma potência menor também e no caso de uma ddp
Série
maior, pode ser que ela queime.
Req = 4 + 2 + 6
Req = 12

CURTO-CIRCUITO 4

36V 2

6
Se um elemento do circuito tiver os seus terminais ligados por
um fio de resistência elétrica desprezível, ocorrerá neste aparelho
um curto-circuito, pois toda corrente que o atravessava passará
a percorrer o fio; e em termos práticos, é como se o elemento Req = 12(antes)
não estivesse associado, isto é, como se ele fosse eliminado.
Quando ligamos dois pontos por um fio ideal (sem resistência), os
potenciais desses dois pontos rapidamente se igualam. Depois:
4 Paralelo 4
R’ = 6 . 3
eq
6+3 i
i i
R’eq=2
36V 6 3 36V 6 3

i i i

6 6
EXERCÍCIO RESOLVIDO
Série Série
Req = 4alteração
01. No esquema abaixo é mostrado uma + 2 + 6 no circuito, Req = 4 + 6
observe: Req = 12 Req = 10
4 4Ω 4

36V 2 36V
36V 6Ω 3Ω

6Ω
6 6

4Ω
Req = 12(antes) Req = 10(depois)

→ antes 12 6
36V 6Ω 3Ω
= =
depois 10 5

6Ω

2 PROENEM.COM.BR PRÉ-VESTIBULAR
19 ELEMENTOS DE UM CIRCUITO FÍSICA II

PONTE DE WHEATSTONE EXERCÍCIOS


Na figura abaixo, um gerador alimenta os quatro resistores
e um galvanômetro (amperímetro) que constituem o sistema. O PROTREINO
resistor R2 é um reostato. Percebe-se que para um certo valor da
resistência do reostato, o galvanômetro não registra a passagem
de corrente elétrica, e o circuito constitui a chamada Ponte de 01. A imagem abaixo representa um circuito elétrico composto por
Wheatstone. Isso é muito utilizado quando se pretende encontrar o três resistores R1, R2 e R3, um amperímetro e um gerador ideal.
valor de uma dada resistência, R1 e R3 são conhecidas, R2 é variável Considerando U = 190 V, R1 = 12 Ω, R2 = 4 Ω e R3 = 7 Ω determine a
e R4 é desconhecida. marcação no amperímetro.

Ao deixar de passar corrente pelo galvanômetro, equilibrando a


ponte de Wheatstone, os potenciais dos pontos C e D se igualam,
com isso teremos uma corrente i passando pelos resistores R1 e R2 02. A imagem abaixo representa um circuito elétrico composto por
e outra corrente i` passando pelos resistores R3 e R4. três resistores R1, R2 e R3, um amperímetro e um gerador ideal.
Considerando U = 190 V, R1 = 12 Ω, R2 = 4 Ω e R3 = 7 Ω determine
a marcação no amperímetro quando a chave está na posição A.

Mas VC = VD
Então
R1i = R3i’ 03. No circuito abaixo um eletricista conhece os valores de R2 = 30 Ω,
R3 = 45 Ω e R4 = 90 Ω. Deseja descobrir o valor resistência R1,
R2i = R4i’
para isso, com um voltímetro e um gerador ideais, monta o circuito
Dividindo uma igualdade pela outra representado abaixo:
R1 R3
= ⇒ R1 ⋅ R4 = R2 ⋅ R3
R2 R4
Condição para a ponte estar equilibrada

PROEXPLICA

COMO INTERRUPTORES PODE CAIR NO ENEM?


Os interruptores, dispositivos tão comuns em nosso dia a
dia, não passam de chaves que são inseridas em circuitos
para que possamos ligar e desligar determinados aparelhos.
Desta forma, alguns aparelhos podem entrar em curto com a
simples troca da posição do botão do interruptor. A questão
busca a identificação, dentro das opções indicadas, do
circuito descrito no enunciado. O eletricista fica surpreso ao perceber que a marcação no voltímetro
é zero volt. Com essas informações determine o valor estimado,
corretamente, pelo profissional.

PRÉ-VESTIBULAR PROENEM.COM.BR 3
FÍSICA II 19 ELEMENTOS DE UM CIRCUITO

04. Um circuito é composto por cinco resistores idênticos de


R = 10 Ω, como representado na imagem abaixo:

a) 0,0
b) 0,2
c) 0,3
d) 0,4
Calcule a resistência equivalente do circuito acima, considerando
os fios ideais. 02. (IFPE) O circuito elétrico representado no diagrama abaixo
contém um gerador ideal de 21 Volts com resistência interna
05. O circuito é composto por resistores do tipo R1, R2 e R3, todos desprezível alimentando cinco resistores.
ôhmicos de 4 Ω, 2 Ω e 1 Ω, respectivamente, também compõe o
circuito fios, um amperímetro, um voltímetro e um gerador, todos
ideais.

Qual o valor da medida da intensidade da corrente elétrica, expressa


em amperes, que percorre o amperímetro A conectado ao circuito
Determine:
elétrico representado?
a) a resistência equivalente do circuito;
a) 0,5 A
b) as marcações nos aparelhos de medição;
b) 1,0 A
c) o sentido da corrente em cada fio.
c) 1,5 A
d) 2,0 A
e) 2,5 A

EXERCÍCIOS 03. (CN) Considere que um determinado estudante, utilizando


PROPOSTOS resistores disponíveis no laboratório de sua escola, montou os
circuitos apresentados abaixo:

01. (EEAR) Em uma aula de laboratório o professor montou um


circuito com 3 resistores ôhmicos R1, R2 e R3 associados a uma
fonte de alimentação ideal (Vt) conforme o circuito abaixo. E
solicitou ao aluno que, usando um amperímetro ideal, medisse o
valor da intensidade de corrente elétrica que flui através de R2.

Querendo fazer algumas medidas elétricas, usou um voltímetro (V)


para medir a tensão e um amperímetro (A) para medir a intensidade
da corrente elétrica. Considerando todos os elementos envolvidos
como sendo ideais, os valores medidos pelo voltímetro (situação 1)
O aluno, porém fez a ligação do amperímetro (A) da maneira e pelo amperímetro (situação 2) foram, respectivamente:
indicada na figura a seguir. Com base nisso, assinale a alternativa
que representa o valor indicado, em ampères, no amperímetro.

4 PROENEM.COM.BR PRÉ-VESTIBULAR
19 ELEMENTOS DE UM CIRCUITO FÍSICA II

a) 2 V e 1,2 A
b) 4 V e 1,2 A
c) 2 V e 2,4 A
d) 4 V e 2,4 A
e) 6 V e 1,2 A

04. (ENEM) Um sistema de iluminação foi construído com um


circuito de três lâmpadas iguais conectadas a um gerador (G) de
tensão constante. Esse gerador possui uma chave que pode ser
ligada nas posições A ou B.

Para um valor de temperatura em que RS = 100 Ω, a leitura


apresentada pelo voltímetro será de
a) +6,2 V. c) +0,3 V. e) –6,2 V.
b) +1,7 V. d) –0,3 V.

07. (ENEM) Um eletricista analisa o diagrama de uma instalação


elétrica residencial para planejar medições de tensão e corrente
em uma cozinha. Nesse ambiente existem uma geladeira (G), uma
tomada (T) e uma lâmpada (L), conforme a figura. O eletricista
deseja medir a tensão elétrica aplicada à geladeira, a corrente total
e a corrente na lâmpada. Para isso, ele dispõe de um voltímetro (V)
e dois amperímetros (A).

Considerando o funcionamento do circuito dado, a lâmpada 1


brilhará mais quando a chave estiver na posição
a) B, pois a corrente será maior nesse caso.
b) B, pois a potência total será maior nesse caso.
Para realizar essas medidas, o esquema da ligação desses
c) A, pois a resistência equivalente será menor nesse caso. instrumentos está representado em:
d) B, pois o gerador fornecerá uma maior tensão nesse caso. a)
e) A, pois a potência dissipada pelo gerador será menor nesse
caso.

05. (ENEM) Um circuito em série é formado por uma pilha, uma


lâmpada incandescente e uma chave interruptora. Ao se ligar a
chave, a lâmpada acende quase instantaneamente, irradiando
calor e luz. Popularmente, associa-se o fenômeno da irradiação b)
de energia a um desgaste da corrente elétrica, ao atravessar o
filamento da lâmpada, e à rapidez com que a lâmpada começa a
brilhar. Essa explicação está em desacordo com o modelo clássico
de corrente.
De acordo com o modelo mencionado, o fato de a lâmpada acender
quase instantaneamente está relacionado à rapidez com que c)
a) o fluido elétrico se desloca no circuito.
b) as cargas negativas móveis atravessam o circuito.
c) a bateria libera cargas móveis para o filamento da lâmpada.
d) o campo elétrico se estabelece em todos os pontos do circuito.
e) as cargas positivas e negativas se chocam no filamento da d)
lâmpada.

06. (ENEM) Medir temperatura é fundamental em muitas aplicações,


e apresentar a leitura em mostradores digitais é bastante prático.
O seu funcionamento é baseado na correspondência entre valores
de temperatura e de diferença de potencial elétrico. Por exemplo, e)
podemos usar o circuito elétrico apresentado, no qual o elemento
sensor de temperatura ocupa um dos braços do circuito (RS) e a
dependência da resistência com a temperatura é conhecida.

PRÉ-VESTIBULAR PROENEM.COM.BR 5
FÍSICA II 19 ELEMENTOS DE UM CIRCUITO

08. (ENEM) Um curioso estudante, empolgado com a aula de A intensidade da corrente elétrica indicada pelo amperímetro, em
circuito elétrico que assistiu na escola, resolve desmontar sua A, é de
lanterna. Utilizando-se da lâmpada e da pilha, retiradas do a) 3.
equipamento, e de um fio com as extremidades descascadas, faz
as seguintes ligações com a intenção de acender a lâmpada: b) 4.
c) 8.
d) 12.
e) 15.

11. (PUCRJ) Calcule a corrente em ampères medida no amperímetro


(A) do circuito apresentado na figura.

Tendo por base os esquemas mostrados, em quais casos a


lâmpada acendeu?
a) (1), (3), (6) c) (1), (3), (5) e) (1), (2), (5)
a) 1,6
b) (3), (4), (5) d) (1), (3), (7)
b) 3,3
09. (UFMG) Um professor pediu a seus alunos que ligassem uma c) 5,0
lâmpada a uma pilha com um pedaço de fio de cobre. d) 8,3
Nestas figuras, estão representadas as montagens feitas por e) 20,0
quatro estudantes:
12. (UNESP) A figura mostra o circuito elétrico que acende a
lâmpada de freio e as lanternas traseira e dianteira de um dos lados
de um automóvel.

Considerando-se essas quatro ligações, é CORRETO afirmar que a


lâmpada vai acender apenas
a) na montagem de Mateus.
b) na montagem de Pedro.
c) nas montagens de João e Pedro. Considerando que as três lâmpadas sejam idênticas, se o circuito
for interrompido no ponto P, estando o automóvel com as lanternas
d) nas montagens de Carlos, João e Pedro.
apagadas, quando o motorista acionar os freios,

10. (UFTM) No circuito mostrado no diagrama, todos os resistores a) apenas a lanterna dianteira se acenderá.
são ôhmicos, o gerador e o amperímetro são ideais e os fios de b) nenhuma das lâmpadas se acenderá.
ligação têm resistência elétrica desprezível. c) todas as lâmpadas se acenderão, mas com brilho menor que
seu brilho normal.
d) apenas a lanterna traseira se acenderá.
e) todas as lâmpadas se acenderão com o brilho normal.

13. (ENEM (LIBRAS)) Durante a reforma de sua residência, um


casal decidiu que seria prático poder acender a luz do quarto
acionando um interruptor ao lado da porta e apagá-la com outro
interruptor próximo à cama. Um eletrotécnico explicou que esse
sistema usado para controlar uma lâmpada a partir de dois pontos
é conhecido como circuito de interruptores paralelos.

6 PROENEM.COM.BR PRÉ-VESTIBULAR
19 ELEMENTOS DE UM CIRCUITO FÍSICA II

fusível queima. Dessa forma, evita que a corrente elevada danifique


os aparelhos do circuito. Suponha que o circuito elétrico mostrado
seja alimentado por uma fonte de tensão U e que o fusível suporte
uma corrente nominal de 500 mA.

Como deve ser feita a montagem do circuito da lâmpada no quarto


desse casal?
a)

b)

Qual é o máximo valor da tensão U para que o fusível não queime?


a) 20 V
b) 40 V
c)
c) 60 V
d) 120 V
e) 185 V

15. (PUCRS) Na figura abaixo, estão representadas quatro


lâmpadas idênticas associadas por fios condutores ideais a uma
bateria ideal B. Uma chave interruptora C e três amperímetros ideais
também fazem parte do circuito. Na figura, a chave interruptora
está inicialmente fechada, e os amperímetros A1, A2 e A3 medem
intensidades de correntes elétricas, respectivamente, iguais a i1, i2 e
i3. Considere o campo gravitacional uniforme.

d)

e)

Quando a chave interruptora C é aberta, as leituras indicadas por


A1, A2 e A3 passam a ser, respectivamente,
a) menor que i1, menor que i2 e igual a i3.
b) menor que i1, igual a i2 e igual a i3.
c) igual a i1, maior que i2 e maior que i3.
14. (ENEM) Fusível é um dispositivo de proteção contra d) igual a i1, igual a i2 e menor que i3.
sobrecorrente em circuitos. Quando a corrente que passa por esse e) maior que i1, maior que i2 e maior que i3.
componente elétrico é maior que sua máxima corrente nominal, o

PRÉ-VESTIBULAR PROENEM.COM.BR 7
FÍSICA II 19 ELEMENTOS DE UM CIRCUITO

16. (FUVEST) O arranjo experimental representado na figura é 18. (ENEM PPL) Fusíveis são dispositivos de proteção de um
formado por uma fonte de tensão F, um amperímetro A, um voltímetro circuito elétrico, sensíveis ao excesso de corrente elétrica. Os
V, três resistores, R1, R2 e R3, de resistências iguais, e fios de ligação. modelos mais simples consistem de um filamento metálico de
baixo ponto de fusão, que se funde quando a corrente ultrapassa
determinado valor, evitando que as demais partes do circuito sejam
danificadas. A figura mostra um diagrama de um circuito em que
o fusível F protege um resistor R de 12 Ω, uma lâmpada L de 6 W e
um alto-falante que conduz 1 A.

Sabendo que esse fusível foi projetado para trabalhar com uma
corrente até 20% maior que a corrente nominal que atravessa esse
circuito, qual é o valor, em ampères, da corrente máxima que o
fusível F permite passar?
a) 1,0
Quando o amperímetro mede uma corrente de 2 A, e o voltímetro,
uma tensão de 6 V, a potência dissipada em R2 é igual a b) 1,5
Note e adote: c) 2,0
- A resistência interna do voltímetro é muito maior que a dos d) 2,5
resistores (voltímetro ideal). e) 3,0
- As resistências dos fios de ligação devem ser ignoradas.
19. (ENEM) Para ligar ou desligar uma mesma lâmpada a partir
a) 4 W
de dois interruptores, conectam-se os interruptores para que a
b) 6 W mudança de posição de um deles faça ligar ou desligar a lâmpada,
c) 12 W não importando qual a posição do outro. Esta ligação é conhecida
d) 18 W como interruptores paralelos. Este interruptor é uma chave de
duas posições constituída por um polo e dois terminais, conforme
e) 24 W mostrado nas figuras de um mesmo interruptor. Na Posição I a
chave conecta o polo ao terminal superior, e na Posição II a chave o
17. (ENEM) Um estudante, precisando instalar um computador, conecta ao terminal inferior.
um monitor e uma lâmpada em seu quarto, verificou que precisaria
fazer a instalação de duas tomadas e um interruptor na rede
elétrica. Decidiu esboçar com antecedência o esquema elétrico.
“O circuito deve ser tal que as tomadas e a lâmpada devem
estar submetidas à tensão nominal da rede elétrica e a lâmpada
deve poder ser ligada ou desligada por um interruptor sem afetar
os outros dispositivos” — pensou. O circuito que cumpre a finalidade de funcionamento descrita no
Símbolos adotados: texto é:
a)

Qual dos circuitos esboçados atende às exigências?


a) d)
b)

b)
e)

c)

c)

8 PROENEM.COM.BR PRÉ-VESTIBULAR
19 ELEMENTOS DE UM CIRCUITO FÍSICA II

d) e 80 W cada. A chave pode ser ligada no ponto A ou no ponto B,


fazendo funcionar apenas uma parte do circuito de cada vez.

e)

20. (ENEM PPL) Um eletricista precisa medir a resistência elétrica


de uma lâmpada. Ele dispõe de uma pilha, de uma lâmpada (L),
de alguns fios e de dois aparelhos: um voltímetro (V), para medir Considerando desprezíveis as resistências elétricas dos fios de
a diferença de potencial entre dois pontos, e um amperímetro (A), ligação e de todas as conexões utilizadas, calcule as potências
para medir a corrente elétrica. O circuito elétrico montado pelo dissipadas pelas lâmpadas L1 e L2, quando a chave é ligada no ponto
eletricista para medir essa resistência é A. Em seguida, calcule as potências dissipadas pelas lâmpadas L1 e
L2, quando a chave é ligada no ponto B.
a) d)
03. (UNESP) Três lâmpadas idênticas (L1, L2 e L3), de resistências
elétricas constantes e valores nominais de tensão e potência iguais
a 12 V e 6 W, compõem um circuito conectado a uma bateria de
12 V. Devido à forma como foram ligadas, as lâmpadas L2 e L3 não
brilham com a potência para a qual foram projetadas.

e)

b)

c)

Considerando desprezíveis as resistências elétricas das conexões


EXERCÍCIOS DE e dos fios de ligação utilizados nessa montagem, calcule a
resistência equivalente, em ohms, do circuito formado pelas três
APROFUNDAMENTO lâmpadas e a potência dissipada, em watts, pela lâmpada L2.

01. (PUCRJ) Uma estudante tem uma pequena lâmpada LED 04. (UNESP) Um estudante pretendia construir o tetraedro regular
vermelha em cujas especificações lê-se o seguinte: “Queda de BCDE, representado na figura 1, com seis fios idênticos, cada um
tensão 1,8 V corrente máxima 0,02 A”. Ela quer ligar essa lâmpada com resistência elétrica constante de 80 Ω no intuito de verificar
a duas pilhas AA em série, cada uma delas com voltagem de 1,5 experimentalmente as leis de Ohm em circuitos de corrente
V. Mas percebe que, para isso, deve acrescentar algum resistor ao contínua.
circuito.
a) Qual deve ser a resistência mínima do resistor para que a
lâmpada LED não queime?
b) A estudante tem à sua disposição até quatro resistores de 120
Ω. Quantos resistores ela deve usar para que a lâmpada opere,
seguramente, em sua corrente máxima? Justifique e faça um
esquema do circuito.

02. (UNESP) O circuito representado é constituído por quatro


resistores ôhmicos, um gerador ideal, uma chave Ch de resistência
elétrica desprezível e duas lâmpadas idênticas, L1 e L2, que
apresentam valores nominais de tensão e potência iguais a 40 V

PRÉ-VESTIBULAR PROENEM.COM.BR 9
FÍSICA II 19 ELEMENTOS DE UM CIRCUITO

Acidentalmente, o fio DE rompeu-se; com os cinco fios restantes e EXERCÍCIOS DE APROFUNDAMENTO


um gerador de 12 V um amperímetro e um voltímetro, todos ideais, 01. a) 60 Ω
o estudante montou o circuito representado na figura 2, de modo b) Associação em paralelo de dois resistores.
que o fio BC permaneceu com o mesmo comprimento que tinha
na figura 1.

02. P1A = 0; P2A = 12,8 W; P2B = 0; P1B = 80 W


03. R = 16 Ω; P2 = 1,5 W.
04. iA = 0,15 A; UV = 6 V.
05. a) 1.200 W
b) R1 = 4 Ω; R2 = 8 Ω
c) 4,5

ANOTAÇÕES
Desprezando a resistência dos fios de ligação dos instrumentos
ao circuito e das conexões utilizadas, calcule as indicações do
amperímetro, em A, e do voltímetro, em V, na situação representada
na figura 2.

05. (FUVEST) Uma jovem, para aquecer uma certa quantidade de


massa M de água, utiliza, inicialmente, um filamento enrolado, cuja
resistência elétrica R0 é igual a 12 Ω, ligado a uma fonte de 120 V
(situação I).
Desejando aquecer a água em dois recipientes, coloca, em cada um,
metade da massa total de água (M/2), para que sejam aquecidos
por resistências R1 e R2, ligadas à mesma fonte (situação II). A
jovem obtém essas duas resistências, cortando o filamento inicial
em partes não iguais, pois deseja que R1 aqueça a água com duas
vezes mais potência que R2. Para analisar essas situações:

a) Estime a potência P0, em watts, que é fornecida à massa total


de água, na situação I.
b) Determine os valores de R1 e R2, em ohms, para que no recipiente
onde está R1 a água receba duas vezes mais potência do que
no recipiente onde está R2, na situação II.
c) Estime a razão P/P0, que expressa quantas vezes mais potência
é fornecida na situação II (P), ao conjunto dos dois recipientes,
em relação à situação I (P0).
NOTE E ADOTE: V = RI; P = VI

GABARITO

EXERCÍCIOS PROPOSTOS
01. C 05. D 09. C 13. B 17. E
02. B 06. D 10. E 14. D 18. E
03. B 07. E 11. C 15. B 19. E
04. C 08. D 12. C 16. A 20. C

10 PROENEM.COM.BR PRÉ-VESTIBULAR
20
FÍSICA II
GERADORES

INTRODUÇÃO U = 0 volt
 (Correntequando
(Corrente quandooogerador
geradoréé colocado em curto − circ
Um gerador é um aparelho que transforma algum tipo de  E
 icc = r colocado em curto-circuito)
energia em energia elétrica.
Simbologia:

POTÊNCIA DE UM GERADOR
Já sabemos calcular a potência de uma associação de
resistores utilizando a ddp, a resistência e a corrente do circuito.
Baseado nos resultados obtidos anteriormente, temos:

Pgerada = ε ⋅ i 

FORÇA ELETROMOTRIZ (FEM Ε) Pútil = U⋅i Pg =
Pu + Pd
Pdissipada = r ⋅ i²
A força eletromotriz de um gerador é a ddp máxima que pode
ser gerada por ele em seus terminais. P → watt(W)
Porém, nem sempre a ddp fornecida ao circuito é igual ao valor
da fem, pois em alguns casos há uma perda em relação à ddp que
pode ser gerada, como é o caso do gerador não ideal. RENDIMENTO DE UM GERADOR
Quando se compara a ddp que o gerador fornece ao circuito
GERADOR IDEAL × GERADOR NÃO IDEAL com sua força eletromotriz, estamos medindo o rendimento (n) do
gerador, que pode ser expresso de forma decimal ou percentual,
O gerador ideal é aquele em que toda sua força eletromotriz é
essa grandeza é calculada pela razão entre a ddp útil e a f.e.m. do
aproveitada pelo circuito, não há nenhum tipo de perda.
gerador, como mostrado abaixo:
U=ε

ddp útil = ddp produzida


O gerador ideal é teórico usado para facilitar o estudo, na
prática, não existem geradores ideais pois sempre há dissipação
interna de energia. Todo gerador possui uma resistência interna ASSOCIAÇÃO DE GERADORES
(r) que já provoca uma queda de potencial (perda) em seu interior. Assim como os resistores, os geradores podem se associar,
Para os geradores reais, vale a equação conhecida como equação quando há mais de um gerador no circuito, e essa associação pode
do Gerador. ser em série ou em paralelo.
U=ε–r.i
Entenda o que ela nos diz: a ddp que o gerador põe para o GERADORES EM SÉRIE
circuito externo é igual ao valor da força eletromotriz (ε) menos
a queda de tensão que os próprios elementos internos, devido à
resistência interna do gerador, provocam.
Podemos observar o gráfico dessa equação, dado por:

i = 0A (Circuito de chave aberta)



U = E

Em problemas de associação de geradores é recorrente


a necessidade de determinar o gerador equivalente, que seria
um único gerador que funciona de modo a substituir de forma
equivalente todos os geradores da associação.
A força eletromotriz equivalente será dada pela soma das
forças eletromotrizes.
εeq = ε1 + ε2 + ε3 +... + εn
A resistência interna equivalente será também obtida pela
soma de cada resistência interna.
req = r1 + r2 + r3 +... + rn
tan(x) = r(resistência interna)

PRÉ-VESTIBULAR PROENEM.COM.BR 1
FÍSICA II 20 GERADORES

GERADORES EM PARALELO

No caso de geradores em paralelo, estudaremos apenas o


caso em que os geradores são idênticos, mesma fem e mesma
resistência interna.
Nesse tipo de associação, a fem do gerador equivalente será
a mesma dos geradores da associação. Porém, para se obter a
resistência interna equivalente, deve-se imaginar n resistores iguais
em paralelo, isto é, a resistência interna do gerador equivalente é o Dados: E1 = 120V, r1 = 4 Ω
valor da resistência dividido pelo número de geradores associados. E2 = 120V, r2 = 4 Ω
R1 = R2 = 2 Ω
R3 = 4 Ω
R4 = 3 Ω
Considerando os fios ideais determine a marcação no amperímetro.
EXERCÍCIOS
PROTREINO 03. O circuito abaixo é composto por quatro geradores E1, E2, E3 e
E4 de resistências internas r1, r2, r3 e r4, respectivamente, um resistor
ôhmico R e um amperímetro ideal A.
01. O circuito abaixo é composto por dois geradores E1 e E2, um
resistor ôhmico R e um voltímetro ideal V.

Dados: E1 = 20 V, r1 = 2 Ω
E2 = 30 V, r2 = 4 Ω
E3 = E4 = 50 V e r3 = r4 = 8 Ω
R = 10 Ω
Considere os fios ideais e determine a marcação no amperímetro.

04. O gráfico abaixo representa a curva característica de um gerador.

Dados: E1 = 100 V, r1 = 2 Ω
E2 = 20 V, r2 = 4 Ω
R = 34 Ω
Considerando os fios ideais calcule a intensidade da corrente
elétrica i e determine a marcação do voltímetro.

02. O circuito abaixo é composto por dois geradores E1 e E2, de


resistências internas r1 e r2, respectivamente, quatro resistores
ôhmicos R1, R2, R3 e R4 e um amperímetro ideal A.

Determine a força eletromotriz, a resistência interna e a corrente


elétrica em curto circuito do gerador representado pelo gráfico
acima.

2 PROENEM.COM.BR PRÉ-VESTIBULAR
20 GERADORES FÍSICA II

05. O gráfico abaixo representa a curva característica de um gerador Desconsiderando perdas internas, se em uma descarga a raia-
torpedo conseguir produzir uma corrente elétrica total de 50 A
durante um curto intervalo de tempo, a potência elétrica gerada por
ela, nesse intervalo de tempo, será de
a) 3.500 W.
b) 3.000 W.
c) 2.500 W.
d) 4.500 W.
e) 4.000 W.

02. (FAMERP) Quando um gerador de força eletromotriz 12 V é


ligado a um resistor R de resistência 5,8 Ω, uma corrente elétrica
i de intensidade 2,0 A circula pelo circuito.
Determine a força eletromotriz, a resistência interna e a corrente
elétrica em curto circuito do gerador representado pelo gráfico acima.

EXERCÍCIOS
PROPOSTOS
01. (UNESP) Na maioria dos peixes elétricos as descargas
são produzidas por órgãos elétricos constituídos por células,
chamadas eletroplacas, empilhadas em colunas. Suponha que
cada eletroplaca se comporte como um gerador ideal.
A resistência interna desse gerador é igual a
a) 0,40 Ω.
b) 0,20 Ω.
Suponha que o sistema elétrico de um poraquê, peixe elétrico de
água doce, seja constituído de uma coluna com 5.000 eletroplacas c) 0,10 Ω.
associadas em série, produzindo uma força eletromotriz total de d) 0,30 Ω.
600 V. e) 0,50 Ω.

03. (UERJ) Observe o gráfico, que representa a curva característica


de operação de um gerador:

Considere que uma raia-torpedo, que vive na água do mar, possua


um sistema elétrico formado por uma associação em paralelo
de várias colunas, cada uma com 750 eletroplacas iguais às do
poraquê, ligadas em série, constituindo mais da metade da massa
corporal desse peixe.

Com base nos dados, a resistência interna do gerador, em ohm, é


igual a:
a) 1,0
b) 3,0
c) 4,0
d) 6,0

PRÉ-VESTIBULAR PROENEM.COM.BR 3
FÍSICA II 20 GERADORES

04. (ENEM PPL) Baterias de lítio, utilizadas em dispositivos 07. (EBMSP) Unidades hospitalares utilizam geradores elétricos
eletrônicos portáteis, são constituídas de células individuais com para se prevenir de interrupções no fornecimento de energia elétrica.
ddp de 3,6 V. É comum os fabricantes de computadores utilizarem Considerando-se um gerador elétrico de força eletromotriz 120,0 V
as células individuais para a obtenção de baterias de 10,8 V ou 14,4 e resistência interna 4,0 Ω que gera potência elétrica de 1.200,0 W,
V. No entanto, fazem a propaganda de seus produtos fornecendo a quando ligado a um circuito externo, é correto afirmar, com base
informação do número de células da bateria e sua capacidade de nessas informações e nos conhecimentos de eletricidade, que
carga em mAh, por exemplo, 4.400 mAh. a) o gerador elétrico transforma energia elétrica em outras formas
Disponível em: http://laptopbattery.net. Acesso em: 15 nov. 2011 (adaptado). de energia.
Dentre as baterias de 10,8 V e 14,4 V, constituídas por 12 células b) a diferença de potencial elétrico entre os terminais do gerador
individuais, qual possui maior capacidade de carga? é igual a 110,0 V.
a) A bateria de 10,8 V, porque possui combinações em paralelo de c) a intensidade da corrente elétrica que circula através do
4 conjuntos com 3 células em série. gerador é igual a 8,0 A.
b) A bateria de 14,4 V, porque possui combinações em paralelo de d) a potência dissipada em outras formas de energia no interior
3 conjuntos com 4 células em série. do gerador é igual a 512,0 W.
c) A bateria de 14,4 V, porque possui combinações em série de 3 e) a potência elétrica que o gerador lança no circuito externo para
conjuntos com 4 células em paralelo. alimentar as instalações é igual a 800,0 W.
d) A bateria de 10,8 V, porque possui combinações em série de 4
08. (UECE) Considere duas pilhas alcalinas de 1,5 V ligadas
conjuntos com 3 células em paralelo.
em paralelo, com polos de mesmo sinal ligados entre si. Nessa
e) A bateria de 10,8 V, porque possui combinações em série de 3 configuração, a tensão entre os terminais da associação é, em
conjuntos com 4 células em série. Volts,
a) 0,5. c) 1,5.
05. (UEG) Considere uma bateria de força eletromotriz ε e
resistência interna desprezível. Qual dos gráficos a seguir melhor b) 7,5. d) 3,0.
representa a bateria?
09. (ENEM PPL) Uma lâmpada é conectada a duas pilhas de tensão
a) d)
nominal 1,5 V, ligadas em série. Um voltímetro, utilizado para medir
a diferença de potencial na lâmpada, fornece uma leitura de 2,78 V
e um amperímetro indica que a corrente no circuito é de 94,2 mA.
O valor da resistência interna das pilhas é mais próximo de
a) 0,021 Ω.
b) 0,22 Ω.
b) e) c) 0,26 Ω.
d) 2,3 Ω.
e) 29 Ω.

10. (ESPCEX (AMAN)) No circuito elétrico desenhado abaixo, todos


os resistores ôhmicos são iguais e têm resistência R = 1,0 Ω. Ele é
alimentado por uma fonte ideal de tensão contínua de E = 5,0 V. A
c) diferença de potencial entre os pontos A e B é de:

06. (ENEM) Em algumas residências, cercas eletrificadas são


utilizadas com o objetivo de afastar possíveis invasores. Uma cerca
eletrificada funciona com uma diferença de potencial elétrico de
aproximadamente 10.000 V. Para que não seja letal, a corrente que
pode ser transmitida através de uma pessoa não deve ser maior do
que 0,01 A. Já a resistência elétrica corporal entre as mãos e os pés
de uma pessoa é da ordem de 1.000 Ω. a) 1,0 V

Para que a corrente não seja letal a uma pessoa que toca a cerca b) 2,0 V
eletrificada, o gerador de tensão deve possuir uma resistência c) 2,5 V
interna que, em relação à do corpo humano, é d) 3,0 V
a) praticamente nula. e) 3,3 V
b) aproximadamente igual.
c) milhares de vezes maior. 11. (ENEM PPL) Em um laboratório, são apresentados aos alunos
uma lâmpada, com especificações técnicas de 6 V e 12 W, e um
d) da ordem de 10 vezes maior.
conjunto de 4 pilhas de 1,5 V cada.
e) da ordem de 10 vezes menor.
Qual associação de geradores faz com que a lâmpada produza
maior brilho?

4 PROENEM.COM.BR PRÉ-VESTIBULAR
20 GERADORES FÍSICA II

a) 13. (UNESP) Uma espécie de peixe-elétrico da Amazônia, o


Poraquê, de nome científico Electrophorous electricus, pode gerar
diferenças de potencial elétrico (ddp) entre suas extremidades, de
tal forma que seus choques elétricos matam ou paralisam suas
presas. Aproximadamente metade do corpo desse peixe consiste
em células que funcionam como eletrocélulas. Um circuito elétrico
de corrente contínua, como o esquematizado na figura, simularia o
circuito gerador de ddp dessa espécie. Cada eletrocélula consiste
em um resistor de resistência R = 7,5 Ω e de uma bateria de fem ε.

b)

c)

Sabendo-se que, com uma ddp de 750 V entre as extremidades A e


B, o peixe gera uma corrente I = 1,0 A, a fem ε em cada eletrocélula,
em volts, é
a) 0,35.
b) 0,25.
d)
c) 0,20.
d) 0,15.
e) 0,05.

14. (MACKENZIE) No laboratório de Física, um aluno observou que


ao fechar a chave ch do circuito a seguir, o valor fornecido pelo
voltímetro ideal passa a ser 3 vezes menor. Analisando esse fato, o
aluno determinou que a resistência interna do gerador vale:
e)

a) 4 Ω
b) 6 Ω
12. (ESPCEX (AMAN)) A pilha de uma lanterna possui uma força c) 8 Ω
eletromotriz de 1,5 V e resistência interna de 0,05 Ω. O valor da d) 10 Ω
tensão elétrica nos polos dessa pilha quando ela fornece uma
corrente elétrica de 1,0 A a um resistor ôhmico é de e) 12 Ω

a) 1,45 V
15. (MACKENZIE) Quando as lâmpadas L1, L2 e L3 estão ligadas
b) 1,30 V ao gerador de f.e.m. ε, conforme mostra a figura ao lado, dissipam,
c) 1,25 V respectivamente, as potências 1,00 W, 2,00 W e 2,00 W, por efeito
d) 1,15 V Joule. Nessas condições, se o amperímetro A, considerado ideal,
indica a medida 500 mA, a força eletromotriz do gerador é de:
e) 1,00 V

PRÉ-VESTIBULAR PROENEM.COM.BR 5
FÍSICA II 20 GERADORES

a) 23,0 Ω
b) 36,6 Ω
c) 12,5 Ω
d) 5,00 Ω
e) 19,2 Ω

19. (UEFS) Um circuito elétrico é constituído por um gerador de


força eletromotriz E e resistência interna r = 2 Ω e por um resistor
ôhmico de resistência R. Se por esse circuito circular uma corrente
elétrica de intensidade i = 2 A, a diferença de potencial entre os
pontos A e B será 16 V.

a) 2,25 V
b) 3,50 V
c) 3,75 V
d) 4,00 V
e) 4,25 V

16. (FGV) A unidade de medida de potencial elétrico do Sistema


Internacional é o volt (V), que também é unidade da grandeza física
chamada
a) força elétrica.
b) carga elétrica.
c) corrente elétrica.
d) força eletromotriz.
e) campo magnético.
Considerando desprezíveis as resistências dos fios e das conexões
17. (FUVEST) Uma bateria possui força eletromotriz å e resistência utilizados na montagem desse circuito, os valores de E e de R são
interna R0. Para determinar essa resistência, um voltímetro foi a) 20 V e 8 Ω.
ligado aos dois polos da bateria, obtendo-se V0 = ε (situação I). b) 10 V e 8 Ω.
Em seguida, os terminais da bateria foram conectados a uma
lâmpada. Nessas condições, a lâmpada tem resistência R = 4 Ω e o c) 32 V e 8 Ω.
voltímetro indica VA (situação II), de tal forma que V0/VA = 1,2. Dessa d) 32 V e 10 Ω.
experiência, conclui-se que o valor de R0 é e) 20 V e 10 Ω.

20. (UFU) O circuito elétrico (fig. 1) é utilizado para a determinação


da resistência interna r e da força eletromotriz ε do gerador. Um
resistor variável R (também conhecido como reostato) pode
assumir diferentes valores, fazendo com que a corrente elétrica
no circuito também assuma valores diferentes para cada valor
escolhido de R.
Ao variar os valores de R, foram obtidas leituras no voltímetro V e
no amperímetro A, ambos ideais, resultando no gráfico (fig. 2).

a) 0,8 Ω
b) 0,6 Ω
c) 0,4 Ω
d) 0,2 Ω
e) 0,1 Ω

18. (ITA) Uma bateria composta por 50 células voltaicas em série é


carregada por uma fonte de corrente contínua ideal de 220 V. Cada
célula tem uma força eletromotriz de 2,30 V e resistência interna
de 0,100 Ω. Sendo a corrente de carregamento de 6,00 A, indique Com base nessas informações, assinale a alternativa que
o valor da resistência extra que deve ser inserida em série com a corresponde aos valores corretos, respectivamente, da resistência
fonte. interna e da força eletromotriz do gerador.

6 PROENEM.COM.BR PRÉ-VESTIBULAR
20 GERADORES FÍSICA II

a) 2 Ω e 7 V.
b) 1 Ω e 4 V.
c) 3 Ω e 12 V.
d) 4 Ω e 8 V.

EXERCÍCIOS DE
APROFUNDAMENTO
01. (UFPR) Um dado gerador elétrico real fornece uma tensão
V entre seus terminais quando percorrido por uma corrente i.
O gráfico apresenta a curva V x i para esse gerador.
Determine
a) a corrente I, em amperes, que alimenta o equipamento eletrônico C.
b) o número mínimo N, de baterias, necessário para manter o
sistema, supondo que as baterias armazenem carga de 50 A·h
cada uma.
c) a tensão V, em volts, que deve ser fornecida pelo gerador, para
carregar as baterias em 4 h.
NOTE E ADOTE:
(1 ampere × 1 segundo = 1 coulomb)
O parâmetro usado para caracterizar a carga de uma bateria,
produto da corrente pelo tempo, é o ampere·hora (A·h).
Suponha que a tensão da bateria permaneça constante até o final
a) Determine a resistência interna r desse gerador. de sua carga.
b) Um resistor de resistência R0 = 6 Ω é ligado aos terminais
desse gerador, formando um circuito fechado em que gerador
04. (UFRRJ) Um estudante deseja medir a resistência interna de
e resistor estão ligados em série. Determine o rendimento do
um gerador, cuja f.e.m. pode ser ajustada para diferentes valores.
gerador quando funcionando nessa configuração.
Para tanto, ele constrói um circuito com o próprio gerador - um
amperímetro A e um resistor de resistência R = 18 Ω - e obtém o
02. (UFPR) Foi feito um estudo com uma associação de resistores gráfico a seguir, relacionando a f.e.m. do gerador a corrente medida
(de acordo com a figura ao lado), a qual foi conectada a uma fonte pelo amperímetro.
de tensão com força eletromotriz de 7,5 V e resistência interna “r”.
Os valores dos resistores da associação estão indicados na figura
a seguir.

Com base no gráfico:


a) Calcule a resistência interna do gerador.
b) Para uma f.e.m. igual a 12 V, calcule a potência dissipada pela
resistência interna do gerador.

05. (UERJ) Na tabela abaixo, são apresentadas as resistências e


as d.d.p. relativas a dois resistores, quando conectados, separada-
Todos os fios condutores são ideais e os resistores são ôhmicos.
mente, a uma dada bateria.
Verificou-se uma intensidade de corrente elétrica no resistor R2 de
0,5 A. Assim, determine:
Resistência (Ω) d.d.p. (v)
a) O resistor equivalente da associação.
5,8 11,6
b) A tensão elétrica nos extremos da associação de resistores.
c) A resistência interna do gerador. 3,8 11,4
Considerando que os terminais da bateria estejam conectados a
03. (FUVEST) Em uma ilha distante, um equipamento eletrônico de um resistor de resistência igual a 11,8 Ω, calcule a energia elétrica
monitoramento ambiental, que opera em 12 V e consome 240 W, é dissipada em 10 segundos por esse resistor.
mantido ligado 20h por dia. A energia é fornecida por um conjunto
de N baterias ideais de 12 V. Essas baterias são carregadas por um
gerador a diesel, G, através de uma resistência R de 0,2 Ω. Para
evitar interferência no monitoramento, o gerador é ligado durante
4h por dia, no período em que o equipamento permanece desligado.

PRÉ-VESTIBULAR PROENEM.COM.BR 7
FÍSICA II 20 GERADORES

GABARITO

EXERCÍCIOS PROPOSTOS
01. D 05. A 09. D 13. C 17. A
02. B 06. C 10. B 14. E 18. C
03. B 07. E 11. C 15. E 19. A
04. A 08. C 12. A 16. D 20. C

EXERCÍCIOS DE APROFUNDAMENTO
01. a) 4 Ω c) 1 Ω 04. a) 2 Ω
b) 0,6 03. a) 20 A b) 0,72 W
02. a) 4 Ω b) 8 baterias 05. 118 J
b) U = 6 V c) 14 V

ANOTAÇÕES

8 PROENEM.COM.BR PRÉ-VESTIBULAR
21
FÍSICA II
CAPACITORES

Um capacitor consiste em um dispositivo elétrico que possui


como principal característica armazenar cargas elétricas e, PROEXPLICA
consequentemente, energia potencial eletrostática (ou elétrica).
Praticamente todos os aparelhos que você conhece como: rádios, Propriedade gráfica
computadores, geladeiras, secadores de cabelo, etc., possuem
capacitores em sua estrutura ou placas principais. Uma importante
diferença entre um capacitor e um gerador (que também fornece
carga a um circuito) é o tempo de fornecimento dessas cargas.
O capacitor fornece energia em um tempo muito mais curto, se
descarregando logo a seguir. Como exemplo temos o flash de
uma câmera fotográfica, que possui um capacitor responsável por
liberar boa parte da carga no momento do “disparo”, provocando
um brilho muito intenso da lâmpada.

A CAPACIDADE ELÉTRICA
Para um condutor isolado, em equilíbrio eletrostático, eletrizado
com uma quantidade de carga Q e potencial V, observa-se que Q e
CAPACITOR OU CONDENSADOR
V são diretamente proporcionais, ou seja: Conjunto de dois condutores, denominados armaduras,
eletrizados com quantidades de cargas de mesmo valor absoluto,
Q mas de sinais contrários.
= constante
V Função: armazenar cargas elétricas.
É a essa constante de proporcionalidade que se chama de Carga do Capacitor: é a carga da armadura positiva.ddp do
Capacidade Elétrica (C). capacitor: U = VA – VB
Q Q
C= Capacidade Elétrica ou Capacitância: C =
V U
Q2 Q ⋅ U C ⋅ U2
PROEXPLICA Energia Potencial Elétrica: EPOT
= = =
2C 2 2

1) Capacidade Elétrica e Capacitância são sinônimos.


coulomb C
CAPACITOR PLANO
2) Unidade SI: = farad(F) ⇒ 1F = 1 É o capacitor cujas armaduras são planas, de mesma área e
volt V
paralelas entre si, separadas por uma distância d.
3) Um caso importante é o de uma esfera carregada, como
k ⋅Q R Entre as armaduras coloca-se um material isolante para que
seu potencial é dado pela equação V = , sua C = não haja fluxo de cargas elétricas entre as placas, chamado de
R K
dielétrico.
capacidade elétrica só depende do seu raio e do meio onde
ela se encontra: Pode-se, no lugar de um material isolante, usar o vácuo entre
as armaduras, e é o que faremos em nosso estudo.
4) É muito comum usar o μF: 1 μF = 10 F. –6

ENERGIA POTENCIAL ELÉTRICA (EPOT)


Para um condutor isolado, em equilíbrio eletrostático, de Para carregar o capacitor, liga-se ele a um gerador:
capacidade elétrica C, quantidade de carga Q e potencial V, tem-se
que:
Q2
EPOT =
2C

Se combinarmos essa equação com Q = C·V

C ⋅ V2
teremos: EPOT =
2
Q Q⋅V
Se combinamos com C =
V
, teremos: EPOT =
2 CAPACITÂNCIA DE CAPACITOR PLANO
A capacitância de um capacitor plano é:
1º) diretamente proporcional à área A de suas armaduras;
2º) inversamente proporcional à distância d entre as armaduras.

PRÉ-VESTIBULAR PROENEM.COM.BR 1
FÍSICA II 21 CAPACITORES

CαAeCα
1 A
, logo C α ; para que se transformem em uma
ASSOCIAÇÃO EM PARALELO
d d
equação, devemos multiplicar por uma constante:
Esta constante de proporcionalidade é denominada

A
C = ε0
d

permissividade elétrica do isolante colocado entre as


armaduras do capacitor, no caso do vácuo seu valor é:
ε = 8,85 × 10-12 F/m

PROEXPLICA
• Tem como objetivo aumentar a quantidade de carga
Entre as armaduras de um capacitor plano forma-se um armazenada sob a mesma ddp.
campo elétrico (E) uniforme, logo, é válida a expressão vista • Atenção: numa associação em paralelo, todos ficam
em eletrostática: U = E · d. submetidos à mesma ddp.
• O capacitor equivalente sob a mesma ddp acumula o
somatório das cargas.
• O cálculo da capacidade elétrica do capacitor equivalente
ASSOCIAÇÃO DE CAPACITORES (Ce) é feito por:
Ceq = C1 + C2 + C3 + ... + Cn
ASSOCIAÇÃO EM SÉRIE
EXERCÍCIO RESOLVIDO

02. Vamos calcular a capacitância equivalente do circuito


abaixo.
C1=3F

• Tem como objetivo dividir a ddp entre os capacitores C2=6F


associados, sem que se queimem.
• Atenção: numa associação em série, todos os capacitores
ficam carregados com a mesma carga.
• Capacitor Equivalente é o capacitor que nas mesmas
condições (U) é carregado com a mesma carga (Q).
• O cálculo da Capacidade Elétrica do capacitor equivalente
(Ce) é feito por:
1 1 1 1 1 
= + + + ... +
Ceq C1 C2 C3 Cn
Resolução:
Ceq= C1 + C2
Ceq= 3 + 6
EXERCÍCIO RESOLVIDO
Ceq = 9µF
01. Vamos calcular a capacitância equivalente do circuito Ceq= 9 ⋅ 10−6 F
abaixo.

C2=6F C1=3F PROEXPLICA

COMO CAPACITOR PODE CAIR NO ENEM?


Os capacitores elétricos são dispositivos que armazenam
 energia elétrica para ser utilizada em um momento futuro,
Resolução: quando necessário. Em alguns circuitos, capacitores e
resistores são utilizados juntos buscando o funcionamento
C1= 3 µF C ⋅C 3 ⋅6
 ⇒ Ceq = 1 2 = ⇒ Ceq =2 µF adequado. Na questão abaixo é explorada a identificação do
C2 = 6 µF C1 + C2 3 + 6 uso cotidiano deste dispositivo.

2 PROENEM.COM.BR PRÉ-VESTIBULAR
36V
21 CAPACITORES FÍSICA II

Considerando U = 100 V, determine:


EXERCÍCIOS a) a capacitância equivalente do circuito, em µF;
PROTREINO b) a carga acumulada pelo conjunto, em coulomb;
c) a carga acumulada em cada capacitor, em µC;
01. Um dado capacitor apresenta uma certa quantidade de carga
d) a d.d.p. em cada capacitor, em volt.
Q em suas placas quando submetido a uma tensão V. O gráfico
abaixo o apresenta o comportamento da carga Q (em coulombs)
desse capacitor para uma tensão de V = 220 volts. 04. Calcule a capacitância equivalente entre os pontos A e B.

Com base no gráfico, assinale a alternativa que expressa


corretamente a energia U armazenada, em joules, nesse capacitor
quando submetido a uma tensão de 220V.
Dados: C1 = 6 mF; C2 = 3 mF; C3 = 0,5 mF; C4 = 1,5 mF
02. Três capacitores C1, C2 e C3 de capacitâncias 3 µF, 4 µF e 12 µF,
05. A curva característica de um gerador está representada abaixo:
respectivamente. Conforme esquema abaixo:

Considerando U = 120V, determine:


a) capacitância equivalente do circuito, em µF;
b) a carga acumulada pelo conjunto, em coulomb; Nos terminais desse gerador foi conectado um capacitor de
capacitância C = 4 µF e um resistor R = 8Ω conforme o esquema
c) a d.d.p. em cada capacitor, em volt.
abaixo.

03. Três capacitores C1, C2 e C3 de capacitâncias 4 µF, 6 µF e 12 µF,


respectivamente. Conforme esquema abaixo:

Calcule a carga armazenada, em coulomb, no capacitor quando ele


está carregado.

PRÉ-VESTIBULAR PROENEM.COM.BR 3
FÍSICA II 21 CAPACITORES

04. (INSPER) No circuito ideal esquematizado na figura, o gerador


fornece uma tensão contínua de 200 V. As resistências dos
resistores ôhmicos são R1 = R3 = 20 Ω, R2 = 60 Ω e a capacitância
EXERCÍCIOS do capacitor é C = 2,0 · 10-6 F.
PROPOSTOS
01. (EFOMM) A professora Ana Clara, com intuito de determinar a
capacitância de um capacitor que estava com suas especificações
ilegíveis, realizou o seguinte procedimento: carregou um segundo
capacitor de 150 pF com uma tensão de 100 V, utilizando uma
fonte de alimentação. Em seguida, desligou o capacitor da fonte
e o conectou em paralelo com o capacitor de valor desconhecido.
Nessas condições, ela observou que os capacitores apresentavam
uma tensão de 60 V. Com esse procedimento, a professora pôde
calcular o valor do capacitor desconhecido, que é de
a) 45 pF
b) 70 pF
c) 100 pF
d) 150 pF
e) 180 pF
Nessas condições, a quantidade de carga acumulada no capacitor
02. (UFPR) Um dado capacitor apresenta uma certa quantidade será, em C, igual a
de carga Q em suas placas quando submetido a uma tensão
V. O gráfico ao lado apresenta o comportamento da carga Q a) 2,4 · 10-3
(em microcoulombs) desse capacitor para algumas tensões V b) 2,4 · 10-4
aplicadas (em volts). c) 1,2 · 10-3
d) 1,2 · 10-4
e) 2,0 · 10-3

05. (IFSUL) Cinco capacitores, todos de mesma capacitância C,


são associados entre os pontos A e B da associação, conforme a
configuração indicada na figura a seguir.

Com base no gráfico, assinale a alternativa que expressa


corretamente a energia U armazenada nesse capacitor quando
submetido a uma tensão de 3 V.
a) U = 24 µJ. d) U = 96 µJ.
b) U = 36 µJ. e) U = 144 µJ.
c) U = 72 µJ.

03. (MACKENZIE) Um estagiário do curso de Engenharia Elétrica


da UPM – Universidade Presbiteriana Mackenzie – montou um
circuito com o objetivo de acumular energia da ordem de mJ
(milijoule). Após algumas tentativas, ele vibrou com a montagem
do circuito abaixo, cuja energia potencial elétrica acumulada vale,
em mJ,

Se esses cinco capacitores fossem substituídos por um único


capacitor que, submetido à mesma diferença de potencial elétrico
armazenasse a mesma quantidade de carga elétrica, esse capacitor
deveria ter capacitância igual a:
a) 5C
a) 2 d) 6 b) C/5
b) 3 e) 9 c) 3C/7
c) 4 d) 7C/3

4 PROENEM.COM.BR PRÉ-VESTIBULAR
21 CAPACITORES FÍSICA II

06. (MACKENZIE) a) a tensão entre os terminais do de maior capacitância é menor.


b) a tensão entre os terminais dos dois capacitores é a mesma.
c) a corrente fornecida pela bateria é sempre maior que zero.
d) a corrente fornecida pela bateria é sempre menor que zero.

11. (UECE) Um resistor de 3 Ω é ligado em série a um capacitor


de 4 µF, e a associação assim obtida é conectada aos terminais
de uma bateria de 12 V. Após o capacitor estar completamente
carregado, é correto afirmar que a diferença de potencial (em Volts)
nos terminais do capacitor e do resistor é, respectivamente,
a) 12 e 0.
b) 48 e 4.
c) 4 e 3.
d) 3 e 4.
Na associação de capacitores, esquematizada acima, a capacitância
está indicada na figura para cada um dos capacitores. Assim, a 12. (ENEM 2ª APLICAÇÃO) Um cosmonauta russo estava a
capacitância equivalente, entre os pontos A e B no circuito, é bordo da estação espacial MIR quando um de seus rádios de
comunicação quebrou. Ele constatou que dois capacitores do rádio
a) C.
de 3 µF e 7 µF ligados em série estavam queimados. Em função
b) 2C. da disponibilidade, foi preciso substituir os capacitores defeituosos
c) 3C. por um único capacitor que cumpria a mesma função.
d) 4C. Qual foi a capacitância, medida em µF, do capacitor utilizado pelo
e) 8C. cosmonauta?
a) 0,10
07. (UECE) Considere dois capacitores ligados em série e conectados b) 0,50
a uma bateria. Um dos capacitores tem capacitância maior que a do c) 2,1
outro. É correto afirmar que a capacitância equivalente
d) 10
a) é menor que qualquer uma das capacitâncias individuais.
e) 21
b) é maior que qualquer uma das capacitâncias individuais.
c) tem valor entre as duas capacitâncias da associação. 13. (PUCPR) Fibrilação ventricular é um processo de contração
d) depende da tensão na bateria. desordenada do coração que leva à falta de circulação sanguínea no
corpo, chamada parada cardiorrespiratória. O desfibrilador cardíaco
08. (UECE) Considere dois capacitores, C1 = 2 µF e C2 = 3 µF, ligados é um equipamento que aplica um pulso de corrente elétrica através
em série e inicialmente descarregados. Supondo que os terminais do coração para restabelecer o ritmo cardíaco. O equipamento é
livres da associação foram conectados aos polos de uma bateria, é basicamente um circuito de carga e descarga de um capacitor
correto afirmar que, após cessar a corrente elétrica, (ou banco de capacitores). Dependendo das características da
emergência, o médico controla a energia elétrica armazenada no
a) as cargas nos dois capacitores são iguais e a tensão elétrica
capacitor dentro de uma faixa de 5 a 360 J. Suponha que o gráfico
é maior em C2.
dado mostra a curva de carga de um capacitor de um desfibrilador.
b) a carga é maior em C2 e a tensão elétrica é igual nos dois. O equipamento é ajustado para carregar o capacitor através de uma
c) as cargas nos dois capacitores são iguais e a tensão elétrica diferença de potencial de 4 kV. Qual o nível de energia acumulada
é maior em C1. no capacitor que o médico ajustou?
d) a carga é maior em C1 e a tensão elétrica é igual nos dois.

09. (IFSUL) Capacitores são componentes eletrônicos que têm por


função básica armazenar cargas elétricas e, consequentemente,
energia potencial elétrica. Em circuitos elétricos compostos apenas
por capacitores, eles podem ser associados em série, em paralelo
ou de forma mista.
Em relação às características desses tipos de associação, quando
associados em série,
a) os capacitores armazenam cargas iguais.
b) os capacitores submetem-se sempre à mesma diferença de
potencial.
c) a carga total estabelecida na associação é igual à soma das
cargas de cada capacitor.
d) a capacitância equivalente da associação é igual à soma das
capacitâncias individuais. a) 100 J.
b) 150 J.
10. (UECE) Considere dois capacitores com diferentes capacitâncias, c) 200 J.
ligados em paralelo e conectados a uma bateria. É correto afirmar
d) 300 J.
que, após carregados,
e) 400 J.

PRÉ-VESTIBULAR PROENEM.COM.BR 5
FÍSICA II 21 CAPACITORES

14. (UFPR) No circuito esquematizado abaixo, deseja-se que o Assinale a alternativa correta para a energia elétrica armazenada
capacitor armazene uma energia elétrica de 125 µJ. no capacitor C.
a) 625 x 10-6 J.
b) 225 x 10-6 J.
c) 25 x 10-6 J.
d) 50 x 10-6 J.
e) 75 x 10-6 J.

18. (UNICAMP) Quando um rolo de fita adesiva é desenrolado,


As fontes de força eletromotriz são consideradas ideais e de ocorre uma transferência de cargas negativas da fita para o rolo,
valores ε1 = 10 V e ε2 = 5 V. conforme ilustrado na figura a seguir.
Assinale a alternativa correta para a capacitância C do capacitor
utilizado.
a) 10 µF. d) 12,5 µF.
b) 1 µF. e) 50 µF.
c) 25 µF.

15. (UERN) O capacitor equivalente de uma associação em série,


constituída por 3 capacitores iguais, tem capacitância 2 µF.
Utilizando-se 2 destes capacitores para montar uma associação
em paralelo, a mesma apresentará uma capacitância de
a) 3 µF.
b) 6 µF.
c) 12 µF. Quando o campo elétrico criado pela distribuição de cargas é maior
d) 18 µF. que o campo elétrico de ruptura do meio, ocorre uma descarga
elétrica. Foi demonstrado recentemente que essa descarga pode
16. (IFPE) Um circuito elétrico é constituído por três capacitores, ser utilizada como uma fonte econômica de raios-X.
quatro resistores e um gerador ideal, conforme a figura abaixo. Para um pedaço da fita de área A = 5,0×10−4 m2 mantido a uma
O circuito é submetido a uma tensão elétrica de 220 V. A carga distância constante d = 2,0 mm do rolo, a quantidade de cargas
elétrica armazenada pelo capacitor de 10 µF, em µC, vale: acumuladas é igual a Q = CV, sendo V a diferença de potencial entre
A C
a fita desenrolada e o rolo e C = ε0 em que ε0 ≈ 9,0x10−12 .
d Vm
Nesse caso, a diferença de potencial entre a fita e o rolo para
Q = 4,5×10−9 C é de
a) 1,2×102 V.
b) 5,0×10−4 V.
c) 2,0×103 V.
d) 1,0×10−20 V.

19. (UPE) No circuito A, considere os três resistores com


resistências iguais e, no circuito B, considere os três capacitores
com capacitâncias iguais.

a) 154 c) 462 e) 924


b) 308 d) 716

17. (UFPR) Capacitores são dispositivos que podem armazenar


energia quando há um campo elétrico em seu interior, o qual é
produzido por cargas elétricas depositadas em suas placas. O
circuito ao lado é formado por um capacitor C de capacitância 2
µF e por duas fontes de fem, consideradas ideais, com ε1 = 10 V e
ε2 = 15 V.
É CORRETO afirmar que a resistência equivalente é igual a
a) 3R, e a capacitância equivalente é igual a 3C.
b) R/3, e a capacitância equivalente é igual a 3C.
c) 3R, e a capacitância equivalente é igual a C/3.
d) R/3, e a capacitância equivalente é igual a C/3.
e) R, e a capacitância equivalente é igual a C.

6 PROENEM.COM.BR PRÉ-VESTIBULAR
21 CAPACITORES FÍSICA II

20. (ENEM 2ª APLICAÇÃO) Atualmente, existem inúmeras opções


de celulares com telas sensíveis ao toque (touchscreen). Para
decidir qual escolher, é bom conhecer as diferenças entre os
principais tipos de telas sensíveis ao toque existentes no mercado.
Existem dois sistemas básicos usados para reconhecer o toque de
uma pessoa:
• O primeiro sistema consiste de um painel de vidro normal,
recoberto por duas camadas afastadas por espaçadores.
Uma camada resistente a riscos é colocada por cima de todo
o conjunto. Uma corrente elétrica passa através das duas
camadas enquanto a tela está operacional. Quando um usuário
toca a tela, as duas camadas fazem contato exatamente
naquele ponto. A mudança no campo elétrico é percebida, e
as coordenadas do ponto de contato são calculadas pelo
computador.
• No segundo sistema, uma camada que armazena carga
elétrica é colocada no painel de vidro do monitor. Quando um
a) Qual é a diferença de potencial no capacitor em t = 0,2 s?
usuário toca o monitor com seu dedo, parte da carga elétrica
é transferida para o usuário, de modo que a carga na camada b) Num outro instante, a corrente no capacitor é ic = 150 µA.
que a armazena diminui. Esta redução é medida nos circuitos Quanto vale a corrente iB no resistor RB nesse instante?
localizados em cada canto do monitor. Considerando as
diferenças relativas de carga em cada canto, o computador 02. (UNICAMP) Numa tela de televisor de plasma, pequenas células
calcula exatamente onde ocorreu o toque. contendo uma mistura de gases emitem luz quando submetidas
Disponível em: http://eletronicos.hsw.uol.com.br. Acesso em: 18 set. 2010 (adaptado). a descargas elétricas. A figura a seguir mostra uma célula com
dois eletrodos, nos quais uma diferença de potencial é aplicada
O elemento de armazenamento de carga análogo ao exposto no para produzir a descarga. Considere que os eletrodos formam
segundo sistema e a aplicação cotidiana correspondente são, um capacitor de placas paralelas, cuja capacitância é dada por
respectivamente, ε0 A
C= , onde ε0 = 8,9 x 10-12 F/m, A é a área de cada eletrodo e d
a) receptores — televisor. d
b) resistores — chuveiro elétrico. é a distância entre os eletrodos.
c) geradores — telefone celular.
d) fusíveis — caixa de força residencial.
e) capacitores — flash de máquina fotográfica.

EXERCÍCIOS DE
APROFUNDAMENTO
01. (UNICAMP) Capacitores são componentes de circuitos
elétricos que têm a função de armazenar carga. O tempo a) Calcule a capacitância da célula.
necessário para carregar ou descarregar um capacitor depende b) A carga armazenada em um capacitor é proporcional à
da sua capacitância C, bem como das características dos outros diferença de potencial aplicada, sendo que a constante de
componentes a que ele está ligado no circuito. É a relativa demora proporcionalidade é a capacitância. Se uma diferença de
na descarga dos capacitores que faz com que o desligamento de potencial igual a 100 V for aplicada nos eletrodos da célula,
certos eletrodomésticos não seja instantâneo. O circuito da figura qual é a carga que será armazenada?
µC
A apresenta um capacitor de capacitância = C 20 = 20 µF ligado c) Se a carga encontrada no item b) atravessar o gás em 1 µs
V
(tempo de descarga), qual será a corrente média?
a dois resistores de resistências RA = 40 Ω e RB = 60 kΩ, e a uma
bateria de força eletromotriz ε = 12 V. A chave S é ligada no instante
t = 0 e o gráfico da figura B mostra a carga q(t) no capacitor em 03. (UNESP) São dados um capacitor de capacitância (ou
função do tempo. capacidade) C, uma bateria de f.e.m. ε e dois resistores cujas
resistências são, respectivamente, R1 e R2. Se esses elementos
forem arranjados como na figura adiante, a carga armazenada no
capacitor será nula. Justifique esta afirmação.

PRÉ-VESTIBULAR PROENEM.COM.BR 7
FÍSICA II 21 CAPACITORES

04. (EBMSP) A era digital acabou por alterar hábitos da ANOTAÇÕES


comunicação dentro da família. Se por um lado a internet rompe
barreiras da comunicação e permite a interação com pessoas de
partes distintas do país e do mundo, por outro ela quebra diálogos
rotineiros. Filhos que antes sentavam à mesa com os pais, hoje
preferem a internet e o “bate-papo” de amigos.
Disponível em: <http://www.lagoinha.com/ibl-noticia/familias-do-seculo-xxi
-nao-sao-mais-as-mesmas/>. Acesso em: 6 out. 2015.

Sabe-se que as teclas de computadores utilizadas para digitar


mensagens se comportam como os capacitores de placas planas
e paralelas imersas no ar.
Considerando
- a área média de cada tecla de um computador igual a 1,0 cm²,
- a distância entre uma tecla e a base do seu teclado igual a 1,0 mm,
- a permissividade do ar, ε0, igual a 9,0 · 10-12 F/m,
- a tensão aplicada em cada tecla igual a 6,0 V, no instante que uma
tecla é empurrada para baixo cerca de 0,4 mm da sua posição de
origem, determine a carga armazenada na armadura do capacitor

05. (UFG) Um capacitor de placas paralelas é parcialmente


preenchido por um líquido dielétrico de constante dielétrica k = 2.
Para otimizar o armazenamento de energia, um técnico resolve
medir a capacitância do capacitor deitado e em pé, conforme
ilustrado na figura a seguir.

Considerando o exposto, determine:


a) uma expressão para a capacitância de cada um dos capacitores
nas situações ilustradas nas figuras (a) e (b);
b) o valor numérico da razão entre as capacitâncias das situações
ilustradas nas figuras (a) e (b) respectivamente, considerando
que a razão H/h = 3.

GABARITO

EXERCÍCIOS PROPOSTOS
01. C 05. C 09. A 13. C 17. A
02. C 06. A 10. B 14. A 18. C
03. E 07. A 11. A 15. C 19. C
04. B 08. C 12. C 16. E 20. E

EXERCÍCIOS DE APROFUNDAMENTO
01. a) 2,5 V
b) 60 µA
02. a) 1,068 x 1014 F
b) 1,068 x 10-12 C
c) 1,068 x 10-6 A
03. Os terminais do capacitor estão ligados por fio condutor de resistência nula, logo a
diferença de potencial (ddp) entre seus suportes será zero. Já que Q = C.U, a carga no
capacitor será nula.
04. 9,0 x 10-12 C
2 εL w ; ε w (L +  )
05. a) Ca = Cb = 0
(2H − h) H
b) 0,9

8 PROENEM.COM.BR PRÉ-VESTIBULAR
22 CIRCUITOS ELÉTRICOS - FÍSICA II

REVISÃO

1ª LEI DE OHM EM PARALELO


A ddp entre os terminais de um circuito gera uma força elétrica Quando dois ou mais resistores são ligados nos mesmos
nos elétrons do condutor submetido a ddp. Para resistores ôhmicos, potenciais, ou seja, submetidos à mesma ddp.
a razão entre ddp e a intensidade da corrente elétrica é constante.
U1 U2 U3 Un
= = = … = = R (resistência constante)
i1 i2 i3 in

U=R.i

U (v)

U2
Tg = R
U1 i = i1 + i2 + i3 + ... + in
U U=
= 1 U=
2 U=
3 ... Un
=

1 1 1 1 1
= + + + ... +
0 i1 i2 i (A) Req R1 R2 R3 Rn

Para dois resistores em paralelo


U – tensão, ddp, diferença de potencial [volt (V)]
R – Resistência [ohm (Ω)] R1 ⋅ R2
Req =
i – Corrente elétrica [ampére (A)] R1 + R2

Para n resistores idênticos


2ª LEI DE OHM R
A resistência elétrica de um condutor ôhmico é constante e Req =
n
depende de suas dimensões e do material que constitui o condutor.
ρ ⋅L AMPERÍMETRO: aparelho que mede a corrente elétrica, deve
R= ser ligado em série. O amperímetro ideal possui resistência nula.
A

VOLTÍMETRO: aparelho que mede a ddp, deve ser ligado em


paralelo. O voltímetro ideal possui resistência infinita.

FUSÍVEL: aparelho que protege o circuito, permite a passagem


L – Comprimento (m) de uma corrente limite.
A – Área da secção transversal (m²)
ρ – Resistividade (Ωm) CURTO-CIRCUITO: quando dois pontos do circuito são ligados
por um condutor de resistência desprezível, serão percorridos por
ASSOCIAÇÃO DE RESISTORES toda a corrente, não ocorrendo divisão das cargas nesse ponto.

EM SÉRIE PROEXPLICA
Quando dois ou mais resistores são percorridos pela mesma
corrente elétrica (cargas elétricas). COMO POTÊNCIA ELÉTRICA PODE CAIR NO ENEM?
A potência de determinados aparelhos está diretamente
relacionada com a voltagem que este aparelho irá receber.
Devemos sempre ficar atentos à voltagem indicada nas
i i i
especificações técnicas, pois uma voltagem superior àquela
indicada pode causar uma sobrecarga e queimar o aparelho.
A questão relaciona a potência do aparelho descrito com a
voltagem na qual este será ligado, percebendo se ocorrerá
aumento ou redução da potência.
i = i1 = i2 = i3 = ...= in
U = U1 + U2 + U3 + ...+ Un

Req = R1 + R2 + R3 + ...+ Rn

PRÉ-VESTIBULAR PROENEM.COM.BR 1
FÍSICA II 22 CIRCUITOS ELÉTRICOS - REVISÃO

EXERCÍCIO RESOLVIDO

01. (ENEM) Observe a tabela seguinte. Ela traz especificações


técnicas constantes no manual de instruções fornecido pelo
fabricante de uma torneira elétrica.

03. Num balde contendo 2,7 litros de água, a uma temperatura


inicial de 50°C existe um resistor ôhmico, imerso na água, de
resistência elétrica R = 18 Ω alimentado por um gerador de força
(http://www.cardeal.com.br.manualprod/Manuais/Torneira%20
Suprema/”Manual…Torneira…Suprema…roo.pdf) eletromotriz E = 200 V e resistência interna r = 2 Ω, conforme o
desenho abaixo:
Considerando que o modelo de maior potência da versão
220 V da torneira suprema foi inadvertidamente conectada a
uma rede com tensão nominal de 127 V, e que o aparelho está
configurado para trabalhar em sua máxima potência, qual o
valor aproximado da potência ao ligar a torneira?
a) 1.830 W c) 3.200 W e) 5.500 W
b) 2.800 W d) 4.030 W

Resolução: A
Relação de potência: P = U²/R
Considerando a resistência R um valor constante do chuveiro,
temos que P varia de forma diretamente proporcional a U².
Na tabela, observamos que o modelo de 220 V com o maior
valor de potência (Quente) opera com a potência máxima de
5500 W. O experimento foi feito ao nível do mar. Considerando que a energia
dissipada pelo resistor imerso foi totalmente absorvida pela água
Com os dados apresentados podemos fazer a razão entre e que o resistor permaneceu ligado por dois minutos. Calcule a
a situação antes e depois. Colocaremos ‘ para identificar as temperatura final da água.
grandezas depois.
Dados:
P’/P = (U’/U)² calor específico da água: 1 cal/g°C
P’/5500 = (127/220)² calor latente de vaporização da água: 540 cal/g
P’ = 1833 W. densidade da água = 1 kg/L
1 cal = 4 J

04. O circuito é construído com resistores idênticos de resistência


R = 1,6Ω. Os fios, o amperímetro, o voltímetro e o gerador são ideais.
EXERCÍCIOS
Observe o esquema abaixo:
PROTREINO
01. A tabela abaixo representa dois aparelhos eletrodomésticos e
seu tempo de uso em horas por semana.

Aparelho Potência (W) Tempo (horas/semana)

Chuveiro 2000 10
Geladeira 500 140

Considerando o custo de 1 kWh igual a R$ 0,50 calcule o custo


mensal desses aparelhos.

02. O gráfico abaixo apresenta o comportamento da intensidade


da corrente elétrica quando aumentamos a d.d.p. em um resistor a Determine:
uma temperatura constante. a) A resistência equivalente;
Determine o tipo de resistor, calcule sua resistência elétrica e b) A marcação no amperímetro;
determine a voltagem para que ele seja atravessado por uma c) A marcação no voltímetro.
corrente elétrica de 0,4 A.

2 PROENEM.COM.BR PRÉ-VESTIBULAR
22 CIRCUITOS ELÉTRICOS - REVISÃO FÍSICA II

05. Um modelo de gerador foi submetido a testes e sua curva Dados: o calor específico e a densidade do ar da sala são iguais a
característica foi plotada em um gráfico fora de escala representado 1,0 kJ (kg°K) e 1,1 kg/m³, respectivamente.
abaixo: a) 1 min
b) 6 min
c) 10 min
d) 220 min
e) 360 min

02. (PUCRJ) Uma lâmpada é ligada a uma bateria de 120 V e


dissipa 40,0 W. A resistência dessa lâmpada, em Ω, é:
a) 8,00 x 10-2
b) 0,33
c) 3,00
d) 80,0
Para montar um projeto foram usados dois geradores, desse e) 360
mesmo modelo, resistores ôhmicos, um fio e um voltímetro, ambos
ideais, conforme representado no esquema abaixo: 03. (UNESP) O poraquê é um peixe elétrico que vive nas águas
amazônicas. Ele é capaz de produzir descargas elétricas elevadas
pela ação de células musculares chamadas eletrócitos. Cada
eletrócito pode gerar uma diferença de potencial de cerca de 0,14
V. Um poraquê adulto possui milhares dessas células dispostas
em série que podem, por exemplo, ativar-se quando o peixe se
encontra em perigo ou deseja atacar uma presa.

A corrente elétrica que atravessa o corpo de um ser humano pode


causar diferentes danos biológicos, dependendo de sua intensidade
e da região que ela atinge. A tabela indica alguns desses danos em
Sabendo que a marcação no voltímetro é zero determine o valor
função da intensidade da corrente elétrica.
de R e calcule a intensidade da corrente elétrica i que atravessa os
geradores e a d.d.p. entre os pontos A e B.
intensidade de corrente elétrica dano biológico
Até 10 mA apenas formigamento
Atenção Proaluno!
De 10 mA até 20 mA contrações musculares
A questão Protreino 5 é uma grande revisão de vários conteúdos
apresentados nos últimos módulos. Curva característica do De 20 mA até 100 mA convulsões e parada respiratória
gerador, associação de geradores, associação mista de resistores, De 100 mA até 3 A fibrilação ventricular
curto-circuito e ponte de Wheatstone. Todos esses elementos
estão presentes para que seja possível relembrar cada detalhe acima de 3 A parada cardíaca e queimaduras graves
que foi estudado até aqui. Se necessário revise esses conteúdos e (José Enrique R. Duran. Biofísica: fundamentos e aplicações, 2003. Adaptado.)
retorne para essa questão, o desenho pode assustar num primeiro
momento, mas você consegue! Considere um poraquê que, com cerca de 8000 eletrócitos, produza
uma descarga elétrica sobre o corpo de uma pessoa. Sabendo que
a resistência elétrica da região atingida pela descarga é de 6000 Ω,
de acordo com a tabela, após o choque essa pessoa sofreria
EXERCÍCIOS a) parada respiratória.

PROPOSTOS b) apenas formigamento.


c) contrações musculares.
01. (UPE) Uma resistência de 440 Ω utilizada por um aquecedor d) fibrilação ventricular.
está conectada a uma tomada de 220 V de tensão. e) parada cardíaca.
Sabendo que o aquecedor deve elevar a temperatura do ar de uma
sala de dimensões 2,0 m x 2,0 m x 2,5 m em 6,0 °C, determine por 04. (UFRGS) No circuito esquematizado abaixo R1 e R2 são resistores
quanto tempo aproximadamente o aquecedor deve permanecer com a mesma resistividade p. R1 tem comprimento 2 L e seção
ligado. Considere que as paredes são termicamente isolantes. transversal A, e R2 tem comprimento L e seção transversal 2 A.

PRÉ-VESTIBULAR PROENEM.COM.BR 3
FÍSICA II 22 CIRCUITOS ELÉTRICOS - REVISÃO

Nessa situação, a corrente elétrica que percorre o circuito é


a) 2 AV / (5pL). Por que uma lâmpada incandescente de 100 W a 110 V, como a
usada pelo personagem da tira, queima quando ligada em uma
b) 2 AV / (3pL). rede de 220 V?
c) AV / (pL). a) Ao dobrar a tensão, a lâmpada dissipa energia a uma taxa
d) 3 AV / (2pL). quatro vezes maior.
e) 5 AV / (2pL). b) Ao dobrar a tensão, a lâmpada dissipa energia a uma taxa duas
vezes maior.
05. (MACKENZIE) c) Ao dobrar a corrente, a lâmpada dissipa energia a uma taxa
duas vezes maior.
d) Ao dobrar a corrente, a resistência da lâmpada cai à metade.
e) Ao dobrar a corrente, a potência da lâmpada cai à metade.

07. (UNICAMP) A figura 1 apresentada a seguir representa a


potência elétrica dissipada pelo filamento de tungstênio de uma
lâmpada incandescente em função da sua resistência elétrica.
Já a figura 2 apresenta a temperatura de operação do filamento
em função de sua resistência elétrica. Se uma lâmpada em
funcionamento dissipa 150 W de potência elétrica, a temperatura
do filamento da lâmpada é mais próxima de:

A ponte de fio mostrada acima é constituída por uma bateria,


um galvanômetro G, dois resistores, um de resistência elétrica
R1 = 10,0 Ω e outro de resistência elétrica R2 = 40,0 Ω, um fio
condutor homogêneo de resistividade r, área de secção transversal
A e comprimento L = 100,0 cm e um cursor C que desliza sobre
o fio condutor. Quando o cursor é colocado de modo a dividir o
fio condutor em dois trechos de comprimentos L1 e L2 a corrente
elétrica no galvanômetro é nula.
Os comprimentos L1 e L2 valem, respectivamente,
a) 50,0 cm e 50,0 cm
b) 60,0 cm e 40,0 cm
c) 40,0 cm e 60,0 cm
d) 80,0 cm e 20,0 cm
e) 20,0 cm e 80,0 cm

06. (UFSM) Uma vez que a produção de energia elétrica, em qualquer


de suas modalidades, tem impactos ambientais, inovações que
levem à diminuição do consumo de energia são necessárias. Assim,
as antigas lâmpadas incandescentes vêm sendo substituídas por
alternativas energeticamente mais eficientes. Naquele tipo de
lâmpada, a emissão de luz ocorre quando a temperatura de um
filamento de tungstênio é elevada a valores entre 2.700 e 3.300
K. Esse aquecimento ocorre como resultado da dissipação da
energia dos elétrons ao serem transportados através do condutor.
Aquecimento e emissão de radiação infravermelha consomem
cerca de 90% da energia elétrica fornecida para a lâmpada. Com
base nesse conhecimento, considere a situação representada na
tira a seguir. a) 325°C. d) 3.750°C.
b) 1.250°C.
c) 3.000°C.

4 PROENEM.COM.BR PRÉ-VESTIBULAR
22 CIRCUITOS ELÉTRICOS - REVISÃO FÍSICA II

08. (ENEM PPL) Um eletricista projeta um circuito com três 10. (PUCRS) No circuito abaixo, estão representadas três lâmpadas
lâmpadas incandescentes idênticas, conectadas conforme a idênticas, L1, L2 e L3, associadas por fios condutores ideais a uma
figura. Deseja-se que uma delas fique sempre acesa, por isso é bateria ideal B. Cada uma das lâmpadas dissipa, respectivamente,
ligada diretamente aos polos da bateria, entre os quais se mantém uma potência P1, P2 e P3.
uma tensão constante. As outras duas lâmpadas são conectadas
em um fio separado que contém uma chave. Com a chave aberta
(desligada), a bateria fornece uma potência X.

Sobre a potência elétrica dissipada pelas lâmpadas, é correto


afirmar que:
a) P1 = P2 = P3 d) P1 < P2 = P3
b) P1 < P2 < P3 e) P1 > P2 > P3
Assumindo que as lâmpadas obedeçam à Lei de Ohm, com a chave c) P1 > P2 = P3
fechada, a potência fornecida pela bateria, em função de X, é:
2 3 e) 3X. 11. (UECE) O poder calorífico da gasolina é 34,86 kJ/L. Isso equivale
a) X. c) X.
3 2 a dizer que 1L desse combustível tem armazenados 34,86 kJ de
b) X. d) 2X. energia no caso de sua utilização em uma combustão perfeita.
Suponha que 1L de gasolina pudesse ser utilizado com 100% de
eficiência para produção de calor. Isso seria suficiente para manter
09. (EPCAR -AFA) Em um chuveiro elétrico, submetido a uma tensão
por 3486 s um aquecedor de que potência, em W?
elétrica constante de 110 V, são dispostas quatro resistências
ôhmicas, conforme figura abaixo. a) 1.
b) 10.
c) 100.
d) 1000.

12. (UNICAMP) Por sua baixa eficiência energética, as lâmpadas


incandescentes deixarão de ser comercializadas para uso
doméstico comum no Brasil. Nessas lâmpadas, apenas 5% da
energia elétrica consumida é convertida em luz visível, sendo o
restante transformado em calor. Considerando uma lâmpada
incandescente que consome 60 W de potência elétrica, qual a
energia perdida em forma de calor em uma hora de operação?
a) 10.800 J. c) 250.200 J.
b) 34.200 J. d) 216.000 J.

13. (UEMA)
A LÂMPADA COM INTERNET
Ela é do tipo LED, mas funciona nos soquetes comuns que
você tem em casa e emite luz equivalente à de uma lâmpada
Faz-se passar pelas resistências um fluxo de água, a uma mesma convencional de 50 watts (mas gasta apenas 8,5 W). É preciso
temperatura, com uma vazão constante de 1,32 litros por minuto. conectar um adaptador – que já vem com o produto – à rede Wi-Fi
Considere que a água tenha densidade de 1,0 g/cm³ e calor da sua casa, e instalar um app no seu iPad ou iPhone.
específico de 1,0 cal/g°C, que 1 cal = 4 J e que toda energia elétrica Fonte: BADÔ, Fernando; GARATTONI, Bruno. Lâmpada com Internet.
fornecida ao chuveiro seja convertida em calor para aquecer, In: Revista Superinteressante. v. 229. São Paulo: Abril, 2013.
homogeneamente, a água.
A grandeza física que está relacionada aos números citados é
Nessas condições, a variação de temperatura da água, em °C, ao denominada
passar pelas resistências é
a) tensão elétrica.
a) 25
b) resistência elétrica.
b) 28
c) intensidade de corrente elétrica.
c) 30
d) energia elétrica.
d) 35
e) potência elétrica.

PRÉ-VESTIBULAR PROENEM.COM.BR 5
FÍSICA II 22 CIRCUITOS ELÉTRICOS - REVISÃO

14. (UPF) Em uma aula no laboratório de Física, o professor solicita


aos alunos que meçam o valor da resistência elétrica de um
resistor utilizando um voltímetro ideal e um amperímetro ideal. Dos
esquemas abaixo, que representam arranjos experimentais, qual o
mais indicado para a realização dessa medição?
a) Esquema A A resistência elétrica equivalente entre os
pontos A e B é igual a
R
a)
4
b) 3R
4
c) 4R
3
d) 4R

16. (CEFET MG) Analise o circuito abaixo.

b) Esquema B

c) Esquema C
Sabendo-se que a corrente I é igual a 500mA, o valor da tensão
fornecida pela bateria, em volts, é
a) 10.
b) 20.
c) 30.
d) 40.
e) 50.

17. (UFRGS) Considere o circuito formado por três lâmpadas


d) Esquema D idênticas ligadas em paralelo à bateria, conforme representa a
figura (1).

e) Esquema E

Como a chave C foi aberta na figura (2), considere as afirmações


abaixo sobre a figura (2), em comparação à situação descrita na
15. (IFSUL) Quatro resistores, todos de mesma Resistência Elétrica
figura (1).
R, são associados entre os pontos A e B de um circuito elétrico,
conforme a configuração indicada na figura.

6 PROENEM.COM.BR PRÉ-VESTIBULAR
22 CIRCUITOS ELÉTRICOS - REVISÃO FÍSICA II

I. A potência fornecida pela bateria é a mesma.


II. A diferença de potencial aplicada a cada lâmpada acesa é a
mesma.
III. As correntes elétricas que percorrem as lâmpadas acesas são
menores.
Quais estão corretas?
a) Apenas II. d) Apenas I e III.
b) Apenas III. e) I, II e III.
c) Apenas I e II. Nessa situação, quais são as três lâmpadas que acendem com
o mesmo brilho por apresentarem igual valor de corrente fluindo
nelas, sob as quais devem se posicionar os três atores?
18. (ENEM PPL) No manual de uma máquina de lavar, o usuário vê
o símbolo: a) L1, L2 e L3. d) L4, L5 e L6.
b) L2, L3 e L4. e) L4, L7 e L8.
c) L2, L5 e L7.

EXERCÍCIOS DE
APROFUNDAMENTO
Este símbolo orienta o consumidor sobre a necessidade de a 01. (UEMA) Poraquê é o nome popular do peixe elétrico da Amazônia.
máquina ser ligada a Seu nome vem da língua tupi e significa “o que coloca pra dormir”.
Ele é comparado com uma pilha e pode produzir descarga de até
a) um fio terra para evitar sobrecarga elétrica.
1500 volts, a denominação não podia ser mais apropriada.
b) um fio neutro para evitar sobrecarga elétrica.
c) um fio terra para aproveitar as cargas elétricas do solo.
d) uma rede de coleta de água da chuva.
e) uma rede de coleta de esgoto doméstico.

19. (ENEM) Todo carro possui uma caixa de fusíveis, que são
utilizados para proteção dos circuitos elétricos. Os fusíveis são
constituídos de um material de baixo ponto de fusão, como o
estanho, por exemplo, e se fundem quando percorridos por uma
corrente elétrica igual ou maior do que aquela que são capazes
de suportar. O quadro a seguir mostra uma série de fusíveis e os
valores de corrente por eles suportados.
Caso uma pessoa mergulhe a uma distância de 20 cm desse
peixe, considerando uma descarga elétrica de 1500 V, calcule a
Fusível Corrente Elétrica (A)
intensidade
Azul 1,5 a) do campo elétrico produzido por esse peixe para essa distância.
Amarelo 2,5 b) da corrente elétrica, considerando a resistência elétrica da
água desse local de 120 Ω.
Laranja 5,0
Preto 7,5 02. (PUCRJ) Um circuito elétrico é composto por um conjunto de
Vermelho 10,0 dois resistores de mesma resistência R e uma bateria regulável V.
Ao medirmos a corrente no circuito em função da tensão aplicada,
Um farol usa uma lâmpada de gás halogênio de 55 W de potência obtemos a curva apresentada na figura abaixo.
que opera com 36 V. Os dois faróis são ligados separadamente,
com um fusível para cada um, mas, após um mau funcionamento,
o motorista passou a conectá-los em paralelo, usando apenas um
fusível. Dessa forma, admitindo-se que a fiação suporte a carga
dos dois faróis, o menor valor de fusível adequado para proteção
desse novo circuito é o
a) azul. d) amarelo.
b) preto. e) vermelho.
c) laranja.

20. (ENEM) Considere a seguinte situação hipotética: ao preparar


o palco para a apresentação de uma peça de teatro, o iluminador
deveria colocar três atores sob luzes que tinham igual brilho e
os demais, sob luzes de menor brilho. O iluminador determinou,
então, aos técnicos, que instalassem no palco oito lâmpadas
incandescentes com a mesma especificação (L1 a L8), interligadas
em um circuito com uma bateria, conforme mostra a figura.

PRÉ-VESTIBULAR PROENEM.COM.BR 7
FÍSICA II 22 CIRCUITOS ELÉTRICOS - REVISÃO

a) A partir do gráfico, determine a resistência equivalente do circuito. 05. (UNIFESP) Para compor sua decoração de Natal, um
b) Sabendo que, nesse circuito, as resistências estão em série, comerciante decide construir uma estrela para pendurar na
determine qual seria a corrente em um circuito, cuja tensão fachada de sua loja. Para isso, utilizará um material que, quando
aplicada fosse de 12 V, conectado a essas resistências percorrido por corrente elétrica, brilhe emitindo luz colorida. Ele tem
colocadas em paralelo. à sua disposição barras de diferentes cores desse material, cada
uma com resistência elétrica constante R = 20 Ω.
03. (UFJF-PISM 3) Durante uma aula de projetos elétricos,
o professor pediu que os alunos construíssem um circuito
elétrico como mostrado abaixo. Os resistores R1, R2, R3 e R4 têm
resistências iguais a 2,0 Ω, 4,0 Ω, 5,0 Ω e 7,0 Ω, respectivamente. Utilizando dez dessas barras, ele montou uma estrela e conectou
O circuito é alimentado por uma bateria de 6,0 V com resistência os pontos A e B a um gerador ideal de força eletromotriz constante
interna desprezível. e igual a 120 V.

a) Qual a corrente total que atravessa esse circuito? Justifique


sua resposta.
b) Qual a diferença de potencial entre as extremidades do resistor
R3? Justifique sua resposta.

04. (UFU) Uma pessoa pretende montar um circuito elétrico,


conforme o esquematizado a seguir:

Considerando desprezíveis as resistências elétricas dos fios


utilizados e das conexões feitas, calcule:
a) a resistência equivalente, em ohms, da estrela.
b) a potência elétrica, em watts, dissipada em conjunto pelas
pontas de cores laranja (CAD), azul (DEF) e vermelha (FBG) da
estrela, quando ela se encontrar acesa.

GABARITO

EXERCÍCIOS PROPOSTOS
01. C 05. E 09. A 13. E 17.A
02. E 06. A 10. C 14. A 18. A
Nele, essa pessoa irá instalar um fusível (F), que interrompe a 03. D 07. C 11. B 15. C 19. C
passagem de corrente pelo circuito, caso ela seja superior a 0,6 04. A 08. C 12. C 16. C 20. B
A. Para tal montagem, ele dispõe de dois cilindros condutores, de
EXERCÍCIOS DE APROFUNDAMENTO
material e dimensão distintos, conforme as especificações a seguir:
01. a) 7500 V/m
b) 12,5 A
02. a) 4 Ω
b) 12 A
03. a) 5 A
b) 2,5 V
04. a) R1 = 6 Ω; R2 = 2 Ω
b) Deve ser usado o cilindro condutor (1) por ter a maior resistência, deixa passar menos
corrente elétrica, permitindo que a corrente não ultrapasse o valor limite de 0,6 A.
05. a) 48 Ω
A resistividade elétrica do material (1) é 3 x 10-5 Ω·m e a do material b) 220 W
(2) é 8 x 10-5 Ω·m e “A” representa a área da secção reta de cada
cilindro condutor.
ANOTAÇÕES
a) Com base nas especificações indicadas, qual a resistência
elétrica de cada um dos cilindros condutores?
b) Considerando desprezível a resistência dos demais fios
indicados no circuito, exceto a dos cilindros condutores, qual
deles (1 ou 2) deve ser empregado no referido circuito, de tal
modo que o fusível não interrompa a passagem da corrente
elétrica gerada?

8 PROENEM.COM.BR PRÉ-VESTIBULAR
23 MAGNETISMO: IMÃS FÍSICA II

E EXPERIMENTO DE OERSTED

ÍMÃS Ao aproximar um material ferromagnético, como, por exemplo o


ferro, níquel ou cobalto, de um ímã este passa a exibir propriedades
A palavra magnetismo é oriunda de Magnésia, que era o nome de ímã de forma temporária, diz-se que este metal está imantado
de uma região da Grécia antiga onde eram facilmente encontrados artificialmente.
imãs naturais. Tales de Mileto, matemático e filósofo que viveu no
século VI a.C., afirmava que a substância tinha “alma” e podia atrair





pedaços de matéria inanimada, “aspirando-os”, para os antigos




pensadores as substâncias tinham vontades e desejos como se
fossem seres vivos.     
 S 
   N
Em 1820, o professor de física da Universidade de Copenhague, S N
  
Hans Christian Oersted (1777-1851) provou de forma experimental sul norte
induzido induzido
uma relação entre fenômenos elétricos e magnéticos, desde
então ficou comprovado que tanto ímãs naturais quanto correntes Indução do prego pelo ímã natural
elétricas geravam campo magnético, daí o nome eletromagnetismo.
Os ímãs naturais ou magnetos são corpos caracterizados pela
propriedade de atrair certos materiais além de interagirem entre si.
Essas propriedades magnéticas são possuídas por pedras naturais
que contêm um minério denominado magnetita.
Hoje em dia, por um processo chamado de imantação, já é
possível criar um ímã artificial.

POLOS DE UM ÍMÃ
Quando aproximamos limalhas de ferro de um ímã, vemos
uma concentração maior de limalhas em certas regiões. Todo ímã
possui regiões em que as ações magnéticas são mais acentuadas
e essas regiões recebem o nome de polos magnéticos. Todo ímã
possui dois polos: um polo norte (N) e um polo sul (S).

LEIS DAS AÇÕES MAGNÉTICAS ÍMÃS ELEMENTARES OU


Polos de mesmo nome se repelem e polos de nomes diferentes
se atraem na razão inversa do quadrado da distância que os separa. DOMÍNIOS MAGNÉTICOS
Apelando para o modelo clássico do átomo, podemos supor
que o movimento do elétron em torno do núcleo atômico se
comporta como uma pequena corrente elétrica infinitesimal.
Graças à essa corrente é gerado um campo magnético elementar
que respeita uma certa orientação. Em um ímã, essa orientação
possui uma direção privilegiada, tornando a peça imantada.

INSEPARABILIDADE DOS
POLOS DE UM ÍMÃ
Devido a inúmeras experiências, os físicos concluíram que é
impossível separar os dois polos de um ímã. Portanto não existe a
carga magnética (monopolo), quando se quebra um ímã ao meio, Elétron em torno do núcleo.
na verdade, nós passamos a ter dois novos ímãs, cada um com um
polo Norte e um polo Sul.

Ímãs elementares de uma peça imantada.

PRÉ-VESTIBULAR PROENEM.COM.BR 243


FÍSICA II 23 MAGNETISMO: IMÃS E EXPERIMENTO DE OERSTED

MAGNETISMO NA SUBSTÂNCIA
FERROMAGNÉTICA: substâncias ferromagnéticas sofrem uma
forte atração do campo magnético, independente do polo, seus
ímãs elementares se alinham e o corpo passa a ter propriedades
magnéticas. Como exemplo: ferro, níquel, cobalto e etc.
PARAMAGNÉTICA: sofrem uma pequena atração do campo, O professor segura o parafuso e gira o imã, de modo que o lado
seus ímãs elementares se alinham com dificuldade e facilmente A fique próximo do parafuso e pergunta para os alunos se será
retornam à condição de desordem. Como exemplo: Alumínio, observado novamente uma atração ou se ocorrerá uma repulsão.
Náilon, Magnésio e etc. Usando o conhecimento aprendido no módulo responda à pergunta
DIAMAGNÉTICA: sofrem uma pequena repulsão do campo do professor, justifique sua resposta.
magnético, seus ímãs elementares se alinham no sentido oposto
ao das linhas de indução. Como exemplo: Ouro, Prata, Cobre e etc. 04. Um estudante dispõe de um imã de polaridades conhecidas,
pintado de vermelho a região Norte e de azul a região Sul.

EXPERIÊNCIA DE OERSTED
Após várias experiências, o cientista dinamarquês Hans Christian
Oersted, constatou que uma corrente elétrica é capaz de gerar um
campo magnético circundante. A direção e o sentido do campo
magnético podem ser determinados pela regra da mão direita.

Ao quebrar o imã no meio acredita ter separado os polos, entretanto,


ao fazer experimentos suas constatações o surpreendem.
Explique se é possível que o aluno tenha conseguido criar
monopolos magnéticos.

05. Em um laboratório aproxima-se um imã de uma esfera em dois


momentos.
a) b)

EXERCÍCIOS
PROTREINO
01. Três imãs estão representados abaixo:

Nos dois casos a esfera foi levemente repelida pelo imã, de qual
material é feita a esfera?
Considere as informações a seguir:
• A é um polo norte;
• D é atraído por B;
• E é repelido por D. EXERCÍCIOS
A partir das informações determine a polaridade das regiões PROPOSTOS
representadas pelas letras B, C, D, E e F.
01. (CPS) Pela primeira vez, cientistas detectaram a presença de
02. Três objetos estão representados abaixo: partículas de poluição que interferem no funcionamento do cérebro,
podendo inclusive ser uma das causas de Alzheimer. A conexão
entre esses materiais e o mal de Alzheimer ainda não é conclusiva.
Um desses materiais poluentes encontrados no cérebro é a
magnetita, um óxido de ferro que constitui um ímã natural.
<http://tinyurl.com/hzvm3fh> Acesso em: 30.09.16. Adaptado.
Considere as informações a seguir:
• AB é um imã e a região A representa um polo norte; Sobre o óxido citado no texto, é correto afirmar que ele apresenta
• A região E é fortemente repelida pela região A; a) dois polos magnéticos: norte e sul, e ambos atraem o ferro.
• A região C é fortemente atraída pela região B; b) dois polos magnéticos: norte e sul, mas apenas o polo sul atrai
• A região C é fortemente atraída pela região E. o ferro.
A partir das informações determine se CD e EF são imãs e em que c) dois polos magnéticos: norte e sul, mas apenas o polo norte
região se encontram os polos Norte e Sul. atrai o ferro.
d) quatro polos magnéticos: norte, sul, leste e oeste, e todos
03. Um professor aproxima o lado B de um imã e um parafuso atraem o ferro.
de ferro, os alunos presentes no momento podem observar uma e) quatro polos magnéticos: norte, sul, leste e oeste, mas apenas
atração entre os objetos. o norte e o sul atraem o ferro.

244 PROENEM.COM.BR PRÉ-VESTIBULAR


23 MAGNETISMO: IMÃS E EXPERIMENTO DE OERSTED FÍSICA II

02. (CPS) Ímãs podem ser utilizados em muitas brincadeiras. Não


é a toa que há uma série de brinquedos em que figuras planas
ou tridimensionais podem ser montadas utilizando-se ímãs. Um
desses brinquedos consiste em uma grande quantidade de ímãs
em formato de bastão.
A figura 1 mostra o perfil de um desses ímãs sendo que a parte
escurecida corresponde ao polo Norte, enquanto a parte em branco
corresponde ao polo Sul. O vagão foi colocado inicialmente em repouso e no meio de uma
caixa de papelão de comprimento maior, porém de largura muito
próxima à da caixa de creme dental. Na caixa de papelão também
foram colados ímãs permanentes idênticos aos do vagão.
Admitindo-se que não haja atrito entre as laterais da caixa de
Carlos vai dispor alguns ímãs de acordo com a figura 2, de modo creme dental, em que se desenhou o vagão, e a caixa de papelão,
que eles fiquem unidos apenas pela ação da força magnética, sem para se obter o efeito de levitação e ainda um pequeno movimento
a ação de atritos ou outras forças. horizontal do vagão sempre para a esquerda, em relação à figura
desenhada, a disposição dos ímãs permanentes, no interior da
caixa de papelão, deve ser a que se encontra representada em:
a)

Assinale a alternativa que apresenta corretamente uma


possibilidade de arranjo dos ímãs para que Carlos consiga montar
a disposição apresentada na figura 2. b)
a)

b)
c)

c)

d)

d)

e)
e)

03. (CPS) O Maglev é uma espécie de trem sem rodas que possui
eletroímãs em sua base, e há também eletroímãs no trilho que ele
04. (PUCCAMP) Para que se possa efetuar a reciclagem do lixo,
percorre. As polaridades desses eletroímãs são controladas por
antes é necessário separá-lo. Uma dessas etapas, quando não se
computador, e esse controle permite que o trem levite sobre o trilho
faz a coleta seletiva, é colocar o lixo sobre uma esteira, para que
bem como seja movido para frente ou para trás.
passe, por exemplo, por um imã. Esse processo permite que sejam
Para demonstrar o princípio do funcionamento do Maglev, um separados materiais magnéticos, como o metal
estudante desenhou um vagão de trem em uma caixa de creme
a) alumínio. d) zinco.
dental e colou em posições especiais ímãs permanentes, conforme
a figura. b) ferro. e) magnésio.
c) cobre.

PRÉ-VESTIBULAR PROENEM.COM.BR 245


FÍSICA II 23 MAGNETISMO: IMÃS E EXPERIMENTO DE OERSTED

05. (ENEM) A magnetohipertermia é um procedimento terapêutico barras QR, ST, UV e WX aparentemente idênticas. Verifica-se,
que se baseia na elevação da temperatura das células de uma região experimentalmente, que Q atrai T, repele U e atrai W; R repele V,
específica do corpo que estejam afetadas por um tumor. Nesse atrai T e atrai W.
tipo de tratamento, nanopartículas magnéticas são fagocitadas
pelas células tumorais, e um campo magnético alternado externo
é utilizado para promover a agitação das nanopartículas e
consequente aquecimento da célula.
Diante do exposto, assinale a alternativa correta.
A elevação de temperatura descrita ocorre porque
a) QR e ST são ímãs.
a) o campo magnético gerado pela oscilação das nanopartículas
é absorvido pelo tumor. b) QR e UV são ímãs.
b) o campo magnético alternado faz as nanopartículas girarem, c) RS e TU são ímãs.
transferindo calor por atrito. d) QR, ST e UV são ímãs.
c) as nanopartículas interagem magneticamente com as células e) As quatro barras são ímãs.
do corpo, transferindo calor.
d) o campo magnético alternado fornece calor para as 09. (IFSP) Os ímãs têm larga aplicação em nosso cotidiano tanto
nanopartículas que o transfere às células do corpo. com finalidades práticas, como em alto-falantes e microfones, ou
e) as nanopartículas são aceleradas em um único sentido em como meramente decorativas. A figura mostra dois ímãs, A e B, em
razão da interação com o campo magnético, fazendo-as colidir forma de barra, com seus respectivos polos magnéticos.
com as células e transferir calor.

06. (PUCMG) Um ímã permanente, em forma de “ferradura”, cujos


polos norte e sul estão indicados na figura a seguir, é dividido em
três partes.

Analise as seguintes afirmações sobre ímãs e suas propriedades


magnéticas.
I. Se quebrarmos os dois ímãs ao meio, obteremos quatro
pedaços de material sem propriedades magnéticas, pois
teremos separados os polos norte e sul um do outro.
II. A e B podem tanto atrair-se como repelir-se, dependendo da
posição em que os colocamos, um em relação ao outro.
É CORRETO concluir que: III. Se aproximarmos de um dos dois ímãs uma pequena esfera de
a) a parte 1 terá apenas o polo norte e a parte 2 terá apenas o ferro, ela será atraída por um dos polos desse ímã, mas será
polo sul. repelida pelo outro.
b) as partes 1 e 2 formarão novos ímãs, mas a parte 3 não. É correto o que se afirma em
c) as partes 1, 2 e 3 perderão suas propriedades magnéticas. a) I, apenas.
d) as partes 1, 2 e 3 formarão três novos ímãs, cada uma com b) II, apenas.
seus polos norte e sul. c) I e II, apenas.
d) I e III, apenas.
07. (IFSP) Dispõe-se de três ímãs em formato de barra, conforme e) II e III, apenas.
mostra a figura a seguir:
10. (IFSP) Um professor de Física mostra aos seus alunos 3 barras
de metal AB, CD e EF que podem ou não estar magnetizadas.
Com elas faz três experiências que consistem em aproximá-las e
observar o efeito de atração e/ou repulsão, registrando-o na tabela
a seguir.
Sabe-se que o polo A atrai o polo C e repele o polo E. Se o polo F é
sul, pode-se dizer que:
a) A é polo sul e B polo Sul.
b) A é polo sul e C é polo norte.
c) B é polo norte e D é polo norte.
d) A é polo norte e C é polo sul.
e) A é polo norte e E é polo sul.

08. (IFSP) No mundo, existe uma grande variedade de elementos


químicos metálicos, cujas propriedades físicas e químicas são Após o experimento e admitindo que cada letra pode corresponder
similares ou bastante distintas. Comumente, os metais são a um único polo magnético, seus alunos concluíram que
separados em dois grandes grupos: os ferrosos (compostos por a) somente a barra CD é ímã.
ferro) e os não ferrosos (ausência de ferro). O primeiro grupo é b) somente as barras CD e EF são ímãs.
considerado magnético, enquanto que o segundo não. Desta forma,
c) somente as barras AB e EF são ímãs.
uma maneira eficiente e rápida para fazer a separação destes
elementos é pela utilização de eletroímãs, que são dispositivos que d) somente as barras AB e CD são ímãs.
atraem apenas os metais ferromagnéticos. Considere as quatro e) AB, CD e EF são ímãs.

246 PROENEM.COM.BR PRÉ-VESTIBULAR


23 MAGNETISMO: IMÃS E EXPERIMENTO DE OERSTED FÍSICA II

11. (EEAR) Quanto à facilidade de imantação, podemos afirmar que: 15. (CPS) Uma das hipóteses, ainda não comprovada, sobre os
“Substâncias __________ são aquelas cujos ímãs elementares modos como se orientam os animais migratórios durante suas
se orientam em sentido contrário ao vetor indução magnética, longas viagens é a de que esses animais se guiam pelo campo
sendo, portanto, repelidas pelo ímã que criou o campo magnético”. magnético terrestre. Segundo essa hipótese, para que ocorra essa
O termo que preenche corretamente a lacuna é: orientação, esses animais devem possuir, no corpo, uma espécie
de ímã que, como na bússola, indica os polos magnéticos da Terra.
a) diamagnéticas c) ultramagnéticas
De acordo com a Física, se houvesse esse ímã que pudesse se
b) paramagnéticas d) ferromagnéticas movimentar como a agulha de uma bússola, orientando uma ave
que migrasse para o hemisfério sul do planeta, local em que se
12. (MACKENZIE) Considere as seguintes afirmações. encontra o polo norte magnético da Terra, esse ímã deveria
I. Quando se coloca um ímã em contato com limalha (fragmentos) a) possuir apenas um polo, o sul.
de ferro, estes não aderem a ele em toda a sua extensão, mas
b) possuir apenas um polo, o norte.
predominantemente nas regiões próximas das extremidades.
c) apontar seu polo sul para o destino.
II. Cortando-se um ímã em duas partes iguais, que por sua vez
podem ser redivididas em outras tantas, observa-se que cada d) apontar seu polo norte para o destino.
uma dessas partes constitui um novo ímã, que embora menor e) orientar-se segundo a linha do Equador.
tem sempre dois polos.
III. Polos de mesmo nome se atraem e de nomes diferentes se 16. (UERN) A agulha de uma bússola ao ser colocada entre dois
repelem. imãs sofre um giro no sentido anti-horário. A figura que ilustra
Com relação às afirmações, podemos dizer que corretamente a posição inicial da agulha em relação aos imãs é
a) apenas I é correta. a)
b) apenas I e II são corretas.
c) apenas I e III são corretas.
d) apenas II e III são corretas.
e) todas são corretas.
b)
13. (EEAR) Entre as substâncias magnéticas, aquelas que ao serem
colocadas próximas a um imã, cujo campo magnético é intenso,
são repelidas por ambos os polos do imã, são classificadas como
a) diamagnéticas. c) ferromagnéticas.
b) paramagnéticas. d) imãs permanentes.
c)
14. (UFU) Três carrinhos idênticos são colocados em um trilho,
porém, não se encostam, porque, na extremidade de cada um
deles, conforme mostra o esquema abaixo, é acoplado um ímã, de
tal forma que um de seus polos fica exposto para fora do carrinho
(polaridade externa).
d)

Considerando que as polaridades externas dos ímãs (N – norte e 17. (UFPEL) Considere um ímã permanente e uma barra de ferro
S – sul) nos carrinhos são representadas por números, conforme inicialmente não imantada, conforme a figura a seguir.
o esquema a seguir, assinale a alternativa que representa a ordem
correta em que os carrinhos foram organizados no trilho, de tal
forma que nenhum deles encoste no outro:

Ao aproximarmos a barra de ferro do ímã, observa-se a formação


de um polo ______ em A, um polo ______ em B e uma ______ entre o
ímã e a barra de ferro.
A alternativa que preenche respectiva e corretamente as lacunas
da afirmação anterior é
a) norte, sul, repulsão
b) sul, sul, repulsão.
c) sul, norte, atração.
a) 1 – 2 – 4 – 3 – 6 – 5. c) 3 – 4 – 6 – 5 – 2 – 1.
d) norte, sul, atração
b) 6 – 5 – 4 – 3 – 1 – 2. d) 2 – 1 – 6 – 5 – 3 – 4.
e) sul, norte, repulsão.

PRÉ-VESTIBULAR PROENEM.COM.BR 247


FÍSICA II 23 MAGNETISMO: IMÃS E EXPERIMENTO DE OERSTED

18. (PUCPR) Um pedaço de ferro é colocado próximo de um ímã,


conforme a figura a seguir: EXERCÍCIOS DE
APROFUNDAMENTO
01. (UNIFESP) Uma bonequinha está presa, por um ímã a ela
Assinale a alternativa correta: colado, à porta vertical de uma geladeira.
a) é o ferro que atrai o ímã. a) Desenhe esquematicamente essa bonequinha no caderno de
respostas, representando e nomeando as forças que atuam
b) a atração do ferro pelo ímã é igual à atração do ímã pelo ferro.
sobre ela.
c) é o ímã que atrai o ferro.
b) Sendo m = 20g a massa total da bonequinha com o ímã e
d) a atração do ímã pelo ferro é mais intensa do que a atração do μ = 0,50 o coeficiente de atrito estático entre o ímã e a porta da
ferro pelo ímã. geladeira, qual deve ser o menor valor da força magnética entre
e) a atração do ferro pelo ímã é mais intensa do que a atração do o ímã e a geladeira para que a bonequinha não caia? Dado:
ímã pelo ferro. g = 10m/s2.

19. (UFSCAR) Um menino encontrou três pequenas barras 02. (FUVEST) A figura esquematiza um ímã permanente, em forma
homogêneas e, brincando com elas, percebeu que, dependendo de cruz de pequena espessura, e oito pequenas bússolas, colocadas
da maneira como aproximava uma da outra, elas se atraiam ou sobre uma mesa. As letras N e S representam, respectivamente,
se repeliam. Marcou cada extremo das barras com uma letra e polos norte e sul do ímã e os círculos representam as bússolas
manteve as letras sempre voltadas para cima, conforme indicado nas quais você irá representar as agulhas magnéticas. O ímã é
na figura. simétrico em relação às retas NN e SS. Despreze os efeitos do
campo magnético terrestre.
a) Desenhe na própria figura algumas linhas de força que
permitam caracterizar a forma do campo magnético criado
pelo ímã, no plano da figura.
Passou, então, a fazer os seguintes testes: b) Desenhe nos oito círculos da figura a orientação da agulha da
I. aproximou o extremo B da barra 1 com o extremo C da barra 2 bússola em sua posição de equilíbrio. A agulha deve ser repre-
e percebeu que ocorreu atração entre elas; sentada por uma flecha (→) cuja ponta indica o seu polo norte.
II. aproximou o extremo B da barra 1 com o extremo E da barra 3
e percebeu que ocorreu repulsão entre elas;
III. aproximou o extremo D da barra 2 com o extremo E da barra 3
e percebeu que ocorreu atração entre elas.
Verificou, ainda, que nos casos em que ocorreu atração, as
barras ficaram perfeitamente alinhadas. Considerando que, em
cada extremo das barras representado por qualquer uma das
letras, possa existir um único polo magnético, o menino concluiu,
corretamente, que
a) as barras 1 e 2 estavam magnetizadas e a barra 3
desmagnetizada.
b) as barras 1 e 3 estavam magnetizadas e a barra 2
desmagnetizada.
c) as barras 2 e 3 estavam magnetizadas e a barra 1 03. (UFRJ) Um ímã permanente cai por ação da gravidade através
desmagnetizada. de uma espira condutora circular fixa, mantida na posição horizontal,
d) as barras 1, 2 e 3 estavam magnetizadas. como mostra a figura. O polo norte do ímã esta dirigido para baixo e
e) necessitaria de mais um único teste para concluir sobre a a trajetória do ímã é vertical e passa pelo centro da espira.
magnetização das três barras.

20. (FATEC) Dispõe-se de três barras, idênticas nas suas geome-


trias, x, y e z, e suas extremidades são nomeadas por x1, x2, y1, y2,
z1 e z2.

Aproximando-se as extremidades, verifica-se que x2 e y2 se


repelem; x1 e z1 se atraem; y1 e z2 se atraem e x1 e y2 se atraem. É
correto concluir que somente
a) x e y são ímãs permanentes.
Use a lei de Faraday e mostre por meio de diagramas e justifique
b) x e z são ímãs permanentes. suas respostas:
c) x é ímã permanente. a) o sentido da corrente induzida na espira no momento ilustrado
d) y é ímã permanente. na figura;
e) z é ímã permanente. b) a direção e o sentido da força resultante exercida sobre o ímã.

248 PROENEM.COM.BR PRÉ-VESTIBULAR


23 MAGNETISMO: IMÃS E EXPERIMENTO DE OERSTED FÍSICA II

04. (UEMA) Um professor de física, para construir um eletroímã, 02.


montou um circuito com as seguintes características: valor da a) b)
resistência R = 15 Ω, solenoide com 8π x 10-2 m de comprimento,
5.000 espiras e resistência r = 85 Ω, conforme ilustrado:

03. a)

b) O campo gerado pela corrente induzida na espira comporta-se como um ímã cujo polo
norte está dirigido para cima tendendo a repelir verticalmente o ímã. Portanto, a força
resultante é vertical para baixo e tem módulo menor do que o peso do ímã.
Determine o módulo do vetor indução magnética no interior do solenoide 04. 0,015 T
quando a d.d.p. for de 60 V, considerando µ0 = 4π x 10-7 T·m/A. 05.

05. (UFPR) Em 1820, Hans Cristian Oersted aproximou de uma bússola


um fio condutor percorrido por uma corrente elétrica e não observou
qualquer alteração na direção da agulha dessa bússola. Mais tarde, ao
refazer o experimento, porém agora com o fio condutor posicionado
em outra direção, ele constatou que ocorria uma alteração na direção
da agulha da bússola. Essa experiência histórica fez a conexão entre - Na fig. 1, a corrente está entrando. O sentido das linhas de indução magnética é horário,
a eletricidade e o magnetismo, criando o que nós conhecemos hoje desviando a extremidade imantada da agulha no sentido leste.
por eletromagnetismo. Suponha uma bússola posicionada sobre esta - Na fig. 2, a corrente está saindo. O sentido das linhas de indução magnética é anti-
folha de papel com sua agulha apontando para a parte superior da horário, desviando a extremidade imantada da agulha no sentido oeste.
folha, o que corresponde à direção norte.
Utilizando a figura a seguir, desenhe a direção em que deverá ser ANOTAÇÕES
posicionado o fio condutor, passando exatamente sobre o centro
da bússola, para que se obtenha o maior desvio possível da sua
agulha. Escolha um sentido para a corrente no fio, marcando-o
com uma seta na figura. Indique na figura para qual lado ocorrerá
esse desvio, se para leste ou para oeste, de modo compatível com
o sentido da corrente escolhido. Justifique suas respostas.

GABARITO
EXERCÍCIOS PROPOSTOS
01. A 05. B 09. B 13. A 17. C
02. B 06. D 10. B 14. D 18. B
03. A 07. D 11. A 15. C 19. B
04. B 08. B 12. B 16. C 20. A

EXERCÍCIOS DE APROFUNDAMENTO
01. a)

b) 0,40 N

PRÉ-VESTIBULAR PROENEM.COM.BR 249


FÍSICA II 23 MAGNETISMO: IMÃS E EXPERIMENTO DE OERSTED

ANOTAÇÕES

250 PROENEM.COM.BR PRÉ-VESTIBULAR


24
FÍSICA II
CAMPO MAGNÉTICO

Como vimos, o ímã possui a propriedade de atrair corpos


ferromagnéticos (ferro, níquel, cobalto etc.), portanto, podemos afirmar
que um ímã exerce uma influência na região ao seu redor. Nesse caso,
dizemos que o ímã cria um campo magnético nessa região.
O campo magnético, como se trata de uma grandeza
vetorial, será representado por um vetor, denominado vetor
indução magnética e simbolizado por B, sua unidade no Sistema
Internacional (S.I.) é o tesla (T).

LINHAS DE INDUÇÃO DE UM ÍMÃ


Para representar o campo magnético em todos os pontos ao PROEXPLICA
redor de um ímã são utilizadas linhas de indução. O vetor campo
magnético (B) é tangente às linhas de indução e seu módulo será
Representações do Campo Magnético
mais intenso com o aumento da densidade dessas linhas.

Nesta representação vemos as Nesta representação vemos os


extremidades das setas do vetor pontos de aplicação das setas do
campo. O vetor aponta para o leitor vetor campo. O vetor aponta para
e forma um ângulo de 90º com o dentro do papel e forma um ângulo
plano do papel. de 90º com o plano do papel

Por convenção, as linhas são orientadas “saindo” do polo norte


e “entrando” no polo sul na região externa ao imã. CÁLCULO DO CAMPO MAGNÉTICO
Para se obter o vetor campo magnético em qualquer ponto
representado por uma linha de indução, basta traçar uma tangente,
respeitando sempre o sentido da linha de indução.
FIO RETILÍNEO
Vamos considerar um fio retilíneo a analisar regiões próximas
do fio e afastadas de suas extremidades. Ao passar uma corrente
elétrica de intensidade i, observamos linhas de indução do campo
magnético sendo geradas, essas linhas são circunferências
dispostas em planos perpendiculares ao condutor, com centros
neste, cujos sentidos são dados pela regra da mão direita
envolvente. Para aplicar essa regra, “segure” o fio com a mão
direita, e alinhe seu polegar de forma que a ponta do dedo fique
no mesmo sentido da corrente elétrica i, os outros dedos darão,
automaticamente, o sentido das linhas de indução.

Veja as características das linhas de indução.


1. São sempre linhas fechadas: saem e voltam a um mesmo
ponto.
2. As linhas nunca se cruzam.
3. Região externa do ímã, as linhas saem do polo norte e se
dirigem para o polo sul.
4. Região interna do ímã, as linhas são orientadas do polo sul
para o polo norte.
5. Saem e entram na direção perpendicular às superfícies dos
polos.
6. Nos polos, a concentração das linhas é maior: quanto
maior concentração de linhas, mais intensa será a indução
magnética numa dada região.

PRÉ-VESTIBULAR PROENEM.COM.BR 1
FÍSICA II 24 CAMPO MAGNÉTICO

CAMPO MAGNÉTICO
TERRESTRE E BÚSSOLA


i A Terra funciona como um gigantesco ímã (consequência das
 
correntes elétricas existentes na parte central da Terra, constituída
B B
. . x x
de ferro fundido), porém, há uma diferença em relação ao nome dos
polos. Próximo ao polo norte geográfico existe o polo sul magnético
e próximo ao polo sul geográfico existe o polo norte magnético. Por
isso, o polo norte de uma bússola (ímã especialmente utilizado para
orientação) sempre aponta para norte geográfico (sul magnético).
Polos com nomes diferentes se atraem. Essas diferenças ocorrem
devido a convenções (geográficas e magnéticas) distintas adotadas.

Já que um fio percorrido por uma corrente elétrica pode gerar


um campo magnético à sua volta, podemos supor facilmente que
quanto maior for essa corrente, maior será esse campo magnético,
porém seu módulo diminui para pontos mais afastados.

PROEXPLICA
ESPIRA
Cinturão de Van Allen
Agora coloquemos um fio, percorrido por uma corrente elétrica,
em formato de circunferência. O cálculo do módulo do campo O campo magnético terrestre gera uma proteção das
magnético no centro da Espira será dado por: radiações provindas do espaço. Partículas de alta energia
são deflexionadas, ou capturadas pelos dois cinturões
que blindam a Terra contra as altas energias provindas do
Cosmos e do Sol. Os íons de hélio trazidos pelo vento solar
ficam sob influência do campo magnético terrestre e entram
em movimento de espiral freneticamente, entre as regiões de
campo magnético intenso próximo aos polos magnéticos do
ímã Terra. Estas partículas são responsáveis pelos chamados
cinturões de Van Allen que envolvem a Terra, causando o
belíssimo efeito visual da aurora boreal (norte) e austral (sul).

µ ⋅i
B=
2 ⋅r

SOLENOIDE
Um enrolamento de fios percorridos por uma corrente elétrica.
Para um solenoide infinito, o campo magnético externo é nulo, só
há campo magnético no interior e seu módulo é calculado por:

µ ⋅n⋅i
B=
L

O módulo do campo será proporcional ao número de espiras do


enrolamento (número n de voltas dadas pelo fio).

2 PROENEM.COM.BR PRÉ-VESTIBULAR
24 CAMPO MAGNÉTICO FÍSICA II

04. Uma espira circular de raio R = 1,5 πm paralela ao plano do


EXERCÍCIOS papel é percorrida por uma corrente elétrica de intensidade de 12A,
PROTREINO conforme esquema abaixo:

Para as questões Protreino, adote: µ0 = 4·π·10-7 Tm/A

01. Num desenho fora de escala uma linha de indução magnética


é representada num imã.

Determine a direção, o sentido e a intensidade do campo magnético


no centro da espira, representado pelo ponto P.

05. Um circuito é composto por um gerador ideal, dois resistores


ôhmicos e um solenoide de 50 espiras e 20 cm de comprimento,
Determine o lado Norte e Sul do imã representado acima. conforme esquema abaixo:

02. Um fio retilíneo, muito longo, é atravessado por uma corrente


de intensidade 8A.

Determine a intensidade do campo magnético no interior do


solenoide. Adote: π = 3.

EXERCÍCIOS
PROPOSTOS
Calcule a intensidade do campo magnético no ponto P. 01. (ESPCEX (AMAN)) Duas espiras circulares, concêntricas
e coplanares de raios R1 = 2π m e R2 = 4π m são percorridas,
03. Dois fios condutores retilíneos, muito longos e paralelos entre respectivamente, por correntes de intensidades i1 = 6 A e i2 = 8 A,
si, são percorridos por correntes elétricas de intensidade distintas, conforme mostra o desenho.
conforme representado na imagem abaixo:

Calcule o sentido e a intensidade do campo magnético resultante


no ponto P.

PRÉ-VESTIBULAR PROENEM.COM.BR 3
FÍSICA II 24 CAMPO MAGNÉTICO

A intensidade (módulo) do vetor indução magnética no centro das a) perpendicular às linhas de indução do campo magnético
espiras “O” é da Terra e ao fato de o polo norte magnético terrestre estar
Dado: o meio é o vácuo e a permeabilidade magnética do vácuo próximo ao polo sul geográfico da Terra.
T ⋅m b) tangente à Linha do Equador e ao fato de o eixo de rotação da
µ0 = 4 π ⋅ 10−7 Terra coincidir com o eixo magnético que atravessa a Terra.
A
c) tangente às linhas de indução do campo magnético da Terra
a) 2·10-7 T. d) 8·10-7 T.
e ao fato de o polo norte magnético terrestre estar próximo ao
b) 3·10-7 T. e) 9·10-7 T. polo norte geográfico da Terra.
c) 6·10 T.
-7
d) tangente às linhas de indução do campo magnético da Terra
e ao fato de o polo norte magnético terrestre estar próximo ao
02. (UNESP) A configuração do campo magnético terrestre causa polo sul geográfico da Terra.
um efeito chamado inclinação magnética. Devido a esse fato, a e) paralela ao eixo magnético terrestre e ao fato de o polo sul
agulha magnética de uma bússola próxima à superfície terrestre, magnético terrestre estar próximo ao polo norte geográfico da
se estiver livre, não se mantém na horizontal, mas geralmente Terra.
inclinada em relação à horizontal (ângulo á, na figura 2). A
inclinação magnética é mais acentuada em regiões de maiores
03. (UECE) O módulo do vetor campo magnético gerado por uma
latitudes. Assim, no equador terrestre a inclinação magnética fica
corrente elétrica constante passando por um fio retilíneo depende
em torno de 0°, nos polos magnéticos é de 90°, em São Paulo é de
da distância do ponto de medição do campo ao fio. Assim, é correto
cerca de 20°, com o polo norte da bússola apontado para cima,
afirmar que a direção desse vetor é
e em Londres é de cerca de 70°, com o polo norte da bússola
apontado para baixo. a) perpendicular ao fio somente para um dos sentidos da corrente.
b) perpendicular ao fio independente do sentido da corrente.
c) paralela ao fio independente do sentido da corrente.
d) paralela ao fio somente para um dos sentidos da corrente.

04. (EEAR) Uma espira circular com 10π cm de diâmetro, ao ser


percorrida por uma corrente elétrica de 500 mA de intensidade,
produz no seu centro um vetor campo magnético de intensidade
igual a ______ · 10-6 T. Obs. Utilize µ0 = 4π·10-7 t·m/A
a) 1 c) 4
b) 2 d) 5

05. (ESPCEX (AMAN)) Dois fios condutores retilíneos, muito longos


e paralelos entre si, são percorridos por correntes elétricas de
intensidade distintas, i1 e i2, de sentidos opostos.
Uma espira circular condutora de raio R é colocada entre os dois
fios e é percorrida por uma corrente elétrica i.
A espira e os fios estão no mesmo plano. O centro da espira dista
de 3R de cada fio, conforme o desenho abaixo.

Para que o vetor campo magnético resultante, no centro da espira,


seja nulo, a intensidade da corrente elétrica i e seu sentido, tomando
como referência o desenho, são respectivamente:
i +i i +i
a) 1 2 e horário d) 1 2 e horário
3 3π
i1 − i2
b) e anti-horário e) i1 + i2 e anti-horário
3π 3π
i1 − i2
Esse efeito deve-se ao fato de a agulha magnética da bússola c) e horário

alinhar-se sempre na direção

4 PROENEM.COM.BR PRÉ-VESTIBULAR
24 CAMPO MAGNÉTICO FÍSICA II

06. (FUVEST) As figuras representam arranjos de fios longos, a) Diâmetro do fio condutor.
retilíneos, paralelos e percorridos por correntes elétricas de mesma b) Distância entre as espiras.
intensidade. Os fios estão orientados perpendicularmente ao plano
desta página e dispostos segundo os vértices de um quadrado. A c) Densidade linear de espiras.
única diferença entre os arranjos está no sentido das correntes: d) Corrente que circula pelo fio.
os fios são percorridos por correntes que entram ⊗ ou saem  do e) Permeabilidade relativa do núcleo.
plano da página.
10. (UNESP) Um ímã em forma de barra, com seus polos Norte
e Sul, é colocado sob uma superfície coberta com partículas de
limalha de ferro, fazendo com que elas se alinhem segundo seu
campo magnético. Se quatro pequenas bússolas, 1, 2, 3 e 4, forem
colocadas em repouso nas posições indicadas na figura, no mesmo
plano que contém a limalha, suas agulhas magnéticas orientam-se
segundo as linhas do campo magnético criado pelo ímã.
O campo magnético total é nulo no centro do quadrado apenas em
a) I.
b) II.
c) I e II.
d) II e III.
e) III e IV.

07. (UECE) No caso hipotético de uma corrente elétrica por um


condutor retilíneo, há geração de um campo magnético
a) na mesma direção do condutor.
b) que aumenta proporcionalmente à distância do condutor.
c) que é constante e uniforme em torno da direção do condutor.
d) em direções perpendiculares à do condutor.

Desconsiderando o campo magnético terrestre e considerando que


08. (EEAR) Um fio condutor é percorrido por uma corrente i como
a agulha magnética de cada bússola seja representada por uma
mostra a figura.
seta que se orienta na mesma direção e no mesmo sentido do vetor
campo magnético associado ao ponto em que ela foi colocada,
assinale a alternativa que indica, correta e respectivamente, as
configurações das agulhas das bússolas 1, 2, 3 e 4 na situação
descrita.
a)

b)

Próximo ao condutor existe um ponto P, também representado na c)


figura. A opção que melhor representa o vetor campo magnético
no ponto P é:
a) c) d)

e)

b) d)

11. (UECE) Em um experimento A, sobre eletromagnetismo, um


fio condutor muito fino é disposto em linha reta sobre uma mesa
isolante horizontal. Pelo fio passa uma corrente elétrica constante.
Em um segundo experimento, B, o mesmo fio é disposto na forma
09. (ENEM - LIBRAS) Um guindaste eletromagnético de um
de uma circunferência também sobre a mesa. Em ambas as
ferro-velho é capaz de levantar toneladas de sucata, dependendo
situações o fio está contido no plano da mesa.
da intensidade da indução em seu eletroímã. O eletroímã é um
dispositivo que utiliza corrente elétrica para gerar um campo É correto afirmar que, no plano da mesa, os campos magnéticos
magnético, sendo geralmente construído enrolando-se um fio produzidos pela corrente elétrica nos dois experimentos são
condutor ao redor de um núcleo de material ferromagnético (ferro, a) verticais.
aço, níquel, cobalto).
b) horizontais.
Para aumentar a capacidade de carga do guindaste, qual
c) vertical e horizontal, respectivamente.
característica do eletroímã pode ser reduzida?
d) horizontal e vertical, respectivamente.

PRÉ-VESTIBULAR PROENEM.COM.BR 5
FÍSICA II 24 CAMPO MAGNÉTICO

12. (PUCSP) A figura representa dois fios condutores retilíneos e Dado: Permeabilidade magnética do vácuo: µ0 = 4π · 10-7 T·m/A
muito compridos, paralelos e percorridos por correntes elétricas de a) 3,0 · 10-7 T d) 7,5 · 10-7 T
mesma intensidade (iF), porém, de sentidos contrários. Entre os fios
há uma espira circular de raio R percorrida por uma corrente elétrica b) 4,5 · 10 T
-7
e) 8,0 · 10-7 T
c) 6,5 · 10 T
-7
iF
de intensidade (iE). Determine a razão e o sentido da corrente
iE
15. (UNESP)
elétrica na espira circular para que o campo de indução magnética A bússola interior
resultante no centro da espira seja nulo. Os fios condutores e a
espira circular estão situados no mesmo plano. A comunidade científica, hoje, admite que certos animais
detectam e respondem a campos magnéticos. No caso das trutas
arco-íris, por exemplo, as células sensoriais que cobrem a abertura
nasal desses peixes apresentam feixes de magnetita que, por sua
vez, respondem a mudanças na direção do campo magnético da
Terra em relação à cabeça do peixe, abrindo canais nas membranas
celulares e permitindo, assim, a passagem de íons; esses íons,
a seu turno, induzem os neurônios a enviarem mensagens ao
cérebro para qual lado o peixe deve nadar. As figuras demonstram
esse processo nas trutas arco-íris:

a) π e o sentido da corrente na espira deve ser anti-horário.


b) π e o sentido da corrente na espira deve ser horário.
c) 1,5π e o sentido da corrente na espira deve ser horário.
d) 1,5π e o sentido da corrente na espira deve ser anti-horário.

13. (UDESC) Considere um longo solenoide ideal composto por


Na situação da figura 2, para que os feixes de magnetita voltem a se
10.000 espiras por metro, percorrido por uma corrente contínua
orientar como representado na figura 1, seria necessário submeter
de 0,2 A. O módulo e as linhas de campo magnético no interior do
as trutas arco-íris a um outro campo magnético, simultâneo ao da
solenoide ideal são, respectivamente:
Terra, melhor representado pelo vetor
a) Nulo, inexistentes.
a) c) e)
b) 8π x 10-4 T, circunferências concêntricas.
c) 4π x 10-4 T, hélices cilíndricas.
d) 8π x 10-3 T, radiais com origem no eixo do solenoide.
e) 8π x 10-4 T, retas paralelas ao eixo do solenoide. b)

d)
14. (ESPCEX (AMAN)) Dois fios “A” e “B” retos, paralelos e extensos,
estão separados por uma distância de 2 m. Uma espira circular de
raio igual a π/4 m encontra-se com seu centro “O” a uma distância
de 2 m do fio “B”, conforme desenho abaixo. 16. (COTUCA) Podemos dizer que a Terra e a agulha de uma
bússola comportam-se como barras de ímãs, que possuem polos
magnéticos Norte e Sul. Na figura a seguir, o Polo Norte magnético
da agulha da bússola aponta de forma aproximada para o Polo
Norte geográfico terrestre.

A espira e os fios são coplanares e se encontram no vácuo. Os fios


“A” e “B” e a espira são percorridos por correntes elétricas de mesma
intensidade i = 1 A com os sentidos representados no desenho. A
intensidade do vetor indução magnética resultante originado pelas
três correntes no centro “O” da espira é:

6 PROENEM.COM.BR PRÉ-VESTIBULAR
24 CAMPO MAGNÉTICO FÍSICA II

Desta forma, podemos concluir que: 18. (FAMERP) Três ímãs idênticos, em forma de barra, estão
a) o Polo Sul magnético da agulha da bússola aponta para o Polo dispostos com uma de suas extremidades equidistantes de um
Norte geográfico terrestre, aproximadamente. ponto P, como mostra a figura.
b) o Polo Sul magnético da agulha da bússola é atraído pelo Polo
Sul magnético da Terra.
c) o Polo Sul magnético da agulha da bússola aponta para o Polo
Sul geográfico terrestre, aproximadamente.
d) o Polo Norte magnético da agulha da bússola aponta para o
Polo Sul geográfico terrestre, aproximadamente.
e) os Polos magnéticos da agulha são atraídos pelos Polos
magnéticos terrestres de mesma denominação.

17. (CPS) Leia as afirmações e a imagem.


- Todo ímã possui dois polos magnéticos, conhecidos como norte
e sul.
- Nos ímãs, as linhas de campo magnético saem do polo norte,
circundam externamente o ímã e entram no polo sul.
- Quando dois ímãs são aproximados, os polos de nomes diferentes
se atraem enquanto que os polos de nomes iguais se repelem.
- A agulha de uma bússola é um ímã. A ponta dessa agulha O campo de indução magnética resultante da ação dos três ímãs
corresponde ao polo norte desse ímã. no ponto P é representado pelo vetor
- O interior do planeta Terra pode ser comparado a um grande ímã, a)
mantendo um forte campo magnético em torno do planeta.
- O Polo Norte Geográfico se encontra no hemisfério do planeta
que abriga o Polo Sul Magnético. Do mesmo modo, o Polo Sul
Geográfico se encontra no hemisfério do planeta que abriga o Polo b)
Norte Magnético. c)

d)

e) nulo

19. (UEFS) A figura representa um ímã em forma de barra, seus


dois polos magnéticos Norte e Sul e algumas linhas de indução,
contidas no plano da figura, do campo magnético criado pelo ímã.
Sobre essas linhas estão assinalados os pontos de A até H.

Considerando cada afirmação e tendo como referência a posição


do planeta na imagem apresentada, o ímã que poderia substituir o
interior da Terra é
a) d)

b) e)
Desprezando a ação de quaisquer outros campos magnéticos, o
vetor campo magnético criado por esse ímã tem a mesma direção
e o mesmo sentido em
a) B e H.
b) B e D.
c) c) E e G.
d) A e C.
e) D e H.

PRÉ-VESTIBULAR PROENEM.COM.BR 7
FÍSICA II 24 CAMPO MAGNÉTICO

20. (CFTRJ) Sabendo que o vermelho da agulha corresponde ao polo norte da


bússola, qual alternativa melhor representa a orientação da agulha?
a) c)

b) d)

EXERCÍCIOS DE
APROFUNDAMENTO
01. (UERJ) A questão a seguir aborda situações relacionadas
ao ambiente do metrô, referindo-se a uma mesma composição,
formada por oito vagões de dois tipos e movida por tração elétrica.
Para seus cálculos, sempre que necessário, utilize os dados e as
“X-MEN APOCALIPSE: fórmulas abaixo.
Magneto ainda mais poderoso neste filme”
O título de *X-Men: Apocalipse* não se refere apenas ao vilão Características da composição
do filme com fome de poder, mas também à grande quantidade de velocidade máxima 100 km/h
destruição que será mostrada nas telas.
aceleração constante 1,10 m/s²
Despertado depois de milhares de anos, o antigo mutante
Apocalipse acredita que o mundo precisa de um recomeço massivo, desaceleração constante 1,25 m/s²
Gerais
e com a ajuda de seus quatro cavaleiros – Magneto, Tempestade, quantidade de tipo I 2
Psylocke e Anjo – irão iniciar uma aniquilação épica. E cabe aos vagões tipo II 6
X-Men, incluindo Charles Xavier e Mística parar a carnificina, mas
os heróis têm seu trabalho interrompido, pois os quatro cavaleiros massa média por passageiro 60 kg
são impregnados de poder por terem se juntando ao time do vilão. comprimento médio 22,0 m
(Adaptado de: http://universoxmen.com.br/2015/12/x-menapocalipse-magneto-estara-
ainda-mais-poderoso-no-filme/. Acessado em: 13/09/2017) largura 3,00 m
altura 3,60 m
Peça chave desse filme, Magneto é um personagem criado em
1963 e publicado, desde então, pela editora Marvel Comics. Ele é tipo I 38.000 kg
Por vagão massa
um mutante com enormes poderes de manipulação de campos tipo II 35.000 kg
magnéticos, sendo um dos mais poderosos mutantes do Universo
quantidade 4
Marvel. motores
potência por motor 140 kW
Imagine que Magneto tenha manipulado o campo magnético do
seu próprio corpo a ponto de transformá-lo num “ímã humano” capacidade máxima 8 passageiros/m²
com seu polo norte magnético localizado em sua cabeça. Uma
Nas linhas de metrô, o dispositivo conhecido como terceiro
bússola é então colocada no ponto A, em frente a Magneto,
trilho fornece energia elétrica para alimentar os motores das
conforme a figura.
composições, produzindo um campo magnético em seu entorno,
cuja intensidade varia em função da distância. Observe, abaixo, a
imagem da plataforma de uma estação. Nela, uma passageira está
de pé, a 5,0 m de distância do terceiro trilho.

Admita que uma corrente contínua de 5.000 ampères atravesse o


terceiro trilho da linha metroviária.
Determine, em teslas, a intensidade do campo magnético produzido
sobre a passageira na plataforma.

8 PROENEM.COM.BR PRÉ-VESTIBULAR
24 CAMPO MAGNÉTICO FÍSICA II

02. (UNESP) Dois fios longos e retilíneos, 1 e 2, são dispostos no 04. (UDESC) A tabela a seguir apresenta algumas propriedades dos fios
vácuo, fixos e paralelos um ao outro, em uma direção perpendicular de cobre comumente utilizados em circuitos e instalações elétricas.
ao plano da folha. Os fios são percorridos por correntes elétricas
constantes, de mesmo sentido, saindo do plano da folha e Calibre Diâmetro a 20°C (mm) Área (mm²)
apontando para o leitor, representadas, na figura, pelo símbolo . 4 5,2 21,2
Pelo fio 1 circula uma corrente elétrica de intensidade i1 = 9 A e,
pelo fio 2, uma corrente de intensidade i2 = 16 A. A circunferência 8 3,3 8,5
tracejada, de centro C, passa pelos pontos de intersecção entre os 12 2,1 3,5
fios e o plano que contém a figura.
16 1,3 1,3
20 0,8 0,5
Considerando que a resistividade do cobre a 20°C é igual a 1,7 x
10-8 Ωm, e as informações fornecidas na tabela acima, resolva as
questões a seguir:
a) Calcule a resistência por unidade de comprimento de um fio de
cobre de calibre 12.
b) Para a montagem de um circuito elétrico são necessários 10
m de fio de cobre. A resistência máxima oferecida pelo fio não
poderá ser maior do que 2,0 x 10-2 Ω para o bom funcionamento
do circuito. Determine qual o diâmetro mínimo de fio que pode
ser utilizado para a montagem do circuito e identifique qual o
calibre do fio.
T ⋅m
−7 c) Determine o campo magnético a 10 cm de um fio (longo e
Considerando µ0 = 4 ⋅ π ⋅ 10 , calcule o módulo do vetor indução
A reto) de cobre de calibre 20, quando nele estiver passando uma
magnética resultante, em tesla, no centro C da circunferência e no corrente elétrica contínua igual a 2,0 A.
ponto P sobre ela, definido pelas medidas expressas na figura,
devido aos efeitos simultâneos das correntes i1 e i2.
05. (UNIFESP) A figura representa uma bateria, de força eletromotriz
E e resistência interna r = 5,0 Ω, ligada a um solenoide de 200
03. (UNICAMP) Em 2011 comemoram-se os 100 anos da espiras. Sabe-se que o amperímetro marca 200 mA e o voltímetro
descoberta da supercondutividade. Fios supercondutores, que marca 8,0 V, ambos supostos ideais.
têm resistência elétrica nula, são empregados na construção de
bobinas para obtenção de campos magnéticos intensos. Esses
campos dependem das características da bobina e da corrente que
circula por ela.
a) O módulo do campo magnético B no interior de uma bobina
pode ser calculado pela expressão B = µ0ni, na qual i e a corrente
que circula na bobina, n e o número de espiras por unidade de
Tm
comprimento e µ0 = 1,3 × 10−6 . Calcule B no interior de uma
A
bobina de 25000 espiras, com comprimento L = 0,65 m, pela
qual circula uma corrente i = 80 A.
b) Os supercondutores também apresentam potencial de
aplicação em levitação magnética. Considere um ímã de
massa m = 200 g em repouso sobre um material que se torna a) Qual o valor da força eletromotriz da bateria?
supercondutor para temperaturas menores que uma dada b) Qual a intensidade do campo magnético gerado no ponto P,
temperatura crítica TC. Quando o material é resfriado até uma localizado no meio do interior vazio do solenoide?
temperatura
 T < TC, surge sobre o ímã uma força magnética Dados: µ0 = 4π·10-7 T· m/A;

Fm. Suponha que Fm tem a mesma direção e sentido oposto
 B = µ0 (N/L) i (módulo do campo magnético no interior de um solenoide)
ao da força peso P do ímã, e que, inicialmente, o ima sobe
com aceleração constante de módulo aR = 0,5 m/s2, por uma GABARITO
distância d = 2,0 mm , como
 ilustrado na figura abaixo. Calcule
o trabalho realizado por Fm ao longo do deslocamento do ímã. EXERCÍCIOS PROPOSTOS
01. A 05. E 09. B 13. E 17. B
02. D 06. D 10. C 14. D 18. D
03. B 07. D 11. A 15. B 19. E
04. B 08. A 12. D 16. C 20. C

EXERCÍCIOS DE APROFUNDAMENTO
01. 2·10-4 T
02. BC = 5,6 x 10-6 T/ BP = 1 x 10-5 T
03. a) 4,0 T
b) 4,2 x 10-3 J
04. a) 4,86 x 10-3 Ω/m
b) 3,29 mm e Calibre 12
c) 4·10-6 T
05. a) 9,0 V
b) 8,0π·10-5 T

PRÉ-VESTIBULAR PROENEM.COM.BR 9
FÍSICA II 24 CAMPO MAGNÉTICO

ANOTAÇÕES

10 PROENEM.COM.BR PRÉ-VESTIBULAR
25 ELETROMAGNETISMO E FÍSICA II

INDUÇÃO MAGNÉTICA

FORÇA MAGNÉTICA Perceba, no exemplo da imagem anterior, que quando a carga


é positiva a força magnética tem um sentido "saindo" da palma da
Quando uma carga elétrica se move em uma região em que mão. Aqui deve ter uma vírgula e não ponto final no caso de a carga
há campo magnético, essa carga pode sofrer uma força de origem ser negativa, a direção é a mesma, mas o sentido é o contrário ao
magnética chamada força magnética. determinado pela regra da mão.
Essa força, como toda grandeza vetorial, possuirá módulo,
direção e sentido.
O módulo de uma força magnética sofrida por uma partícula
de carga q, em um campo magnético de módulo B e com uma
velocidade de módulo V, pode ser obtido pela expressão:
Fm = |q| · v · B · sen(θ)
Sendo θ o ângulo formado entre o vetor velocidade e o vetor
campo magnético.
Quando a partícula é lançada perpendicularmente ao campo
magnético, tornando o ângulo θ igual a 90° e como sen 90°, = 1, a
expressão se transforma em:

Fm = |q| · v · B O vetor Fm “entra na mão” quando a carga é negativa.
A direção da força magnética é sempre perpendicular ao plano
comum dos vetores velocidade e campo magnético. PROEXPLICA

Resumindo a regra da mão direita



Para determinar o sentido da força magnética Fm deve-se

alinhar o polegar com o sentido da velocidade v  os demais
dedos com o sentido do campo magnético, faça isso
independente do sentido da carga. Então:
• se a carga for positiva, a força magnética terá sentido
“saindo” da palma da mão.
O sentido do vetor representativo da força magnética pode ser
determinado pela “regra da mão”, que pode ser da mão direita ou • se a carga for negativa, a força magnética terá sentido
da mão esquerda. “entrando” na palma da mão.
Para treinar a regra da mão direita estude os exercícios
Protreino 01 e 02.

Para representar vetores perpendiculares ao plano do papel


usa-se a simbologia:

MOVIMENTO DE PARTÍCULAS
No caso de se utilizar a mão direita, o polegar indicará o sentido
da velocidade, os demais dedos, o campo magnético e a palma da DENTRO DE UM CAMPO MAGNÉTICO
mão, o sentido da força magnética, conforme indicado na imagem: Sobre o movimento de partículas carregadas dentro de campos
magnéticos há dois casos recorrentes:

1º caso: A velocidade tem mesma direção de B

PRÉ-VESTIBULAR PROENEM.COM.BR 225


FÍSICA II 25 ELETROMAGNETISMO E INDUÇÃO MAGNÉTICA

Neste caso, a partícula não sofre força magnética já que com


os ângulos de 0° e 180° os senos são iguais a zero.

2º caso: A velocidade tem direção perpendicular a B

FORÇA MAGNÉTICA SOBRE


Neste caso, a força magnética será sempre perpendicular à UM CONDUTOR
velocidade e, dessa maneira, a carga irá realizar um movimento Suponha um condutor, percorrido por uma corrente, imerso em
circular e a força magnética atuará como força centrípeta. uma região que possui campo magnético.

F  q  v  B  sen 
 S
F  i q  B  sen mas v 
t t
F  B  i    sen

A direção da força magnética continua sendo determinada


através da “regra da mão”, só que desta vez o sentido convencional
da corrente (movimento de cargas) substituirá a velocidade (v) da
Observando a expressão, percebemos que o raio da trajetória é partícula.
diretamente proporcional à massa da partícula. Portanto o raio do Força entre dois fios retilíneos paralelos de comprimento L
movimento circular descrito pelo elétron será menor que o raio da percorridos por uma corrente elétrica.
trajetória de um próton atravessando a mesma região, com mesma
(a) (b)
velocidade.
Para determinar o período (T) do movimento circular
I1 I2 I1 I2
devemos antes lembrar da relação entre a velocidade e o período:
2 ⋅ π ⋅R    
V= F21 F12 F21 F12
T
Logo
2 ⋅ π ⋅R m⋅ V
T = Substituindo R
V q⋅B
2⋅ π m⋅ V
T
= ⋅ Quando os fios são atravessados por correntes de mesma
V q ⋅B
direção e mesmo sentido (a), pela regra da mão direita, se atraem
2 ⋅ π ⋅m mutuamente. Por outro lado, quando dois fios retilíneos são
T=
q ⋅B atravessados por correntes de mesma direção e sentidos opostos
(b), pela regra da mão direita, se repelem mutuamente.
Perceba que o período não depende do módulo da velocidade Vamos calcular o módulo da força magnética entre os fios.
que a partícula entra na região do campo magnético. 
Considerando a situação (a) a força F12 (força que o “fio 1 faz
Se o ângulo entre a velocidade e o campo magnético for no fio 2”) é dada por
diferente de 0°, 90° e 180°, assumindo qualquer outro valor, a
trajetória descrita pela partícula será helicoidal (formato de uma F12 = B1 ⋅ i2 ⋅ L ⋅ sen θ nesse caso sen( θ) = sen(90) = 1, então
mola), resultado da soma de dois movimentos simultâneos. F12= B1 ⋅ i2 .L ( eq. 1)
Decompondo a velocidade, encontramos uma componente
paralela ao campo magnético (não gera força magnética, apenas Perceba que é o campo magnético
 gerado pelo fio 1 (B1)
desloca a partícula) e uma componente perpendicular ao campo que induz uma força magnética F12 no fio 2 percorrido por uma
(provocando um movimento circular). corrente i2.

226 PROENEM.COM.BR PRÉ-VESTIBULAR


25 ELETROMAGNETISMO E INDUÇÃO MAGNÉTICA FÍSICA II

µ ⋅i
Estudamos no módulo 24 que B1 = 0 1 , nesse caso R é a 02. (UDESC) Dois fios retilíneos, longos e paralelos, estão
distância entre os fios. 2 ⋅ π ⋅R dispostos, conforme mostra a figura, em duas configurações
Substituindo B1 na equação 1 diferentes: na primeira correntes elétricas de intensidades
iA = 3,0 A e iB = 2,0 A são paralelas; e na segunda, correntes
µ0 ⋅ i1 elétricas também de intensidades iA = 3,0 A e iB = 2,0 A são
F12
= ⋅ i2 ⋅ L
2 ⋅ π ⋅R antiparalelas.
µ ⋅i ⋅i ⋅L
F12 = 0 1 2
2 ⋅ π ⋅R

EXERCÍCIO RESOLVIDO

01. (FAC. ALBERT EINSTEIN - MED) Dois fios condutores


retos, muito compridos, paralelos e muito próximos entre
si, são percorridos por correntes elétricas constantes, de
sentidos opostos e de intensidades 2 A e 6 A, conforme
esquematizado na figura.

A intensidade da força magnética sobre 1,0 m de comprimento


do fio B, e o comportamento dos fios, nas duas configurações
acima, são, respectivamente, iguais a:
a) 6,0 x 10-6 N, repelem-se; 8,0 x 10-6 N, atraem-se.
b) 3,0 x 10-6 N, atraem-se; 3,0 x 10-6 N, repelem-se.
c) 3,0 x 10-6 N, repelem-se; 3,0 x 10-6 N, atraem-se.
d) 9,0 x 10-6 N, atraem-se; 9,0 x 10-6 N, repelem-se.
e) 8,0 x 10-6 N, atraem-se; 6,0 x 10-6 N, repelem-se.
A razão entre os módulos das forças magnéticas de um fio
Resolução: E
sobre o outro e o tipo de interação entre essas forças é igual
a: A intensidade da força magnética imposta a dois fios paralelos
a) 1, repulsiva µ ⋅ i1 ⋅ i2
é dada por: F =
b) 3, atrativa 2⋅π⋅d
µ ⋅i
Equação esta, derivada de outras duas:
= F Bil
= eB
c) 12, atrativa 2πd
Substituindo os valores fornecidos, teremos:
d) a resultante das forças será nula, portanto, não haverá
interação entre elas. 4 π ⋅ 10−7 ⋅ 3 ⋅ 2
F= ∴F = 6 ⋅ 10−6 N
e) 18, atrativa 2 ⋅ π ⋅ 0,2

Resolução: A O sentido da força em cada situação é obtido usando-se,


primeiramente, a regra da mão direita para determinar o
Aplicando as regras práticas do eletromagnetismo, sentido do campo magnético em cada fio, e então, com a
montaram-se as figuras a seguir. regra da mão esquerda definimos o sentido da força em cada
A FIG I mostra
 que na linha em que está o fio 2, o vetor indução caso.
magnética (F1,2 ) devido ao fio 1, é dirigido para fora da página

e que a força magnética que o fio 1 exerce sobre o fio 2 (F1,2 )
é orientada para a direita.
A FIG II mostra que na linha em que está o fio 1, o vetor
indução magnética (B2 ) devido ao fio 2, é dirigido para fora
da página e que a força magnética que o fio 2 exerce sobre
o fio 1 (F2,1) é orientada para a esquerda. Conclusão: essas
forças são repulsivas, conforme mostra a FIG III.

Como essas forças se comportam como um par ação-


F
reação, elas têm mesma intensidade. Então: 1,2 = 1.
F2,1

PRÉ-VESTIBULAR PROENEM.COM.BR 227


FÍSICA II 25 ELETROMAGNETISMO E INDUÇÃO MAGNÉTICA

 
FLUXO MAGNÉTICO Onde θ é o ângulo formado entre os vetores B e A.
Suponha um caminho fechado formado por um condutor, com
um campo magnético (linhas de campo) atravessando a superfície
formada por esse caminho. O fluxo magnético está associado à
quantidade de linhas que atravessam essa determinada área.
Observe a imagem com as áreas S1 e S2 sendo atravessadas pelas
linhas de campo magnético.
S2

S1
O ângulo θ representa a inclinação da reta normal à área em
Φ relação às linhas de campo magnético. Portanto, quando forem
perpendiculares, o fluxo magnético pela superfície será nulo
(cos90° = 0). Entretanto, se forem paralelos o fluxo por tal superfície
será máximo (cos0° = 1).

Na figura acima o número de linhas de campo magnético que


atravessam a área maior (S1) é exatamente o mesmo número de
linhas de campo que atravessam a área menor (S2), seis linhas
em cada caso, por isso, podemos dizer que φ1= φ2 ,ou seja, o fluxo
magnético é o mesmo nas duas áreas.
Entretanto, a campo magnético é mais intenso onde a
densidade de linhas (número de linhas por m²) na região for maior,
o que nos leva a afirmar que
B1> B2
A unidade do Fluxo Magnético no Sistema Internacional (SI) é
Então para que a “contagem de linhas” seja bem-sucedida dada por weber (Wb).
devemos levar em consideração dois fatores:
1 Wb = 1 T . m²
1. A densidade de linhas de campo magnético, isto é, a
intensidade do campo magnético (B).
2. O tamanho da área (A) por onde as linhas passarão. LEI DE FARADAY-LENZ
Com base nessa ideia temos que φ = B.A. Se o fluxo magnético que atravessa um circuito fechado variar

Seja A o “vetor área” definido como um vetor normal com o tempo, será gerada uma força eletromotriz induzida (tensão
(perpendicular) à superfície dessa área, cujo módulo é o próprio induzida), produzindo então uma corrente elétrica que se opõe à
valor dessa área. variação desse fluxo.
FEM induzida



t

Se aproximarmos e afastarmos um ímã de um circuito fechado,


variamos a intensidade do Campo B e consequentemente variamos
o fluxo magnético que atravessa a superfície do circuito.

O fluxo magnético mede o número de linhas de indução que PROEXPLICA


atravessa a área A de uma espira imersa no campo magnético.
Lei de Lenz e o sentido da corrente induzida
A figura acima revela que o fluxo depende de algum ângulo,
O sentido da corrente induzida é tal que o campo magnético
visto que esse fluxo varia à medida que essa área sofre uma
que ela produz se opõem à variação do fluxo magnético do
rotação no interior desse campo.
indutor.
Então vamos reescrever de uma forma mais geral
Veja o esquema abaixo:
 
φ | B | ⋅ | A | ⋅ cos( θ)
= a) Fluxo indutor aumentando → Fluxo induzido contrariando
o aumento do fluxo indutor.
Analisando a relação acima, vemos que ela leva em conta b) Fluxo indutor diminuindo → Fluxo induzido contrariando a
todos os fatores relevantes à contagem do número de linhas diminuição do fluxo indutor.
I. O número de linhas por m² (representado por B); Observe os exemplos de um imã se aproximando e se
II. O tanto de metros quadrados m² (representado por A) afastando de uma espira pelo lado norte.
III. A orientação da área no interior desse campo B, representada
pelo cos(θ).

228 PROENEM.COM.BR PRÉ-VESTIBULAR


25 ELETROMAGNETISMO E INDUÇÃO MAGNÉTICA FÍSICA II

a) EXERCÍCIO RESOLVIDO

03. (UDESC) Na figura abaixo, a barra feita de material condutor


desliza sem atrito, com velocidade constante de 6,0 cm/s para
a direita, sobre trilhos de material também condutor, no plano
horizontal. A barra partiu da extremidade esquerda do trilho
em t = 0s. Nesta região, há um campo magnético uniforme de
intensidade de 10-4 T, como mostra a Figura.

Ao aproximar o polo norte de um imã (indutor) de uma espira


o fluxo magnético através dela (número de linhas de indução
que a atravessa) aumenta, portanto, surge uma corrente na
espira que produz um fluxo induzido contrário ao aumento
do indutor.
b)

Assinale a alternativa que corresponde ao valor absoluto da


tensão induzida, em microvolts, entre os pontos C e D da barra.
a) 600 c) 0,060 e) 0,60
b) 6.000 d) 60

Resolução: E
Vamos aproveitar esse questão para nos aprofundar sobre o
tema de Indução e Transferência de Energia.
Ao afastar o polo norte de um imã (indutor) de uma espira o
fluxo magnético através dela (número de linhas de indução De acordo com a Lei de Lenz, quando um imã é afastado
que a atravessa) diminui, portanto, surge uma corrente na ou aproximado de uma espira, surge uma força magnética
espira que produz um fluxo induzido contrário a diminuição que resiste ao afastamento ou a aproximação. Assim,
do indutor. percebemos que este é um movimento forçado, ou seja,
é necessário que uma força externa execute um trabalho
positivo, a favor do movimento para que o mesmo aconteça.
Além de alterarmos a intensidade de B, aproximando e afastando Esse trabalho representa a energia que é transferida por
o ímã, podemos variar o fluxo magnético pelo circuito alterando essa força ao sistema imã + espira, essa energia devido a
a área exposta às linhas de campo magnético ou modificando o corrente elétrica induzida e a resistência elétrica do material
ângulo entre às linhas de campo e a reta normal à área. acaba se transformando em energia térmica. Quanto maior a
velocidade relativa entre espira e imã, mais depressa a força
aplicada realiza trabalho e menor o tempo em que a energia
se transforma em energia térmica, ou seja, maior a potência
associada à transferência de energia.
Usando a Lei de Faraday-Lenz vamos calcular a força
eletromotriz induzida na espira quando uma força potente
atua na barra, forçando um deslizamento com velocidade
constante para direita.

x x x x x x B

x x x x x x L

x x x x x x
Fp

∆x
i induzido
i +++


x x  x x Fe x V x
B induzido
x x x x x x
 E
x x x Fm x Fb x Fp x
- --
i i

É dessa forma que a energia elétrica é “gerada” nas usinas,


turbinas são acionadas por quedas d’água (hidrelétrica) ou pela
x x x x x x
expansão de vapores aquecidos (termoelétrica) fazendo girar um V

ímã próximo a um enrolado de fios. Isso provoca uma corrente x x x x x x


induzida que vai alternadamente mudando de sentido, a chamada
x x x x x Fm x Fp
corrente alternada.

PRÉ-VESTIBULAR PROENEM.COM.BR 229


FÍSICA II 25 ELETROMAGNETISMO E INDUÇÃO MAGNÉTICA

Perceba que enquanto a barra desliza o fluxo magnético Como estamos analisando a transferência de energia,
aumenta na região interna da área da espira, pela Lei de Lenz, vamos calcular a potência da força externa, lembrando que
surge na espira uma corrente induzida que gera um campo a potência é dada pela razão entre o trabalho da força e o
magnético induzido contrário a esse aumento, nesse caso, intervalo de tempo.
o campo induzido será perpendicular ao plano do papel com t
sentido saindo do papel. (símbolo). P=
∆t
∆∅
ε= O trabalho é dado pela relação t = F . ∆x, então
∆t
B ⋅ ∆A F ⋅ ∆x
ε= P=
∆t ∆t
Perceba que ∆A é a variação da área enquanto a barra de

Perceba que a Força F em módulo é igual a força magnética
comprimento L percorre a distância ∆X, logo ∆A = L . ∆X FB = B . i . L, visto que a velocidade da barra é constante deve
B ⋅ L ⋅ ∆X haver um equilíbrio entre as forças.
ε=
∆t ∆X
Já foi observado que a razão é a velocidade V, portanto
∆X ∆t
Perceba que a razão é a velocidade V com que a barra se podemos escrever que:
∆t
move, logo, temos que: P=B.i.L.V
A força eletromotriz induzida (ε) é obtida pelo equilíbrio entre a B ⋅L ⋅ V
Lembrando que i =
força elétrica e a força magnética, envolvendo a velocidade da R
B ⋅L ⋅ V
barra condutora (v), o comprimento da barra (L) e o módulo P = B⋅ ⋅L ⋅ V
do campo magnético (B), de acordo com a equação abaixo R
ε = B.L.V B2 ⋅ L2 ⋅ V2
P=
R
Usando os dados do problema
(B ⋅ L ⋅ V)2
B = 1⋅ 10−4 T P=
R
=V 6cm= / s 0,06m / s
Não por acaso, encontramos a mesma relação de potência
=L 10cm
= 0,1m
da força motora e da potência dissipada no circuito, a
intenção desse trecho não é fazer o leitor decorar todas essas
fórmulas, mas mostrar matematicamente que a taxa com a
qual realizamos trabalho sobre a espira é igual a taxa com
que o circuito dissipa energia. Esse é um ótimo exemplo da
conservação da energia.
Agora que vimos a teoria que envolve esse fenômeno, vamos
resolver a questão abaixo usando as relações diretamente.

04. (EN) Analise a figura a seguir.


ε=B.L.V
ε = 1 . 10-4 . 0,1 . 0,06
ε = 0,6 . 10-6 V
ε = 0,6 µ V
Indo além do solicitado na questão, para calcular a intensidade
da corrente elétrica induzida devemos usar a relação ε = R . i
onde R é a resistência elétrica do circuito induzido
ε
i=
R

B ⋅L ⋅ V
i=
R
Imersa numa região onde o campo magnético tem direção
Para calcular a potência dissipada no circuito, basta usar a
vertical e módulo B = 6,0 T, uma barra condutora de um metro
relação
de comprimento, resistência elétrica R = 1,0 Ω e massa m
P=i.ε = 0,2 kg desliza sem atrito apoiada sobre trilhos condutores
B ⋅L ⋅ V em forma “U” dispostos horizontalmente, conforme indica a
Substituindo i = e ε = B ⋅ L ⋅ V , temos que
R figura acima. Se uma força externa F mantém a velocidade
B ⋅L ⋅ V da barra constante e de módulo v=2,0 m/s, qual o módulo da
=P ⋅ (B ⋅ L ⋅ V)
R força F, em newtons?
(B ⋅ L ⋅ V)2 a) 6,0 c) 36 e) 72
P=
R b) 18 d) 48

230 PROENEM.COM.BR PRÉ-VESTIBULAR


25 ELETROMAGNETISMO E INDUÇÃO MAGNÉTICA FÍSICA II

Resolução: E EXERCÍCIO RESOLVIDO


Força eletromotriz induzida na barra:
05. (ENEM) A figura mostra o tubo de imagens dos aparelhos
ε = BLv = 6 ⋅ 1⋅ 2 ⇒ ε = 12 V
de televisão usado para produzir as imagens sobre a tela.
Corrente elétrica na barra: Os elétrons do feixe emitido pelo canhão eletrônico são
acelerados por uma tensão de milhares de volts e passam por
ε 12
i= = ⇒ i = 12 A um espaço entre bobinas, onde são defletidos por campos
R 1 magnéticos variáveis, de forma a fazerem a varredura da tela.
Como a barra desliza com velocidade constante, devemos ter
que:
F = Fm = BiL ⇒ F = 6 · 12 · 1 ⇒ F = 72 N

TRANSFORMADORES
Provavelmente você já viu um transformador pendurado
em algum poste pelas ruas da sua cidade. Trata-se de um
dispositivo que tem como função transformar a tensão de Nos manuais que acompanham os televisores é comum
entrada (enrolamento primário) em outro valor de tensão na saída encontrar, entre outras, as seguintes recomendações:
(enrolamento secundário). Por motivos de facilidade, a tensão I. Nunca abra o gabinete ou toque as peças no interior do
é transportada pelo país a uma alta tensão, quando chega aos televisor.
postes próximos às casas e estabelecimentos é reduzida pelos
transformadores para ser utilizada. II. Não coloque seu televisor próximo de aparelhos
domésticos com motores elétricos ou ímãs.
Estas recomendações estão associadas, respectivamente,
aos aspectos de:
a) Riscos pessoais por alta tensão / Perturbação ou
deformação de imagem por campos externos.
b) Proteção dos circuitos contra manipulação indevida / Per-
turbação ou deformação de imagem por campos externos.
c) Riscos pessoais por alta tensão / Sobrecarga dos
circuitos internos por ações externas.
d) Proteção dos circuitos contra a manipulação indevida /
Sobrecarga da rede por fuga de corrente.
e) Proteção dos circuitos contra manipulação indevida /
Um transformador funciona recebendo uma corrente alternada Sobrecarga dos circuitos internos por ação externa.
no enrolamento primário i1, que possui uma tensão eficaz U1.
Essa corrente alternada gera um campo magnético variável que Resolução: A
atravessa o enrolamento secundário, gerando uma corrente
induzida, também alternada, i2, além de submeter uma tensão
eficaz U2. Respeitando a conservação da energia e supondo um
EXERCÍCIOS
transformador ideal, a potência do enrolamento primário tem que
ser igual à potência do secundário, portanto: PROTREINO
U11i = U2 . i2
01. Esferas eletricamente carregadas foram lançadas em uma
Podemos ainda deduzir que a tensão eficaz em cada enrolamento região de campo magnético constante. As velocidades num
é diretamente proporcional ao seu número de espiras. Desta instante estão indicadas na imagem abaixo. Desenhe o vetor força
maneira, se o enrolamento primário possuir U1 = 220V e N1 = 500 magnética em cada uma das esferas.
espiras, para o enrolamento secundário ser submetido à U2 = 110V,
precisaria de N2 = 250 espiras.
U1 U2
=
N1 N2

PROEXPLICA

COMO TUBO DE TELEVISÃO PODE CAIR NO ENEM?


As televisões de tubo já não são as mais modernas, porém
continuam sendo utilizadas em larga escala. Este aparelho
não é nada além de um tubo de raios catódicos que são
desviados por ímãs em seu interior para formar as imagens
que vemos na tela. A questão analisa a necessidade de
determinados cuidados ao se manusear essas televisões.

PRÉ-VESTIBULAR PROENEM.COM.BR 231


FÍSICA II 25 ELETROMAGNETISMO E INDUÇÃO MAGNÉTICA

02. Esferas eletricamente carregadas foram lançadas em uma 05. Desenhe o sentido da corrente induzida na espira devido à
região de campo magnético constante. As velocidades num variação do fluxo magnético em cada caso.
instante estão indicadas na imagem abaixo. Desenhe o vetor força a) Aproximação
magnética em cada uma das esferas.

b) Afastamento

03. Uma partícula de 3 ⋅ 10-14 Kg eletricamente carregada, com


módulo de |q| = 9 nC, que foi acelerada até a velocidade de
300·10² m/s entra em uma região de campo magnético uniforme
de intensidade B = 0,5 T e descreve uma trajetória circular de raio R,
entrando no ponto P e saindo no ponto Q, conforme figura abaixo:

c) Aproximação

Considerando que a única força que atua na partícula entre os


pontos P e Q é a força magnética, determine:
Adote: π = 3
a) Se carga é positiva ou negativa;
b) O módulo da força magnética sobre a partícula, em newtons;
c) O raio (R) da trajetória, em cm;
d) O tempo que a particula leva de P até Q, em segundos;
e) O trabalho da força magnética sore a partícula de P até Q;
f) A velocidade com que a partícula saiu no ponto B.

d) Afastamento
04. A imagem abaixo representa dois fios paralelos, longos,
retilíneos atravessados por correntes elétricas de 3 A e 10 A,
antiparalelas.

Determine a intensidade da
força magnética sobre 1,0
m de comprimento do fio
atravessado pela corrente de
10 A e indique se os fios se
atraem ou se repelem.
Adote: µ0 = 4 ⋅ π ⋅ 10-7 T ⋅ m/A
e π = 3.

232 PROENEM.COM.BR PRÉ-VESTIBULAR


25 ELETROMAGNETISMO E INDUÇÃO MAGNÉTICA FÍSICA II

A fim de se obter uma corrente com o mesmo sentido da


apresentada na figura, utilizando os mesmos materiais, outra
possibilidade é mover a espira para a
EXERCÍCIOS
a) esquerda e o imã para a direita com polaridade invertida.
PROPOSTOS b) direita e o imã para a esquerda com polaridade invertida.
c) esquerda e o imã para a esquerda com mesma polaridade.
01. (EFOMM) Um condutor retilíneo PT, de resistência R = 20,0 Ω,
está em contato com um condutor de resistência desprezível e d) direita e manter o imã em repouso com polaridade invertida.
dobrado em forma de U, como indica a figura.
 O conjunto está imerso e) esquerda e manter o imã em repouso com mesma polaridade.
em um campo de indução
 magnética B, uniforme, de intensidade
15,0 T, de modo que B é ortogonal ao plano do circuito. Seu Demi, 04. (UEFS) Os ímãs, naturais ou artificiais, apresentam determinados
um operador, puxa o condutor PT, de modo que este se move com fenômenos denominados de fenômenos magnéticos.

velocidade constante v, como indica a figura, sendo v = 4,0 m/s. Sobre esses fenômenos, é correto afirmar:
a) A Lei de Lenz estabelece que o sentido da corrente induzida é
tal que se opõe à variação de fluxo magnético através de um
circuito que a produziu.
b) Os pontos da superfície terrestre que possuem inclinação
magnética máxima pertencem a uma linha chamada Equador
Magnético.
c) Sob a ação exclusiva de um campo magnético, o movimento
de uma carga elétrica é retilíneo e uniformemente acelerado.
d) Nas regiões em que as linhas de indução estão mais próximas,
o campo magnético é menos intenso.
e) As linhas de indução são, em cada ponto, perpendiculares ao
vetor indução magnética.
Determine a forma eletromotriz induzida no circuito e o valor da
força aplicada por seu Demi ao condutor PT. 05. (UERJ) O princípio físico do funcionamento de alternadores e
a) 45 V e 80,45 N d) 90 V e 101,25 N transformadores, comprovável de modo experimental, refere-se à
produção de corrente elétrica por meio da variação de um campo
b) 65 V e 90,10 N e) 100,85 V e 110,95 N magnético aplicado a um circuito elétrico.
c) 80 V e 100,65 N Esse princípio se fundamenta na denominada Lei de:
a) Newton b) Ampère c) Faraday d) Coulomb
02. (UEMG) O desenvolvimento tecnológico das últimas décadas
tem exigido a produção cada vez maior de energia, principalmente
de energia elétrica. Além das hidrelétricas, outras fontes como 06. (UERJ) A corrente elétrica no enrolamento primário de um trans-
painéis fotovoltaicos, usinas eólicas, termoelétricas e baterias têm formador corresponde a 10 A, enquanto no enrolamento secundário
sido usadas para produzir energia elétrica. corresponde a 20 A. Sabendo que o enrolamento primário possui
1.200 espiras, o número de espiras do enrolamento secundário é:
São fontes de energia que não se baseiam na indução
eletromagnética para produção de energia elétrica: a) 600 b) 1.200 c) 2.400 d) 3.600

a) pilhas e painéis fotovoltaicos.


07. (UPE) Uma bobina, formada por 5 espiras que possui um
b) termoelétricas e usinas eólicas. raio igual a 3,0 cm é atravessada por um campo magnético
c) pilhas, termoelétricas e painéis fotovoltaicos. perpendicular ao plano da bobina. Se o campo magnético tem seu
d) termoelétricas, painéis fotovoltaicos e usinas eólicas. módulo variado de 1,0 T até 3,5 T em 9,0 m/s, é CORRETO afirmar
que a força eletromotriz induzida foi, em média, igual a
03. (ENEM) O funcionamento dos geradores de usinas elétricas a) 25 mV c) 0,25 V e) 3,75 V
baseia-se no fenômeno da indução eletromagnética, descoberto b) 75 mV d) 1,25 V
por Michael Faraday no século XIX. Pode-se observar esse
fenômeno ao se movimentar um imã e uma espira em sentidos 08. (UFG) Uma nova tecnologia vem sendo desenvolvida para
opostos com módulo da velocidade igual a v, induzindo uma cozinhar alimentos com maior eficiência no aproveitamento da
corrente elétrica de intensidade i, como ilustrado na figura. energia fornecida. Estima-se que esse novo fogão aproveite 90%
da energia, em comparação com 55% do fogão a gás e 65% do
fogão elétrico. Entretanto, para o seu funcionamento, deve-
se usar apenas panelas de metais ferromagnéticos, tais como
ferro fundido ou aço. Essas panelas são aquecidas por meio de
correntes induzidas quando colocadas sobre uma plataforma de
vitrocerâmica. De acordo com o exposto, conclui-se que o princípio
físico que fundamenta o funcionamento deste novo fogão é
a) o efeito fotoelétrico da interação da radiação com a matéria.
b) a lei de Stefan-Boltmann da emissão de radiação de um corpo
negro.
c) a lei de Ohm.
d) a lei de Faraday.
e) a lei de Ampère.

PRÉ-VESTIBULAR PROENEM.COM.BR 233


FÍSICA II 25 ELETROMAGNETISMO E INDUÇÃO MAGNÉTICA

09. (UEL) Em uma usina hidrelétrica, a água do reservatório é 12. (ENEM) A tecnologia de comunicação da etiqueta RFID
guiada através de um duto para girar o eixo de uma turbina. O (chamada de etiqueta inteligente) é usada há anos para rastrear
movimento mecânico do eixo, no interior da estrutura do gerador, gado, vagões de trem, bagagem aérea e carros nos pedágios. Um
transforma a energia mecânica em energia elétrica que chega até modelo mais barato dessas etiquetas pode funcionar sem baterias
nossas casas. Com base nas informações e nos conhecimentos e é constituído por três componentes: um microprocessador de
sobre o tema, é correto afirmar que a produção de energia elétrica silício; uma bobina de metal, feita de cobre ou de alumínio, que
em uma usina hidrelétrica está relacionada é enrolada em um padrão circular; e um encapsulador, que é um
a) à indução de Faraday. d) ao princípio de Arquimedes. material de vidro ou polímero envolvendo o microprocessador e
a bobina. Na presença de um campo de radiofrequência gerado
b) à força de Coulomb. e) ao ciclo de Carnot. pelo leitor, a etiqueta transmite sinais. A distância de leitura é
c) ao efeito Joule. determinada pelo tamanho da bobina e pela potência da onda de
rádio emitida pelo leitor.
10. (ENEM) O manual de funcionamento de um captador de guitarra Disponível em: http:eleletronicos.hsw.uol.com.br. Acesso em: 27 fev. 2012 (adaptado).
elétrica apresenta o seguinte texto:
A etiqueta funciona sem pilhas porque o campo
Esse captador comum consiste de uma bobina, fios condutores
enrolados em torno de um ímã permanente. O campo magnético do a) elétrico da onda de rádio agita elétrons da bobina.
ímã induz o ordenamento dos polos magnéticos na corda da guitarra, b) elétrico da onda de rádio cria uma tensão na bobina.
que está próxima a ele. Assim, quando a corda é tocada, as oscilações
c) magnético da onda de rádio induz corrente na bobina.
produzem variações, com o mesmo padrão, no fluxo magnético que
atravessa a bobina. Isso induz uma corrente elétrica na bobina, que é d) magnético da onda de rádio aquece os fios da bobina.
transmitida até o amplificador e, daí, para o alto-falante. e) magnético da onda de rádio diminui a ressonância no interior
Um guitarrista trocou as cordas originais de sua guitarra, que da bobina.
eram feitas de aço, por outras feitas de náilon. Com o uso dessas
cordas, o amplificador ligado ao instrumento não emitia mais som, 13. (ENEM) Cercas elétricas instaladas nas zonas urbanas são
porque a corda de náilon dispositivos de segurança planejados para inibir roubos e devem
a) isola a passagem de corrente elétrica da bobina para o alto- ser projetadas para, no máximo, assustar as pessoas que toquem
falante. a fiação que delimita os domínios de uma propriedade. A legislação
vigente que trata sobre as cercas elétricas determina que a unidade
b) varia seu comprimento mais intensamente do que ocorre com de controle deverá ser constituída, no mínimo, de um aparelho
o aço. energizador de cercas que apresente um transformador e um
c) apresenta uma magnetização desprezível sob a ação do ímã capacitor. Ela também menciona que o tipo de corrente elétrica
permanente. deve ser pulsante.
d) induz correntes elétricas na bobina mais intensas que a Considere que o transformador supracitado seja constituído
capacidade do captador. basicamente por um enrolamento primário e outro secundário, e
e) oscila com uma frequência menor do que a que pode ser que este último está ligado indiretamente à fiação. A função do
percebida pelo captador. transformador em uma cerca elétrica é
a) reduzir a intensidade de corrente elétrica associada ao
11. (ENEM) O espectrômetro de massa de tempo de voo é um secundário.
dispositivo utilizado para medir a massa de íons. Nele, um íon de b) aumentar a potência elétrica associada ao secundário.
carga elétrica
 q é lançado em uma região de campo magnético c) amplificar a energia elétrica associada a este dispositivo.
constante B, descrevendo uma trajetória helicoidal, conforme
a figura. Essa trajetória é formada pela composição de um d) proporcionar perdas de energia do primário ao secundário.
movimento circular uniforme no plano yz e uma translação ao e) provocar grande perda de potência elétrica no secundário.
longo do eixo x. A vantagem desse dispositivo é que a velocidade
angular do movimento helicoidal do íon é independente de sua 14. (ENEM) Os dínamos são geradores de energia elétrica
velocidade inicial. O dispositivo então mede o tempo t de voo para utilizados em bicicletas para acender uma pequena lâmpada. Para
N voltas do íon. Logo, com base nos valores q, B, N e t, pode-se isso, é necessário que a parte móvel esteja em contato com o pneu
determinar a massa do íon. da bicicleta e, quando ela entra em movimento, é gerada energia
elétrica para acender a lâmpada. Dentro desse gerador, encontram-
se um imã e uma bobina.

A massa do íon medida por esse dispositivo será


qBt qBt 2qBt qBt 2qBt
a) b) c) d) e)
2 πN πN πN N N

234 PROENEM.COM.BR PRÉ-VESTIBULAR


25 ELETROMAGNETISMO E INDUÇÃO MAGNÉTICA FÍSICA II

O princípio de funcionamento desse equipamento é explicado pelo


fato de que a
a) corrente elétrica no circuito fechado gera um campo magnético
nessa região.
b) bobina imersa no campo magnético em circuito fechado gera
uma corrente elétrica.
c) bobina em atrito com o campo magnético no circuito fechado
gera uma corrente elétrica.
d) corrente elétrica é gerada em circuito fechado por causa da
presença do campo magnético.
e) corrente elétrica é gerada em circuito fechado quando há
variação do campo magnético.

15. Imagine um elétron do átomo de hidrogênio girando em órbita 


estável ao redor do núcleo desse átomo. A frequência com que ele O diagramaque representa corretamente as forças elétrica FE e
gira é altíssima. magnética FB atuando sobre a partícula é:
a)

b)

c)
A figura destaca o eixo perpendicular ao plano da trajetória do
elétron e que contém o centro da trajetória e um ponto P do eixo,
próximo ao núcleo do átomo. d)
O movimento desse elétron produz, no ponto P um campo elétrico
a) variável e um campo magnético de intensidade constante, mas
de direção variável. e)
b) de intensidade constante, mas de direção variável, e um campo
magnético constante.
c) e um campo magnético, ambos de intensidades constantes,
mas de direções variáveis. 17. (UFRGS) O fogão mostrado na figura 1 abaixo não produz
chamas nem propaga calor. O cozimento ou aquecimento dos
d) e um campo magnético, ambos de intensidades variáveis, mas
alimentos deve ser feito em panelas de ferro ou de aço e ocorre
de direções constantes.
devido à existência de campos magnéticos alternados, produzidos
e) de intensidade constante, mas de direção variável, e um campo em bobinas, conforme representado no esquema da figura 2.
magnético variável. Os campos magnéticos penetram na base das panelas, criando
correntes elétricas que as aquecem.
16. (FUVEST) Um solenoide muito longo é percorrido por uma
corrente elétrica I, conforme mostra a figura 1.

Em um determinado instante, uma partícula  de carga q positiva


desloca-se com velocidade instantânea v perpendicular ao eixo do
solenoide, na presença de um campo elétrico na direção do eixo
do solenoide. A figura 2 ilustra essa situação, em uma seção reta
definida por um plano que contém o eixo do solenoide.

PRÉ-VESTIBULAR PROENEM.COM.BR 235


FÍSICA II 25 ELETROMAGNETISMO E INDUÇÃO MAGNÉTICA

20. (UDESC) Na figura abaixo, a barra feita de material condutor


desliza sem atrito, com velocidade constante de 6,0 cm/s para
a direita, sobre trilhos de material também condutor, no plano
horizontal. A barra partiu da extremidade esquerda do trilho em
t = 0 s. Nesta região, há um campo magnético uniforme de
intensidade de 10-4 T, como mostra a Figura.

Assinale a alternativa que corresponde ao valor absoluto da tensão


Assinale a alternativa que preenche corretamente as lacunas do
induzida, em microvolts, entre os pontos C e D da barra.
enunciado abaixo, na ordem em que aparecem.
a) 600 d) 60
O processo físico que fundamenta essa aplicação tecnológica
é conhecido como __________ e é regido pela lei de __________. b) 6.000 e) 0,60
a) convecção – Faraday-Lenz c) 0,060
b) indução – Faraday-Lenz
c) indução – Ampère
EXERCÍCIOS DE
d) radiação – Gauss
e) radiação – Ampère
APROFUNDAMENTO
01. (UNICAMP) Julho de 2019 marcou o cinquentenário da
18. (EEAR) Uma partícula com carga elétrica igual a 3,2 µC e
chegada do homem à Lua com a missão Apollo 11. As caminhadas
velocidade de 2 · 104 m/s é lançada perpendicularmente a um
dos astronautas em solo lunar, com seus demorados saltos, são
campo magnético uniforme e sofre a ação de uma força magnética
imagens emblemáticas dessa aventura humana.
de intensidade igual a 1,6 · 10² N. Determine a intensidade do
campo magnético (em Tesla) no qual a partícula foi lançada. a) A aceleração da gravidade na superfície da Lua é gL = 1,6 m/s².
Calcule o tempo de queda de um corpo solto a partir do repouso
a) 0,25 ⋅ 10³
de uma altura de 1,8 m com relação à superfície lunar.
b) 2,5 ⋅ 10³
b) A espectrometria de massas é uma técnica que pode ser
c) 2,5 ⋅ 104 usada na identificação de moléculas da atmosfera e do solo
d) 0,25 ⋅106 lunar. A figura a seguir mostra a trajetória (no plano do papel)
de uma determinada molécula ionizada (carga q = 1,6 × 10-19 C)
19. (UEG) A figura a seguir descreve uma região do espaço que que entra na região de campo magnético do espectrômetro,
 
contém um vetor campo elétrico E e um vetor campo magnético B. sombreada na figura, com velocidade de módulo
V = 3,2 × 105 m/s. O campo magnético é uniforme e
perpendicular ao plano do papel, dirigido de baixo para cima, e
tem módulo B = 0,4 T. Como ilustra a figura, na região de campo
magnético a trajetória é circular de raio R = 36 cm, e a força
centrípeta é dada pela força magnética de Lorentz, cujo módulo
vale F = qVB. Qual é a massa m da molécula?

Mediante um ajuste, percebe-se que, quando os campos elétricos


e magnéticos assumem valores de 1,0 × 10³ N/C e 2,0 × 10-2 T,
respectivamente, um íon positivo, de massa desprezível, atravessa
os campos em linha reta. A velocidade desse íon, em m/s, foi de
a) 5,0 × 104
b) 1,0 × 105
c) 2,0 × 10³
d) 3,0 × 10³
e) 1,0 × 104

236 PROENEM.COM.BR PRÉ-VESTIBULAR


25 ELETROMAGNETISMO E INDUÇÃO MAGNÉTICA FÍSICA II

02. (FUVEST) Adotando g = 10 m/s² e considerando que, para manter o objeto


preso à balança em repouso, será necessário ajustar a tensão
do gerador para U = 200 V, calcule, quando a balança estiver em
funcionamento,
a) a diferença de potencial, em V, nos terminais do resistor de
40 Ω e a potência dissipada por ele, em W.
b) a intensidade da corrente elétrica, em ampères, que atravessa a
barra AC e a massa M, em kg, do objeto preso a balança.

04. (UNESP) Em um equipamento utilizado para separar partículas



eletrizadas atuam
 dois campos independentes, um elétrico, E, e um
magnético, B, perpendiculares entre si. Uma partícula de massa
m = 4 × 10-15 kg e carga q = 8 × 10-6 C parte do repouso no ponto
P, é acelerada pelo campo elétrico e penetra, pelo ponto Q, na
Em um ambiente do qual se retirou praticamente todo o ar,
região onde atua o campo magnético, passando a descrever uma
as placas de um capacitor estão arranjadas paralelamente e
trajetória circular de raio R, conforme a figura.
carregadas com cargas de mesma magnitude Q e sinais contrários,
produzindo, na região entre as placas, um campo elétrico que pode
ser considerado uniforme, com módulo igual a 106 V/m. Uma
partícula carregada negativamente, com carga de módulo igual a
10-9 C, é lançada com velocidade de módulo V0 igual a 100 m/s ao
longo da linha que passa exatamente pelo centro da região entre as
placas, como mostrado na figura. A distância d entre as placas é
igual a 1 mm. Despreze os efeitos gravitacionais.
a) Aponte, entre as trajetórias 1 e 2 mostradas na figura, aquela
que mais se aproxima do movimento da partícula na região
entre as placas.
b) Sabendo que a massa da partícula é igual a 10 µg, determine a
que distância horizontal x a partícula atingirá uma das placas,
supondo que elas sejam suficientemente longas.
c) Quais seriam o sentido e o módulo de um eventual campo
magnético a ser aplicado na região entre as placas,
perpendicularmente ao plano da página, para que a partícula, Sabendo que entre os pontos P e Q existe uma diferença de
em vez de seguir uma trajetória curva, permaneça movendo-se potencial de 40 V, que a intensidade do campo magnético é B = 10-3 T
na mesma direção e no mesmo sentido com que foi lançada? e desprezando ações gravitacionais sobre a partícula eletrizada,
calcule:

03. (UNIFESP) A figura representa uma balança eletromagnética a) a intensidade do campo elétrico E, em N/C.
utilizada para determinar a massa M do objeto preso a ela. Essa
b) o raio R, em m, da trajetória circular percorrida
 pela partícula na
balança é constituída por um gerador ideal cuja tensão U pode ser
região em que atua o campo magnético B.
ajustada, por um resistor ôhmico de resistência R = 40 Ω e por uma
barra condutora AC, de massa e resistência elétrica desprezíveis,
conectada ao gerador por fios ideais. A barra AC mede 50 cm e 05. (FUVEST) Uma espira quadrada, de lado L, constituída por
está totalmente imersa em um campo magnético uniforme de barras rígidas de material condutor, de resistência
 elétrica total R,
intensidade B = 1,6 T, perpendicular à barra e ao plano desta folha se desloca no plano xy com velocidade v constante, na direção do
e apontado para dentro dela. O objeto, cuja massa pretende-se eixo x. No instante t = 0, representado na figura, a espira começa a
determinar, está preso por um fio isolante e de massa desprezível entrar em uma região do espaço, de  seção reta quadrada, de lado

no centro da barra AC. 2 L, onde há um campo magnético B perpendicular a v; a velocidade
da espira é mantida constante por meio da ação de um agente
externo. O campo B é uniforme, constante e tem a direção do eixo
z, entrando no plano xy.

PRÉ-VESTIBULAR PROENEM.COM.BR 237


FÍSICA II 25 ELETROMAGNETISMO E INDUÇÃO MAGNÉTICA
LBv
F = B ⋅ i3 ⋅ L = B ⋅ ⋅L
R
a) A figura abaixo representa a situação para o instante t1 = L/(2v). BLv2 2
c)
∴F =
Indique nessa figura o sentido da corrente elétrica i1 que circula R
pela espira e determine o seu valor.

ANOTAÇÕES

b) Determine a corrente i1 na espira para o instante t2 = (3 L)/(2 v).



c) Determine a força eletromagnética F (módulo, direção e
sentido) que atua na espira no instante t3 = (5 L)/(2 v).
Note e adote:
Força eletromotriz na espira parcialmente imersa no campo
magnético: ε = L B v

GABARITO
EXERCÍCIOS PROPOSTOS
01. D 05. C 09. A 13. A 17. B
02. A 06. A 10. C 14. E 18. B
03. A 07. E 11. A 15. B 19. A
04. A 08. D 12. C 16. A 20. E

EXERCÍCIOS DE APROFUNDAMENTO
01.
a) t = 1,5 s
b) m = 7,2 ⋅ 10–26 kg
02. a) Dado que a partícula lançada está carregada negativamente, a força elétrica sobre
ela estará no sentido oposto ao do campo elétrico. Ou seja, a trajetória 1 é a que melhor
descreve o seu movimento.
b) ∆sx = 10-2 e m = 1 cm
c) B = 104 T. E pela regra da mão esquerda, o campo magnético deverá estar na direção
perpendicular à página, com sentido entrando nela.

03. a) P = 100 W
b) i = 5 A e M = 0,4 kg
04. a) E = 200 N/C
ε
b) R i=1 0,2 Ω=
= ( com ε LBv )
R
LBv
05. a)
∴i1 =
R

b) i2 = 0

238 PROENEM.COM.BR PRÉ-VESTIBULAR


26
FÍSICA II
REVISÃO: ELETROMAGNETISMO

SOBRE UMA PARTÍCULA


Para que uma partícula sofra ação da força magnética é
necessário que essa partícula possua carga, esteja dentro de uma
região com campo magnético e tenha uma velocidade que não seja
paralela às linhas de campo.

 
θ é o ângulo entre os vetores v e b,. Quando os vetores são
paralelos, temos que θ = 0° ou θ = 180°, lembrando que: sen(0°) 0 e
sen(180°) = 0. Em consequência a força magnética é nula quando
o vetor velocidade é paralelo ao vetor indução magnética (campo
magnético).
A direção da força magnética será sempre perpendicular ao
plano formado por v e B e podemos determina-la através da regra
da mão direita:
Polegar aponta na direção e sentido da velocidade, indicador na A direção da força magnética novamente será determinada
direção e sentido do campo magnético. pela regra da mão direita, mas agora a corrente (movimento das
cargas) estará no lugar da velocidade:

Quando a carga é positiva a força tem a mesma direção e o


mesmo sentido em que a mão direita daria um empurrão, ou seja,
“saindo da mão”.
FLUXO MAGNÉTICO
O fluxo magnético está associado à quantidade de linhas que
atravessam uma determinada área.

Caso a partícula seja negativa a força magnética tem sentido A unidade do Fluxo Magnético no Sistema Internacional (SI) é
“entrando” na palma da mão. dada por Weber (Wb).
1 Wb = 1 T.m²
SOBRE UM CONDUTOR A reta normal à área é utilizada para determinar o ângulo θ
Suponha um condutor imerso em um campo magnético, sendo entre às linhas de campo magnético e a “direção da área”, é fácil
percorrido por uma corrente elétrica i. Atuará sobre ele uma força observar na figura acima que quando o ângulo θ = 90° o fluxo será
magnética, perpendicular ao campo e à corrente, cujo módulo pode nulo (cos90° = 0), pois não teremos linhas de campo atravessando
ser calculado por: a superfície.

PRÉ-VESTIBULAR PROENEM.COM.BR 239


FÍSICA II 26 REVISÃO: ELETROMAGNETISMO

LEI DE FARADAY-LENZ TRANSFORMADORES


Se o fluxo magnético que atravessa um circuito fechado variar
com o tempo, será gerada uma força eletromotriz induzida (tensão
induzida), produzindo então uma corrente elétrica que se opõe à
variação desse fluxo.
FEM induzida

O sentido da corrente induzida pode ser resumido em:


• Polo Norte se aproximando – a corrente possui sentido
anti-horário, fazendo com que a face da espira se comporte
como um polo Norte, repelindo o ímã e se opondo ao
aumento do fluxo magnético.

• Polo Norte se afastando – a corrente possui sentido horário,


fazendo com que a face da espira se comporte como um Um transformador funciona recebendo uma corrente alternada
polo Sul, atraindo o ímã e se opondo à diminuição do fluxo no enrolamento primário i1, que possui uma tensão eficaz U1.
magnético. Essa corrente alternada gera um campo magnético variável que
atravessa o enrolamento secundário, gerando uma corrente
induzida, também alternada, i2, além de submeter uma tensão
eficaz U2. Respeitando a conservação da energia e supondo um
transformador ideal, a potência do enrolamento primário tem que
ser igual à potência do secundário, portanto:

U11i = U2 . i2

Podemos ainda deduzir que a tensão eficaz em cada


enrolamento é diretamente proporcional ao seu número de espiras.
• Polo Sul se aproximando – a corrente possui sentido
Desta maneira, se o enrolamento primário possuir U1 = 220V e
horário, fazendo com que a face da espira se comporte
N1 = 500 espiras, para o enrolamento secundário ser submetido à
como um polo Sul, repelindo o ímã e se opondo ao aumento
U2 = 110V, precisaria de N2 = 250 espiras.
do fluxo magnético.
U1 U2
=
N1 N2

EXERCÍCIOS
PROTREINO
01. O gráfico relaciona o a variação do fluxo magnético que
• Polo Sul se afastando – a corrente possui sentido anti- atravessa espira num intervalo de tempo. Calcule o módulo da
horário, fazendo com que a face da espira se comporte força eletromotriz induzida nesse intervalo.
como um polo Norte, atraindo o ímã e se opondo à
diminuição do fluxo magnético.

240 PROENEM.COM.BR PRÉ-VESTIBULAR


26 REVISÃO: ELETROMAGNETISMO FÍSICA II

02. Um transformador que fornece energia elétrica a um ventilador


está conectado a uma rede elétrica de tensão eficaz igual a 240 V.
A tensão eficaz no enrolamento secundário é de 12 V, e a corrente EXERCÍCIOS
eficaz no ventilador é de 4 A.
Determine o valor eficaz da corrente no enrolamento primário do PROPOSTOS
transformador.
01. (UNESP) Uma espira, locomovendo-se paralelamente ao solo
03. Uma partícula foi lançada a 100 m/s no interior de um campo e com velocidade constante, atravessa uma região onde existe um
magnético de intensidade B = 2 T, descrevendo um movimento campo magnético uniforme, perpendicular ao plano da espira e ao
circular de raio R = 2 m. solo. O fluxo magnético registrado, a partir do instante em que a
espira entra nessa região até o instante de sua saída, é apresentado
no gráfico da figura.

Determine a razão entre a massa e a carga (m/q) da partícula.

04. O esquema abaixo mostra um fio retilíneo fixo de 80 cm sobre Analisando o gráfico, pode-se dizer que a força eletromotriz
uma diferença de potencial, que é atravessado por uma corrente induzida, em volts, no instante t = 0,2 s, é
de 30 A. a) 80. d) 20.
b) 60. e) 0.
c) 40.

02. (UFOP) O cíclotron é um acelerador em que partículas carregadas


executam movimento circular em um plano perpendicular a um
campo magnético uniforme de módulo B. Se o campo magnético
for o único campo aplicado, a velocidade angular do movimento
circular resultante depende somente da razão carga/massa e de
B. Em um acelerador típico, o valor de B é de 1 tesla e as partículas
percorrem uma trajetória de raio de 50 cm.
Qual a ordem de grandeza da velocidade da partícula (dados: carga
Desenho meramente ilustrativo
igual a 1,6 x 10-19C e massa igual 1,67 x 10-27 kg)?
Determine a direção, o sentido e o módulo da força magnética a) 103 m/s
sobre o fio, sabendo que o campo magnético na região é uniforme b) 105 m/s
e possui módulo de 4T.
c) 107 m/s
d) 109 m/s
05. Uma partícula P eletrizada de massa desprezível foi lançada a
120 m/s paralelamente a duas placas eletrizadas A e B, que geram
um campo elétrico uniforme entre elas. Essa região também tem 03. (UFOP) Para escoar a energia elétrica produzida em suas
influência de um campo magnético de intensidade B = 20 T turbinas, a hidrelétrica de Itaipu eleva a tensão de saída para
aproximadamente 700.000 V. Em sua residência, as tomadas
apresentam uma tensão de 127 V e/ou 220 V. O equipamento que
realiza essa tarefa de elevar e abaixar a tensão é o transformador.
É correto afirmar que
a) o princípio de funcionamento de um transformador exige que a
tensão/corrente seja contínua.
b) o princípio de funcionamento de um transformador exige que a
tensão/corrente seja alternada.
c) o transformador irá funcionar tanto em uma rede com tensão/
corrente alternada quanto em uma com tensão/corrente
Sabendo que a partícula está carregada positivamente e que contínua.
atravessa a região sem sofrer nenhuma alteração no seu vetor d) o transformador irá funcionar quando, no enrolamento primário,
velocidade, portanto, não sofre desvios, determine a direção, o houver uma tensão/corrente contínua e, no secundário, uma
sentido e o módulo do campo elétrico entre as placas. Indique qual alternada.
das placas é positiva.

PRÉ-VESTIBULAR PROENEM.COM.BR 241


FÍSICA II 26 REVISÃO: ELETROMAGNETISMO

04. (ENEM 2ª APLICAÇÃO) Há vários tipos de tratamentos de 08. (FUVEST) Aproxima-se um ímã de um anel metálico fixo em um
doenças cerebrais que requerem a estimulação de partes do suporte isolante, como mostra a figura. O movimento do ímã, em
cérebro por correntes elétricas. Os eletrodos são introduzidos no direção ao anel,
cérebro para gerar pequenas correntes em áreas específicas. Para
se eliminar a necessidade de introduzir eletrodos no cérebro, uma
alternativa é usar bobinas que, colocadas fora da cabeça, sejam
capazes de induzir correntes elétricas no tecido cerebral.
Para que o tratamento de patologias cerebrais com bobinas seja
realizado satisfatoriamente, é necessário que
a) haja um grande número de espiras nas bobinas, o que diminui
a voltagem induzida.
b) o campo magnético criado pelas bobinas seja constante, de
forma a haver indução eletromagnética.
c) se observe que a intensidade das correntes induzidas depende a) não causa efeitos no anel.
da intensidade da corrente nas bobinas. b) produz corrente alternada no anel.
d) a corrente nas bobinas seja contínua, para que o campo c) faz com que o polo sul do ímã vire polo norte e vice versa.
magnético possa ser de grande intensidade. d) produz corrente elétrica no anel, causando uma força de
e) o campo magnético dirija a corrente elétrica das bobinas para atração entre anel e ímã.
dentro do cérebro do paciente. e) produz corrente elétrica no anel, causando uma força de
repulsão entre anel e ímã.
05. (UERJ - ADAPTADO) Um transformador que fornece energia
elétrica a um computador está conectado a uma rede elétrica de 09. (UNESP) O freio eletromagnético é um dispositivo no qual
tensão eficaz igual a 120 V. interações eletromagnéticas provocam uma redução de velocidade
A tensão eficaz no enrolamento secundário é igual a 10V, e a num corpo em movimento, sem a necessidade da atuação de forças
corrente eficaz no computador é igual a 1,2A. de atrito. A experiência descrita a seguir ilustra o funcionamento de
Estime o valor eficaz da corrente no enrolamento primário do um freio eletromagnético.
transformador. Na figura 1, um ímã cilíndrico desce em movimento acelerado por
a) 0,1A dentro de um tubo cilíndrico de acrílico, vertical, sujeito apenas à
ação da força peso. Na figura 2, o mesmo ímã desce em movimento
b) 0,2A
uniforme por dentro de um tubo cilíndrico, vertical, de cobre, sujeito
c) 0,3A à ação da força peso e da força magnética, vertical e para cima, que
d) 0,4A surge devido à corrente elétrica induzida que circula pelo tubo de
cobre, causada pelo movimento do ímã por dentro dele. Nas duas
06. (UFAL) Numa certa região, o campo magnético gerado pela situações, podem ser desconsiderados o atrito entre o ímã e os
Terra possui uma componente Bx paralela à superfície terrestre, tubos, e a resistência do ar
com intensidade de 2 × 10−5 T, e uma componente Bz perpendicular
à superfície terrestre, com intensidade de 5 × 10−5 T. Nessa região,
uma linha de transmissão paralela à componente Bx é percorrida
por uma corrente elétrica de 5000 A. A força magnética por unidade
de comprimento que o campo magnético terrestre exerce sobre
essa linha de transmissão possui intensidade igual a:
a) 0,10 N/m
b) 0,25 N/m
c) 1,0 N/m
d) 2,5 N/m
e) 10 N/m

07. (UFAL) Uma corda metálica de uma guitarra elétrica se


Considerando a polaridade do ímã, as linhas de indução magnética
comporta como um pequeno ímã, com polaridades magnéticas
criadas por ele e o sentido da corrente elétrica induzida no tubo
norte e sul. Quando a corda é tocada, ela se aproxima e se afasta
condutor de cobre abaixo do ímã, quando este desce por dentro
periodicamente de um conjunto de espiras metálicas enroladas
do tubo, a alternativa que mostra uma situação coerente com o
numa bobina situada logo abaixo. A variação do fluxo do campo
aparecimento de uma força magnética vertical para cima no ímã é
magnético gerado pela corda através da bobina induz um sinal
a indicada pela letra
elétrico (d.d.p. ou corrente), que muda de sentido de acordo com a
vibração da corda e que é enviado para um amplificador. a) b) c)
Qual o cientista cujo nome está associado à lei física que explica
o fenômeno da geração de sinal elétrico pela variação do fluxo
magnético através da bobina?
a) Charles Augustin de Coulomb
b) André Marie Ampère
c) Hans Christian Oersted
d) Georg Ohm
e) Michael Faraday

242 PROENEM.COM.BR PRÉ-VESTIBULAR


26 REVISÃO: ELETROMAGNETISMO FÍSICA II

13. (COTIL) O crescimento desordenado, a falta de infraestrutura, os


d) e) problemas sociais e a desonestidade de alguns acarretam um tipo de
furto na rede elétrica conhecido como “gato”. Há dois tipos de “gato”:
1. Dos que alteram o medidor para pagarem menos energia
elétrica do que realmente gastaram;
2. Dos que fazem ligações clandestinas na rede elétrica,
puxando fios diretamente dos postes da rua para o interior das
casas, sem pagar qualquer valor pela energia.
Em ambos os casos, a população é duplamente prejudicada:
podem ocorrer interrupções no fornecimento, devido ao aumento
descontrolado no consumo, e o valor correspondente a energia
roubada é rateado pelos outros pagantes.
10. (UFSM) A tecnologia das grandes usinas hidroelétricas depende
(Disponível em: www.mundodaeletrica.com.br/perigo-dos-gatos-na-rede-eletrica,
de extensas linhas de transmissão. As linhas de transmissão acesso em 26/08/2018. Adaptado. Acessado em 18/09/18.)
usualmente transportam energia elétrica em __________ tensão.
O transformador é um dispositivo que permite transformar baixa Com base nas informações fornecidas, responda ao que se pede:
tensão e __________ corrente em alta tensão e __________ corrente
Um transformador, presente nos postes da rua, é dimensionado
e vice-versa. No transformador, o fluxo magnético associado
para atender a um determinado número de residências e poderá
ao campo criado pela corrente __________ no primário gera uma
ter um mau funcionamento devido à sobrecarga, caso tenha que
corrente no secundário, conforme a lei de Faraday.
atender a uma carga extra de várias ligações clandestinas.
A alternativa que completa, corretamente, as lacunas é
A função do transformador é:
a) alta – alta – baixa – contínua.
a) transformar a corrente contínua (CC) em corrente alternada (AC).
b) alta – baixa – alta – alternada.
b) transformar a corrente alternada (AC) em corrente contínua (CC).
c) baixa – baixa – baixa – contínua.
c) baixar a alta tensão que vem das subestações para baixa
d) alta – alta – baixa – alternada. tensão usada nas residências.
e) baixa – baixa – alta – contínua. d) subir a tensão das subestações para alta tensão usada nas
residências.
11. (EFOMM) Uma partícula de massa m = 1,0 x 10-26 kg e carga
q = 1,0 nC, com energia cinética de 1,25 KeV, movendo-se na 14. (EN) Analise a figura a seguir.
direção positiva do eixo x, penetra em uma região do espaço onde
existe um campo elétrico uniforme de módulo 1,0 KV/m orientado
no sentido positivo do eixo y. Para que não ocorra nenhum desvio
da partícula nessa região, é necessária a existência de um campo
magnético de intensidade
Dado: 1 eV = 1,6 x 10-19 J
a) 1,0 mT c) 3,0 mT e) 5,0 mT
b) 2,0 mT d) 4,0 mT

12. (ESPCEX - AMAN) Dois fios longos e retilíneos 1 e 2, fixos e


paralelos entre si, estão dispostos no vácuo, em uma direção
perpendicular a um ·plano α. O plano α contém o ponto C conforme
representado no desenho abaixo. Os fios são percorridos por O gráfico da figura acima registra a variação do fluxo magnético,
correntes elétricas constantes, de mesmo sentido, saindo do plano Φ, através de uma bobina ao longo de 5 segundos. Das opções a
α para o observador. O fio 1 é percorrido por uma corrente elétrica seguir, qual oferece o gráfico da f.e.m induzida, ε, em função do
de intensidade i1 = 6A e o fio 2 por uma corrente de intensidade tempo?
i2 = 8A. O módulo do vetor indução magnética resultante no ponto a) d)
C devido às correntes i1 e i2 é

b) e)

c)

Dado: considere a permeabilidade magnética do vácuo igual a


4·π·10-7 T·m/A.
a) 8 ⋅ 10−7 T. c) 4 ⋅ 2 ⋅ 10−7 T. e) 2 ⋅ 2 ⋅ 10−7 T.
b) 6 ⋅ 2 ⋅ 10−7 T. d) 4 ⋅ 10−7 T.

PRÉ-VESTIBULAR PROENEM.COM.BR 243


FÍSICA II 26 REVISÃO: ELETROMAGNETISMO

15. (IFPE) Uma bobina chata representa um conjunto de N 17. (CPS)


espiras que estão justapostas, sendo essas espiras todas iguais
e de mesmo raio. Considerando que a bobina da figura abaixo tem
resistência de R = 8 Ω, possui 6 espiras, o raio mede 10 cm, e ela
é alimentada por um gerador de resistência interna de 2 Ω e força
eletromotriz de 50 V, a intensidade do vetor indução magnética no
centro da bobina, no vácuo, vale:
Dado: µo = 4π · 10-7 T·m/A (permeabilidade magnética no vácuo)

a) 2π · 10-5 T
b) 4π · 10-5 T Para vender a fundições que fabricam aço, as grandes indústrias
c) 6π · 10-5 T de reciclagem separam o ferro de outros resíduos e, para realizar a
separação e o transporte do ferro, elas utilizam grandes guindastes
d) 8π · 10-5 T
que, em lugar de possuírem ganchos em suas extremidades,
e) 9π · 10-5 T possuem
a) bobinas que geram corrente elétrica.
16. (FUVEST) Em uma aula de laboratório, os estudantes foram
divididos em dois grupos. O grupo A fez experimentos com o b) bobinas que geram resistência elétrica.
objetivo de desenhar linhas de campo elétrico e magnético. Os c) dínamos que geram campo magnético.
desenhos feitos estão apresentados nas figuras I, II, III e IV abaixo. d) eletroímãs que geram corrente elétrica.
e) eletroímãs que geram campo magnético.

18. (UFRGS) A figura abaixo representa três posições, P1, P2 e P3,


de um anel condutor que se desloca com velocidade v constante
numa região em que há um campo magnético B, perpendicular ao
plano da página.

Com base nestes dados, é correto afirmar que uma corrente elétrica
induzida no anel surge
a) apenas em P1.
b) apenas em P3.
c) apenas em P1 e P3.
d) apenas em P2 e P3.
e) em P1, P2 e P3.
Aos alunos do grupo B, coube analisar os desenhos produzidos
pelo grupo A e formular hipóteses. Dentre elas, a única correta é 19. (UFSM) O alto-falante, usado na comunicação, em megafones,
que as figuras I, II, III e IV podem representar, respectivamente, rádios, televisões, tem o seu princípio de funcionamento ligado à
linhas de campo lei de
a) eletrostático, eletrostático, magnético e magnético. a) Coulomb.
b) magnético, magnético, eletrostático e eletrostático. b) Ohm.
c) eletrostático, magnético, eletrostático e magnético. c) Joule.
d) magnético, eletrostático, eletrostático e magnético. d) Ampère.
e) eletrostático, magnético, magnético e magnético. e) Faraday.

244 PROENEM.COM.BR PRÉ-VESTIBULAR


26 REVISÃO: ELETROMAGNETISMO FÍSICA II


20. (ESPCEX - AMAN) Sob a ação exclusiva de um campo 02. (UERJ) Em um campo magnético uniforme B de intensidade
magnético uniforme de intensidade 0,4 T, um próton descreve igual a 2,0 x 10-3 T, um fio condutor com 50 cm de comprimento
um movimento circular uniforme de raio 10 mm em um plano é posicionado perpendicularmente à direção do campo, conforme
perpendicular à direção deste campo. A razão entre a sua massa e mostra o esquema.
a sua carga é de 10-8 kg/C. A velocidade com que o próton descreve
este movimento é de:
a) 4·105 m/s
b) 2·105 m/s
c) 8·104 m/s
d) 6·104 m/s
e) 5·103 m/s

EXERCÍCIOS DE
APROFUNDAMENTO
01. (EBMSP) A espectrometria de massas é uma poderosa
ferramenta física que caracteriza as moléculas pela medida da Sabendo que a corrente elétrica i estabelecida no condutor é
relação massa/carga de seus íons. Ela foi usada, inicialmente, contínua e igual a 300 mA, determine, em newtons, a intensidade
na determinação de massas atômicas e vem sendo empregada da força F que age no condutor.
na busca de informações sobre a estrutura de compostos
orgânicos, na análise de misturas orgânicas complexas, na 03. (FMJ) Duas placas longas, planas e eletrizadas com sinais
análise elementar e na determinação da composição isotópica opostos e de mesmo módulo, dispostas paralelamente e
dos elementos. A espectrometria de massas acoplada, MS/MS, é distanciadas de 20 cm uma da outra, apresentam entre si diferença
uma técnica analítica poderosa, usada para identificar compostos de potencial 200 V. Uma carga elétrica q, de sinal negativo e
desconhecidos, quantificar compostos conhecidos e auxiliar na peso desprezível, é mantida em movimento entre as placas,
elucidação estrutural de moléculas. A MS/MS apresenta uma vasta paralelamente a elas e com velocidade v igual a 100 m/s, como
gama de aplicações, como por exemplo: na ecologia, na toxicologia, mostra a figura.
na geologia, na biotecnologia, e na descoberta e desenvolvimento
de fármacos.
Disponível em: <http://www.ufrgs.br/uniprote-ms/Content/
02PrincipiosDeAnalise/espectometria.html>. Acesso em: set. 2017.

a) Represente na figura abaixo os vetores campo elétrico e força


elétrica atuantes na carga, enquanto ela estiver na região
central entre as duas placas.

Considere a figura que representa, na forma de um esquema


simplificado, um espectrômetro de massa, sendo F a fonte de
íons, que são acelerados pela diferença de potencial
 ∆V, entram b) Considere desprezíveis os efeitos de bordas das placas
na região onde existe o campo magnético B e descrevem uma eletrizadas e que a intensidade da força magnética atuante na
trajetória semicircular. carga q seja dada por Fmag = Bqv senθ, em que B é a intensidade
Sabendo que os íons são compostos de partículas idênticas, do campo magnético e θ é o ângulo formado entre as linhas
cada uma eletrizada com a carga igual a 1,0·10-6 C e com massa, do campo magnético com a direção de v. Determine  o módulo,
1,0·10-14 kg, que penetram, perpendicularmente, na região do em tesla, e o sentido do vetor campo magnético B que deve ser
campo magnético uniforme com velocidade de módulo 106 m/s e aplicado na região central entre as placas e perpendicularmente
descrevem trajetória semicircular de raio 1,0 mm, ao plano da figura, para manter a velocidade da carga constante
Determine a intensidade do campo magnético. em módulo e direção.

PRÉ-VESTIBULAR PROENEM.COM.BR 245


FÍSICA II 26 REVISÃO: ELETROMAGNETISMO

04. (UEMA) A formação de imagem em um tubo de uma televisão EXERCÍCIOS DE APROFUNDAMENTO


é uma importante aplicação da força magnética que atua sobre 01. 10 T
uma carga elétrica em movimento. Suponha que uma partícula 02. 3,0 · 10–4 N
carregada penetre num tubo de imagem em que existe um campo  
03. a) Representações dos vetores campo elétrico E e força elétrica Fe :
magnético uniforme com velocidade “v”, perpendicular às linhas de
campo. A partir daí, realiza um movimento circular uniforme de raio
R = 1,0 cm, cujo período é T = 3,14 x 10-6 s.
a) Ilustre por meio de um desenho “esquema” o fenômeno
descrito acima.
b) Explique o porquê de a carga descrever um MCU.
c) Determine a intensidade do campo, considerando a carga da
partícula q = 2,0 x 10-15 C e sua massa m = 6,0 x 10-25 kg. b) Como a velocidade da partícula é constante, temos um equilíbrio dinâmico e a resultante
d) Calcule o módulo da velocidade da partícula para os valores de: das forças é nula. Portanto, em módulo, as forças elétrica e magnética são iguais:
q = 4,0 x 10-15 C, B = 4 x 10-4 T e m = 8,0 x 10-25 kg.  
Fmag = Fe ⇒ q B v sen θ = q E ⇒ B =
V
⇒B =
200 V
∴B = 10 T
d v sen θ 0,2 m ⋅ 100 m / s ⋅ sen 90°

05. (UNESP) Em muitos experimentos envolvendo cargas elétricas, Usando a regra da mão direita, determinamos a direção e sentido do campo magnético B
é conveniente que elas mantenham sua velocidade vetorial que aponta perpendicular ao plano da folha entrando nela, representada abaixo:
constante. Isso pode ser conseguido fazendo a carga movimentar-
se em umaregião onde atuam um campo elétrico E e um campo
magnético B, ambos uniformes e perpendiculares entre si. Quando
as magnitudes desses campos são ajustadas convenientemente, a
carga atravessa a região em movimento retilíneo e uniforme.
A figura representa um dispositivo cuja finalidade é fazer com
que uma partícula eletrizada com carga elétrica q > 0 atravesse
uma região entre duas placas paralelas P1 e P2, eletrizadas com
cargas de sinais opostos, seguindo a trajetória indicada pela linha

tracejada. O símbolo x representa um campo magnético uniforme 04. a) Esquema de uma partícula de carga q negativa entrando com velocidade v
B = 0,004 T, com direção horizontal, perpendicular ao plano que perpendicularmente em uma região onde o campo magnético B é de intensidade 
constante, sofrendo mudança de trajetória devido à ação de uma força magnética Fm .
contém a figura e com sentido para dentro dele. As linhas verticais,
ainda não orientadas e paralelas entre si, representam as linhas de
força de um campo elétrico uniforme de módulo E = 20 N/C.

b) O motivo para a partícula descrever um MCU (movimento circular uniforme) é


que, nestas condições, há a presença de uma força magnética constante atuando
perpendicularmente à velocidade e ao campo magnético, no caso para a esquerda
da partícula, causando uma mudança de trajetória para o MCU pela presença de uma
aceleração centrípeta constante que aponta para o centro da curva que é a força
magnética.
c) 6 · 10–4 T

Desconsiderando a ação do campogravitacional sobre a partícula d) 2 · 104 m/s



e considerando que os módulos de B e E sejam ajustados para que 05. 5 x 103 m/s
a carga não desvie quando atravessar o dispositivo, determine,
justificando, se as linhas de força do campo elétrico devem ser
ANOTAÇÕES
orientadas no sentido da placa P1 ou da placa P2 e calcule o módulo
da velocidade v da carga, em m/s.

GABARITO
EXERCÍCIOS PROPOSTOS
01. E 06. B 11. E 16. A
02. C 07. E 12. E 17. E
03. B 08. E 13. C 18. C
04. C 09. A 14. B 19. E
05. A 10. D 15. C 20. A

246 PROENEM.COM.BR PRÉ-VESTIBULAR


27 PRINCÍPIOS DA ÓPTICA, FÍSICA II

REFLEXÃO E ESPELHOS PLANOS

OS FUNDAMENTOS DA ÓPTICA • Feixe Cônico Convergente: todos os raios de luz convergem


para um mesmo ponto. Consegue-se um feixe convergente
GEOMÉTRICA com a ajuda de uma lente convergente, por exemplo.
A Óptica Geométrica é a parte da Física que estuda a
propagação da luz.

A LUZ V
Definimos como luz uma energia radiante que através dos
nossos órgãos visuais nos causa a sensação da visão. Diz-se que
a luz é uma energia radiante, pois ela se propaga por meio de ondas
eletromagnéticas.
Uma característica importantíssima das ondas eletromagnéti-
cas é a sua enorme velocidade. Todas as ondas eletromagnéticas • Feixe Cônico Divergente: todos os raios de luz divergem
possuem a mesma velocidade no vácuo que é representada pela de um mesmo ponto. Num farol de automóvel o feixe é
letra c e corresponde a maior velocidade possível na natureza. A divergente, emitida numa direção tal que não atrapalhe a
velocidade da luz no vácuo vale, aproximadamente: visão do motorista que vem em sentido contrário.
c = 300.000 km/s = 3 · 105 km/s = 3 · 108 m/s
Como já foi dito, a velocidade da luz possui este valor no vácuo.
Nos meios materiais, a velocidade será menor e diferente para cada
onda eletromagnética.
V
Ano-luz é uma unidade de distância e corresponde à distância
percorrida pela luz no vácuo em um ano, e vale aproximadamente:
1 ano-luz = 9,5 x 1012 km

Fontes de luz
Uma fonte de luz é qualquer corpo visível, isto é, que emite luz • Feixe Cilíndrico Paralelo: os raios luminosos são paralelos.
para os nossos olhos. Nos holofotes, o feixe de luz é cilíndrico paralelo.
As fontes podem ser classificadas em:
• Corpo luminoso ou fonte primária: são os que possuem
luz própria: O Sol, uma lâmpada ou uma vela acesa etc.
• Corpo iluminado ou fonte secundária: são os que não
possuem luz própria, eles apenas refletem a luz que incide
sobre eles: a Lua, um livro, um ser humano etc.
• Fonte pontual ou puntiforme: são aquelas que possuem
dimensões desprezíveis em face de outras dimensões
envolvidas: uma estrela nos parecerá um pontinho
luminoso, uma lâmpada bem distante do observador
também será uma fonte pontual.
• Fonte extensa: uma fonte é dita extensa quando não Meio de propagação
é possível desprezar suas dimensões: uma lâmpada Quanto à propagação da luz, os meios podem ser classificados
fluorescente próxima do observador. em:
• transparente - é aquele meio que permite a visualização
Raios de luz e feixes de luz nítida dos objetos, pois a luz se propaga em trajetórias bem
• Raio de Luz: é a representação da trajetória da luz, que definidas;
indica sua direção e sentido. De cada ponto de uma fonte • translúcido - permite a visualização não claramente,
de luz saem infinitos raios de luz. devido às trajetórias distorcidas da luz;
• opaco - não ocorre propagação da luz.

PRINCÍPIOS DA ÓPTICA GEOMÉTRICA


Os princípios que serão estudados são necessários no estudo
das trajetórias descritas pela luz na sua propagação. Estudaremos
os três princípios mais importantes.

PRÉ-VESTIBULAR PROENEM.COM.BR 247


FÍSICA II 27 PRINCÍPIOS DA ÓPTICA, REFLEXÃO E ESPELHOS PLANOS

PRINCÍPIO DA INDEPENDÊNCIA DOS Entre o disco e o anteparo, observa-se uma área que não é
iluminada pela fonte F e, considerando esta como a única fonte
RAIOS LUMINOSOS luminosa, esta região não iluminada consistirá em uma sombra.
Afirma que os raios de luz são independentes, isto é, mesmo No anteparo, como está na figura, ocorre uma região denominada
ocorrendo o cruzamento, um raio não afeta a trajetória do outro. de sombra projetada.

SOMBRA E PENUMBRA
Se um sistema for iluminado por uma fonte extensa ao invés de
uma fonte pontual, ocorrerá, além das regiões de sombra e sombra
projetada, áreas de penumbra e penumbra projetada, que consistem
em regiões que são parcialmente iluminadas e serão uma transição
entre áreas perfeitamente iluminadas a áreas de sombra total.

A figura anterior, em que duas áreas são ilumindas por


holofotes, exemplifica bem a independência dos raios luminosos.

PRINCÍPIO DA PROPAGAÇÃO RETILÍNEA


DA LUZ
Nos meios transparentes, homogêneos e isótropos, a luz se
propaga em linha reta. Um grande exemplo para este princípio
acontece quando raios de luz solar penetram através de uma janela
de uma sala.
ECLIPSES
Os casos mais importantes de sombra e penumbra são os
PRINCÍPIO DA REVERSIBILIDADE DOS eclipses solares e lunares.
RAIOS LUMINOSOS O eclipse lunar acontece quando a Lua penetra no cone de
Por um princípio conhecido como da mínima-ação, sabemos sombra da Terra, como mostra a figura abaixo. Os eclipses lunares
que a luz percorre entre dois pontos o caminho de menor “esforço”. ocorrem somente em Lua cheia.
Portanto, se a luz descreve uma trajetória do ponto A ao ponto B, o
caminho contrário, ou seja, do ponto B ao ponto A, será exatamente
o mesmo.
Cone de sombra da terra

Os eclipses solares são mais amplos. Existem os eclipses


anulares, que ocorrem quando o cone de sombra da lua se fecha
antes da Terra e permanece apenas a penumbra projetada. A figura
abaixo exemplifica um eclipse anular.
APLICAÇÕES DOS PRINCÍPIOS DA
ÓPTICA
A seguir, veremos alguns fenômenos físicos que podem ser
explicados através dos princípios da óptica geométrica:
O eclipse do Sol ocorre na Lua nova e, neste caso, a Lua projeta
sobre a Terra uma região de sombra projetada, eclipse total, e uma
SOMBRA região de penumbra projetada.
Na figura abaixo podemos identificar uma fonte luminosa
puntiforme representada por F, um disco opaco D e um anteparo A.

• Região 1 = cone de sombra da Lua


• Região 2 = penumbra
• Região 3 = sombra da Lua projetada na Terra. Nessa região
ocorre o eclipse total do Sol.
• Região 4 = penumbra projetada. Nessa região ocorre o
eclipse parcial do Sol, caso em que uma parte do “disco
solar” permanece visível.

248 PROENEM.COM.BR PRÉ-VESTIBULAR


27 PRINCÍPIOS DA ÓPTICA, REFLEXÃO E ESPELHOS PLANOS FÍSICA II

FASES DA LUA CÂMARA ESCURA


A Lua gira em torno da Terra em uma órbita elíptica levando Na figura a seguir podemos identificar uma câmara escura
aproximadamente 27,3 dias para completar uma volta. Durante com um orifício e diante desta caixa, no lado que contém o orifício,
este período, dependendo de sua posição, a Lua é vista de formas há um objeto luminoso AB. Os raios que partem do objeto AB
diferentes da Terra. Essas formas diferentes como a Lua é atravessam o orifício O, penetram na câmara e projetam na parede
iluminada denominam-se de fases da Lua. oposta uma imagem semelhante ao objeto, porém invertida.
Na Lua Nova, o hemisfério voltado para a Terra não é iluminado,
e na Lua Cheia, o hemisfério em questão é totalmente iluminado.
Nas fases denominadas de Quarto- Crescente e Quarto-Minguante,
o hemisfério voltado para a Terra é parcialmente iluminado. Na
figura abaixo, estão todas as fases da Lua.

Pela geometria concluímos que os triângulos OAB e OA’B’ são


semelhantes, desta forma:
a AB
=
b A'B'
Sendo AB e A’B’ os comprimentos do objeto e da imagem e a
e b as distâncias entre o objeto e o orifício e a imagem e o orifício,
respectivamente.
Apenas uma das faces da Lua permanece voltada para a Terra,
isso acontece pois o período de rotação em torno do seu próprio ÂNGULO VISUAL
eixo é igual ao seu período de translação em torno da Terra. O lado
oculto ou lado negro da Lua já foi tema de muitas abordagens na O ângulo visual representado na figura a seguir por α,
Arte, como o cinema e a música. corresponde ao ângulo formado pelos raios que ligam as
extremidades do objeto e os olhos do observador.
PROEXPLICA

COMO ECLIPSE PODE CAIR NO ENEM?


O eclipse solar é um fenômeno que ocorre de tempos em
tempos, porém, só é possível observá-lo em determinadas
posições do planeta, devido às dimensões do Sol e da Lua. A
questão aborda a relação entre a sombra vista em um eclipse
solar e a posição em que se encontra o observador.
REFLEXÃO DA LUZ E ESPELHOS PLANOS
A reflexão da luz ocorre quando um raio luminoso atinge uma
EXERCÍCIO RESOLVIDO superfície e retorna para o meio no qual se propagava.
Os elementos que devem ser conhecidos:
01. (ENEM) A figura abaixo mostra um eclipse solar no instante RI → Raio Incidente.
em que é fotografado em cinco diferentes pontos do planeta. RR → Raio Refletido.
N → Normal no ponto de incidência.
(S)→ Símbolo de superfície plana refletora: o espelho plano.
As reflexões podem ser classificadas em:
• Reflexão Regular ou Especular: Raios incidentes paralelos
refletem paralelos. É a que ocorre numa superfície polida. É
a que estudaremos no nosso curso.
• Reflexão Difusa: Raios incidentes paralelos não refletem
paralelos.
A reflexão regular ou especular é regida pelas seguintes leis:
• 1ª LEI: O raio incidente, a normal no ponto de incidência
Três dessas fotografias estão reproduzidas abaixo. e o raio refletido estão no mesmo plano (são coplanares).
• 2ª LEI: O ângulo de incidência (i) é igual ao ângulo de
reflexão (r).

As fotos poderiam corresponder, respectivamente, aos pontos:


a) III, V e II c) II, IV e III e) I, II e V
b) II, III e V d) I, II e III

Resolução: A

PRÉ-VESTIBULAR PROENEM.COM.BR 249


FÍSICA II 27 PRINCÍPIOS DA ÓPTICA, REFLEXÃO E ESPELHOS PLANOS

EXERCÍCIO RESOLVIDO
ESPELHOS PLANOS
O espelho plano é o sistema óptico mais simples de ser estudado
e é também o mais facilmente encontrado no dia a dia. Constitui-
02. (UECE) Dois espelhos planos são dispostos paralelos um
se de uma superfície polida onde ocorre predominantemente a
ao outro e com as faces reflexivas viradas uma para outra.
reflexão da luz. No estudo de suas propriedades utilizaremos,
Em um dos espelhos incide um raio de luz com ângulo de
basicamente, os princípios da óptica geométrica.
incidência de 45º. Considerando que haja reflexão posterior
no outro espelho, o ângulo de reflexão no segundo espelho é
a) 45º. c) 90º. e) 10,0°. Imagem conjugada por um espelho plano
b) 180º. d) 22,5º. Para estudarmos o comportamento da imagem de um ponto
material produzida por um espelho plano, consideraremos um
Resolução: A ponto P diante um espelho. A luz que parte de P incide obliquamente
no espelho e sofre reflexão. A figura ao lado ilustra a situação e
Construindo os raios incidentes e refletidos:
podemos perceber que os prolongamentos dos raios refletidos irão
se cruzar no ponto P’, por onde também passa a reta normal (reta
perpendicular ao sistema óptico). O observador enxerga o ponto P’
como imagem do ponto P.

De acordo com a figura, podemos concluir que o ângulo de


reflexão no segundo espelho é de 45º.

REFLEXÃO SELETIVA DA COR DOS CORPOS Sobre a imagem é importante saber que ela e o objeto são
simétricos em relação ao espelho plano, ou seja, a distância do
A luz branca (policromática) é a mistura de todas as cores objeto ao espelho é igual a deste à imagem e pertencem a mesma
(monocromáticas) básicas do “arco-íris”: vermelho, alaranjado, reta normal à superfície do espelho. Esta propriedade dos espelhos
amarelo, verde, azul, anil e violeta. Lembrando que existem vários tons, planos, da imagem ser simétrica ao objeto, é conhecida como
portanto as sete cores do arco-íris na verdade são diversas cores Propriedade Fundamental dos Espelhos Planos.
formando um espectro contínuo.
O estudo torna-se um pouco mais complexo se tratarmos
Um corpo que apresenta aos nossos olhos a cor vermelha de corpos extensos, porém a propriedade da simetria continuará
é porque ele tem a capacidade de refletir apenas a luz vermelha, a ser aplicada. Quando tratamos de corpos extensos dizemos
qualquer cor que incida sobre ele será absorvida, assim é também que o objeto e a imagem são enantiomorfos e isso significa, que
com outras cores: um corpo é azul se tem a capacidade de o lado direito do objeto corresponde ao esquerdo da imagem. Na
refletir o azul e absorver as outras cores. Um corpo verde se figura acima um triângulo ABC foi colocado diante de um espelho
apresentará com a cor verde se for iluminado pela luz branca plano formando uma imagem dele enantiomorfa. Observe que as
(já que ela contém a verde) ou por uma mistura qualquer que distâncias de A, B e C ao espelho são iguais, respectivamente, as
contenha verde, caso contrário ele se apresentará aos nossos distâncias do espelho as suas imagens A’, B’ e C’.
olhos como negro.

PROEXPLICA

Existe também o fenômeno da refração seletiva e acontece


quando uma luz policromática incide em uma superfície
monocromática que funciona como um filtro. Neste caso, se S
a luz policromática possuir a cor do filtro, este permitirá que
somente esta luz passe. Na figura abaixo, a luz branca incide em
um filtro vermelho e somente a luz vermelha emergirá do filtro.

Pegaremos como exemplo agora a palavra FÍSICA. Se


colocarmos um espelho à sua direita, teremos a seguinte imagem:

250 PROENEM.COM.BR PRÉ-VESTIBULAR


27 PRINCÍPIOS DA ÓPTICA, REFLEXÃO E ESPELHOS PLANOS FÍSICA II

O aluno fatalmente encontrará questões que se referem a Translação de um espelho plano


palavras, figuras ou objetos, como relógios diante de espelhos e
O movimento de translação de um espelho plano consiste
perguntas a respeito da imagem conjugada. Algumas figuras que
em deslocar o espelho no mesmo eixo onde se encontra o objeto.
possuem um eixo de simetria, como a letra A, por exemplo, não
Na figura abaixo temos um espelho plano que foi deslocado
apresentarão problemas, mas na maioria deve-se identificar a
horizontalmente para direita de uma distância x. Com isso a
imagem enantiomorfa.
imagem se deslocou de y. Pela figura deduz-se que:
Uma maneira prática de obtermos a imagem corretamente
seria escrevermos a palavra ou figura em uma folha de papel,
girarmos a folha ao redor da borda onde estaria o espelho e
olharmos as costas da folha.

A “sombra” da palavra vista por trás corresponde à imagem do


objeto obtida pelo espelho.

EXERCÍCIO RESOLVIDO
PROEXPLICA
03. (UERN) Na noite do réveillon de 2013, Lucas estava
usando uma camisa com o ano estampado na mesma. Ao Observe o esquema que mostra um espelho E que se
visualizá-la através da imagem refletida em um espelho desloca uma distância x, produzindo um deslocamento y
plano, o número do ano em questão observado por Lucas se na imagem de um objeto que permanece em repouso em
apresentava da seguinte forma relação ao solo.
a) c)
b) d)

Resolução: B
No espelho plano, objeto e imagem são simétricos em
relação ao plano do espelho. Como consequência, a imagem
é enantiomorfa em relação ao objeto.

Campo visual
O campo visual corresponde à área que pode ser visualizada
pelo observador olhando para o espelho.

Conseguimos pela imagem, lembrando da propriedade


de simetria, duas relações:
D = d + x (eq.1)
O campo visual pode ser facilmente determinado traçando-se
d + y = D + x (eq.2)
raios ligando o observador aos extremos do espelho que sofrerão
reflexão. A área compreendida entre os raios refletidos determinará Somando as equações 1 e 2, temos:
o campo visual. Para traçarmos os raios refletidos devemos obter D+d+y=d+x+D+x
a posição simétrica do observador e traçar retas que liguem esta y = 2·x
posição aos extremos dos espelhos, como mostra a figura abaixo:
Observa-se que a distância y percorrida pela imagem é o
dobro da distância x percorrida pelo espelho. Resumindo, se o
espelho se afastar 1 metro do objeto, a imagem vai se afastar 2
metros do objeto; caso o espelho se afaste 4 metros do objeto, a
imagem vai se afastar 8 metros do objeto.

PRÉ-VESTIBULAR PROENEM.COM.BR 251


FÍSICA II 27 PRINCÍPIOS DA ÓPTICA, REFLEXÃO E ESPELHOS PLANOS

Quando giramos um espelho de um ângulo igual a θ o raio


PROEXPLICA refletido também sofre um desvio. Representaremos esse desvio
por Δ.
Caso o objeto se movimente em relação ao espelho
com velocidade v de aproximação, a imagem também se
aproximará com velocidade v, em módulo.

Caso o objeto se afaste, a imagem também se afastará


do espelho, porém, o módulo da velocidade será sempre o
mesmo. Se o objeto permanece parado em relação ao solo
e o espelho se desloca com velocidade de módulo igual a v, a Na figura, os ângulos internos do triângulo CAB são, θ, 90°+ α
imagem se deslocará com velocidade igual a 2v em relação e β, logo:
ao objeto e V em relação ao próprio espelho.
θ + 90º + α + β = 180° ⇒ θ = 90° – α – β
No triângulo ABD temos:
EXERCÍCIO RESOLVIDO Δ + 2α + 2β = 180° ⇒ Δ = 180° – 2α – 2β ⇒ Δ = 2 (90° – α – β)
Logo: Δ = 2q
04. (UEMG) Um espelho reflete raios de luz que nele incidem. Além do raio refletido, a imagem também se altera. A figura
Se usássemos os espelhos para refletir, quantas reflexões ao lado P’ é a primeira imagem formada e P’’ é a imagem formada
interessantes poderíamos fazer. Enquanto a filosofia se após o deslocamento angular do espelho. A imagem executa um
incumbe de reflexões internas, que incidem e voltam para movimento circular. O ângulo q corresponde ao deslocamento
dentro da pessoa, um espelho trata de reflexões externas. angular da imagem e vale o dobro do deslocamento do espelho.
Mas, como escreveu Luiz Vilela, “você verá.”
Você está diante de um espelho plano, vendo-se
totalmente. Num certo instante, e é disso que é feita a vida, de
instantes, você se aproxima do espelho a 1,5 m/s e está a 2,0 m
de distância do espelho.
Nesse instante, a sua imagem, fornecida pelo espelho, estará
a) a 2,0 m de distância do espelho, com uma velocidade de
3,0 m / s em relação a você.
b) a 2,0 m de distância do espelho, com uma velocidade de
1,5 m / s em relação a você.
c) a uma distância maior que 2,0 m do espelho, com uma
velocidade de 3,0 m / s em relação ao espelho.
d) a uma distância menor que 2,0 m do espelho, com uma
velocidade de 1,5 m / s em relação ao espelho.
e) a 4,0 m de distância do espelho, com uma velocidade de
1,5 m/s em relação a você.
Dessa forma, se imaginarmos a velocidade angular do espelho
Resolução: A (wE) e da imagem (wi), teremos a seguinte relação:
Num espelho plano, objeto e respectiva imagem são
wi = 2wE
simétricos em relação ao plano do espelho. Portanto, quando
você está a 2 m do espelho sua imagem também está a 2
m dele. Devido a essa mesma propriedade (simetria) a EXERCÍCIO RESOLVIDO
velocidade da imagem em relação ao espelho é, em módulo,
igual à do objeto, porém em sentido oposto.
05. (UNESP) Um raio de luz, vertical, incide num espelho
Assim, se você se aproxima do espelho com velocidade de plano horizontal. Se o espelho girar 20 graus em torno de
módulo 1,5 m/s sua imagem também se aproxima com 1,5 m/s. um eixo horizontal, o raio refletido se desviará de sua direção
Então, relativamente a você, a velocidade de sua imagem tem original de
módulo 3,0 m/s.
a) 0° c) 10° e) 40°
b) 20° d) 60°

Rotação do espelho plano Resolução: E


O movimento de rotação do espelho plano acontece quando ∆=2.θ
um espelho plano gira ao redor de um ponto fixo (ponto C da ∆ = 2 . 20º
figura). Sobre este movimento podem ser feitas duas perguntas: o
desvio angular do raio incidente e a velocidade angular da imagem. ∆ = 40º

252 PROENEM.COM.BR PRÉ-VESTIBULAR


27 PRINCÍPIOS DA ÓPTICA, REFLEXÃO E ESPELHOS PLANOS FÍSICA II

Associação de espelhos planos


Resolução: C
Podemos obter inúmeras imagens através da associação de
espelhos planos. Isso se deve a propriedade dos espelhos planos Fazendo o desenho do movimento da Mônica e rebatendo-o
de sua imagem também se constituir em um espelho. Na figura nos espelhos, temos:
abaixo, temos um ponto P entre dois espelhos planos que formam
um ângulo α.

O número de imagens formadas pode ser obtido através da


fórmula:

360°
n= −1
α
Assim, a imagem em R2 sofre dois rebatimentos totalizando
Porém, deve-se ter atenção, pois caso n seja ímpar, a fórmula uma rotação de 180º, conforme a figura acima.
é aplicada para qualquer posição de P. Se n for par, seu valor será
correto somente se P estiver a uma mesma distância dos espelhos.
Pela figura a seguir verificamos que a conjugação de imagens só Dimensão do espelho plano
não ocorre quando a imagem se encontra na chamada zona morta. Vamos obter o tamanho mínimo do espelho para que uma
pessoa possa se ver por inteiro.

Os pontos P’’2 e P’’1 constituem somente uma imagem. H – Altura da pessoa.


É válido lembrar que as imagens formadas por reflexão simples h – Altura dos olhos em relação ao solo.
(somente uma reflexão) serão enantiomorfas, porém as imagens x – Tamanho do espelho.
formadas por duas reflexões serão iguais ao objeto. Assim sendo, Y – Distância do espelho ao solo.
as imagens formadas por número ímpar de reflexões serão
enantiomorfas e as formadas por número par serão iguais ao objeto. Por semelhança de triângulos, temos:

H 2d H h 2d h
EXERCÍCIO RESOLVIDO = → x= = → Y=
x d 2    Y d 2

06. (FCMMG) Dois espelhos perpendiculares entre si estão O espelho precisa ter, no mínimo, metade da altura da pessoa,
posicionados em paredes verticais de um shopping. Mônica além de estar localizado a uma distância do solo que igual à metade
move-se entre eles na direção de Pedro, que está sentado da altura dos olhos da pessoa em relação ao chão.
num banco, também entre os espelhos, como mostrado na
figura, vista do alto.
EXERCÍCIO RESOLVIDO

07. (UFJF-PISM 2) Manuela deve comprar um espelho para


instalar em seu quarto. Ela pretende comprar um espelho que
permita ver sua imagem completa refletida nele. Sabendo que
Manuela tem 1,70 m de altura e que seus olhos estão a 1,55 m
do chão, ajude-a a realizar sua escolha, calculando o que se pede.
a) A máxima altura em relação ao solo onde pode ser
colocada a base do espelho.
b) A altura mínima em relação ao solo onde pode ser
Pedro observa três imagens da Mônica, através dos espelhos, colocado o topo do espelho.
nas regiões R1, R2 e R3. O sentido do movimento de Mônica
observado por Pedro na região R2 é representado pela seta:

a) b) c) d)

PRÉ-VESTIBULAR PROENEM.COM.BR 253


FÍSICA II 27 PRINCÍPIOS DA ÓPTICA, REFLEXÃO E ESPELHOS PLANOS

02. Neymar está à frente de uma mesa, um copo de suco e um


Resolução: espelho plano vertical, conforme representado na imagem:
Dados: H = 1,70 m; h = 1,55 m.
Considerando que o referido espelho seja plano, objeto e
imagem são simétricos em relação ao plano do espelho. A
figura ilustra a situação.

a) Seja y a distância da borda inferior do espelho ao solo e d


a distância de Manuela ao espelho.
Por semelhança de triângulos:
y h h 1,55
ΔNQP' ≈ ΔGPP' ⇒ = ⇒ y= = ⇒ y= 0,775 m.
d 2d 2 2

b) Por semelhança de triângulos, encontra-se também a


altura mínima x desse espelho: Imagem fora de escala
x H H 1,70
Δ GMN ≈ ΔGC'P' ⇒ = ⇒ x= = ⇒ x = 0,85 m. Determine a distância da imagem do copo até os olhos do jovem
d 2d 2 2 Neymar. Considere o copo um objeto pontual sobre a mesa.
A distância do topo do espelho ao solo é:
03. Usain Bolt corre na direção de um espelho plano com velocidade
He = x + y = 0,85 + 0,775 ⇒ He = 1,625 m. de 12 m/s, conforme representado na imagem:

EXERCÍCIOS
PROTREINO
01. A imagem representa a reflexão de um raio luminoso num
espelho plano.

Considerando que o espelho está parado em relação ao solo,


determine:
a) A velocidade da imagem de Bolt em relação ao espelho;
b) A velocidade da imagem de Bolt em relação ao próprio Bolt.

04. Uma esfera é coloca entre dois espelhos planos, conforme


representado na imagem:

Determine:
a) O ângulo de incidência;
b) O ângulo de reflexão;
c) O desvio de raio refletido se o espelho rotacionar 25° no sentido Calcule o número de imagens formadas quando o ângulo entre os
horário. espelhos é de 45°.

254 PROENEM.COM.BR PRÉ-VESTIBULAR


27 PRINCÍPIOS DA ÓPTICA, REFLEXÃO E ESPELHOS PLANOS FÍSICA II

05. Um jovem nerd viaja com sua família para acompanhar um 03. (IFCE) Um garoto parado na rua vê sua imagem refletida por um
eclipse Solar que será visto do Egito. Seu irmão mais novo pede que espelho plano preso verticalmente na traseira de um ônibus que se
olhem no calendário em qual fase lunar acontecerá o fenômeno afasta com velocidade escalar constante de 36 km/h.
e sem olhar o jovem nerd responde corretamente que será na lua Em relação ao garoto e ao ônibus, as velocidades da imagem são,
nova. respectivamente,
Explique como o jovem conseguiu responder sem precisar conferir a) 20 m/s e 10 m/s. d) 10 m/s e 20 m/s.
no calendário.
b) Zero e 10 m/s. e) 20 m/s e 20 m/s.
c) 20 m/s e zero.

04. (UNIFOR) Ao acordar pela manhã, Camilla levantou-se e saiu


EXERCÍCIOS em direção perpendicular ao espelho plano colado à parede de seu
PROPOSTOS quarto, com velocidade constante de 45,0 cm/s. Nesta situação,
pode-se afirmar que
a) a imagem de Camilla aproximou-se dela a 45,0 cm/s.
01. (CFTMG) A figura abaixo mostra uma sequência de
experimentos em que um feixe de luz incide sobre um espelho b) a imagem de Camilla aproximou-se do espelho a 90,0 cm/s.
plano. No experimento I, o espelho está na horizontal e, nos c) a imagem de Camilla aproximou-se dela a 90,0 cm/s.
experimentos II e III, o espelho é inclinado de um ângulo α, para d) a imagem de Camilla afasta-se do espelho a 45,0 cm/s.
esquerda e para a direita, respectivamente. As linhas tracejadas
mostram três possíveis trajetórias que o feixe pode seguir, após e) a imagem de Camilla afasta-se dela a 90,0 cm/s.
refletir-se no espelho.
05. (UPF) Uma pessoa com visão perfeita observa um adesivo, de
tamanho igual a 6 mm, grudado na parede na altura de seus olhos.
A distância entre o cristalino do olho e o adesivo é de 3 m. Supondo
que a distância entre esse cristalino e a retina, onde se forma a
imagem, é igual a 20 mm, o tamanho da imagem do adesivo
formada na retina é:

As trajetórias corretas, observadas na sequência dos experimentos


I, II e III, após a reflexão, são, respectivamente,
a) 1, 2 e 3.
b) 2, 1 e 3. a) 4 x 10-3 mm. d) 5 x 10-4 mm.
c) 2, 3 e 1. b) 5 x 10-3 mm. e) 2 x 10-4 mm.
d) 2, 3 e 2. c) 4 x 10-2 mm.

02. (FAC. ALBERT EINSTEIN - MED) Um pequeno boneco está 06. (ENEM PPL) A figura mostra, de forma esquemática, uma
diante de um espelho plano, conforme a figura abaixo. representação comum em diversos livros e textos sobre eclipses.
Apenas analisando essa figura, um estudante pode concluir que
os eclipses podem ocorrer duas vezes a cada volta completa da
Lua em torno da Terra. Apesar de a figura levar a essa percepção,
algumas informações adicionais são necessárias para se concluir
que nem o eclipse solar, nem o lunar ocorrem com tal periodicidade.

A periodicidade dos eclipses ser diferente da possível percepção do


estudante ocorre em razão de
Em relação à imagem conjugada pelo espelho, podemos classificá- a) eclipses noturnos serem imperceptíveis da Terra.
la como tendo as seguintes características: b) planos das órbitas da Terra e da Lua serem diferentes.
a) real, direita e do mesmo tamanho do objeto. c) distância entre a Terra e a Lua variar ao longo da órbita.
b) virtual, invertida lateralmente e maior que o objeto. d) eclipses serem visíveis apenas em parte da superfície da Terra.
c) virtual, direita e do mesmo tamanho do objeto. e) o Sol ser uma fonte de luz extensa comparado ao tamanho da
d) real, invertida lateralmente e do mesmo tamanho do objeto. lua.

PRÉ-VESTIBULAR PROENEM.COM.BR 255


FÍSICA II 27 PRINCÍPIOS DA ÓPTICA, REFLEXÃO E ESPELHOS PLANOS

07. (ENEM PPL) Algumas crianças, ao brincarem de esconde- Esse instrumento simula o que acontece quando, devidamente
esconde, tapam os olhos com as mãos, acreditando que, ao protegidos, estamos observando, daqui da Terra, o Sol no momento
adotarem tal procedimento, não poderão ser vistas. Essa percepção em que ocorre um eclipse
da criança contraria o conhecimento científico porque, para serem a) lunar total, com a Lua se interpondo entre a Terra e o Sol.
vistos, os objetos
b) lunar parcial, com a Terra se interpondo entre a Lua e o Sol.
a) refletem partículas de luz (fótons), que atingem os olhos.
c) solar total, com a Lua se interpondo entre a Terra e o Sol.
b) geram partículas de luz (fótons), convertidas pela fonte externa.
d) solar total, com a Terra se interpondo entre a Lua e o Sol.
c) são atingidos por partículas de luz (fótons), emitidas pelos
olhos. e) solar parcial, com a Lua se interpondo entre a Terra e o Sol.

d) refletem partículas de luz (fótons), que se chocam com os


11. (FAMEMA) Ao entrar no banheiro de um shopping, uma pessoa
fótons emitidos pelos olhos.
se depara com uma parede onde se encontra afixado um grande
e) são atingidos pelas partículas de luz (fótons), emitidas pela espelho plano. Enquanto caminha com velocidade de 1 m/s
fonte externa e pelos olhos. em uma direção perpendicular a esse espelho e no sentido de
aproximar-se dele, essa pessoa observa que, relativamente a seu
08. (ENEM) É comum aos fotógrafos tirar fotos coloridas em corpo, sua imagem
ambientes iluminados por lâmpadas fluorescentes, que contêm a) se afasta com velocidade 1 m/s.
uma forte composição de luz verde. A consequência desse fato na
fotografia é que todos os objetos claros, principalmente os brancos, b) se aproxima com velocidade 2 m/s.
aparecerão esverdeados. Para equilibrar as cores, deve-se usar c) se aproxima com velocidade 4 m/s.
um filtro adequado para diminuir a intensidade da luz verde que d) se aproxima com velocidade 1 m/s.
chega aos sensores da câmera fotográfica. Na escolha desse filtro,
e) se afasta com velocidade 2 m/s.
utiliza-se o conhecimento da composição das cores-luz primárias:
vermelho, verde e azul; e das cores-luz secundárias: amarelo =
vermelho + verde, ciano = verde + azul e magenta = vermelho + azul. 12. (EEAR) Um dado, comumente utilizado em jogos, cujos números
Disponível em: http://nautilus.fis.uc.pt. Acesso em 20 maio 2014 (adaptado).
nas faces são representados pela quantidade de pontos pretos é
colocado frente a dois espelhos planos que formam entre si um
Na situação descrita, qual deve ser o filtro utilizado para que a ângulo de 60°. Nesses espelhos é possível observar nitidamente as
fotografia apresente as cores naturais dos objetos? imagens de apenas uma das faces do dado, sendo que a soma de
todos os pontos pretos observados nos espelhos, referentes a essa
a) Ciano.
face, totalizam 20 pontos. Portanto, a face voltada para os espelhos
b) Verde. que gera as imagens nítidas é a do número ____.
c) Amarelo. a) 1 b) 2 c) 4 d) 5
d) Magenta.
e) Vermelho. 13. (CFTRJ) Durante o ano de 2018, ocorreram 5 eclipses:
3 eclipses solares e 2 eclipses lunares. No Brasil, só nos foi possível
observar o eclipse lunar total que ocorreu em 27 de julho.
09. (ENEM 2ª APLICAÇÃO)
O eclipse lunar ocorre porque a Terra se alinha com o Sol e com Lua.
O Sol emite seus raios para a Terra que, devido a suas dimensões,
cria uma sombra na Lua. Este efeito será mais visível quanto melhor
for a visibilidade da Lua e, por isso, em alguns casos, o eclipse lunar
deixa a Lua com uma coloração próxima ao vermelho.
Podemos afirmar que a fase em que se encontrava a Lua para o
fenômeno do eclipse lunar total, observado no Brasil, era a:
a) Nova. c) Cheia.
b) Crescente. d) Minguante.

14. (FAMEMA) Tomando como referência a sombra gerada por uma


cadeira de 60 cm de altura, uma pessoa decidiu determinar a altura
Os quadrinhos mostram, por meio da projeção da sombra da árvore de um muro construído próximo à lateral de sua casa por meio de
e do menino, a sequência de períodos do dia: matutino, meio-dia e métodos geométricos. A casa, o muro e a cadeira estavam sobre o
vespertino, que é determinada mesmo chão horizontal e, como não era possível obter uma sombra
a) pela posição vertical da árvore e do menino. completa do muro, a pessoa providenciou um espelho plano que
prendeu paralelamente à lateral da casa, como mostra a figura, que
b) pela posição do menino em relação à árvore.
representa os resultados obtidos em um mesmo instante.
c) pelo movimento aparente do Sol em torno da Terra.
d) pelo fuso horário específico de cada ponto da superfície da
Terra.
e) pela estação do ano, sendo que no inverno os dias são mais
curtos que no verão.

10. (CPS) SOHO, abreviação de Solar & Heliospheric Observatory, é


uma sonda que orbita o Sol e coleta informações de nossa estrela
utilizando vários instrumentos. Um deles registra o comportamento
da coroa solar e, para isso, conta com um pequeno disco opaco que
fica estrategicamente posicionado à frente da câmera, ocultando a
visão do disco solar.

256 PROENEM.COM.BR PRÉ-VESTIBULAR


27 PRINCÍPIOS DA ÓPTICA, REFLEXÃO E ESPELHOS PLANOS FÍSICA II

A pessoa concluiu que o muro tinha uma altura de


a) 2,1 m.
b) 3,2 m.
c) 3,0 m.
d) 2,4 m.
e) 2,7 m.

15. (UNIOESTE) No dia 27 de julho deste ano de 2018, aconteceu


um fenômeno celeste denominado de “Lua de Sangue”.
Considerado o eclipse lunar com maior duração já ocorrido no
século 21, o fenômeno acontece devido à luz do Sol, que é refratada
pela atmosfera da Terra e chega à superfície da Lua no espectro do
vermelho.
(REVISTA GALILEU, Lua de Sangue: por que o eclipse será o mais longo do século?
Disponível em: https://revistagalileu.globo.com/Ciencia/noticia/2018/07/lua-de-sangue-
por-que-o-eclipse-sera-o-mais-longo-doseculo.html. Acesso em: 20 de agosto de 2018. Determine a menor distância entre o cliente e a imagem que ele
observa da sua nuca no espelho E1, considerando que a cabeça do
Sobre o fenômeno dos eclipses, a propagação da luz e as cores dos cliente também possui dimensões desprezíveis.
objetos, assinale a alternativa CORRETA.
a) 1,6 m
a) O eclipse lunar ocorre quando a Lua se encontra entre o Sol e a
Terra, em perfeito alinhamento, projetando sua sombra sobre a b) 2,0 m
superfície do planeta. c) 2,2 m
b) Eclipses são fenômenos que acontecem como consequência d) 2,6 m
imediata do princípio de propagação retilínea da luz. e) 3,2 m
c) O fenômeno da interferência explica a decomposição da luz
branca nas diversas cores que formam o espectro da luz visível 18. (ACAFE) Em tempos de crise econômica, uma pessoa deseja
quando essa atravessa a atmosfera terrestre. empreender montando uma pequena loja de roupas. Um dos itens
d) Dentre as cores visíveis, a vermelha é a que possui maior essenciais é colocar um espelho em uma parede vertical, de modo
energia, por isso ela consegue atravessar a atmosfera terrestre que qualquer cliente de média altura (h) possa se ver inteiro nesse
e atingir a superfície da Lua durante o eclipse. espelho a certa distância horizontal (d). Mas para economizar, o
espelho deverá ter a menor altura possível.
e) No fenômeno da “Lua de Sangue”, a Lua absorve apenas a
frequência do vermelho e reflete as demais frequências da luz A alternativa correta que indica a altura desse espelho é:
solar. a) h/3
b) h/d
16. (IFSUL) Na figura a seguir, está representado um espelho plano,
c) 2h/3
onde O é um observador, enquanto A, B e C são objetos pontuais.
d) h/2

19. (UECE) Dois espelhos planos são dispostos paralelos um ao


outro e com as faces reflexivas viradas uma para outra. Em um
dos espelhos incide um raio de luz com ângulo de incidência de
45°. Considerando que haja reflexão posterior no outro espelho, o
ângulo de reflexão no segundo espelho é
a) 45°.
b) 180°.
c) 90°.
d) 22,5°.

20. (UNICAMP) O Teatro de Luz Negra, típico da República Tcheca,


é um tipo de representação cênica caracterizada pelo uso do
cenário escuro com uma iluminação estratégica dos objetos
O observador poderá ver, por reflexão no espelho, o(s) objeto(s) exibidos. No entanto, o termo Luz Negra é fisicamente incoerente,
a) A e B. pois a coloração negra é justamente a ausência de luz. A luz
branca é a composição de luz com vários comprimentos de onda
b) B.
e a cor de um corpo é dada pelo comprimento de onda da luz que
c) C. ele predominantemente reflete. Assim, um quadro que apresente
d) B e C. as cores azul e branca quando iluminado pela luz solar, ao ser
iluminado por uma luz monocromática de comprimento de onda
17. (COTUCA) Um barbeiro B segura um espelho plano E2, de correspondente à cor amarela, apresentará, respectivamente, uma
espessura desprezível, paralelamente a outro espelho plano E1, coloração
também de espessura desprezível, permitindo que seu cliente A a) amarela e branca.
observe, no espelho E1, o seu corte de cabelo na parte posterior b) negra e amarela.
da cabeça.
c) azul e negra.
d) totalmente negra.

PRÉ-VESTIBULAR PROENEM.COM.BR 257


FÍSICA II 27 PRINCÍPIOS DA ÓPTICA, REFLEXÃO E ESPELHOS PLANOS

EXERCÍCIOS DE
APROFUNDAMENTO
01. (UERJ) A altura da imagem de um objeto, posicionado a uma
distância P1 do orifício de uma câmara escura, corresponde a 5%
da altura desse objeto. A altura da imagem desse mesmo objeto,
posicionado a uma distância P2 do orifício da câmara escura,
corresponde a 50% de sua altura.
Calcule P2 em função de P1.

02. (UNESP) Uma pessoa de 1,8 m de altura está parada diante de


um espelho plano apoiado no solo e preso em uma parede vertical.
Como o espelho está mal posicionado, a pessoa não consegue ver a
imagem de seu corpo inteiro, apesar de o espelho ser maior do que
o mínimo necessário para isso. De seu corpo, ela enxerga apenas
a imagem da parte compreendida entre seus pés e um detalhe de
sua roupa, que está a 1,5 m do chão. Atrás dessa pessoa, há uma
parede vertical AB, a 2,5 m do espelho. Considerando as medidas indicadas na figura 2, calcule, em metros:
a) a largura (L) da janela.
b) a largura mínima (x) do espelho E para que o observador possa
ver por inteiro a imagem da placa conjugada por ele.

04. (UNESP) Considere um objeto luminoso pontual, fixo no ponto


P, inicialmente alinhado com o centro de um espelho plano E. O
espelho gira, da posição E1 para a posição E2, em torno da aresta
cujo eixo passa pelo ponto O, perpendicularmente ao plano da
figura, com um deslocamento angular de 30°, como indicado:

Sabendo que a distância entre os olhos da pessoa e a imagem da


parede AB refletida no espelho é 3,3 m e que seus olhos, o detalhe
em sua roupa e seus pés estão sobre uma mesma vertical, calcule
a distância d entre a pessoa e o espelho e a menor distância que o
espelho deve ser movido verticalmente para cima, de modo que ela
possa ver sua imagem refletida por inteiro no espelho.

03. (UNIFESP) Dentro de uma casa uma pessoa observa, por meio Em sua resolução, copie o ponto P, o espelho em E1 e em E2 e
de um espelho plano E, uma placa com a inscrição VENDO colocada desenhe a imagem do ponto P quando o espelho está em E1 (P1’)
fora da casa, ao lado de uma janela aberta. A janela e o espelho têm e quando o espelho está em E2 (P2’). Considerando um raio de luz
as dimensões horizontais mínimas para que o observador consiga perpendicular a E1, emitido pelo objeto luminoso em P, determine os
ver a placa em toda sua extensão lateral. A figura 1 representa o ângulos de reflexão desse raio quando o espelho está em E1 (α1’) e
espelho e a janela vistos de dentro da casa. A figura 2 representa quando o espelho está em E2 (α2’).
uma visão de cima da placa, do espelho plano E, do observador O
e de dois raios de luz emitidos pela placa que atingem, depois de 05. (UERJ) As superfícies refletoras de dois espelhos planos, E1 e E2,
refletidos em E, os olhos do observador. formam um ângulo α. O valor numérico deste ângulo corresponde
a quatro vezes o número de imagens formadas.
Determine α.

GABARITO
EXERCÍCIOS PROPOSTOS
01. B 05. C 09. C 13. C 17. E
02. C 06. B 10. C 14. E 18. D
03. A 07. A 11. B 15. B 19. A
04. C 08. D 12. C 16. D 20. B

EXERCÍCIOS DE APROFUNDAMENTO
01. p2 = 0,1 · p1
02. d = 80 cm / x = 15 cm
03. a) 2,2 m
b) 0,6 m
04. 1º caso: α1 = 0° / 2º caso: α2 = 30°
05. α = 36°

258 PROENEM.COM.BR PRÉ-VESTIBULAR


28
FÍSICA II
REFRAÇÃO DA LUZ

INTRODUÇÃO PROEXPLICA
Diz-se que a luz sofre refração quando passa de um meio para
outro, modificando sua velocidade. Normalmente, a variação da
O índice de refração absoluto de um meio nos diz como a
velocidade é acompanhada de um desvio angular na trajetória do
luz será freada ao passar do vácuo para aquele meio. Se o índice
raio incidente para o raio refratado. Somente se a trajetória do raio
de refração de um meio é 2, a luz ao passar do vácuo para este
incidente for normal (perpendicular) à superfície separadora dos
meio terá sua velocidade 300.000 km/s dividida por 2.
meios, não haverá desvio.
O índice de refração relativo entre dois meios nos diz
se a velocidade da luz ao passar de um meio para outro irá
aumentar (se n < 1) ou diminuir (se n > 1).
Quando o índice de refração de um meio é maior que o de
outro, diz-se que o meio é mais refringente (a luz se propaga
com menor velocidade) que o outro.

LEIS DA REFRAÇÃO
São duas as leis da refração:
O fenômeno da refração caracteriza-se pela passagem da
luz de um meio de propagação para outro meio de propagação, O raio incidente, a normal no ponto de incidência e o raio
modificando sua velocidade e, na maioria das vezes, sua direção refratado estão no mesmo plano.
de propagação. Lei de Snell-Descartes: o produto do índice de refração de
um meio pelo seno do ângulo formado pelo raio com a normal é
constante.
Na figura a seguir:

Mudança de velocidade e de direção de propagação na


incidência oblíqua, enquanto que na incidência normal só há
mudança de velocidade.

ÍNDICE DE REFRAÇÃO
Índice de refração é uma grandeza criada para compa-rarmos n1 . sen i = n2 . sen r
o comportamento da velocidade, não só da luz, mas de todas as
formas de onda, quando estas passam de um meio para outro.
c PROEXPLICA
n=
V
A combinação da Lei de Snell-Descartes com a definição
O índice de refração absoluto é representado pela letra n e é dado de índice de refração absoluto com índice de refração relativo
pela razão entre a velocidade da luz no vácuo (igual a 3 . 108 m/s) nos permite escrever:
e a velocidade da luz no meio V em questão. n2 V1 sen i
n2,1
= = =
O índice de refração absoluto de um meio nos diz como será n1 V2 sen r
modificada a velocidade da onda ao passar do vácuo para este
meio.
Para que se possa comparar um meio com outro, existe o índice
de refração relativo. O índice de refração relativo entre dois meios ANÁLISE DO DESVIO DO
A e B é dado pela razão entre seus índices de refração absolutos.
RAIO REFRATADO
nA
nAB = Como já foi dito, na incidência oblíqua o raio ao mudar de meio
nD de propagação sofre um certo desvio, além de sua velocidade
sofrer uma variação. É importante ressaltar que não há variação da
c c
=
Como: nA = e nB frequência da onda, ela se mantém constante. Como a velocidade
VA VB varia, o comprimento de onda também irá variar. Existem dois
casos possíveis: passagem de um meio menos refringente para
nA VB outro mais refringente ou, o contrário, a passagem de um meio
Teremos: nAB
= =
nB VA mais refringente para outro menos refringente.

PRÉ-VESTIBULAR PROENEM.COM.BR 259


FÍSICA II 28 REFRAÇÃO DA LUZ

1) Incidência de um meio menos refringente para outro mais


refringente (n2 > n1):
DISPERSÃO DA LUZ
A dispersão luminosa ocorre quando um feixe de luz
policromática incide em uma superfície separadora de dois
meios. Na figura, temos um feixe de luz branca que incide em uma
superfície, após mudar de meio. A luz se decompõe nas sete cores
monocromáticas que a formam.

Quando um raio luminoso passa de um meio menos refringente


para outro mais refringente:
1ª Ele se aproxima da normal.
2ª O ângulo de refração é menor que o ângulo de incidência.
3ª A velocidade de propagação diminui após a refração.

2) Incidência de um meio mais refringente para outro menos


Isso ocorre devido às diferenças entre os índices de refração
refringente (n2 < n1):
das diferentes cores em um mesmo meio. Podemos afirmar que
quanto maior o seu índice de refração, maior será o desvio. Na
dispersão da luz, a luz monocromática de maior frequência sempre
sofre o maior desvio.

Quando um raio luminoso passa de um meio mais refringente


para outro menos refringente:
1ª Ele se afasta da normal. ARCO-ÍRIS
2ª O ângulo de refração é maior que o ângulo de incidência. O arco-íris é um fenômeno observado quando ocorre a
presença de gotas de água em suspensão diante da luz branca
3ª A velocidade de propagação aumenta após a refração. (luz do sol). Pode ser facilmente explicado pela dispersão da luz.
Na incidência NORMAL, já vimos que não há mudança de A luz branca incide nas gotas de água e os feixes monocromáticos
direção do raio refratado, mas ocorrerá variação de velocidade de sofrem desvios diferenciados, como mostra a figura, e assim
propagação: se a luz passa de um meio mais para outro menos temos a formação do arco-íris.
refringente sua velocidade aumenta, caso contrário, sua velocidade
diminui.

REFRAÇÃO ATMOSFÉRICA
Quando a luz que vem dos astros celestes penetram na camada
atmosférica ela vai passando para camadas cada vez mais densas
e consequentemente cada vez mais refringente, sofrendo inúmeros
desvios que os aproximam da normal, desse fato resulta vermos os
astros numa posição na qual eles realmente não se encontram, por
isso se fala em posição aparente dos astros.

LÂMINAS DE FACES PARALELAS


Consiste em uma lâmina transparente e limitada por duas
faces paralelas. Na maioria das questões que envolvem lâminas,
ela aparece envolvida por um único meio e as observações que
serão feitas também levam isto em consideração. Normalmente, a
lâmina é constituída de vidro.

260 PROENEM.COM.BR PRÉ-VESTIBULAR


28 REFRAÇÃO DA LUZ FÍSICA II

Na figura acima, como só há um meio envolvente, o ângulo x é


igual ao ângulo θ1. EXERCÍCIO RESOLVIDO

01. (ENEM) Um grupo de cientistas liderado por pesqui-sadores


Existe também uma expressão matemática utilizada para do Instituto de Tecnologia da Califórnia (Caltech), nos Estados
calcular o desvio lateral sofrido pelo raio. Unidos, construiu o primeiro metamaterial que apresenta
valor negativo do índice de refração relativo para a luz visível.
Denomina-se metamaterial um material óptico artificial,
tridimensional, formado por pequenas estruturas menores do
que o comprimento de onda da luz, o que lhe dá propriedades
e comportamentos que não são encontrados em materiais
naturais. Esse material tem sido chamado de “canhoto”.
(Disponível em: http://inovacaotecnologica.com.br.
Acesso em: 28 abr. 2010 [adaptado].)

Considerando o comportamento atípico desse meta-material,


qual é a figura que representa a refração da luz ao passar do
ar para esse meio?
a)

PRISMA ÓPTICO
Na figura abaixo, temos um prisma óptico visto obli-quamente
e frontalmente.

b)

c)
Podemos identificar alguns elementos:
• A: ângulo de abertura;
• n1 e n3: índices de refração dos meios exteriores, que
normalmente são iguais;
• η2: índice de refração do prisma.
d)

e)

Na figura anterior, um raio penetra no prisma, sofre um desvio


e incide na outra face. Sofre novo desvio saindo do prisma óptico.
A letra δ representa o desvio angular total sofrido pelo raio e é dado Resolução: D
pela fórmula: Nos materiais naturais, quando ocorre incidência oblíqua da luz,
δ = i1 + i2 – A os raios incidente e refratado estão em meios diferentes e em
quadrantes opostos, definidos pela superfície e pela normal a
essa superfície. No metamaterial, esses raios estão em meios
PROEXPLICA
diferentes, mas em quadrantes adjacentes.

COMO REFRAÇÃO DA LUZ PODE CAIR NO ENEM?


A refração da luz é um fenômeno presente em nosso
dia a dia que nos causa, muitas vezes, algumas ilusões de
ótica. Esse fato acontece devido ao desvio que a luz sofre,
em algumas situações, ao mudar de meio de propagação. A
questão a seguir explora uma situação de refração incomum
na natureza.

PRÉ-VESTIBULAR PROENEM.COM.BR 261


FÍSICA II 28 REFRAÇÃO DA LUZ

04. Um raio de luz monocromático incide na superfície de separação


EXERCÍCIOS horizontal entre o ar e um líquido, de índices de refração 1 e 2,
PROTREINO respectivamente, conforme representado na imagem abaixo:

01. A imagem abaixo representa um raio de luz monocromático


que se propaga no ar a 300 000Km/s, passando do ar para o vidro.
Os ângulos Q e P são tais que sen(Q) = 0,75 e sen(P) = 0,5. Sabendo
que o índice de refração do ar é 1, determine o índice de refração
do vidro e a velocidade de propagação do raio luminoso no vidro.

Imagem fora de escala

Calcule o ângulo de refração (P) e o desvio (D) sofrido pelo raio


Imagem fora de escala passar do ar para o líquido.

02. Dois raios luminosos monocromáticos R1 e R2 se propagam no 05. Um LASER a prova d’água emite um raio de luz monocromático
ar e incidem num acrílico transparente em forma de disco, os raios no ângulo limite conforme representado na imagem abaixo:
e o disco pertencem ao mesmo plano.

Esboce a trajetória dos raios no interior do disco.

03. Um raio luminoso monocromático incide na superfície de


separação horizontal entre o ar e o líquido, de índices de refração Imagem fora de escala
1 e 3 , respectivamente, conforme representado na imagem abaixo:
Sabendo que os índices de refração do ar e da água são,
respectivamente, 1 e 1,4, determine, aproximadamente, o ângulo
limite (L) indicado na imagem.
Dados: 2 = 1,4 e 3 = 1,7

30° 45° 60°


1 2 3
sen
2 2 2

3 2 1
cos
2 2 2

3
Imagem fora de escala tg 1 3
3
Calcule o ângulo de refração (P) e o ângulo entre o raio refratado e
o raio refletido.

262 PROENEM.COM.BR PRÉ-VESTIBULAR


28 REFRAÇÃO DA LUZ FÍSICA II

03. (EEAR) A tirinha abaixo utiliza um fenômeno físico para a


construção da piada. Que fenômeno é esse?
EXERCÍCIOS
PROPOSTOS
01. (ESPCEX - AMAN) Um raio de luz monocromática propagando-
se no ar incide no ponto O, na superfície de um espelho, plano e
horizontal, formando um ângulo de 30° com sua superfície.
Após ser refletido no ponto O desse espelho, o raio incide na
superfície plana e horizontal de um líquido e sofre refração. O raio
refratado forma um ângulo de 30° com a reta normal à superfície
do líquido, conforme o desenho abaixo. a) Reflexão
b) Refração
c) Difração
d) Propagação retilínea da luz

04. (UFRGS) Um feixe de luz monocromática atravessa a interface


entre dois meios transparentes com índices de refração n1 e n2,
respectivamente, conforme representa a figura abaixo.

Sabendo que o índice de refração do ar é 1, o índice de refração do


líquido é:
3 3
Dados: sen30° = 1/2 e cos60° = 1/2; sen60° = e cos30° = .
2 2

3 d) 2 3
a)
3 3

3 e) 2 3
b)
2
Com base na figura, é correto afirmar que, ao passar do meio com
c) 3 n1 para o meio com n2, a velocidade, a frequência e o comprimento
de onda da onda, respectivamente,
02. (ENEM – LIBRAS) No Hemisfério Sul, o solstício de verão
a) permanece, aumenta e diminui.
(momento em que os raios solares incidem verticalmente sobre
quem se encontra sobre o Trópico de Capricórnio) ocorre no dia b) permanece, diminui e aumenta.
21 ou 23 de dezembro. Nessa data, o dia tem o maior período de c) aumenta, permanece e aumenta.
presença de luz solar. A figura mostra a trajetórias da luz solar nas
d) diminui, permanece e diminui.
proximidades do planeta Terra quando ocorre o fenômeno ótico
que possibilita que o Sol seja visto por mais tempo pelo observador. e) diminui, diminui e permanece.

05. (EEAR) Considerando as velocidades de propagação da luz


em dois meios homogêneos e distintos, respectivamente iguais
a 200.000 km/s e 120.000 km/s, determine o índice de refração
relativo do primeiro meio em relação ao segundo. Considere a
velocidade da luz no vácuo, igual a 300.000 km/s.
a) 0,6
b) 1,0
Qual é o fenômeno ótico mostrado na figura? c) 1,6
a) A refração da luz solar ao atravessar camadas de ar com d) 1,7
diferentes densidades.
b) A polarização da luz solar ao incidir sobre a superfície dos 06. (EEAR) O vidro tem índice de refração absoluto igual a 1,5.
oceanos. Sendo a velocidade da luz no ar e no vácuo aproximadamente igual
a 3·108 m/s, pode-se calcular que a velocidade da luz no vidro é
c) A reflexão da luz solar nas camadas mais altas da ionosfera.
igual a
d) A difração da luz solar ao contornar a superfície da Terra.
a) 2·105 m/s
e) O espalhamento da luz solar ao atravessa a atmosfera.
b) 2·105 km/s
c) 4,5·108 m/s
d) 4,5·108 km/s

PRÉ-VESTIBULAR PROENEM.COM.BR 263


FÍSICA II 28 REFRAÇÃO DA LUZ

07. (UNESP) Dois raios luminosos monocromáticos, um azul e um 09. (ENEM) A figura representa um prisma óptico, constituído
vermelho, propagam-se no ar, paralelos entre si, e incidem sobre de um material transparente, cujo índice de refração é crescente
uma esfera maciça de vidro transparente de centro C e de índice com a frequência da luz que sobre ele incide. Um feixe luminoso,
de refração 3, nos pontos A e V. Após atravessarem a esfera, os composto por luzes vermelha, azul e verde, incide na face A,
raios emergem pelo ponto P, de modo que o ângulo entre eles é emerge na face B e, após ser refletido por um espelho, incide num
igual a 60°. filme para fotografia colorida, revelando três pontos.

Considerando que o índice de refração absoluto do ar seja igual a Observando os pontos luminosos revelados no filme, de baixo para
3 1 cima, constatam-se as seguintes cores:
1, que sen60° = que sen30° = , o ângulo α indicado na figura
2 2 a) Vermelha, verde, azul.
é igual a b) Verde, vermelha, azul.
a) 90°. c) 120°. e) 150°. c) Azul, verde, vermelha.
b) 165°. d) 135°. d) Verde, azul, vermelha.
e) Azul, vermelha, verde.
08. (ENEM PPL) A fotografia feita sob luz polarizada é usada por
dermatologistas para diagnósticos. Isso permite ver detalhes 10. (ENEM PPL) As miragens existem e podem induzir à percepção
da superfície da pele que não são visíveis com o reflexo da luz de que há água onde não existe. Elas são a manifestação de um
branca comum. Para se obter luz polarizada, pode-se utilizar a luz fenômeno óptico que ocorre na atmosfera.
transmitida por um polaroide ou a luz refletida por uma superfície Disponível em: www.invivo.fiocruz.br. Acesso em: 29 fev. 2012.
na condição de Brewster, como mostra a figura. Nessa situação,
o feixe da luz refratada forma um ângulo de 90° com o feixe da Esse fenômeno óptico é consequência da
luz refletida, fenômeno conhecido como Lei de Brewster. Nesse
a) refração da luz nas camadas de ar próximas do chão quente.
caso, o ângulo da incidência θp, também chamado de ângulo de
polarização, e o ângulo de refração θr estão em conformidade com b) reflexão da luz ao incidir no solo quente.
a Lei de Snell. c) reflexão difusa da luz na superfície rugosa.
d) dispersão da luz nas camadas de ar próximas do chão quente.
e) difração da luz nas camadas de ar próximas do chão quente.

11. (UEFS) Um raio luminoso incide sobre a superfície de separação


entre o ar e o vidro com um ângulo α = 80º e refrata com um ângulo
β = 30º, como mostra a figura.

Considere um feixe de luz não polarizada proveniente de um meio


com índice de refração igual a 1, que incide sobre uma lâmina e faz Considerando sen30º = cos60º=0,5; sen60º = cos30º = 0,87; o
um ângulo de refração θr de 30°. índice de refração do ar igual a 1 e o índice de refração do vidro
Nessa situação, qual deve ser o índice de refração da lâmina para igual a n, então o valor de n é igual a
que o feixe refletido seja polarizado? a) 1,48 c) 1,63 e) 1,83
a) 3 c) 2 3 b) 1,57 d) 1,74
e)
3 1 2
b) d)
3 2

264 PROENEM.COM.BR PRÉ-VESTIBULAR


28 REFRAÇÃO DA LUZ FÍSICA II

12. (ENEM PPL) Em um experimento, coloca-se glicerina dentro de Utilizando-se da palavra latina spectrum, ele descreveu o conjunto
um tubo de vidro liso. Em seguida, parte do tubo é colocada em um de cores que resultou dessa dispersão da luz branca ao atravessar
copo de vidro que contém glicerina e a parte do tubo imersa fica o prisma. A explicação para o observado por Newton encontra-se
invisível. Esse fenômeno ocorre porque a associada ao fato de que cada cor que compõe o spectrum sofre
a) intensidade da luz é praticamente constante no vidro. um desvio diferente em virtude
b) parcela de luz refletida pelo vidro é praticamente nula. a) da sua polarização. d) da sua velocidade no vácuo.
c) luz que incide no copo não é transmitida para o tubo de vidro. b) da sua difusão. e) da sua interferência.
d) velocidade da luz é a mesma no vidro e na glicerina. c) do seu índice de refração.
e) trajetória da luz é alterada quando ela passa da glicerina para
o vidro. 17. (FMP) A luz do sol, após atravessar a água em um aquário,
projeta um arco-íris na parede de uma residência.
13. (IFSUL) Quando uma onda luminosa atravessa dois meios A decomposição da luz branca do sol, ao atravessar os meios ar –
diferentes, por exemplo o ar e uma parede de vidro, qual das água – ar, ocorre porque cada componente da luz possui, na água,
quantidades permanece constante? diferentes índices de
a) A velocidade de propagação. a) polarização d) interferência
b) A amplitude. b) refração e) coloração
c) A frequência. c) difração
d) O comprimento de onda.
18. (MACKENZIE) Um raio de luz monocromática de frequência
f = 1,0 x 1015 Hz, com velocidade v = 3,0 x 105 km/s, que se propaga
14. (UECE) A energia solar fotovoltaica é uma das fontes de energia no ar, cujo índice de refração é igual a 1, incide sobre uma lâmina
em franca ascensão no Brasil. Dentre os diversos componentes de de vidro (nvidro = 2), formando um ângulo 45° com a superfície da
um sistema solar fotovoltaico, destaca-se o painel solar. De modo lâmina. O seno do ângulo de refração é
simplificado, esse componente é constituído por uma camada de
vidro para proteção mecânica, seguida de uma camada formada a) 0,5. b) 0,7. c) 1,0. d) 3,0. e) 2.
por células solares e uma última camada, na parte inferior, também
para proteção e isolamento. 19. (UNICAMP) Uma lente de Fresnel é composta por um conjunto
Sendo o vidro um material semitransparente, um raio solar que de anéis concêntricos com uma das faces plana e a outra inclinada,
chega ao painel é como mostra a figura (a). Essas lentes, geralmente mais finas que
as convencionais, são usadas principalmente para concentrar um
a) parcialmente refletido e totalmente refratado pelo vidro. feixe luminoso em determinado ponto, ou para colimar a luz de
b) parcialmente refletido e parcialmente refratado pelo vidro. uma fonte luminosa, produzindo um feixe paralelo, como ilustra
c) totalmente refratado pelo vidro. a figura (b). Exemplos desta última aplicação são os faróis de
automóveis e os faróis costeiros. O diagrama da figura (c) mostra
d) totalmente refletido pelo vidro. um raio luminoso que passa por um dos anéis de uma lente de
Fresnel de acrílico e sai paralelamente ao seu eixo.
15. (UFPR) Um dado meio tem um índice de refração n1. Um outro
meio tem um índice de refração n2. Assinale a alternativa que
expressa corretamente a relação entre os módulos das velocidades
da luz nos dois meios, quando n2 = 2n1.
a) v2 = 4v1
b) v2 = 2v1
c) v2 = v1
v1
d) v2 = .
2
v
e) v2 = 1 .
4
16. (UPF) Conta a história que Isaac Newton, trabalhando no
polimento de algumas peças de vidro, conseguiu obter um prisma
triangular, o qual utilizou para a sua famosa experiência da
dispersão da luz branca, ilustrada na figura a seguir.

Se sen(θ1) = 0,5 e sen(θ2) = 0,75, o valor do índice de refração do


acrílico é de
a) 1,50. c) 1,25.
b) 1,41. d) 0,66.

PRÉ-VESTIBULAR PROENEM.COM.BR 265


FÍSICA II 28 REFRAÇÃO DA LUZ

20. (FUVEST) Em uma aula de laboratório de física, utilizando-se o Um feixe de luz monocromático, refletido pelo cartaz, incide sobre a
arranjo experimental esquematizado na figura, foi medido o índice interface de separação entre a lâmina e o ar, formando com a vertical
de refração de um material sintético chamado poliestireno. Nessa um ângulo de 53°. Ao se refratar, esse feixe forma um ângulo de 30°
experiência, radiação eletromagnética, proveniente de um gerador com a mesma vertical. Observe o esquema ampliado a seguir, que
de micro-ondas, propaga-se no ar e incide perpendicularmente representa a passagem do raio de luz entre a lâmina e o ar.
em um dos lados de um bloco de poliestireno, cuja seção reta
é um triângulo retângulo, que tem um dos ângulos medindo
25°, conforme a figura. Um detetor de micro-ondas indica que a
radiação eletromagnética sai do bloco propagando-se no ar em
uma direção que forma um ângulo de 15° com a de incidência.

A partir desse resultado, conclui-se que o índice de refração


do poliestireno em relação ao ar para essa micro-onda é,
aproximadamente,
Note e adote:
- índice de refração do ar: 1,0
- sen 15° ≈ 0,3
- sen 25° ≈ 0,4 Determine o índice de refração da lâmina.
- sen 40° ≈ 0,6
02. (UNIFESP) Para demonstrar o fenômeno da refração luminosa,
a) 1,3 d) 2,0 um professor faz incidir um feixe monocromático de luz no ponto A
b) 1,5 e) 2,2 da superfície lateral de um cilindro reto constituído de um material
c) 1,7 homogêneo e transparente, de índice de refração absoluto igual a
1,6 (figura 1).

EXERCÍCIOS DE
APROFUNDAMENTO
01. (UERJ) A questão a seguir aborda situações relacionadas
ao ambiente do metrô, referindo-se a uma mesma composição,
formada por oito vagões de dois tipos e movida por tração elétrica.
Para seus cálculos, sempre que necessário, utilize os dados e as A figura 2 representa a secção transversal circular desse cilindro,
fórmulas abaixo. que contém o plano de incidência do feixe de luz. Ao incidir no
ponto A, o feixe atravessa o cilindro e emerge no ponto B, sofrendo
Características da composição um desvio angular α.
velocidade máxima 100 km/h
aceleração constante 1,10 m/s²
desaceleração constante 1,25 m/s²
Gerais
quantidade de tipo I 2
vagões tipo II 6
massa média por passageiro 60 kg
comprimento médio 22,0 m
largura 3,00 m
altura 3,60 m

Por tipo I 38.000 kg


massa
vagão tipo II 35.000 kg
Sabendo que a velocidade da luz no vácuo é igual a 3 x 108 m/s,
quantidade 4
motores que o índice de refração absoluto do ar é igual a 1,0 e adotando
potência por motor 140 kW sen53° = 0,8, calcule:
capacidade máxima 8 passageiros/m² a) a velocidade escalar do feixe luminoso, em m/s, no interior do
cilindro.
Em uma estação, um cartaz informativo está protegido por uma
lâmina de material transparente. b) o desvio angular α, em graus, sofrido pelo feixe luminoso ao
atravessar o cilindro.

266 PROENEM.COM.BR PRÉ-VESTIBULAR


28 REFRAÇÃO DA LUZ FÍSICA II

03. (UNIFESP) O pingente de um colar é constituído por duas


Ângulo θ senθ tgθ
peças, A e B, feitas de materiais homogêneos e transparentes, de
índices de refração absolutos n=
A 1,6 ⋅ 3 e nB = 1,6. A peça A tem o 30º 0,50 0,58
formato de um cone reto e a peça B, de uma semiesfera.
40º 0,64 0,84
Um raio de luz monocromático R propaga-se pelo ar e incide,
paralelamente ao eixo do cone, no ponto P da superfície cônica, 42º 0,67 0,90
passando a se propagar pelo material da peça A. Atinge o ponto 53º 0,80 1,33
C, no centro da base do cone, onde sofre nova refração, passando
a propagar-se pelo material da peça B, emergindo do pingente no 60º 0,87 1,73
ponto Q da superfície esférica. Desde a entrada até a sua saída do
pingente, esse raio propaga-se em um mesmo plano que contém 05. (UNICAMP) Há atualmente um grande interesse no desenvolvi-
o vértice da superfície cônica. A figura 1 representa o pingente mento de materiais artificiais, conhecidos como metamateriais, que
pendurado verticalmente e em repouso e a figura 2, a intersecção têm propriedades físicas não convencionais. Este é o caso de meta-
do plano que contém o raio R com o pingente. As linhas tracejadas, materiais que apresentam índice de refração negativo, em contraste
indicadas na figura 2, são paralelas entre si e α = 30°. com materiais convencionais que têm índice de refração positivo.
Essa propriedade não usual pode ser aplicada na camuflagem de
objetos e no desenvolvimento de lentes especiais.
a) Na figura a seguir é representado um raio de luz A que se
propaga em um material convencional (Meio 1) com índice de
refração n1 = 1,8 e incide no Meio 2 formando um ângulo θ1 =
30° com a normal. Um dos raios B, C, D ou E apresenta uma
trajetória que não seria possível em um material convencional
e que ocorre quando o Meio 2 é um metamaterial com índice
de refração negativo. Identifique este raio e calcule o módulo
do índice de refração do Meio 2, n2, neste caso, utilizando a
lei de Snell na forma: n1 sen=θ1 n2 senθ2 . Se necessário use
= 2 1,4
= e 3 1,7.
Calcule o valor do ângulo β indicado na figura 2, em graus.
a) Considere que a peça B possa ser substituída por outra peça B’, b) O índice de refração de um meio material, n, é definido pela
com o mesmo formato e com as mesmas dimensões, mas de razão entre as velocidades da luz no vácuo e no meio. A
1
maneira que o raio de luz vertical R sempre emerja do pingente velocidade da luz em um material é dada por v = , em que
pela superfície esférica. εµ
b) Qual o menor índice de refração do material de B’ para que o ε é a permissividade elétrica e μ é a permeabilidade magnética
raio R não emerja pela superfície cônica do pingente? do material. Calcule o índice de refração de um material que
2
−11 C N.s2
= tenha ε 2,0x10= 2
e µ 1,25x10−6 2 . A velocidade da
04. (FUVEST) Um jovem pesca em uma lagoa de água transparente, N.m C
utilizando, para isto, uma lança. Ao enxergar um peixe, ele atira sua luz no vácuo é c = 3,0×108 m/s.
lança na direção em que o observa. O jovem está fora da água e o
peixe está 1 m abaixo da superfície. A lança atinge a água a uma
distância x = 90 cm da direção vertical em que o peixe se encontra,
como ilustra a figura abaixo. Para essas condições, determine:
a) O ângulo α, de incidência na superfície da água, da luz refletida
pelo peixe.
b) O ângulo β que a lança faz com a superfície da água.
c) A distância y, da superfície da água, em que o jovem enxerga
o peixe.

GABARITO
EXERCÍCIOS PROPOSTOS
01. C 05. A 09. A 13. C 17. B
02. A 06. B 10. A 14. B 18. A
03. B 07. C 11. D 15. D 19. A
04. D 08. A 12. D 16. C 20. B

EXERCÍCIOS DE APROFUNDAMENTO
01. 1,45
02. a) 1,9 x 108 m/s; b) 46°
NOTE E ADOTE:
03. a) β = 60°; b) nB = 0,8√3
Índice de refração do ar = 1 04. a) θ = 42°; b) β = 30°; c) y = 0,52 m
Índice de refração da água = 1,3 05. a) 1,29; b) 1,5

Lei de Snell: v1/v2 = senθ1/senθ2

PRÉ-VESTIBULAR PROENEM.COM.BR 267


FÍSICA II 28 REFRAÇÃO DA LUZ

ANOTAÇÕES

268 PROENEM.COM.BR PRÉ-VESTIBULAR


29 REFLEXÃO TOTAL FÍSICA II

E DIOPTRO PLANO

REFLEXÃO TOTAL / ÂNGULO LIMITE Os índices de refração de cinco líquidos diferentes estão
Vimos que, quando a luz passa de um meio mais refringente indicados na tabela abaixo.
para um meio menos refringente o raio refratado se afasta da
normal, este afastamento pode ocasionar um fenômeno conhecido Líquido Índice de refração
como reflexão total. Líquido 1 1,1
Observe a figura a seguir:
Líquido 2 1,3
Líquido 3 1,5
Líquido 4 1,7
Líquido 5 1,8
O índice de refração de qual líquido se aproxima mais do
obtido pelo experimento de Maria Meitner?
a) Do líquido 5.
b) Do líquido 4.
A luz, saindo da fonte F, que se encontra num meio mais c) Do líquido 3.
refringente (a água, por exemplo) sai para um meio menos d) Do líquido 2.
refringente (o ar, por exemplo). Observe que o raio refratado se
e) Do líquido 1.
afasta cada vez mais da normal, chegando numa situação em que
ele se torna rasante à superfície de separação dos meios, dizemos
Resolução: C
que este ângulo de incidência é o ângulo limite a partir daí, um
outro raio que incida com um ângulo de incidência maior que o Aplicando a Lei de Snell, temos:
ângulo limite, não mais refratará, voltará para o meio 1, ocorrendo a nLsen
= θL narsen90°
reflexão total. Observe que para o cálculo do ângulo limite teremos:
nL ⋅ 0,67 =
1⋅ 1
n2 sen i n sen L n 1
= ⇒ 2= ⇒ sen L= 2 ∴ nL= ≅ 1,5
n1 sen r n1 sen 90° n1 0,67

Quando surgir dúvida sobre o seno do ângulo limite para um


par de meios, a saída é dividir o menor índice de refração pelo maior.
Isso pois o seno de um ângulo nunca pode assumir um valor FIBRA ÓPTICA
maior que 1. As fibras ópticas são, geralmente, de um núcleo central
cilíndrico e transparente de vidro puro, envolvido por uma camada
EXERCÍCIO RESOLVIDO de material com menor índice de refração (fator que viabiliza a
reflexão total). Ou seja, a fibra óptica é composta por um material
com maior índice de refração (núcleo) envolto por um material com
01. (UFJF-PISM 2) Em um experimento realizado em
menor índice de refração (casca).
um laboratório, Maria Meitner colocou uma caneta laser
adequadamente protegida no fundo de um aquário e depois
o encheu com um líquido desconhecido. Ao instalar o laser,
ela mediu o ângulo limite, θL, para que ocorra a reflexão total
na interface com o ar, encontrando o valor de 42º. A figura a
seguir representa o experimento, sendo que a seta no fundo
do aquário representa a caneta laser e as outras, por sua vez,
indicam a direção de propagação do feixe.
Dados: cos42° = 0,74; sen42° = 0,67; nar = 1,0 (índice de
refração do ar).

fonte: https://www.iteel.it/installazione-configurazione-fibra-ottica-roma

A passagem destes sinais luminosos é feita através da reflexão


total que acontece no interior da fibra.

PRÉ-VESTIBULAR PROENEM.COM.BR 269


FÍSICA II 29 REFLEXÃO TOTAL E DIOPTRO PLANO

DIOPTRO PLANO Considere: tgθ ≈ sen θ ≈ θ e o índice de refração da água


Entende-se por dioptro plano dois meios opticamente diferentes n = 1,33. Então, a profundidade aparente h vista pelo pescador
separados por uma superfície. O seu estudo é muito importante é igual a
para entendermos certos fenômenos, como, por exemplo, quando
a) 2,5 m
olhamos para o fundo de uma piscina e ela parece ser mais rasa
do que realmente é. Isto se deve ao fato de os raios luminosos que b) 5,0 m
partem do fundo da piscina sofrerem refração ao passarem da c) 7,5 m
água para o ar, afastando-se do normal. d) 8,0 m
e) 9,0 m

Resolução: C
nar n água
=
h H
1 1,33
=
h 10
É importante notar que o raio sempre parte do objeto.
1,33 ⋅ h =10
Agora, imaginemos o observador dentro da piscina. Os objetos
h ≅ 7,5m
para ele parecerão estar acima do que realmente estão.

PROEXPLICA

COMO POSIÇÃO APARENTE DE OBJETOS PODE CAIR NO ENEM?


É comum em nosso dia a dia termos a sensação de que
objetos que estejam dentro da água estão tortos ou quebrados.
Essa ilusão de ótica é causada pelo desvio sofrido pela luz ao
sair da água e iniciar sua propagação pelo ar. A questão analisa
Se soubermos os índices de refração dos meios e posição uma situação onde esta ilusão de ótica ocorre.
real do objeto, podemos descobrir a posição aparente através da
expressão matemática:
p' nEM
= 03. (UFMG) O empregado de um clube está varrendo o fundo
p Ninc
da piscina com uma vassoura que tem um longo cabo de
Em que: alumínio. Ele percebe que o cabo de alumínio parece entortar-
p é a posição real do objeto; se ao entrar na água, como mostra a figura a seguir.
p’ é a posição aparente do objeto;
nEM é o índice de refração para onde o raio emerge, isto é, o
meio do observador;
nINC é o índice de refração do meio de onde o raio parte.

PROEXPLICA

Essa relação é válida para pequenos ângulos, menores


que 10°, onde sen(x) é aproximadamente igual a tg (x).
Isso ocorre porque:
a) a luz do Sol, refletida na superfície da água, interfere com
EXERCÍCIO RESOLVIDO a luz do Sol refletida pela parte da vassoura imersa na
água;
02. (ITA) Um pescador deixa cair uma lanterna acesa em um b) a luz do Sol, refletida pela parte da vassoura imersa na
lago a 10,0 m de profundidade. No fundo do lago, a lanterna água, sofre reflexão parcial na superfície de separação
emite um feixe luminoso formando um pequeno ângulo è água–ar;
com a vertical (veja figura).
c) a luz do Sol, refletida pela parte da vassoura imersa na
água, sofre reflexão total na superfície de separação
água–ar;
d) a luz do Sol, refletida pela parte da vassoura imersa
na água, sofre refração ao passar pela superfície de
separação água–ar;
e) o cabo de alumínio sofre uma dilatação na água, devido à
diferença de temperatura entre a água e o ar.

Resolução: D

270 PROENEM.COM.BR PRÉ-VESTIBULAR


29 REFLEXÃO TOTAL E DIOPTRO PLANO FÍSICA II

03. Explique o funcionamento de miragens observadas em


EXERCÍCIOS desertos em dias de alta temperatura.
PROTREINO 04. Veja o esquema da fibra óptica.

01. Uma fonte de luz monocromática está fixada no fundo de um


balde de profundidade 32,5 cm e índice de refração 1,3. Determine
a profundidade aparente quando a fonte é observada por uma
pessoa próxima ao balde.

O núcleo transparente de alto índice de refração revestido por


camadas plásticas transparentes cujos índices de refração são
mais baixos que os do núcleo permitem a transmissão de dados
em alta velocidade.
Explique o fenômeno físico que permite a utilização das fibras
ópticas.

05. Considerando o funcionamento das fibras ópticas, explique o


Desenho ilustrativo – fora de escala motivo do núcleo sempre ter um índice de refração maior que as
camadas que os revestem.
Considere nar = 1 e os ângulos de incidência e refração menores
que 10°.

02. Um LASER a prova d’água emite um raio de luz monocromático


no ângulo limite conforme representado na imagem abaixo: EXERCÍCIOS
PROPOSTOS
01. (ACAFE) O uso de fibras ópticas em aplicações médicas tem
evoluído bastante desde as aplicações pioneiras do Fiberscope,
onde um feixe de fibras de vidro servia basicamente para iluminar
e observar órgão no interior do corpo humano. Hoje em dia, tem-
se uma variedade de aplicações de sistemas sensores com fibras
ópticas em diagnóstico e cirurgia.

Imagem fora de escala


Assinale a alternativa correta que completa as lacunas das frases
Sabendo que os índices de refração do ar e da água são, a seguir.
respectivamente, 1 e 1,4, determine, aproximadamente, o ângulo O princípio é que quando lançado um feixe de luz numa extremidade
limite (L) indicado na imagem. da fibra e, pelas características ópticas do meio (fibra), esse feixe
Dados:
= 2 1,4= e 3 1,7 percorre a fibra por meio de __________ sucessivas. A fibra possui
no mínimo duas camadas: o núcleo (filamento de vidro) e o
revestimento (material eletricamente isolante). No núcleo, ocorre a
30° 45° 60° transmissão da luz propriamente dita. A transmissão da luz dentro
da fibra é possível graças a uma diferença de índice de __________
1 2 3 entre o revestimento e o núcleo, sendo que o núcleo possui sempre
sen
2 2 2 um índice de refração mais elevado, característica que, aliada ao
ângulo de __________ do feixe de luz, possibilita o fenômeno da
3 2 1 __________ total.
cos a) refrações – refração – incidência – reflexão
2 2 2
b) reflexões – refração – incidência – reflexão
3 c) reflexões – incidência – refração – refração
tg 1 3
3 d) interferências -– refração – incidência – reflexão

PRÉ-VESTIBULAR PROENEM.COM.BR 271


FÍSICA II 29 REFLEXÃO TOTAL E DIOPTRO PLANO

02. (UFRN) Ainda hoje, no Brasil, alguns índios pescam em rios 06. (ESPCEX - AMAN) Uma fonte luminosa está fixada no fundo de
de águas claras e cristalinas, com lanças pontiagudas, feitas uma piscina de profundidade igual a 1,33 m.
de madeira. Apesar de não saberem que o índice de refração da Uma pessoa na borda da piscina observa um feixe luminoso
água é igual a 1,33, eles conhecem, a partir da experiência do seu monocromático, emitido pela fonte, que forma um pequeno ângulo
dia, a lei da refração (ou da sobrevivência da natureza) e, por isso, α com a normal da superfície da água, e que, depois de refratado,
conseguem fazer a sua pesca. forma um pequeno ângulo β com a normal da superfície da água,
conforme o desenho.

A figura acima é apenas esquemática. Ela representa a visão que


o índio tem da posição em que está o peixe. Isto é, ele enxerga o
peixe como estando na profundidade III. As posições I, lI, III e IV
correspondem a diferentes profundidades numa mesma vertical.
Considere que o peixe está praticamente parado nessa posição.
Para acertá-lo, o índio deve jogar sua lança em direção ao ponto: A profundidade aparente “h” da fonte luminosa vista pela pessoa
a) I c) III é de:
b) II d) IV Dados: sendo os ângulos α e β pequenos, considere tgα ≅ senα e
tgβ ≅ senβ.
03. (UECE) Uma escova de dentes tem seu cabo feito de plástico índice de refração da água: nágua =1,33
azul, no qual estão presas cerdas de nylon incolor. As pontas das índice de refração do ar: nar = 1
cerdas parecem azuis quando a escova é iluminada com a luz do
a) 0,80 m
dia. O fenômeno ótico responsável principal por essa coloração
azul nas pontas das cerdas é denominado b) 1,00 m
a) interferência construtiva. c) 1,10 m
b) reflexão total. d) 1,20 m
c) difração. e) 1,33 m
d) interferência destrutiva.
07. (IFPE) Quando olhamos uma piscina, estando em pé e do
lado de fora da mesma, sempre temos a impressão de que ela
04. (UFJF-PISM 2) No seu laboratório de pesquisa, o aluno Pierre
tem uma profundidade diferente da que percebemos quando nela
de Fermat utiliza um sistema de fibras ópticas para medir as
mergulhamos. Isso se deve ao fato de que o ar atmosférico e a água
propriedades ópticas de alguns materiais. A fibra funciona como
têm índices de refração absolutos diferentes. Se a profundidade
um guia para a luz, permitindo que esta se propague por reflexões
real de uma piscina é 2,0 m e os índices de refração absolutos do ar
totais sucessivas. Em relação aos fenômenos de reflexão e
atmosférico e da água da piscina valem 1,0 e 1,3, respectivamente,
refração, assinale a alternativa CORRETA:
é correto dizer que um observador em pé, fora da piscina, verá que
a) A reflexão total só pode ocorrer quando a luz passa de um meio a sua profundidade será, aproximadamente, em metros:
menos refringente para um mais refringente;
a) 1,5 d) 2,6
b) A reflexão total só pode ocorrer quando a luz passa de um meio
b) 1,2 e) 1,0
mais refringente para um menos refringente;
c) 2,4
c) A luz não sofre reflexões no interior da fibra óptica, ela
simplesmente se curva junto com a curvatura da fibra;
08. (COTIL) Ao pescar com arco e flecha, um índio aprendeu com
d) O efeito de reflexão total só ocorre em função da proteção
sua experiência que não deve lançar sua arma na direção do peixe.
plástica que envolve as fibras; sem a proteção, a luz irá se
Considerando que a “normal” é a reta que forma com a superfície
perder;
um ângulo de 90°, para que o índio atinja seu alvo ele deve lançar
e) A Lei de Snell não prevê que ocorra o fenômeno de refração. a flecha:
a) Um pouco mais abaixo da imagem que ele está vendo, pois o
05. (CEFET MG) No vácuo, um determinado meio material raio de luz que emerge da água se afasta da “normal”, dando
isotrópico e transparente com índice de refração absoluto igual a impressão de que o peixe está mais próximo da superfície.
a 2 apresentará a condição de reflexão total para um raio de luz
b) Um pouco mais acima da imagem que ele está vendo, pois o
com ângulo limite de incidência igual a _______, propagando-se do
raio de luz que emerge da água se afasta da “normal”, dando
______________________ para o ______________________ .
a impressão de que o peixe está mais próximo da superfície.
Os termos que preenchem, corretamente, as lacunas são
c) Um pouco mais abaixo da imagem que ele está vendo, pois o
a) 30°, material, vácuo. raio de luz que emerge da água se aproxima da “normal”, dando
b) 30°, vácuo, material. a impressão de que o peixe está mais longe da superfície.
c) 60°, material, vácuo. d) Um pouco mais acima da imagem que ele está vendo, pois o
d) 60°, vácuo, material. raio de luz que emerge da água se aproxima da “normal”, dando
a impressão de que o peixe está mais longe da superfície.
e) 90°, vácuo, material.

272 PROENEM.COM.BR PRÉ-VESTIBULAR


29 REFLEXÃO TOTAL E DIOPTRO PLANO FÍSICA II

09. (UFJF-PISM 2) As fibras ópticas podem ser usadas em 12. (UNESP) Dentro de uma piscina, um tubo retilíneo luminescente,
telecomunicações, quando uma única fibra, da espessura de um com 1 m de comprimento, pende, verticalmente, a partir do centro de
fio de cabelo, transmite informação de vídeo equivalente a muitas uma boia circular opaca, de 20 cm de raio. A boia flutua, em equilíbrio,
imagens simultaneamente. Também são largamente aplicadas em na superfície da água da piscina, como representa a figura.
medicina, permitindo transmitir luz para visualizar vários órgãos
internos, sem cirurgias. Um feixe de luz pode incidir na extremidade
de uma fibra óptica de modo que nenhuma ou muito pouca energia
luminosa será perdida através das paredes da fibra. O princípio
ou fenômeno que explica o funcionamento das fibras ópticas é
denominado:
a) reflexão interna total da luz.
b) refração total da luz.
c) independência da velocidade da luz.
d) reflexão especular da luz.
e) dispersão da luz.

10. (FATEC) Um professor do curso de Materiais da Fatec


apresentou aos alunos a seguinte citação:
Sabendo que o índice de refração absoluto do ar é 1,00 e que o
“As fibras ópticas podem ser usadas para guiar a luz ao índice de refração absoluto da água da piscina é 1,25, a parte visível
longo de um determinado caminho. A ideia é fazer um raio de luz desse tubo, para as pessoas que estiverem fora da piscina, terá
percorrer uma fibra transparente, ricocheteando entre as suas comprimento máximo igual a
paredes. Desde que o ângulo de incidência do raio na parede da
a) 45 cm. c) 15 cm. e) 65 cm.
fibra seja sempre maior que o ângulo crítico, o raio permanecerá
sempre dentro dela mesmo que ela esteja curva”. b) 85 cm. d) 35 cm.
KIRK, Tim. Physics for the IB Diploma. Oxford University Press, 2003. Livre tradução.
13. (ENEM PPL) A banda larga brasileira é lenta. No Japão já
Em seguida, pediu para que os alunos respondessem, de maneira existem redes de fibras ópticas, que permitem acessos à internet
assertiva, à qual conceito físico a citação se refere. com velocidade de 1 gigabit por segundo (Gbps), o suficiente para
A resposta correta esperada pelo professor é baixar em um minuto, por exemplo, 80 filmes. No Brasil a maioria
das conexões ainda é de 1 megabit por segundo (Mbps), ou seja,
a) difração.
menos de um milésimo dos acessos mais rápidos do Japão. A fibra
b) polarização. óptica é composta basicamente de um material dielétrico (sílica ou
c) ângulo limite. plástico), segundo uma estrutura cilíndrica, transparente e flexível.
d) espalhamento. Ela é formada de uma região central envolta por uma camada,
também de material dielétrico, com índice de refração diferente
e) dispersăo luminosa. ao do núcleo. A transmissão em uma fibra óptica acontecerá de
forma correta se o índice de refração do núcleo, em relação ao
11. (FGV) São dados os índices de refração absolutos (n) dos revestimento, for
seguintes meios ópticos: nar = 1,0, nágua = 1,3, nvidro c = 1,5, nvidro p = 1,8. a) superior e ocorrer difração.
Um raio de luz monocromática foi emitido sobre um sistema óptico b) superior e ocorrer reflexão interna total.
formado por 3 desses meios, obtendo-se a configuração seguinte.
I e II são dióptros planos, que separam os meios A de B e B de C, c) inferior e ocorrer reflexão interna parcial.
respectivamente. d) inferior e ocorrer interferência destrutiva.
e) inferior e ocorrer interferência construtiva.

14. (EFOMM) O aquário da figura abaixo apresenta bordas bem


espessas de um material cujo índice de refração é igual a 3. Um
observador curioso aponta uma lanterna de forma que seu feixe de
luz forme um ângulo de incidência de 60°, atravessando a borda
do aquário e percorrendo a trajetória AB. Em seguida, o feixe de
luz passa para a região que contém o líquido, sem sofrer desvio,
seguindo a trajetória BC.

A possível, correta e respectiva relação entre os meios A, B e C é


a) água, vidro p e ar.
b) ar, vidro c e vidro p.
c) água, vidro c e vidro p.
d) vidro c, ar e água.
e) ar, água e vidro p.

PRÉ-VESTIBULAR PROENEM.COM.BR 273


FÍSICA II 29 REFLEXÃO TOTAL E DIOPTRO PLANO

Considere o índice de refração do ar igual a 1,0. O feixe de luz Dentre as afirmativas acima, as únicas corretas são:
emergirá do líquido para o ar no ponto C? a) I e II
3 b) III e IV
a) Sim, e o seno do ângulo refratado será .
3 c) II e III
3 d) I e IV
b) Sim, e o seno do ângulo refratado será .
2 e) I e III
3
c) Não, e o seno do ângulo limite será .
2 17. (CN) Observe a figura a seguir.
d) Não, pois o seno do ângulo refratado é menor que o seno do
ângulo limite.
e) Não, pois o seno do ângulo refratado é maior que o seno do
ângulo limite.
Uma das maiores revoluções ocorridas nas últimas décadas foi o
15. (ENEM) Será que uma miragem ajudou a afundar o Titanic? O uso de cabos de fibra óptica para o tráfego de dados (voz, imagem,
fenômeno ótico conhecido como Fata Morgana pode fazer com som, ...) através das redes de telecomunicação.
que uma falsa parede de água apareça sobre o horizonte molhado.
O maior desses cabos, atualmente, é o SeaMewe 3 que sai da
Quando as condições são favoráveis, a luz refletida pela água fria
Alemanha e chega até a Coreia do Sul, passando por 32 países,
pode ser desviada por uma camada incomum de ar quente acima,
num total de 39.000 km de comprimento. Considerando a
chegando até o observador, vinda de muitos ângulos diferentes.
trajetória da luz pela fibra óptica (ver figura) e que o tempo médio
De acordo com estudos de pesquisadores da Universidade de San
de transmissão de dados entre a Alemanha e a Coreia do Sul seja
Diego, uma Fata Morgana pode ter obscurecido os icebergs da
de, aproximadamente, 0,195 s, pode-se afirmar que na fibra óptica
visão da tripulação que estava a bordo do Titanic. Dessa forma,
ocorre o fenômeno da
a certa distância, o horizonte verdadeiro fica encoberto por uma
névoa escurecida, que se parece muito com águas calmas no a) dispersão e a luz tem velocidade de 200.000 km/s.
escuro. b) reflexão e a luz tem velocidade de 200.000 km/s.
Disponível em: http://apod.nasa.gov. Acesso em: 6 set. 2012 (adaptado).
c) refração e a luz tem velocidade de 200.000 km/s.
O fenômeno ótico que, segundo os pesquisadores, provoca a Fata d) reflexão e a luz tem velocidade de 300.000 km/s.
Morgana é a e) refração e a luz tem velocidade de 300.000 km/s.
a) ressonância. c) difração. e) difusão.
b) refração. d) reflexão. 18. (MACKENZIE)

16. (ESPCEX - AMAN) Uma fibra óptica é um filamento flexível,


transparente e cilíndrico, que possui uma estrutura simples
composta por um núcleo de vidro, por onde a luz se propaga, e uma
casca de vidro, ambos com índices de refração diferentes.
Um feixe de luz monocromático, que se propaga no interior
do núcleo, sofre reflexão total na superfície de separação entre o
núcleo e a casca segundo um ângulo de incidência α, conforme
representado no desenho abaixo (corte longitudinal da fibra).

A flor Vitória Régia em um lago amazonense calmo

A vitória régia é uma flor da Amazônia que tem forma de círculo.


Tentando guardar uma pepita de ouro, um índio a pendurou em
um barbante prendendo a outra extremidade bem no centro de
uma vitória régia de raio R = 0,50 m, dentro da água de um lago
Com relação à reflexão total mencionada acima, são feitas as
amazonense muito calmo. Considerando-se o índice de refração
afirmativas abaixo.
do ar igual a 1,0, o da água nA e o comprimento do barbante, depois
I. O feixe luminoso propaga-se do meio menos refringente para o de amarrado no centro da flor e solto, 50 cm, pode-se afirmar que o
meio mais refringente. valor de nA, de modo que, do lado de fora do lago, ninguém consiga
II. Para que ela ocorra, o ângulo de incidência α deve ser inferior ver a pepita de ouro é:
ao ângulo limite da superfície de separação entre o núcleo e a a) 2,0
casca.
b) 3
III. O ângulo limite da superfície de separação entre o núcleo e a
c) 2
casca depende do índice de refração do núcleo e da casca.
d) 1,0
IV. O feixe luminoso não sofre refração na superfície de separação
entre o núcleo e a casca. e) 0,50

274 PROENEM.COM.BR PRÉ-VESTIBULAR


29 REFLEXÃO TOTAL E DIOPTRO PLANO FÍSICA II

19. (ENEM) Alguns povos indígenas ainda preservam suas tradições


realizando a pesca com lanças, demonstrando uma notável EXERCÍCIOS DE
habilidade. Para fisgar um peixe em um lago com águas tranquilas o
índio deve mirar abaixo da posição em que enxerga o peixe.
APROFUNDAMENTO
Ele deve proceder dessa forma porque os raios de luz
01. (UFU) Considere um raio de luz que parte do ponto 1 e vai até
a) refletidos pelo peixe não descrevem uma trajetória retilínea no o ponto 2, seguindo por um caminho retilíneo, justamente porque
interior da água. é aquele em que tal raio o percorre em menor tempo possível. Na
b) emitidos pelos olhos do índio desviam sua trajetória quando mesma situação, um raio sai do ponto 1 e chega a 3, mas, em vez
passam do ar para a água. de fazer o caminho seguindo a linha tracejada, ele atravessa a
c) espalhados pelo peixe são refletidos pela superfície da água. lâmina de vidro, passando por a e b.

d) emitidos pelos olhos do índio são espalhados pela superfície


da água.
e) refletidos pelo peixe desviam sua trajetória quando passam da
água para o ar.

20. (UNESP) Ao meio-dia, a areia de um deserto recebe grande


quantidade de energia vinda do Sol. Aquecida, essa areia faz com
que as camadas de ar mais próximas fiquem mais quentes do que
as camadas de ar mais altas. Essa variação de temperatura altera
o índice de refração do ar e contribui para a ocorrência de miragens
no deserto, como esquematizado na figura 1. a) Explique por que o raio de luz não segue a linha tracejada, e sim
desvia-se, passando por a e b.
b) Sabendo-se que o índice de refração do vidro é 1,5, qual a
velocidade com que o raio de luz o atravessa?

02. (UFMG) Ariete deseja estudar o fenômeno da dispersão da


luz branca, ou seja, a sua decomposição em várias cores devido
à dependência do índice de refração do material com a frequência.
Para isso, ela utiliza um prisma de vidro cuja seção reta tem a
forma de um triângulo retângulo isósceles.
O índice de refração desse vidro é n = 1,50 para a luz branca e varia
em torno desse valor para as várias cores do espectro visível.
Ela envia um feixe de luz branca em uma direção perpendicular a
uma das superfícies do prisma que formam o ângulo reto, como
mostrado na figura.

Para explicar esse fenômeno, um professor apresenta a seus alunos


o esquema da figura 2, que mostra um raio de luz monocromático
partindo do topo de uma palmeira, dirigindo-se para a areia e
sofrendo refração rasante na interface entre as camadas de ar B e C.

(Dados: sen 45° = cos 45° = 0,707.)


a) COMPLETE, na figura, a trajetória do feixe até sair do prisma.
b) EXPLIQUE, detalhando seu raciocínio, o que acontece com
esse feixe na superfície oposta ao ângulo reto.
c) Ariete observa a dispersão da luz branca nesse experimento?
JUSTIFIQUE sua resposta.

03. (UFG) Um raio de luz monocromático incide perpendicularmente


na face A de um prisma e sofre reflexões internas totais com
toda luz emergindo pela face C, como ilustra a figura a seguir.
Considerando o exposto e sabendo que o meio externo é o ar
(nar = 1), calcule o índice de refração mínimo do prisma.
Sabendo que nesse esquema as linhas que delimitam as camadas
de ar são paralelas entre si, que nA, nB e nC são os índices de refração
das camadas A, B e C, e sendo α o ângulo de incidência do raio na
camada B, o valor de senα é
nC nB nC
a) c) e)
nB nA nA
nB
b) nA d)
nB nC

PRÉ-VESTIBULAR PROENEM.COM.BR 275


FÍSICA II 29 REFLEXÃO TOTAL E DIOPTRO PLANO

04. (UFG) Atividades como falar ao telefone, assistir à TV a cabo, c) Como na reflexão não há dispersão da luz, e na refração com incidência normal também
não ocorre esse fenômeno, Ariete não observa dispersão da luz nesse experimento.
navegar na internet ou mesmo realizar um exame de endoscopia
03. Como ocorre reflexão total, np > 2.
digestiva etc. são possíveis graças à tecnologia associada às fibras
04. a) θc = arcsen(nc/nn).
ópticas. Algumas das vantagens dessa tecnologia são a imunidade
a interferências, grande capacidade de transmissão de dados,= b) Nr Inteiro (L d)(x2 − 1) + 1.
ausência de ruídos, isolação elétrica e sigilo nas comunicações. A 05. d = 6 cm
figura a seguir mostra uma secção de uma fibra óptica, onde ela ANOTAÇÕES
é basicamente constituída de casca e núcleo, ambos de vidro, de
índices de refração diferentes.

1
Dado: sen=
θ , x > 1.
x
a) Calcule o valor do ângulo crítico θc, para que haja a transmissão
da luz, dados os índices de refração nc da casca e nn do núcleo,
com nc < nn.
b) Considerando que as reflexões internas totais em toda fibra se
comportem conforme a secção da figura (θ > θc), determine o
número de reflexões num comprimento L da fibra, em função
de x, L e d (diâmetro do núcleo).

05. (UERJ) Uma caixa d’água cilíndrica, com altura h = 36 cm e


diâmetro D = 86 cm, está completamente cheia de água. Uma
tampa circular, opaca e plana, com abertura central de diâmetro d,
é colocada sobre a caixa. No esquema a seguir, R representa o raio
da tampa e r o raio de sua abertura.

Determine o menor valor assumido por d para que qualquer raio de


luz incidente na abertura ilumine diretamente o fundo da caixa, sem
refletir nas paredes verticais internas.

GABARITO

EXERCÍCIOS PROPOSTOS
01. B 05. A 09. A 13. B 17. B
02. D 06. B 10. C 14. E 18. C
03. B 07. A 11. A 15. B 19. E
04. B 08. A 12. B 16. B 20. E

EXERCÍCIOS DE APROFUNDAMENTO
01. a) Devido à diferença entre os índices de refração entre o vidro e o ar, pela Lei de
Snell-Descartes, o raio de luz que incide com ângulo não nulo sofre desvio que pode ser
observado pela mudança entre os seus ângulos de incidência e refração.
b) 2 ·108 m/s

02. a)
b) Concluímos que sen i > sen L. Logo, ocorre reflexão total.

276 PROENEM.COM.BR PRÉ-VESTIBULAR


30 ESPELHOS E LENTES: FÍSICA II

RAIOS PARTICULARES

ESPELHOS ESFÉRICOS ESPELHOS ESFÉRICOS GAUSSIANOS


Os espelhos esféricos consistem em superfícies refletoras na O estudo da imagem conjugada pelos espelhos esféricos é
forma de calotas esféricas, ou seja, são obtidos seccionando-se muito complicado na prática. Isto se deve ao fato dos raios refletidos
uma superfície esférica através de um plano. pelos espelhos esféricos não se cruzarem em um mesmo ponto
(como mostra a figura a seguir), o que faz com que chamemos os
espelhos esféricos de astigmáticos, e assim, há formação de uma
mancha e não uma imagem nítida.

Existem dois tipos de espelhos esféricos: o espelho côncavo


possui a parte voltada para o centro de curvatura, como a superfície
espelhada e refletora. No espelho convexo a face espelhada está
voltada para o lado externo oposto ao centro de curvatura.
Nos primeiros anos do século XVIII, o físico e matemático
Gauss percebeu que se considerarmos condições especiais,
a imagem pode ser mais facilmente estudada. Chamamos de
espelhos esféricos gaussianos aqueles que respeitam as condições
enumeradas por Gauss que são:
• os raios de luz devem ser pouco inclinados em relação ao
eixo principal;
• os raios de luz devem incidir próximos ao vértice do
espelho.
A não ser que esteja claramente especificado, iremos
Existem alguns elementos de suma importância cujos
considerar sempre os espelhos como gaussianos.
conceitos devem ser muito bem entendidos:
• Raio de curvatura (R): consiste no raio da superfície
esférica que originou a calota, ou seja, o próprio espelho. RAIOS PARTICULARES
• Centro de curvatura (C): é o centro da esfera original. Na determinação da imagem iremos considerar sempre os
• Vértice (V): é o polo da calota. raios particulares. A imagem irá se formar no ponto de encontro
dos raios refletidos ou no ponto de encontro dos prolongamentos
• Foco Principal (F): ponto para onde os raios pa-ralelos ao
dos raios refletidos. Os raios particulares são:
eixo principal são refletidos.
• Todo raio que incide numa direção que passa pelo centro
• Distância focal (f): consiste na distância do vértice ao foco
de curvatura reflete sobre si mesmo. Isto se deve ao fato de
principal e equivale aproximadamente à metade do raio de
o raio possuir a mesma direção da reta normal;
curvatura.
• Eixo principal: é a reta que passa pelo centro de curvatura
e pelo vértice.
• Eixos secundários: são retas perpendiculares à superfície
do espelho que passam pelo centro de curvatura.
Assim como o eixo principal determina o foco princi-pal, devido
à existência dos eixos secundários, também existirão os focos
secundários. Cada foco secundário depende dos raios incidentes,
pois, paralelo a esses, traça-se o eixo secundário próprio e o foco
secundário será determinado pelo cruzamento do eixo secundário • Todo raio que incide no vértice do espelho reflete
com os raios refletidos, como mostra a figura ao lado. simetricamente ao eixo óptico principal. Nesse caso, o eixo
principal se confunde com a normal;

V F C

Eixodário
n
secu Espelho Convexo

PRÉ-VESTIBULAR PROENEM.COM.BR 277


B C B’B’ F V
i
B
B C i F V
V
C iA’ F
FÍSICA II 30 ESPELHOSA’
A’
E LENTES: RAIOS PARTICULARES

• Todo raio que incide paralelamente ao eixo óptico reflete • Objeto Sobre o Centro de Curvatura
em uma direção que passa pelo foco principal do espelho;

A REAL
A INVERTIDA
REAL
A REAL
DO MESMO
INVERTIDA
INVERTIDA
TAMANHO
DO
B DO MESMO
MESMO
TAMANHO
B’B
B C F V TAMANHO
B’
B’ C F V
V
• Todo raio que incide numa direção que passa pelo foco C F
principal reflete paralelamente ao eixo óptico principal. A’
A’
A’

• Objeto Entre o Centro e o Foco


REAL
INVERTIDA
REAL
REAL
MAIOR
INVERTIDA
A INVERTIDA
Classificação das imagens A
MAIOR
MAIOR
A
Uma imagem pode ser:
B’
• direita → se o objeto estiver no mesmo semiplano da
B’
B’ C B F V
imagem, ou seja, em cima ou embaixo;
C B F V
V
• invertida → se o objeto e a imagem estiverem em C B F
i
semiplanos diferentes; ii
• maior → se o tamanho da imagem for maior que o do objeto;
A’
• menor → se a imagem for menor que o objeto; A’
A’
• igual → se o objeto e a imagem possuírem o mesmo tamanho;
• real → se a imagem for formada diante do espelho.
Dependendo do poder refletor do espelho, só poderá ser • Objeto Sobre o Foco
observada através de projeções em anteparos;
• virtual → quando obtemos a imagem a partir do encontro IMPRÓPRIA
DIZ-SE QUE ELA
do prolongamento dos raios luminosos e não a partir IMPRÓPRIA
IMPRÓPRIA
SE FORMA
dos próprios raios, dizemos que a imagem é virtual. Nas A DIZ-SE
DIZ-SE
NO
QUE ELA
QUE ELA
INFINITO
SE FORMA
representações a imagem se forma atrás do espelho. A
A
SE FORMA
NO INFINITO
NO INFINITO
B
DETERMINAÇÃO GRÁFICA DA IMAGEM C B
BF V
Considere um objeto AB colocado diante de um espelho C F V
V
C F
esférico. Utilizaremos os raios particulares para se obter a posição
correta da imagem no eixo principal.
Na determinação gráfica da imagem, consideremos a luz
que parte da extremidade A que sofrerá reflexão e formará a
extremidade A’ da imagem. Basta ligar o ponto A’ ao eixo principal,
obtendo o ponto B’ e, dessa forma, determinando a imagem A’B’.
A cada posição diferente que colocamos o objeto, a imagem se
apresentará diferente. Veremos abaixo todas as diferentes imagens • Objeto Entre o Foco e o Vértice
formadas e suas respectivas classificações:
VIRTUAL
Em espelho côncavo DIREITA
A’
• Objeto Antes do Centro de Curvatura MAIOR
A

A REAL
INVERTIDA B’
MENOR C F B V
B’
B C i F V
A’

VIRTUAL
N’
A REAL
N
DIREITA
INVERTIDA MAIOR
DO MESMO
TAMANHO
B O
278 PROENEM.COM.BR PRÉ-VESTIBULAR
B’ C F V A F M’ M F’ A’
30 ESPELHOS E LENTES: RAIOS PARTICULARES FÍSICA II

Em espelho convexo

A VIRTUAL
DIREITA
MENOR
A’

B V B’ F C

O chamado espelho de aumento é o espelho côncavo com o


objeto colocado entre o foco e o vértice. Este espelho é normalmente
utilizado por dentistas.
O espelho convexo produz sempre uma imagem menor e,
por isso, é muito utilizado em retrovisores pois oferece um maior
campo visual.
Podemos perceber também que sempre que o objeto e a
imagem possuírem a mesma natureza, ou seja, os dois virtuais ou
os dois reais, a imagem será invertida. Caso o objeto e a imagem
possuam naturezas diferentes, um real e o outro virtual, a imagem
será direita. • Quanto às faces: existem seis tipos diferentes de lentes
que podem ser divididos em dois grupos: bordos finos e
grossos.
LENTES ESFÉRICAS • Quanto ao comportamento óptico: são duas classificações
Consiste na união de dois dioptros, sendo um neces-sariamente possíveis: convergente ou divergente.
esférico e outro podendo ser esférico ou plano. Normalmente, as
lentes esféricas são compostas de vidro e serão considerados os
casos em que elas são envolvidas por somente um meio. As lentes,
como constituem um meio de propagação, possuem seu próprio eixo eixo
índice de refração. principal principal

ELEMENTOS GEOMÉTRICOS lente convergente lente divergente


Através da figura abaixo, iremos identificar os elementos
geométricos principais das lentes: Após os raios passarem por uma lente convergente eles
convergem, ou seja, formam um pincel de luz convergente. Do
mesmo modo, se um feixe de luz incidir sobre uma lente divergente,
haverá a formação de um pincel de luz divergente.
Para que uma lente seja classificada de convergente ou
divergente, é preciso comparar seu índice de refração com o índice
de refração do meio que a envolve. Se o índice de refração da lente
for maior que o índice do meio externo, as lentes de bordos finos
serão convergentes e as de bordos grossos serão divergentes.
Caso o meio externo seja mais refringente que a lente, as lentes
Em que: de bordos finos serão divergentes e as de bordos grossos serão
• C1 e C2 são os centros de curvatura das faces S1 e S2 convergentes.
• R1 e R2 são os raios de curvatura das faces S1 e S2; nL > nMEIO
• V1 e V2 são os vértices(interseções do eixo principal com Bordos finos: convergente
as faces S1 e S2); Bordos grossos: divergente
• eixo óptico principal é a reta que passa por C1 e C2; nL > nMEIO
• e = V1V2 é a espessura da lente. Bordos finos: divergente
Bordos grossos: convergente
CLASSIFICAÇÃO DAS LENTES
Apresentaremos, agora, os grupos e as classificações das FOCOS E PONTOS ANTIPRINCIPAIS
lentes. Uma primeira classificação refere-se à espessura. As lentes Nas lentes delgadas existem dois focos principais: o foco
que serão preferencialmente estudadas são as lentes de espessura imagem e o foco objeto. Quando raios paralelos incidem na lente,
desprezível, que são chamadas de lentes delgadas. Além desse ao refratar suas direções passam sempre pelo mesmo ponto
tipo especial de lentes, levaremos em conta a classificação em denominado foco imagem (F’).
duas categorias.

PRÉ-VESTIBULAR PROENEM.COM.BR 279


FÍSICA II 30 ESPELHOS E LENTES: RAIOS PARTICULARES

• Todo raio que incide em uma direção que passa pelo foco
objeto principal refrata paralelamente ao eixo principal.

Lente divergente     Lente convergente

Raios incidentes com sua direção passando pelo foco objeto


(F) refratam paralelamente ao eixo principal.

Lente convergente        Lente divergente

• Todo raio que incide em uma direção que passa pelo ponto
antiprincipal objeto refrata em uma direção que passa pelo
ponto antiprincipal imagem.

Lente divergente     Lente convergente

A distância focal será a distância entre o centro óptico da lente


e qualquer um dos dois focos, pois os focos estão equidistantes
da lente. Na figura, a distância focal pode ser representada por FO
ou F’O.
Além dos focos, existem os pontos antiprincipais, que também
são dois, e estão representados na figura abaixo por A (ponto Lente convergente
antiprincipal objeto) e A’ (ponto antiprincipal imagem).

DETERMINAÇÃO GRÁFICA
DA IMAGEM
A determinação gráfica de uma imagem produzida por uma
lente é um processo análogo ao de espelhos esféricos. Devemos
traçar os raios particulares e a imagem será formada pela
interseção dos raios que emergem da lente. Existe uma grande
diferença na hora de classificarmos as imagens em reais ou
virtuais. Uma imagem é considerada virtual quando ela pode ser
observada atrás do espelho ou lente pelo observador. Porém, nos
RAIOS LUMINOSOS PARTICULARES espelhos esféricos, o observador se encontra do mesmo lado do
Assim como acontece com os espelhos, as lentes também objeto e, dessa forma, uma imagem virtual estará do lado oposto
possuem seus raios particulares, que são úteis na determinação do objeto. Já nas lentes, o observador está do lado oposto do objeto
gráfica das imagens. e, assim sendo, para que ele observe a imagem atrás da lente, ela
deverá estar do mesmo lado do objeto.
• Todo raio de luz que incide no centro óptico não sofre
desvio. Observe os exemplos abaixo em que obteremos a imagem M’N’
do objeto MN:

LENTE DIVERGENTE

N VIRTUAL
DIREITA
MENOR
N’

• Todo raio que incide paralelamente ao eixo principal refrata M A’ F’ M’ 0 F A


em um direção que passa pelo foco imagem principal.

A lente divergente só conjuga um tipo de imagem para objetos


reais.

N REAL
Lente convergente        Lente divergente INVERTIDA
MENOR

M’
280 PROENEM.COM.BR M A F 0 F’ PRÉ-VESTIBULAR
A’
N’
M M’
M A’ F’ M’ 0 F A A F 0 F’ A’
30 ESPELHOS E LENTES: RAIOS PARTICULARES FÍSICA
N’ II

LENTE CONVERGENTE VERGÊNCIA (V)


• Objeto antes do ponto antiprincipal objeto A grandeza vergência está relacionada à capacidade da lente
de convergir ou divergir, ou seja, uma lente com alta vergência
REAL possui altoREAL
poder de convergência ou divergência. A vergência é o
N N
INVERTIDA inverso daINVERTIDA
distância focal.
MENOR MAIOR

M M’
M’ A unidade de vergência
A é Fm-1 ou a0 dioptria F’
(di). Popularmente,
A’ a
M A F 0 F’ A’ dioptria é conhecida como grau.
N’

FÓRMULA DOS FABRICANTES DE LENTESN’


A fórmula dos fabricantes de lentes ou equação de Halley nos
• Objeto no ponto antiprincipal objeto dá a vergência em função de seu índice de refração em relação ao
meio externo (nL,E) e os raios das faces (R1 e R2):
REAL
REAL
INVERTIDA N
REAL
VIRTUAL INVERTIDA
IMPRÓPRIA
DO MESMO INVERTIDA
N
DIREITA DO MESMO
N TAMANHO DO MESMO
MENOR N
TAMANHO TAMANHO
N’ Para colocarmos os valores corretamente, devemos ter
M M’ M
M’ atenção nas seguintes’ observações:
M 0 F’ A’ AM F 0 F’ A’ M
A F • caso uma face seja plana, seu raio é considerado infinito e
F’ M’ 0 F A A F 0 F’ A’ 0 F’ A’
A F 1/R passa a valer zero;
N’ ’
N N’
• se a face for côncava, seu raio é considerado negativo;
• se a face for convexa, seu raio é considerado positivo.
• Objeto entre o Ponto Antiprincipal e o Foco Se a vergência P∞for negativa, a lente será divergente e caso ela
seja positiva, a lente será convergente.
REAL
REAL
INVERTIDA N REAL EXERCÍCIO RESOLVIDO
REAL
INVERTIDA N
MAIOR INVERTIDA INVERTIDA N
MAIOR
MENOR MAIOR
M M’ 01. No dia 20 de junho de 1969, o ser humano caminhou pela
M M’ primeira vez na superfície
A F 0 F’ A’ M M’ lunar. Em uma das fotos registradas
M’ A F 0 F’ A’ nesse dia pode-se ver uma imagem direita e menor formada
A F 0 F’ A’
F 0 F’ A’ pela superfície convexa do visor do capacete do astronauta
N’ Edwin Aldrin, que funciona como um espelho.
N’
N’
N’

• Objeto no Foco
N
N
N
IMPRÓPRIA
VIRTUAL
DIREITA
A’
M M MAIOR
F F 0 F’ F’ A’ A’ M A
0
A F 0 F’ A’

B’
C F B V
Essa imagem é
P∞ P∞ a) real e o objeto se encontra além do centro de curvatura
P∞ do espelho.
b) virtual e independe da localização do objeto.
• Objeto entre o Foco e o Centro Óptico
c) virtual e o objeto se encontra entre o espelho e seu foco
principal.
d) real e o objeto se encontra entre o espelho e seu foco
VIRTUAL principal.
N’
DIREITA
N MAIOR e) real e independe da localização do objeto.

Resolução: B
O
A F M’ M F’ A’ Em todo espelho esférico gaussiano convexo a imagem é
sempre direita, virtual e menor. O foco do espelho convexo é
sempre virtual, independendo da posição do objeto.

PRÉ-VESTIBULAR PROENEM.COM.BR 281


FÍSICA II 30 ESPELHOS E LENTES: RAIOS PARTICULARES

02. (UEL) Certos dispositivos possibilitam visualizar ou 03. (FUVEST) Três amigos vão acampar e descobrem que
demonstrar fenômenos naturais explicados pelas Leis nenhum deles trouxe fósforos. Para acender o fogo e fazer
da Física como o que se encontra no Museu de Ciência e o almoço, resolvem improvisar e prendem um pedaço de
Tecnologia de Londrina, conforme a figura a seguir. filme plástico transparente num aro de “cipó”. Colocam um
pouco de água sobre o plástico, formando uma poça de
aproximadamente 14 cm de diâmetro e 1 cm de profundidade
máxima, cuja forma pode ser aproximada pela de uma calota
esférica. Quando o sol está a pino, para aproveitamento
máximo da energia solar, a distância, em cm, entre o
centro do filme e a palha seca usada para iniciar o fogo, é,
aproximadamente,
Note e adote:
1 1
- Para uma lente plano-convexa, = (n − 1) , sendo n o índice
f R
de refração da lente e R o seu raio de curvatura.
- Índice de refração da água = 1,33.
a) 75
b) 50
c) 25
Nos compartimentos inferiores do dispositivo, há dois d) 14
tipos de lentes, sendo possível observar a convergência e a
e) 7
divergência dos raios de luz que incidem nas lentes e delas
emergem ao se acionar um botão. Resolução: A
Com base na imagem e nos conhecimentos sobre lentes Representação da situação descrita no enunciado:
esféricas, assinale a alternativa que apresenta, corretamente,
o caminho percorrido pelos raios de luz.
a)

b)

c)

Formou-se uma lente plano-convexa, cujo raio de curvatura


R pode ser obtido aplicando-se o teorema de Pitágoras no
d) triângulo retângulo da figura:
(R − 1)
2
R2 = + 72 ⇒ R2 = R2 − 2R + 1+ 49 ⇒
⇒ 2R = 50 ⇒ R = 25 cm
e) Aplicando a equação fornecida:
1 1 1 1
= (n − 1) ⇒ = (1,33 − 1) ⇒
f R f 25
1 0,33 25
⇒= ⇒f≅ ⇒ f ≅ 75 cm
f 25 1
Resolução: C 3
Através dos raios típicos podemos excluir alternativas A distância entre o centro da lente e a fonte de calor deve ser
erradas: igual a distância focal. Logo, a distância entre o centro do
Todo o raio de luz incidente sobre o centro óptico não sofre filme e a palha seca deve ser de aproximadamente 75 cm.
desvio. Descartadas as alternativas [B], [D] e [E].
Todo o raio de luz incidente que passa pelo foco, refrata-se
na lente paralelamente ao eixo principal. Descartadas as
alternativas [A], [D] e [E].
Todo o raio de luz incidente paralelamente ao eixo principal
refrata-se passando pelo foco. Descartadas as alternativas
[B]] e [E].

282 PROENEM.COM.BR PRÉ-VESTIBULAR


30 ESPELHOS E LENTES: RAIOS PARTICULARES FÍSICA II

04. Um espelho côncavo tem distância focal igual a f. Um objeto


EXERCÍCIOS real de altura h é colocado a uma distância d0 defronte do espelho,
PROTREINO sobre o eixo do mesmo. Descreva as características desta imagem
(tamanho, direita ou invertida, real ou virtual), em cada uma das
seguintes condições:
01. Um objeto de comprimento L foi colocado à frente de espelho
a) d0 > 2f
esférico gaussiano côncavo. Sabendo que o centro de curvatura do
espelho é representado pelo ponto R aponte o ponto que representa b) d0 = f
o foco do espelho e usando os raios notáveis determine a posição c) d0 < f
da imagem.
05. Um objeto é colocado sobre o eixo principal de um espelho
esférico gaussiano convexo.

02. Determine graficamente, na figura abaixo, a imagem formada Determine graficamente, na figura acima, a imagem formada,
representando, adequadamente, no mínimo, dois raios “notáveis”, representando, adequadamente, no mínimo, dois raios “notáveis”,
antes e após a ocorrência da reflexão. antes e após a ocorrência da reflexão.

EXERCÍCIOS
PROPOSTOS
01. (IFCE) Como atividade extraclasse, um aluno do IFCE resolveu
gravar um vídeo no qual utilizou-se de um espelho para representar
suas emoções. Num trecho específico do vídeo ele dizia que se
sentia grande, com o dobro de seu tamanho. Em outro momento
ele afirmava que sua vida estava ao contrário do que devia ser e
mostrava uma imagem invertida. Por fim, dizia que na situação
atual do país ele não tinha nenhuma referência política para se
espelhar e, colocava-se a uma posição tal do espelho que sua
imagem se situava no ‘infinito’.
03. Um objeto é coloca à frente de uma lente esférica de borda fina.
De acordo com o enunciado, é correto afirmar-se que
a) o espelho usado pelo aluno era convexo.
b) não é possível saber que tipo de espelho o aluno usava,
podendo ser côncavo ou convexo.
c) o espelho usado pelo aluno era côncavo.
d) não restam dúvidas de que o espelho era plano.
e) não é possível que um único espelho produza todas as imagens
mencionadas no texto.

Determine graficamente, na figura acima, a imagem formada


representando adequadamente, no mínimo, dois raios “notáveis”,
antes e após a ocorrência da refração.

PRÉ-VESTIBULAR PROENEM.COM.BR 283


FÍSICA II 30 ESPELHOS E LENTES: RAIOS PARTICULARES

02. (UEFS) A figura representa um espelho esférico gaussiano (E), Eles contaram que já naquela época o “cientista bélico”
seu centro de curvatura (C), seu foco principal (F) e seu vértice (V). Arquimedes teria inventado várias armas de guerra, incluindo
A figura também mostra quatro regiões (I, II, III e IV) identificadas catapultas, a terrível “Mão de Ferro” (um guindaste que pegava
por cores diferentes. os navios e os levantava), um “Canhão a Vapor” e aquilo que ficou
conhecido como o “Raio da Morte” ou “Raio de Calor”.
O “Raio da Morte” era, na realidade, a concentração dos raios
de luz, advindos do sol, refletidos em um sistema formado por
inúmeros espelhos, formando o que seria um poderoso espelho
esférico côncavo.
Considerando-se ser o eixo principal deste espelho paralelo
ao horizonte; a distância do vértice do espelho ao ponto de
concentração máxima (ponto de queima) dos raios, d, e altura do
ponto de queima ao eixo principal, h, afirma-se corretamente que
o raio de curvatura do espelho esférico é fornecido pela expressão
a) h2 − d2
b) 2 h2 + d2
c) 2 d2 − h2
d) 2 h2 − d2
e) d2 + h2
Se um objeto pontual for colocado sucessivamente nos pontos
1 e 2, as imagens conjugadas pelo espelho se formarão, 05. (FUVEST) Uma pessoa observa uma vela através de uma lente
respectivamente, nas regiões de vidro biconvexa, como representado na figura.
a) II e IV.
b) III e I.
c) III e IV.
d) II e III.
e) II e I.

03. (ESPCEX - AMAN) O espelho retrovisor de um carro e o Considere que a vela está posicionada entre a lente e o seu ponto
espelho em portas de elevador são, geralmente, espelhos esféricos focal F. Nesta condição, a imagem observada pela pessoa é
convexos. Para um objeto real, um espelho convexo gaussiano a) virtual, invertida e maior.
forma uma imagem b) virtual, invertida e menor.
a) real e menor. c) real, direita e menor.
b) virtual e menor. d) real, invertida e maior.
c) real e maior. e) virtual, direita e maior.
d) virtual e invertida.
e) real e direita. 06. (UPF) Muitos instrumentos se utilizam de lentes esféricas
delgadas para seu funcionamento. Tais lentes podem ser do tipo
04. (MACKENZIE) ESPELHO DE ARQUIMEDES – O raio da Morte! convergente ou divergente e formam imagens com características
específicas.
Sobre as imagens formadas por essas lentes, é correto afirmar que
a) quando um objeto é posicionado no foco de uma lente
convergente, se forma uma imagem real, maior e direita.
b) quando um objeto é posicionado entre o foco e o centro ótico
de uma lente convergente, se forma uma imagem real, maior
e direita.
c) quando um objeto é posicionado entre o foco e o centro ótico
de uma lente convergente, não se forma nenhuma imagem.
d) uma lente divergente só pode formar uma imagem virtual,
menor e direita de um objeto.
e) uma lente divergente só pode formar uma imagem real, maior
e direita de um objeto.

07. (IFSUL) Um objeto está localizado a 50,00 cm de uma lente


convergente, cuja distância focal é de 15,00 cm. Com relação à
imagem formada pela lente, é correto afirmar que é
a) virtual, direita e maior que o objeto.
A época era aproximadamente 200 a.C., o local, a cidade-estado
de Siracusa, Sicília, nas Grandes Guerras Púnicas. Os contadores b) real, invertida e menor que o objeto.
da história foram, dentre outros, Tzestes e Luciano de Samosata c) virtual, invertida e menor que o objeto.
em sua obra Hippias. d) real, invertida e maior que o objeto.

284 PROENEM.COM.BR PRÉ-VESTIBULAR


30 ESPELHOS E LENTES: RAIOS PARTICULARES FÍSICA II

08. (FATEC) A figura apresenta a obra de litogravura “Mão com 10. (UNICAMP) A lupa é um instrumento óptico simples formado
esfera refletora” (1935), do artista gráfico holandês Maurits Cornelis por uma única lente convergente. Ela é usada desde a Antiguidade
Escher (1898–1972), que se representou por uma imagem refletida para observar pequenos objetos e detalhes de superfícies. A
em uma esfera. imagem formada pela lupa é direta e virtual. Qual figura abaixo
representa corretamente o traçado dos raios luminosos principais
provenientes de um determinado ponto de um objeto observado
por uma lupa? Nessas figuras, (f) e (f’) representam os pontos
focais, (o) o objeto e (i) a imagem.

a)

b)

Sendo o artista o objeto refletido na superfície dessa esfera, podemos


afirmar corretamente, sobre essa imagem formada, que se
a) assemelha à classificação exata de uma imagem observada
em uma lente delgada convergente.
b) assemelha à classificação exata de uma imagem observada c)
em um espelho côncavo.
c) classifica em menor, direita e real.
d) posiciona entre o foco e o vértice da face refletora.
e) posiciona entre o raio de curvatura e o vértice da face refletora.

09. (ESPCEX - AMAN) Um objeto retilíneo e frontal XY, perpendicular


ao eixo principal, encontra-se diante de uma lente delgada
convergente. Os focos F e F’, os pontos antiprincipais A e A’ e o
centro óptico “O” estão representados no desenho abaixo.
d)

Com o objeto XY sobre o ponto antiprincipal A, pode-se afirmar que


a imagem X' Y ', desse objeto é:
Dados: OF = FA e OF ' = F ' A '
a) real, invertida, e do mesmo tamanho que XY.
b) real, invertida, maior que XY.
c) real, direita, maior que XY.
d) virtual, direita, menor que XY.
e) virtual, invertida, e do mesmo tamanho que XY.

PRÉ-VESTIBULAR PROENEM.COM.BR 285


FÍSICA II 30 ESPELHOS E LENTES: RAIOS PARTICULARES

11. (MACKENZIE) Considerando que o círculo representa a lente, cuja distância focal
é igual a F, a distância entre o centro óptico da lente e o painel é
a) igual a F.
b) maior que 2F.
c) igual a 2F.
d) menor que F.
e) maior que F e menor que 2F.

15. (UNICAMP) Em uma animação do Tom e Jerry, o camundongo


Jerry se assusta ao ver sua imagem em uma bola de Natal cuja
superfície é refletora, como mostra a reprodução abaixo.

O espelho bucal, utilizado por dentistas, é um instrumento que pode


ser feito com um espelho plano ou esférico.
Um dentista, a fim de obter uma imagem ampliada de um dente
específico, deve utilizar um espelho bucal
a) côncavo, sendo colocado a uma distância do dente menor que
a distância focal.
b) côncavo, sendo colocado a uma distância do dente entre o foco
e o centro de curvatura.
c) convexo, sendo colocado a uma distância do dente entre o foco
e o centro de curvatura.
d) plano.
É correto afirmar que o efeito mostrado na ilustração não ocorre na
e) convexo, sendo colocado a uma distância do dente menor que realidade, pois a bola de Natal formaria uma imagem
a distância focal.
a) virtual ampliada.
12. (UECE) Dentre muitas aplicações, a energia solar pode ser b) virtual reduzida.
aproveitada para aquecimento de água. Suponha que para isso c) real ampliada.
seja utilizada uma lente delgada para concentrar os raios solares d) real reduzida.
em um dado ponto que se pretende aquecer. Assuma que os raios
incidentes sejam paralelos ao eixo principal. 16. (IFSUL) No laboratório de Física de uma escola, um aluno
Um tipo de lente que pode ser usada para essa finalidade é a lente observa um objeto real através de uma lente divergente.
a) divergente e o ponto de aquecimento fica no foco. A imagem vista por ele é
b) convergente e o ponto de aquecimento fica no vértice. a) virtual, direita e menor.
c) convergente e o ponto de aquecimento fica no foco. b) real, direita e menor.
d) divergente e o ponto de aquecimento fica no vértice. c) virtual, invertida e maior.
d) real, invertida e maior.
13. (UECE) Sobre lentes convergentes, é correto afirmar que um
raio de luz que incide paralelo ao eixo da lente 17. (ENEM – LIBRAS) Um experimento bastante interessante no
a) passa pelo foco após a refração. ensino de ciências da natureza constitui em escrever palavras em
b) passa pelo foco após a difração. tamanho bem pequeno, quase ilegíveis a olho nu, em um pedaço
de papel e cobri-lo com uma régua de material transparente. Em
c) segue paralelo ao eixo após a refração.
seguida, pinga-se uma gota d’água sobre a régua na região da
d) segue paralelo ao eixo após a difração. palavra, conforme mostrado na figura, que apresenta o resultado
do experimento. A gota adquire o formato de uma lente e permite
14. (UNESP) A figura representa um painel colorido e a imagem de ler a palavra de modo mais fácil em razão do efeito de ampliação.
parte desse painel, observada através de uma lente convergente,
colocada paralelamente à sua frente.

Qual é o tipo de lente formada pela gota d’água no experimento


descrito?
a) Biconvexa. d) Plano-côncava.
b) Bicôncava. e) Convexa-côncava.
c) Plano-convexa.

286 PROENEM.COM.BR PRÉ-VESTIBULAR


30 ESPELHOS E LENTES: RAIOS PARTICULARES FÍSICA II

18. (IFSUL) Um objeto real é colocado perpendicularmente ao 02. (UNICAMP) As vidraças de um arranha-céu em Londres,
eixo principal de um espelho esférico convexo. Nota-se que, nesse conhecido como “Walkie Talkie”, reproduzem a forma de um espelho
caso, a altura da imagem virtual é i1. Em seguida, o mesmo objeto é côncavo. Os raios solares refletidos pelo edifício provocaram danos
aproximado do espelho, formando uma nova imagem com altura i2. em veículos e comércios próximos.
Quando se traz para mais perto o objeto, a imagem se a) Considere um objeto em frente e ao longo do eixo do espelho
a) aproxima do espelho, sendo i1 < i2. côncavo de raio de curvatura R = 1,0 m, conforme mostra a
figura a seguir. Complete os raios luminosos na figura. Em
b) aproxima do espelho, sendo i1 > i2. seguida, calcule a distância d do objeto ao vértice do espelho
c) afasta do espelho sendo i1 = i2. (ponto O), de forma que a intensidade de raios solares,
d) afasta do espelho sendo i1 < i2. incidentes paralelamente ao eixo do espelho, seja máxima na
posição do objeto.
19. (IFSUL) Um objeto linear é colocado diante de um espelho
côncavo, perpendicularmente ao eixo principal. Sabe-se que a
distância do objeto ao espelho é quatro vezes maior que a distância
focal do espelho.
A imagem conjugada por este espelho é
a) virtual, invertida e maior que o objeto.
b) virtual, direita, e menor que o objeto.
c) real, invertida, menor que o objeto.
d) real, direita e maior que o objeto.

20. (MACKENZIE) Uma lente convergente de distância focal f


e centro óptico O conjuga de um objeto real, uma imagem real,
invertida e de mesmo tamanho. Esse objeto encontra-se
a) entre o centro óptico e o foco.
b) sobre o foco.
c) sobre o ponto antiprincipal objeto.
d) entre o foco e o ponto antiprincipal objeto. b) Um objeto metálico de massa m = 200 g e calor específico
c = 480 J/(kg · °C) absorve uma potência P = 60 W de radiação
e) antes do ponto antiprincipal objeto. solar focalizada por um espelho côncavo. Desprezando as
perdas de calor por radiação, condução e convecção, calcule a
variação de temperatura do objeto após ∆t = 32 s de exposição
EXERCÍCIOS DE a essa radiação.
APROFUNDAMENTO 03. (UNIFESP) Em um parque de diversões existem dois grandes
espelhos dispostos verticalmente, um de frente para o outro, a
01. (UERJ) Alguns espelhos retrovisores, instalados nas laterais 10 m de distância um do outro. Um deles é plano, o outro é esférico
dos veículos automotores, apesar de aparentemente planos, são convexo. Uma criança se posiciona, em repouso, a 4 m do espelho
esféricos. Seu uso aumenta a segurança no trânsito uma vez que esférico e vê as duas primeiras imagens que esses espelhos
ampliam o campo de visão dos condutores, conforme ilustrado na formam dela: IP, formada pelo espelho plano, e IC, formada pelo
imagem. espelho esférico, conforme representado na figura.

Calcule:
a) a distância, em metros, entre IP e IC.
b) a que distância do espelho esférico, em metros, a criança
deveria se posicionar para que sua imagem IC tivesse um terço
de sua altura.
Com base nas informações, identifique o tipo de espelho esférico
utilizado como retrovisor lateral. Indique, ainda, três características
das imagens que esse espelho conjuga.

PRÉ-VESTIBULAR PROENEM.COM.BR 287


FÍSICA II 30 ESPELHOS E LENTES: RAIOS PARTICULARES

04. (UFPR) Um espelho côncavo, com raio de curvatura 10 cm e ANOTAÇÕES


centro em C, foi posicionado de acordo com a figura abaixo. Um objeto
O, com 2 cm de altura, está localizado a 3 cm do espelho e orientado
para baixo, a partir do eixo principal. Os segmentos que podem ser
observados sobre o eixo principal são equidistantes entre si.
a) Na figura, assinale o foco do espelho, ressaltando-o por meio
da letra F.

b) Determine graficamente, na figura, a imagem formada, repre-


sentando, adequadamente, no mínimo, dois raios “notáveis”,
antes e após a ocorrência da reflexão.

c) Determine, apresentando os devidos cálculos, o tamanho da


imagem. É sabido que a ampliação corresponde ao simétrico
da razão entre a distância da imagem ao espelho e a distância
do objeto ao espelho, ou a razão entre o tamanho da imagem e
o tamanho do objeto, com as devidas orientações.

05. (FMJ) Um objeto é colocado perpendicularmente sobre o eixo


principal de um espelho esférico de distância focal 2 m, que atende
às condições de nitidez de Gauss. A imagem formada é virtual,
direita e com o dobro do comprimento do objeto.
Nas condições descritas, relativas à natureza e à posição da
imagem formada, determine:
a) o tipo do espelho esférico empregado.
b) a distância, em metros, do objeto ao vértice do espelho esférico.

GABARITO

EXERCÍCIOS PROPOSTOS
01. C 05. E 09. A 13. A 17. C
02. A 06. D 10. A 14. C 18. A
03. B 07. B 11. A 15. B 19. C
04. C 08. D 12. C 16. A 20. C

EXERCÍCIOS DE APROFUNDAMENTO
01. De acordo com as informações do enunciado, o espelho é convexo. E as características
das imagens formadas são: virtuais, menores e direitas.
02. a) 0,5 m; b) 20 °C
03. a) 18 m; b) 8 m
04. a)

b)

c) 5 cm de altura
05. a) O único espelho esférico que fornece uma imagem ampliada e direita é o espelho
côncavo. O objeto deve estar entre a distância focal e o espelho, portanto podemos
também prever que a resposta para o item (b) é menor que 2 m.
b) 1 m

288 PROENEM.COM.BR PRÉ-VESTIBULAR


31 ESPELHOS E LENTES: FÍSICA II

EQUAÇÃO DE GAUSS

INTRODUÇÃO tabela abaixo, temos um resumo com os sinais adotados que


chamamos de referencial gaussiano.
Após a localização gráfica, caso haja necessidade de se obter
As imagens podem se localizar tanto na frente como atrás do
informações precisas sobre a imagem, utilizaremos uma expressão
espelho, então, para que possamos saber sua verdadeira posição,
matemática conhecida como equação de Gauss. Antes de citar a
adotou-se um referencial de sinais positivo e negativo.
expressão iremos definir algumas grandezas:

• p é a distância do objeto ao sistema óptico; abscissa do


objeto;
• p’ é a distância da imagem ao sistema óptico; abscissa da
imagem;
• o é a altura do objeto;
• i é a altura da imagem. PROEXPLICA
A equação de Gauss ou função dos pontos conjugados tem
como finalidade a obtenção das abscissas da imagem ou do objeto COMO ESPELHO CONVEXO PODE CAIR NO ENEM?
e a distância focal, sabendo-se duas delas. A função dos pontos É comum percebermos em portas de lojas e em saídas
conjugados é dada por: de estacionamentos alguns espelhos convexos. Esses
espelhos têm como finalidade a ampliação do campo visual,
melhorando assim a área que é visível. A questão explora as
características da imagem formada por um espelho côncavo.
Porém, ao desenvolvermos a expressão, teremos:

EXERCÍCIO RESOLVIDO

01. (ENEM 2ª APLICAÇÃO) Os espelhos retrovisores, que


AUMENTO LINEAR (A) deveriam auxiliar os motoristas na hora de estacionar ou
O aumento linear transversal é uma grandeza definida pela mudar de pista, muitas vezes causam problemas. É que
razão entre o tamanho da imagem e o tamanho do objeto. Mas o espelho retrovisor do lado direito, em alguns modelos,
também pode ser obtido pela razão entre p’ e p. distorce a imagem, dando a impressão de que o veículo está
a uma distância maior do que a real.
Este tipo de espelho, chamado convexo, é utilizado com o
objetivo de ampliar o campo visual do motorista, já que no Brasil
Através do aumento linear, podemos classificar a imagem. se adota a direção do lado esquerdo e, assim, o espelho da
Caso o aumento seja positivo, a imagem será direita. Se o aumento direita fica muito mais distante dos olhos do condutor.
for negativo, a imagem será invertida. Disponível em: http://noticias.vrum.com.br. Acesso em: 3 nov. 2010 (adaptado).

Se:
Sabe-se que, em um espelho convexo, a imagem formada está
• | A | > 1 → imagem é maior que o objeto; mais próxima do espelho do que este está do objeto, o que
• | A | = 1 → imagem de mesmo tamanho que o objeto; parece estar em conflito com a informação apresentada na
reportagem. Essa aparente contradição é explicada pelo fato de
• | A | < 1 → imagem menor que o objeto.
a) a imagem projetada na retina do motorista ser menor do
que o objeto.
REFERENCIAL GAUSSIANO b) a velocidade do automóvel afetar a percepção da distância.
Para que se obtenha o valor correto das grandezas em questão, c) o cérebro humano interpretar como distante uma imagem
deve-se colocar corretamente os sinais de p, p’ e f. Uma maneira pequena.
prática de se guardar os sinais é lembrar que as grandezas atrás do
d) o espelho convexo ser capaz de aumentar o campo visual
espelho são negativas. Dessa forma, os objetos e imagens virtuais
do motorista.
terão suas abscissas negativas assim como os espelhos convexos
suas distâncias focais negativas. As grandezas diante do espelho e) o motorista perceber a luz vinda do espelho com a parte
terão sinais positivos. Os objetos e imagens reais e as distâncias lateral do olho.
focais dos espelhos côncavos serão considerados positivos. Na

PRÉ-VESTIBULAR PROENEM.COM.BR 289


FÍSICA II 31 ESPELHOS E LENTES: EQUAÇÃO DE GAUSS

Resolução:C Assinale a alternativa que indica CORRETAMENTE o instante


Nossos olhos estão acostumados com imagens em espelhos no qual a imagem do objeto se aproximou 5,0 cm do vértice
planos, onde imagens de objetos mais distantes nos parecem do espelho.
cada vez menores. a) 2,0 s c) 6,0 s e) 10,0 s
Esse condicionamento é levado para o espelho convexo: o b) 4,0 s d) 8,0 s
fato de a imagem ser menor que o objeto é interpretado pelo
cérebro como se o objeto estivesse mais distante do que Resolução: D
realmente está. Aplicando a Equação de Gauss, encontramos a posição inicial
Essa falsa impressão é desfeita quando o motorista está, por da imagem:
exemplo, dando marcha a ré em uma garagem, vendo apenas 1 1 1
a imagem dessa parede pelo espelho convexo. Ele para o carro = + onde:
f p' p
quando percebe pela imagem do espelho convexo que está
quase batendo na parede. Ao olhar para trás, por visão direta, F = distância focal equivalente à metade do raio;
ele percebe que não estava tão próximo assim da parede. p’ = distância da imagem;
p = distância do objeto.
1 1 1 1 1 1 2 1
= + ⇒ − = ⇒ = ∴ p' = 30 cm
02. (EEAR) Uma árvore de natal de 50 cm de altura foi 20 p' 60 20 60 p' 60 p'
colocada sobre o eixo principal de um espelho côncavo, a
uma distância de 25 cm de seu vértice. Sabendo-se que o Agora, aplicando novamente a Equação de Gauss, para uma
espelho possui um raio de curvatura de 25 cm, com relação a aproximação de 5 cm da imagem ao espelho, achamos a
imagem formada, pode-se afirmar corretamente que: nova posição ocupada pelo objeto:
a) É direita e maior do que o objeto, estando a 20 cm do 1 1 1 1 1 1 1 1 1
= + ⇒ = + ⇒ − = ∴ p = 100 cm
vértice do espelho. f p' p 20 25 p 20 25 p
b) É direita e maior do que o objeto, estando a 25 cm do
Finalmente com as posições inicial e final para a imagem mais
vértice do espelho.
a velocidade, usamos a equação do MRU para determinar o
c) É invertida e maior do que o objeto, estando a 25 cm do tempo gasto:
vértice do espelho.
s = s0 + v ⋅ t ⇒ t =
s − s0
⇒t=
(100 − 60 ) cm ∴ t = 8s
d) É invertida e do mesmo tamanho do objeto, estando a
v 5 cm s
25 cm do vértice do espelho.
e) É direita e do mesmo tamanho do objeto, estando a
20 cm do vértice do espelho

Resolução: D 04. (FUVEST) Câmeras digitais, como a esquematizada na


figura, possuem mecanismos automáticos de focalização.
Aplicando a equação de Gauss, vem:
1 1 1 1 1 1 2 1 1
= + ⇒ = + ⇒ − = ⇒ p' =25 cm
f p p' 25 25 p' 25 25 p'
2
Pela equação do aumento linear, obtemos:
i −p' i −25
= ⇒ = ⇒ i =−50 cm
o p 50 25
Portanto, a imagem é invertida, do mesmo tamanho do objeto Em uma câmera digital que utilize uma lente convergente
e está a 25 cm do espelho. com 20 mm de distância focal, a distância, em mm, entre a
lente e o sensor da câmera, quando um objeto a 2 m estiver
corretamente focalizado, é, aproximadamente,
03. (UNIOESTE) Considere um espelho esférico, côncavo e a) 1.
Gaussiano com raio de curvatura R = 40 cm. Um objeto se b) 5.
desloca ao longo do eixo principal que passa pelo vértice do c) 10.
espelho, se afastando do mesmo com velocidade constante de
d) 15.
5,0 cm/s. No instante t = 0 s, o objeto se encontra a 60 cm de
distância do vértice do espelho. e) 20.

Resolução: E
Dados: f = 20 mm; p = 2 m = 2.000 mm.
A distância entre a lente e o sensor da câmera é p’.
Da equação dos pontos conjugados:
1 1 1 pf 2.000 × 20 40.000 4.000
= − ⇒ p' = = = = = 20,02 mm ⇒ p' ≅ 20 mm.
p' f p p − f 2.000 − 20 1.980 198

Nota: os cálculos poderiam ser dispensados, pois a distância


do objeto à lente é muito maior que a distância focal (p>>>f)
Nesse caso, a imagem forma-se, praticamente, sobre o foco.

290 PROENEM.COM.BR PRÉ-VESTIBULAR


31 ESPELHOS E LENTES: EQUAÇÃO DE GAUSS FÍSICA II

04. Um objeto de comprimento 4 cm foi colocado à frente de uma


EXERCÍCIOS lente esférica delgada, conforme imagem abaixo:
PROTREINO
01. Um objeto de 10 cm de comprimento é colocado a 12 cm
do vértice de um espelho esférico gaussiano côncavo, conforme
imagem abaixo:

Determine o tamanho da imagem e a distância da imagem ao


centro óptico O.

05. Um objeto de 5 cm foi colocado à frente de uma lente delgada,


conforme imagem abaixo:

O raio de curvatura e o foco são representados pelos pontos R


e F, respectivamente. Calcule a distância da imagem ao vértice
do espelho p’, a distância da imagem ao objeto d e determine o
tamanho da imagem i.

02. Um objeto de 10 cm de comprimento é colocado a 12 cm


do vértice de um espelho esférico gaussiano convexo, conforme
imagem abaixo:

Calcule:
a) a distância p’ da imagem ao centro óptico;
b) a distância d entre a imagem e o objeto;
c) o tamanho da imagem i.

EXERCÍCIOS

O raio de curvatura e o foco são representados pelos pontos R PROPOSTOS


e F, respectivamente. Calcule a distância da imagem ao vértice
do espelho p’, a distância da imagem ao objeto d e determine o 01. (FAMERP) Um objeto luminoso encontra-se a 40 cm de uma
tamanho da imagem i. parede e a 20 cm de um espelho côncavo, que projeta na parede
uma imagem nítida do objeto, como mostra a figura.
03. A imagem abaixo representa o instante em que um objeto a
8 cm/s se aproxima de um espelho esférico gaussiano côncavo de
distância focal 16 cm passando pelo centro de curvatura.

Considerando que o espelho obedece às condições de nitidez de


Determine a distância da imagem ao vértice do espelho 1 segundo Gauss, a sua distância focal é
depois do instante mostrado na imagem acima e calcule a a) 15 cm. b) 20 cm. c) 30 cm. d) 25 cm. e) 35 cm.
velocidade média da imagem nesse intervalo de tempo.

PRÉ-VESTIBULAR PROENEM.COM.BR 291


FÍSICA II 31 ESPELHOS E LENTES: EQUAÇÃO DE GAUSS

02. (ULBRA) Um objeto está à frente de um espelho e tem sua 05. (UPE) Um objeto foi colocado sobre o eixo principal de um
imagem aumentada em quatro vezes e projetada em uma tela que espelho côncavo de raio de curvatura igual a 6,0 cm. A partir disso,
está a 2,4 m do objeto, na sua horizontal. Que tipo de espelho foi é possível observar que uma imagem real foi formada a 12,0 cm de
utilizado e qual o seu raio de curvatura? distância do vértice do espelho. Dessa forma, é CORRETO afirmar
a) Côncavo; 64 cm. que o objeto encontra-se a uma distância do vértice do espelho
igual a
b) Côncavo; 36 cm.
a) 2,0 cm
c) Côncavo; 128 cm.
b) 4,0 cm
d) Convexo; -128 cm.
c) 5,0 cm
e) Convexo; -64 cm.
d) 6,0 cm
03. (UPE-SSA 2) Uma barra delgada está em uma temperatura na e) 8,0 cm
qual o seu comprimento é igual L0 = 100 cm. A barra, de coeficiente
de dilatação linear 8,0 x 10-5 °C-1, é, então, colocada a uma distância 06. (IFSUL) Uma câmera com uma lente de 50 mm de distância
d = 0,8 m do vértice de um espelho curvo. O espelho possui um focal é utilizada para fotografar uma árvore de 25 m de altura. Se
raio de curvatura de 160 cm. Para se fazer a imagem crescer meio a imagem da árvore no filme tem 25 mm de altura, nas condições
centímetro, pode-se propostas acima, a distância entre a câmera e a árvore vale
a) 20,25 m.
b) 50,05 m.
c) 50,25 m.
d) 25,50 m.

07. (EEAR) Uma lente de vidro convergente imersa no ar tem


distância focal igual a 3 mm. Um objeto colocado a 3 m de distância
conjuga uma imagem através da lente. Neste caso, o módulo do
aumento produzido pela lente vale aproximadamente:
a) 1
b) 1·10-1
c) 1·10-2
d) 1·10-3
a) aproximar a barra em 15 cm.
08. (CEFET MG) Um boneco é colocado em frente a uma lente
b) afastar a barra em 10 cm.
delgada convergente, de distância focal igual a 2,0 m.
c) aquecer a barra em 40 °C.
d) esfriar a barra em 10 °C.
e) aquecer a barra em 125 °C.

04. (UNESP) No centro de uma placa de madeira, há um orifício no


qual está encaixada uma lente delgada convergente de distância
focal igual a 30 cm. Esta placa é colocada na vertical e um objeto
luminoso é colocado frontalmente à lente, à distância de 40 cm. No
lado oposto, um espelho plano, também vertical e paralelo à placa
de madeira, é disposto de modo a refletir a imagem nítida do objeto
sobre a placa de madeira. A figura ilustra a montagem.

Constantes físicas
Aceleração da gravidade: g = 10 m/s²
Velocidade da luz no vácuo: c = 3,00 x 108 m/s
Constante da lei de Coulomb: k0 = 9,0 x 109 N·m²/C²
A posição da imagem sobre o eixo ótico e o fator de ampliação da
imagem do boneco valem, respectivamente,
a) 2,0 m à direita da lente e -2.
b) 2,0 m à esquerda da lente e -1.
c) 4,0 m à direita da lente e -1.
d) 6,0 m à esquerda da lente e -1.
Nessa situação, o espelho plano se encontra em relação à placa de e) 6,0 m à direita da lente e -2.
madeira a uma distância de
a) 70 cm. c) 60 cm. e) 40 cm.
b) 10 cm. d) 30 cm.

292 PROENEM.COM.BR PRÉ-VESTIBULAR


31 ESPELHOS E LENTES: EQUAÇÃO DE GAUSS FÍSICA II

09. (UNESP) Para observar uma pequena folha em detalhes, um 11. (IFSUL) Um objeto real linear é colocado a 60 cm de um espelho
estudante utiliza uma lente esférica convergente funcionando esférico, perpendicularmente ao eixo principal. A altura da imagem
como lupa. Mantendo a lente na posição vertical e parada a 3 cm fornecida pelo espelho é 4 vezes maior que o objeto e é virtual. Com
da folha, ele vê uma imagem virtual ampliada 2,5 vezes. base nisso, é correto afirmar que esse espelho e a medida do seu
raio de curvatura são, respectivamente,
a) convexo e 160 cm.
b) côncavo e 80 cm.
c) convexo e 80 cm.
d) côncavo e 160 cm.

12. (ESPCEX - AMAN) Uma jovem, para fazer sua maquiagem,


comprou um espelho esférico de Gauss. Ela observou que, quando
o seu rosto está a 30 cm do espelho, a sua imagem é direita e três
vezes maior do que o tamanho do rosto.
O tipo de espelho comprado pela jovem e o seu raio de curvatura
são, respectivamente,
a) côncavo e maior do que 60 cm.
b) convexo e maior do que 60 cm.
Considerando válidas as condições de nitidez de Gauss, a distância c) côncavo e igual a 30 cm.
focal, em cm, da lente utilizada pelo estudante é igual a
d) côncavo e menor do que 30 cm.
a) 5.
e) convexo e menor do que 30 cm.
b) 2.
c) 6. 13. (EEAR) Uma árvore de natal de 50 cm de altura foi colocada
d) 4. sobre o eixo principal de um espelho côncavo, a uma distância de
25 cm de seu vértice. Sabendo-se que o espelho possui um raio
e) 3.
de curvatura de 25 cm, com relação a imagem formada, pode-se
afirmar corretamente que:
10. (UNESP) Em uma atividade de sensoriamento remoto,
para fotografar determinada região da superfície terrestre, foi a) É direita e maior do que o objeto, estando a 20 cm do vértice
utilizada uma câmera fotográfica constituída de uma única lente do espelho.
esférica convergente. Essa câmera foi fixada em um balão que b) É direita e maior do que o objeto, estando a 25 cm do vértice
se posicionou, em repouso, verticalmente sobre a região a ser do espelho.
fotografada, a uma altura h da superfície. c) É invertida e maior do que o objeto, estando a 25 cm do vértice
do espelho.
d) É invertida e do mesmo tamanho do objeto, estando a 25 cm
do vértice do espelho.

14. (EEAR) Um objeto é colocado perpendicularmente ao eixo


principal e a 20 cm de uma lente divergente estigmática de
distância focal igual a 5 cm. A imagem obtida é virtual, direita e
apresenta 2 cm de altura. Quando essa lente é substituída por
outra convergente estigmática de distância focal igual a 4 cm e
colocada exatamente na mesma posição da anterior, e mantendo-
se o objeto a 20 cm da lente, a imagem agora apresenta uma altura
de _____ cm.
a) 2,5 c) 5,0
b) 4,0 d) 10,0

15. (FUVEST) Câmeras digitais, como a esquematizada na figura,


possuem mecanismos automáticos de focalização.

Considerando que, nessa atividade, as dimensões das imagens


nas fotografias deveriam ser 5.000 vezes menores do que as
dimensões reais na superfície da Terra e sabendo que as imagens
dos objetos fotografados se formaram a 20 cm da lente da câmera,
a altura h em que o balão se posicionou foi de
a) 1.000 m. Em uma câmera digital que utilize uma lente convergente com
20 mm de distância focal, a distância, em mm, entre a lente e o
b) 5.000 m.
sensor da câmera, quando um objeto a 2 m estiver corretamente
c) 2.000 m. focalizado, é, aproximadamente,
d) 3.000 m. a) 1. c) 10. e) 20.
e) 4.000 m. b) 5. d) 15.

PRÉ-VESTIBULAR PROENEM.COM.BR 293


FÍSICA II 31 ESPELHOS E LENTES: EQUAÇÃO DE GAUSS

16. (MACKENZIE) Dispõe-se de um espelho convexo de Gauss, Em seguida, o professor propõe um exercício com a figura a seguir,
de raio de curvatura R. Um pequeno objeto colocado diante desse que resume o experimento realizado. Nessa figura, a lâmpada
espelho, sobre seu eixo principal, a uma distância R de seu vértice acesa da lanterna é representada pela seta O, a seta I1 representa
V, terá uma imagem conjugada situada no ponto P desse eixo. O a imagem dessa lâmpada formada pela lente L, e I2, representa a
comprimento do segmento VP é imagem da seta I1 formada pelo espelho E.
a) R/4 c) R/2 e) 2R
b) R/3 d) R

17. (UNICAMP) Um objeto é disposto em frente a uma lente


convergente, conforme a figura abaixo. Os focos principais da
lente são indicados com a letra F. Pode-se afirmar que a imagem
formada pela lente

Considerando válidas as condições de nitidez de Gauss, a distância


d, entre O e I2, é
a) 1,6 m.
a) é real, invertida e mede 4 cm. b) 1,4 m.
b) é virtual, direta e fica a 6 cm da lente. c) 1,5 m.
c) é real, direta e mede 2 cm. d) 1,3 m.
d) é real, invertida e fica a 3 cm da lente. e) 1,8 m.

18. (MACKENZIE) Dispõe-se de dois espelhos esféricos, um 20. (UDESC) Um objeto é colocado a 4,0 cm à esquerda de uma
convexo e um côncavo, com raios de curvatura 20,0 cm cada um, lente convergente de distância focal de 2,0 cm. Um espelho
e que obedecem às condições de Gauss. Quando um objeto real é convexo de raio de curvatura de 4,0 cm está 10,0 cm à direita da
colocado perpendicularmente ao eixo principal do espelho convexo, lente convergente, como mostra a figura abaixo.
a 6,0 cm de seu vértice, obtém-se uma imagem conjugada de
1,5 cm de altura. Para que seja obtida uma imagem conjugada,
também de 1,5 cm de altura, colocando esse objeto
perpendicularmente ao eixo principal do espelho côncavo, sua
distância até o vértice desse espelho deverá ser
a) 11,0 cm c) 26,0 cm e) 52,0 cm
b) 15,0 cm d) 30,0 cm
Assinale a alternativa que corresponde à posição da imagem final,
19. (FAC. ALBERT EINSTEIN - MED) Em um laboratório didático, com relação ao vértice V do espelho.
foi montado um banco óptico formado por uma lente esférica
a) 1,5 cm
convergente L de distância focal igual a 20 cm, um espelho plano
E e uma lanterna acesa, funcionando como o objeto O. A fotografia b) -1,5 cm
representa esse sistema com as distâncias entre seus elementos, c) -1,3 cm
fora de escala.
d) 1,3 cm
e) 3,0 cm

EXERCÍCIOS DE
APROFUNDAMENTO
01. (UNESP) Uma placa retangular de espessura desprezível e de
vértices PQRS é posicionada, em repouso, sobre o eixo principal
de um espelho esférico gaussiano de vértice V, foco principal F e
centro de curvatura C, de modo que a posição do vértice R da placa
coincida com a posição do ponto C, conforme figura. O raio de
curvatura desse espelho mede 160 cm e o comprimento da placa
é 40 cm.

294 PROENEM.COM.BR PRÉ-VESTIBULAR


31 ESPELHOS E LENTES: EQUAÇÃO DE GAUSS FÍSICA II

03. (UERJ) Em função de suas características, uma lente


convergente, ao ser exposta à luz do Sol, gera uma concentração
de luz a 60 cm do seu centro óptico, como ilustra a imagem.

Considere que um objeto é colocado a 180 cm do centro óptico


dessa lente para que sua imagem seja projetada com nitidez sobre
uma tela.
a) Na figura apresentada a seguir, construa, traçando raios de
Calcule a distância, em centímetros, em que a tela deve ser
luz, a imagem P’S’ do lado PS dessa placa. Identifique, nessa
colocada, a partir do centro óptico da lente, para obtenção dessa
figura, os pontos P’ e S’ e classifique essa imagem como real
imagem.
ou virtual, justificando sua resposta.
04. (UERJ) Em uma aula prática de óptica, um espelho esférico
côncavo é utilizado para obter a imagem de um prédio. Considere
as seguintes medidas:
- altura do prédio = 20 m;
- distância do prédio ao espelho = 100 m;
- distância focal do espelho = 20 cm;
Admitindo que a imagem conjugada se situa no plano focal do
espelho, calcule, em centímetros, a altura dessa imagem.

05. (UERJ) Um lápis com altura de 20 cm é colocado na posição


vertical a 50 cm do vértice de um espelho côncavo. A imagem
conjugada pelo espelho é real e mede 5 cm. Calcule a distância, em
centímetros, da imagem ao espelho.
b) Calcule, em cm, a distância entre a imagem P’S’, do lado PS, e
a imagem Q’R’, do lado QR.
GABARITO
02. (UNIFESP) Um objeto linear é colocado verticalmente em EXERCÍCIOS PROPOSTOS
repouso sobre o eixo principal de um espelho esférico côncavo E de 01. A 05. B 09. A 13. D 17. A
centro de curvatura C, foco principal F e vértice V. Nessa situação, 02. C 06. B 10. A 14. A 18. C
esse espelho forma uma imagem real, invertida e quatro vezes 03. E 07. D 11. D 15. E 19. D
maior do que o objeto, como representado na figura. 04. C 08. E 12. A 16. B 20.B

EXERCÍCIOS DE APROFUNDAMENTO
01. a)

Portanto, a imagem é real, pois é formada pela convergência dos raios refletidos em frente
ao espelho.
b) 80 cm
Em seguida, o objeto é movimentado horizontalmente, com 02. a) 4
velocidade escalar média VOB até o ponto C, onde é novamente b) 16 cm
mantido em repouso; simultaneamente, sua imagem movimentou- 03. 90 cm
se com velocidade escalar média VM até sua nova posição final.
04. p' = f
Calcule: i f f ⋅o 20 ⋅ 20
= ⇒i= ⇒i= ⇒ i = 4,0 cm
V o p p 100
a) o valor absoluto da razão M . 05. 12,5 cm
VOB
b) a distância focal, em cm, desse espelho.

PRÉ-VESTIBULAR PROENEM.COM.BR 295


FÍSICA II 31 ESPELHOS E LENTES: EQUAÇÃO DE GAUSS

ANOTAÇÕES

296 PROENEM.COM.BR PRÉ-VESTIBULAR


32 INSTRUMENTOS ÓPTICOS FÍSICA II

E ÓPTICA DA VISÃO

O funcionamento de vários instrumentos que utilizamos no dia Se na objetiva, ao invés de utilizarmos uma lente, usarmos um
a dia pode ser justificado pelas associações de diversos tipos de espelho côncavo, teremos um telescópio.
lentes, prismas e espelhos. Veja o esquema de um telescópio abaixo:
Esses inúmeros aparelhos, entre eles máquinas fotográficas,
Objeto no
projetores e lunetas, podem ser divididos em dois grupos de infinito
instrumentos: de projeção e de observação.

}
Os instrumentos de projeção se caracterizam por conjugarem
uma imagem real, ou seja, uma imagem projetada em um anteparo, E’
i1
filme etc. Como exemplo, temos as máquinas fotográficas e os L
i3
projetores.
i2
Aqueles aparelhos em que a imagem pode ser observada O
diretamente dentro do aparelho, isto é, uma imagem virtual,
são chamados de instrumentos de observação. É o caso do
microscópio, da luneta e do telescópio.
E
Veremos agora alguns instrumentos:

LUPA OU LENTE DE AUMENTO A vantagem é que o espelho apresenta menos aberrações


ópticas que as lentes.
O mais simples instrumento óptico. Consiste em uma lente
convergente com o objeto colocado entre o foco e o centro óptico.
MICROSCÓPIO COMPOSTO
Consiste na associação de duas lentes: a objetiva e a ocular.
Seu funcionamento é bem parecido com o da luneta, porém, o
objeto é real.

A ampliação total do microscópio pode ser calculada pelo


LUNETA produto do poder de ampliação de cada lente.
Existem dois tipos de luneta: a luneta terrestre (menor em A = AOB . AOC
tamanho e em alcance) e a luneta astronômica (mais potente), que
é utilizada na observação de astros e, dessa forma, seus objetos
são normalmente impróprios. PROJETOR
Na luneta existem duas lentes. A primeira é denominada O projetor de slides é um instrumento de projeção e precisa
objetiva e a segunda, ocular, sendo ambas lentes convergentes. A de um anteparo, em geral uma tela, para que a imagem possa ser
imagem formada pela objetiva serve como objeto para a ocular. vista. Além da imagem ser real, ela é maior e invertida.

PRÉ-VESTIBULAR PROENEM.COM.BR 297


FÍSICA II 32 INSTRUMENTOS ÓPTICOS E ÓPTICA DA VISÃO

MÁQUINA FOTOGRÁFICA DEFEITOS VISUAIS


A máquina fotográfica é composta de uma lente objetiva que Existem alguns defeitos visuais que são gerados por mal
forma uma imagem no chamado filme fotográfico. A imagem será funcionamento de algum órgão.
invertida e real.

MIOPIA
O míope tem problemas para enxergar objetos distantes. Isto
se deve a um alongamento do globo ocular (como mostra a figura)
ou a um alto poder de convergência do cristalino. Independente
da razão da miopia da pessoa, constata-se que ela apresenta um
problema em seu ponto remoto.

Para que a imagem possa se projetar sempre no filme


fotográfico, a máquina fotográfica possui um dispositivo para
regular a distância da lente ao filme. Existe também um dispositivo,
chamado de diafragma, que regula a entrada de luz.

O OLHO HUMANO No olho míope, a imagem se forma antes da retina e, para que
O globo ocular humano tem a forma aproximada de uma a imagem se conjugue normalmente, devemos utilizar uma lente
esfera, e é constituído por inúmeros elementos, como mostra a divergente.
figura ilustrativa:

HIPERMETROPIA
Um olho hipermétrope tem o problema oposto ao do míope, isto
é, dificuldade para enxergar de perto, devido a um encurtamento do
‘ globo ou a um fraco poder de convergência do cristalino. O ponto
próximo do hipermétrope se encontra a mais de 25 cm do globo.
A parte mais exterior é conhecida como córnea, que tem
como função proteger o cristalino. A íris é a parte que representa
a coloração característica dos olhos e a pupila é o orifício central
(parte escura), responsável pelo controle da luminosidade que
penetra no globo ocular. Atrás da pupila há o cristalino, que funciona
como lente convergente gelatinosa para o olho humano. No fundo
do olho fica a retina, que é onde a imagem é formada quando não há
defeito visual. A retina possui um conjunto de nervos, denominado
nervo óptico, que leva a mensagem de formação de imagens para
o cérebro.
A pupila é responsável pelo fenômeno da adaptação visual, A imagem conjugada pelo cristalino se encontra após a retina
liberando a intensidade luminosa necessária. Assim, quando o e, dessa forma, devemos corrigir este defeito com uma lente
observador se encontra em um lugar pouco iluminado, a pupila convergente.
se abre mais e permite que uma intensidade maior penetre. Da
mesma forma, quando se encontra em lugares exageradamente
iluminados, a pupila se fecha, diminuindo a entrada de luz.
A imagem conjugada pelo cristalino sempre estará na retina (no
nervo óptico), ou seja, p’ terá sempre o mesmo valor. Como há uma
variação de p, pois os objetos se encontram a distâncias diferentes
do olho, ocorrerá uma variação da distância focal provocada pelos
músculos ciliares. Esse processo denomina-se acomodação visual.
Existe um intervalo de acomodação em que o olho humano é capaz
de captar uma imagem; e vai do ponto remoto, onde não há esforço
de acomodação e a distância focal é máxima; até o ponto próximo, ASTIGMATISMO
onde o esforço é máximo e a distância focal é mínima. Para o olho
humano normal, sem nenhuma anomalia, o ponto remoto está no Consiste em uma deformação na córnea, a pessoa que possui
infinito e o ponto próximo a 25 cm de distância. Astigmatismo observa os objetos embaçados. A correção do
Astigmatismo se dá através de lentes cilíndricas.

298 PROENEM.COM.BR PRÉ-VESTIBULAR


32 INSTRUMENTOS ÓPTICOS E ÓPTICA DA VISÃO FÍSICA II

Míope  Correção: Lente divergente (f < 0)


Vergência (V): normalmente representada pela letra V,
nessa questão será representada pela letra “D” conforme
enunciado.

1
D=
f

Ponto remoto de um olho emetrope (sem defeito visual):



Ponto remoto do olho míope do enunciado: 0,4 m.
PRESBIOPIA A lente divergente deve produzir uma imagem que se
Também conhecida como vista cansada, é resultado de forma no ponto remoto do olho míope, portanto, p’ = - 0,4
um enrijecimento dos músculos ciliares, responsáveis pela m (o sinal negativo indica que a imagem é virtual) de um
acomodação visual. Geralmente ocorre a partir dos 40 anos de objeto que se encontra no infinito, portanto, p = ∞ .
idade e para corrigir usamos as lentes convergentes. 1 1 1
= +
f p p′
PROEXPLICA
1 1 1
= +
f ∞ −0,4
COMO LUNETA PODE CAIR NO ENEM?
A luneta astronômica foi um instrumento que trouxe para 1 1
=
os homens a capacidade de observar corpos celestes com f −0,4
uma riqueza de detalhes que não é percebida ao olho nu. As
lunetas são construídas com duas lentes convergentes que f = −0,4 m (a distância focal da lente divergente é de 0,4
formam uma imagem mais próxima do observador. A questão metros)
busca identificar a distância entre os olhos do observador e a 1
imagem formada. D=
f

1
D=
−0,4
EXERCÍCIO RESOLVIDO
D = −2,5 di
01. (UNICAMP - MODIFICADA) Nos olhos das pessoas
míopes, um objeto localizado muito longe, isto é, no infinito, 1
c) Nesse caso = −  4
é focalizado antes da retina. À medida que o objeto se f
1 1 1
aproxima, o ponto de focalização se afasta até cair sobre a = +
retina. A partir deste ponto, o míope enxerga bem. A dioptria D, f p p′
ou "grau", de uma lente é definida como D = 1/(distância focal)
1 1
e 1 grau = 1 m-1. Considere uma pessoa míope que só enxerga −4 = +
bem objetos mais próximos do que 0,4 m de seus olhos. ∞ p'
a) Faça um esquema mostrando como uma lente bem 1
próxima dos olhos pode fazer com que um objeto no −4 =
p′
infinito pareça estar a 40 cm do olho.
b) Qual a dioptria dessa lente? p′ = −0,25 m
c) A partir de que distância uma pessoa míope que usa p' = 25 cm (isso significa que a pessoa com esse grau
óculos de "4 graus" pode enxergar bem sem os óculos? de miopia consegue enxergar com nitidez objetos até
25 cm de distância.)
Resolução:
a) Observe a figura a seguir:

02. (UNESP) Uma pessoa apresenta deficiência visual,


conseguindo ler somente se o livro estiver a uma distância de
75 cm. Qual deve ser a distância focal dos óculos apropriados
para que ela consiga ler, com o livro colocado a 25 cm de
distância?

Resolução:
Hipermetropia  Correção: Lente convergente (f > 0)
Ponto próximo de pessoa emetrope (sem defeito visual): 25
1 cm
b) Deve-se lembrar que se x = ∞ , então, ≈ 0 , ou seja, a
x
Ponto próximo do hipermetrope do enunciado: 75 cm
divisão de um número por ∞ tende a zero.

PRÉ-VESTIBULAR PROENEM.COM.BR 299


FÍSICA II 32 INSTRUMENTOS ÓPTICOS E ÓPTICA DA VISÃO

A lente convergente deve fazer a imagem de um objeto a Dados:


25 cm de distância (p = 25 cm) se formar a uma distância de
i −P' 1 1 1 C
75 cm (p’ = - 75 cm; o sinal negativo indica que a imagem é A= = ; = + ; n = ; n1 ⋅ sen θ1 =n2 ⋅ sen θ2
O P f P P' v
virtual)
a) Com base no esquema do olho abaixo, desenhe a imagem
do objeto (seta) formada em um olho hipermetrope.

b) Que tipo de lente esférica corrige o defeito da hipermetropia


e que fenômeno óptico explica o funcionamento de uma
lente esférica?
Imagem ilustrativa – Fora de escala
c) Considere uma pessoa hipermetrope capaz de enxergar
1 1 1 nitidamente quando seu ponto próximo é de 1,0 m. Nesse
= + caso, qual a vergência da lente corretiva para conjugar a
f p p′
imagem de um objeto no ponto próximo, se esse objeto
1 1 1 estiver a 25 cm do olho?
= +
f 25 −75 Resolução:
1 3 −1 a) Abaixo a construção da imagem para um olho
= hipermetrope.
f 75

1 2
=
f 75

75
f=
2

f = 37,5 cm
b) Para solucionar o problema do olho hipermetrope
Caso a questão perguntasse quantos “graus” a lente deve ter devemos usar uma lente convergente. O fenômeno
basta calcular a vergência (V) óptico que explica o funcionamento das lentes esféricas
é a refração.
=f 37,5 
= cm 0,375 m
c) A distância focal (f) corretiva par o olho hipermetrope é
1 dada pela equação de Gauss, onde a distância da imagem
V=
f (p’) deve ser de 25 cm (ponto próximo do olho normal) e
a distância do objeto (p) deve ser de -1 m (ponto próximo
1 do hipermetrope). A vergência (V) da lente corretiva é o
=V ≅ 2,67 di
0,375 inverso da distância focal.
Ou seja, próximo de 2,7 graus. 1 1 1 1 1 1 1
= + ⇒ = − ∴ = V = 3 di
f P P' f 0,25 m 1m f
1
=f = m 33,3 cm
3
03. (UFSC) Entre os cinco sentidos humanos, a visão é
um dos mais importantes, por isso deve-se cuidar muito
bem dos olhos. Ainda assim, defeitos visuais como miopia,
hipermetropia e astigmatismo aparecem no decorrer da
vida. Mas nada está perdido, pois os óculos são alternativas EXERCÍCIOS
acessíveis e satisfatórias na melhoria da qualidade visual
dos indivíduos. Considere o esquema do olho abaixo para PROTREINO
responder aos itens da questão.
01. Ao examinar uma moeda antiga com o auxílio de uma lupa
de distância focal igual a 12 cm, um colecionador posiciona a
lupa de forma que a moeda fica com as dimensões lineares
ampliadas três vezes na imagem. Calcule a distância entre a
lupa e a moeda nessa situação.

300 PROENEM.COM.BR PRÉ-VESTIBULAR


32 INSTRUMENTOS ÓPTICOS E ÓPTICA DA VISÃO FÍSICA II

02. Complete a imagem com o nome da estrutura apontada.

03. Ao olhar para um objeto, a imagem deste deve se formar sobre


a retina, do contrário, a pessoa não vai enxergar com nitidez. Nas
fotos observamos, à esquerda, um rapaz sem óculos, e à direita, Se fizermos uma analogia entre a ilustração e o olho humano, o
com uns óculos para corrigir sua ametropia e enxergar com nitidez. tecido corresponde ao(à)
a) íris d) córnea
b) retina e) cristalino
c) pupila

02. (UFRGS) Muitas pessoas não enxergam nitidamente objetos


em decorrência de deformação no globo ocular ou de acomodação
defeituosa do cristalino.
Assinale a alternativa que preenche corretamente as lacunas dos
enunciados a seguir, na ordem em que aparecem.
Para algumas pessoas a imagem de um objeto forma-se à frente
da retina, conforme ilustrado na figura I abaixo. Esse defeito de
Identifique se a lente usada para corrigir a ametropia é convergente visão é chamado de __________, e sua correção é feita com lentes
ou divergente, justifique sua resposta. __________.

04. Num olho emetrope, ou seja, um olho livre de ametropias, o


ponto remoto localiza-se no infinito e a distância entre o cristalino
e a retina é de aproximadamente 2 cm. Determine o tipo de lente e o
“grau” adequado para um olho míope cujo ponto remoto encontra-
se a 2 metros.

05. Em um olho emetrope o ponto próximo situa-se a 25 cm, ou


seja, 25 cm é a distância mínima para um olho saudável enxergar
com nitidez. Determine o tipo de lente e o “grau” adequado para um
olho hipermetrope, cujo ponto próximo situa-se a 50 cm, enxergar Em outras pessoas, os raios luminosos são interceptados pela
com nitidez. retina antes de se formar a imagem, conforme representado na
figura II abaixo. Esse defeito de visão é chamado de __________, e
sua correção é feita com lentes __________.

EXERCÍCIOS
PROPOSTOS
01. (ENEM) Entre os anos de 1028 e 1038, Alhazen (lbn al-Haytham:
965-1040 d.C.) escreveu sua principal obra, o Livro da Óptica, que,
com base em experimentos, explicava o funcionamento da visão e
outros aspectos da ótica, por exemplo, o funcionamento da câmara
escura. O livro foi traduzido e incorporado aos conhecimentos
científicos ocidentais pelos europeus. Na figura, retirada dessa
obra, é representada a imagem invertida de edificações em tecido a) presbiopia − divergentes − hipermetropia – convergentes
utilizado como anteparo. b) presbiopia − divergentes − miopia − convergentes
c) hipermetropia − convergentes − presbiopia − divergentes
d) miopia − convergentes − hipermetropia − divergentes
e) miopia − divergentes − hipermetropia − convergentes

PRÉ-VESTIBULAR PROENEM.COM.BR 301


FÍSICA II 32 INSTRUMENTOS ÓPTICOS E ÓPTICA DA VISÃO

03. (UNICAMP) As cirurgias corretivas a laser para a visão estão cada 08. (CN) A visão é um dos principais sentidos usados pelos seres
vez mais eficientes. A técnica corretiva mais moderna é baseada humanos para perceber o mundo e a figura abaixo representa
na extração de um pequeno filamento da córnea, modificando a de forma muito simplificada o olho humano, que é o veículo
sua curvatura. No caso de uma cirurgia para correção de miopia, encarregado de levar essas percepções até o cérebro.
o procedimento é feito para deixar a córnea mais plana. Assinale
a alternativa que explica corretamente o processo de correção da
miopia.
a) Na miopia, a imagem do ponto remoto se forma antes da retina e
a cirurgia visa a aumentar a distância focal da lente efetiva do olho.
b) Na miopia, a imagem do ponto remoto se forma depois da
retina e a cirurgia visa a aumentar a distância focal da lente
efetiva do olho.
Sendo assim, com base na figura acima, é correto afirmar que o
c) Na miopia, a imagem do ponto remoto se forma depois da
olho é
retina e a cirurgia visa a diminuir a distância focal da lente
efetiva do olho. a) míope e a correção é feita com lente convergente.
d) Na miopia, a imagem do ponto remoto se forma antes da retina e b) míope e a correção é feita com lente divergente.
a cirurgia visa a diminuir a distância focal da lente efetiva do olho. c) hipermetrope e a correção é feita com lente convergente.
d) hipermetrope e a correção é feita com lente divergente.
04. (UNISC) Uma pessoa não consegue ver os objetos com nitidez
e) normal e, nesse caso, não precisa de correção.
porque suas imagens se formam entre o cristalino e a retina. Qual é
o defeito de visão desta pessoa e como podemos corrigi-lo?
09. (IFSUL) A grandeza física vergência é medida em dioptrias, o
a) Hipermetropia e a pessoa deverá usar lentes divergentes para que, no cotidiano, é o “grau” de uma lente. Logo, uma pessoa que
a sua correção. usa um óculo com lente para a correção de sua visão de 2,5 graus,
b) Miopia e a pessoa deverá usar lentes divergentes para a sua está usando um óculo com uma lente de vergência igual a 2,5
correção. dioptrias.
c) Miopia e a pessoa deverá usar lentes convergentes para a sua Essa lente tem uma distância focal de
correção. a) 0,30 m. c) 2,50 m.
d) Hipermetropia e a pessoa deverá usar lentes convergentes b) 0,40 m. d) 0,25 m.
para a sua correção.
e) Miopia e a pessoa deverá usar uma lente divergente e outra 10. (UERN) Numa família composta por 4 pessoas, cada uma com
lente convergente para a sua correção. um defeito na visão diferente dos demais, tem-se que:
• o pai apresenta enrijecimento dos músculos ciliares, e com
05. (IFSUL) A receita de óculos para um míope indica que ele deve limitação de sua capacidade de acomodação visual tem
usar lentes de 2,0 graus, isto é, o valor da vergência das lentes dificuldades para enxergar objetos próximos e longínquos;
deve ser 2,0 dioptrias. Com base nos dados fornecidos na receita,
conclui-se que as lentes desses óculos devem ser • a mãe apresenta um alongamento do globo ocular na direção
ântero-posterior com dificuldade para enxergar objetos
a) convergentes, com 2,0 m de distância focal. distantes;
b) convergentes, com 50 cm de distância focal. • a filha apresenta irregularidades na curvatura da córnea
c) divergentes, com 2,0 m de distância focal. e enxerga imagens embaçadas dos objetos próximos ou
d) divergentes, com 50 cm de distância focal. distantes;
• o filho apresenta um encurtamento do globo ocular na direção
06. (ESPCEX - AMAN) Um estudante foi ao oftalmologista, ântero-posterior com dificuldade para enxergar objetos
reclamando que, de perto, não enxergava bem. Depois de realizar próximos.
o exame, o médico explicou que tal fato acontecia porque o ponto As lentes corretivas indicadas para os membros dessa
próximo da vista do rapaz estava a uma distância superior a 25 cm família, considerando-se a ordem em que foram citados, são,
e que ele, para corrigir o problema, deveria usar óculos com “lentes respectivamente,
de 2,0 graus“, isto é, lentes possuindo vergência de 2,0 dioptrias.
a) cilíndricas, bifocais, convergentes e divergentes.
Do exposto acima, pode-se concluir que o estudante deve usar lentes
b) divergentes, bifocais, convergentes e cilíndricas.
a) divergentes com 40 cm de distância focal.
c) bifocais, divergentes, cilíndricas e convergentes.
b) divergentes com 50 cm de distância focal.
d) convergentes, cilíndricas, divergentes e bifocais.
c) divergentes com 25 cm de distância focal.
d) convergentes com 50 cm de distância focal. 11. (ENEM) A maioria das pessoas fica com a visão embaçada
e) convergentes com 25 cm de distância focal. ao abrir os olhos debaixo d’água. Mas há uma exceção: o povo
moken, que habita a costa da Tailândia. Essa característica se
deve principalmente à adaptabilidade do olho e à plasticidade do
07. (FEEVALE) No processo de visão humana, o cristalino
cérebro, o que significa que você também, com algum treinamento,
desempenha um papel importante na formação da imagem.
poderia enxergar relativamente bem debaixo d’água. Estudos
Marque a alternativa correta sobre essa estrutura do olho humano.
mostraram que as pupilas de olhos de indivíduos moken sofrem
a) Controla a quantidade de luz que entra no olho humano. redução significativa debaixo d’água, o que faz com que os raios
b) Controla a energia dos fótons da luz incidente. luminosos incidam quase paralelamente ao eixo óptico da pupila.
c) Atua como lente divergente para acomodar a imagem. GISLÉN, A. et al. Visual Training Improves Underwater Vision in Children.
Vision Research, n. 46, 2006 (adaptado).
d) Atua como lente convergente para acomodar a imagem.
e) Define as cores dos objetos.

302 PROENEM.COM.BR PRÉ-VESTIBULAR


32 INSTRUMENTOS ÓPTICOS E ÓPTICA DA VISÃO FÍSICA II

A acuidade visual associada à redução das pupilas é fisicamente De acordo com o texto, a miopia causada por essa doença deve-
explicada pela diminuição se ao fato de, ao tornar-se mais intumescido, o cristalino ter sua
a) da intensidade luminosa incidente na retina. distância focal
b) da difração dos feixes luminosos que atravessam a pupila. a) aumentada e tornar-se mais divergente.
c) da intensidade dos feixes luminosos em uma direção por b) reduzida e tornar-se mais divergente.
polarização. c) aumentada e tornar-se mais convergente.
d) do desvio dos feixes luminosos refratados no interior do olho. d) aumentada e tornar-se mais refringente.
e) das reflexões dos feixes luminosos no interior do olho. e) reduzida e tornar-se mais convergente.

12. (ACAFE) Alguns instrumentos óticos são formados por lentes. 15. (ENEM PPL) O avanço tecnológico da medicina propicia o
O instrumento ótico formado por lentes objetiva e ocular é: desenvolvimento de tratamento para diversas doenças, como
a) a lupa. as relacionadas à visão. As correções que utilizam laser para o
tratamento da miopia são consideradas seguras até 12 dioptrias,
b) o microscópio. dependendo da espessura e curvatura da córnea. Para valores
c) o retroprojetor. de dioptria superiores a esse, o implante de lentes intraoculares é
d) o periscópio. mais indicado. Essas lentes, conhecidas como lentes fácicas (LF),
são implantadas junto à córnea, antecedendo o cristalino (C), sem
13. (ENEM PPL) A aquisição de um telescópio deve levar em que esse precise ser removido, formando a imagem correta sobre
consideração diversos fatores, entre os quais estão o aumento a retina (R).
angular, a resolução ou o poder de separação e a magnitude limite. O comportamento de um feixe de luz incidindo no olho que possui
O aumento angular informa quantas vezes mais próximo de nós um implante de lentes fácicas para correção do problema de visão
percebemos o objeto observado e é calculado como sendo a apresentado é esquematizado por
razão entre as distâncias focais da objetiva (F1) e da ocular (F2). A a) d)
resolução do telescópio (P) informa o menor ângulo que deve existir
entre dois pontos observados para que seja possível distingui-los. A
magnitude limite (M) indica o menor brilho que um telescópio pode
12
captar. Os valores numéricos de P e M pelas expressões: P =
D
e M = 7,1 + 5(log D), em que D é o valor numérico do diâmetro da
objetiva do telescópio, expresso em centímetro. b) e)
Disponível em: www.telescopiosastronomicos.com.br.
Acesso em: 13 maio 2013 (adaptado).

Ao realizar a observação de um planeta distante e de luminosidade,


não se obteve uma imagem nítida. Para melhorar a qualidade dessa
observação, os valores de D, F1 e F2 devem ser, respectivamente,
a) aumentado, aumentado e diminuído.
c)
b) aumentado, diminuído e aumentado.
c) aumentado, diminuído e diminuído.
d) diminuído, aumentado e aumentado.
e) diminuído, aumentado e diminuído.

14. (UNESP) Dentre as complicações que um portador de diabetes


não controlado pode apresentar está a catarata, ou seja, a perda 16. (ACAFE) A figura abaixo mostra esquematicamente o olho
da transparência do cristalino, a lente do olho. Em situações de humano, enfatizando nos casos I e II os dois defeitos de visão mais
hiperglicemia, o cristalino absorve água, fica intumescido e tem comuns.
seu raio de curvatura diminuído (figura 1), o que provoca miopia
no paciente. À medida que a taxa de açúcar no sangue retorna aos
níveis normais, o cristalino perde parte do excesso de água e volta
ao tamanho original (figura 2). A repetição dessa situação altera
as fibras da estrutura do cristalino, provocando sua opacificação.
(www.revistavigor.com.br. Adaptado.)

PRÉ-VESTIBULAR PROENEM.COM.BR 303


FÍSICA II 32 INSTRUMENTOS ÓPTICOS E ÓPTICA DA VISÃO

Nessa situação, assinale a alternativa correta que completa, em


sequência, as lacunas da frase a seguir. EXERCÍCIOS DE
No caso I trata-se da ___________, que pode ser corrigida com uma APROFUNDAMENTO
lente __________; já no caso II trata-se de ____________, que pode ser
corrigida com uma lente ___________. 01. (UERJ) A altura da imagem de um objeto, posicionado a uma
a) hipermetropía – convergente – miopía – divergente distância P1 do orifício de uma câmara escura, corresponde a 5%
b) hipermetropía – divergente – miopía – convergente da altura desse objeto. A altura da imagem desse mesmo objeto,
posicionado a uma distância P2 do orifício da câmara escura,
c) miopía – divergente – hipermetropía – convergente
corresponde a 50% de sua altura.
d) miopía – convergente – hipermetropía – divergente
Calcule P2 em função de P1.

17. (ENEM PPL) Indivíduos míopes têm dificuldade de enxergar


02. (UFSC) Entre os cinco sentidos humanos, a visão é um dos mais
objetos distantes. Para correção desse problema com lentes, o
importantes, por isso deve-se cuidar muito bem dos olhos. Ainda
oftalmologista deve medir a distância máxima que o indivíduo pode
assim, defeitos visuais como miopia, hipermetropia e astigmatismo
enxergar nitidamente, que corresponde à distância focal da lente.
aparecem no decorrer da vida. Mas nada está perdido, pois os
A vergência (V) de uma lente é numericamente igual ao inverso da
óculos são alternativas acessíveis e satisfatórias na melhoria da
distância focal (f), dada em metros (V = 1/f). A vergência é medida
qualidade visual dos indivíduos. Considere o esquema do olho
em dioptria (di), comumente denominada de graus de uma lente.
abaixo para responder aos itens da questão.
Se a distância máxima a que o indivíduo míope enxerga nitidamente
for 50 cm, para corrigir o problema, o oftalmologista receitará
lentes de vergência
a) -2,00 di. d) 0,20 di.
b) -0,02 di. e) 2,00 di.
c) 0,02 di.

18. (UECE) Uma estudante constrói uma luneta usando uma lente
convergente de 58,2 cm de distância focal como objetiva e uma
lente convergente com 1,9 cm de distância focal como ocular.
Sabendo-se que a distância entre as lentes ocular e objetiva é de
60 cm, qual é, aproximadamente, a distância, em centímetros, entre Dados:
a imagem final de um astro observado e a ocular?
i −P' 1 1 1 C
a) 10,0 c) 34,2 A= = ; = + ; n = ; n1 ⋅ sen θ1 =n2 ⋅ sen θ2
O P f P P' v
b) 30,6 d) 36,4
a) Com base no esquema do olho abaixo, desenhe a imagem do
19. (UNIFESP) Uma das lentes dos óculos de uma pessoa tem objeto (seta) formada em um olho hipermetrope.
convergência +2,0 di. Sabendo que a distância mínima de visão
distinta de um olho normal é 0,25 m, pode-se supor que o defeito
de visão de um dos olhos dessa pessoa é
a) hipermetropia, e a distância mínima de visão distinta desse
olho é 40 cm.
b) miopia, e a distância máxima de visão distinta desse olho é 20
cm.
c) hipermetropia, e a distância mínima de visão distinta desse
olho é 50 cm. b) Que tipo de lente esférica corrige o defeito da hipermetropia
d) miopia, e a distância máxima de visão distinta desse olho é 10 e que fenômeno óptico explica o funcionamento de uma lente
cm. esférica?

e) hipermetropia, e a distância mínima de visão distinta desse c) Considere uma pessoa hipermetrope capaz de enxergar
olho é 80 cm. nitidamente quando seu ponto próximo é de 1,0 m. Nesse caso,
qual a vergência da lente corretiva para conjugar a imagem de
um objeto no ponto próximo, se esse objeto estiver a 25 cm
20. (FGV) Em plena aula, o professor de Física descobriu
do olho?
acertadamente o motivo pelo qual um de seus alunos tinha que
usar óculos. De posse dos óculos desse aluno, verificou que ambas
as lentes possuíam bordos mais espessos que seus centros. Em 03. (UERJ) Um jovem com visão perfeita observa um inseto
seguida, olhando através de cada lente e voltando sua atenção a pousado sobre uma parede na altura de seus olhos. A distância
um friso horizontal na parede, girou-as paralelamente à parede, entre os olhos e o inseto é de 3 metros.
constatando que para ambas, o friso visto através das lentes, não a) Considere que o inseto tenha 3 mm de tamanho e que a
sofria qualquer inclinação. distância entre a córnea e a retina, onde se forma a imagem,
Naturalmente, as lentes em questão eram é igual a 20 mm.

a) cilíndricas e convergentes. b) Determine o tamanho da imagem do inseto.

b) cilíndricas e divergentes.
c) esféricas e convergentes.
d) esféricas e divergentes.
e) parabólicas e convergentes.

304 PROENEM.COM.BR PRÉ-VESTIBULAR


32 INSTRUMENTOS ÓPTICOS E ÓPTICA DA VISÃO FÍSICA II

04. (UNIFESP) Um paciente, que já apresentava problemas de


miopia e astigmatismo, retornou ao oftalmologista para o ajuste GABARITO
das lentes de seus óculos. A figura a seguir retrata a nova receita EXERCÍCIOS PROPOSTOS
emitida pelo médico. 01. B 05. D 09. B 13. A 17. A
02. E 06. D 10. C 14. E 18. C
Nome: Jorge Frederico de Azevedo 03. A 07. D 11. D 15. B 19. C
04. B 08. B 12. B 16. A 20. D
GRAU Esférico Cilíndrico Eixo D. P.
EXERCÍCIOS DE APROFUNDAMENTO

Para OD - 3,00 - 0,75 150º 62,0 01. p2 = 0,1 · p1


longe OE - 3,00 - 0,75 150º mm 02. a) Abaixo a construção da imagem para um olho hipermetrope.

Para OD + 1,00 - 0,75 68,0


perto OE + 1,00 - 0,75 mm

Obs: Óculos para longe e perto separados. Ao pegar seus


óculos é conveniente trazê-los para conferir.
b) Para solucionar o problema do olho hipermetrope devemos usar uma lente convergente.
Próxima consulta:___. 08. 2012. O fenômeno óptico que explica o funcionamento das lentes esféricas é a refração.
c) A distância focal (f) corretiva par o olho hipermetrope é dada pela equação de Gauss,
São Paulo, 30.08.2011. onde a distância da imagem (p’) deve ser de 25 cm (ponto próximo do olho normal) e a
Carlos Figueiredo distância do objeto (p) deve ser de -1 m (ponto próximo do hipermetrope). A vergência (V)
CRM nº 000 00 da lente corretiva é o inverso da distância focal.
1 1 1 1 1 1 1
= + ⇒ = − ∴ = V = 3 di
f P P' f 0,25 m 1m f
a) Caracterize a lente indicada para correção de miopia, identifi- 1
cando a vergência, em dioptrias, e a distância focal, em metros. =f = m 33,3 cm
3
b) No diagrama I, esboce a formação da imagem para um 03. 0,02 mm
paciente portador de miopia e, no diagrama II, a sua correção, 04. a) A correção da miopia é feita com lente divergente que tem vergência (V) negativa.
utilizando-se a lente apropriada. Assim, da tabela dada: V = -3,00 di. Distância focal: -0,33 m
b) Como o olho do míope é alongado, a imagem forma-se antes da retina. Se o objeto está
distante, ele é impróprio, enviando para os olhos um feixe cilíndrico.

05. a) divergente e -1 di
b) 223881 km/s

ANOTAÇÕES

05. (UERJ) Considere uma pessoa míope que só consiga focalizar


objetos situados a, no máximo, 1,0 m de distância de seus olhos.
Determine:
a) o tipo e a dioptria da lente necessária para corrigir esta miopia;
b) a velocidade de propagação da luz no interior do olho, na região
que contém a substância denominada humor vítreo.
Dados: velocidade da luz no vácuo = 300000 km/s e índice de
refração do humor vítreo = 1,34

PRÉ-VESTIBULAR PROENEM.COM.BR 305


FÍSICA II 32 INSTRUMENTOS ÓPTICOS E ÓPTICA DA VISÃO

ANOTAÇÕES

306 PROENEM.COM.BR PRÉ-VESTIBULAR

Você também pode gostar